Sie sind auf Seite 1von 482

Selected Problems on Physics

c.n. MllCBBKOB, T .H. Ocaeoaa

noC06HE no llJH3HKE

1Bwc1uaR mKOJiat MocKBa


S.P. Myasnikov, T.N.Osanova

Selected
Problems
on Physics

Mir Publishers Moscow


Tnmslated from the Ruseiau
hy Natalia Wadhwa

Prlntld ln el11 Union o/ Sol1kt Soclalut Rrpubltc1

ISBN 5-03..001538-7 @ HIAaTeJIJoCTBO cBaemu mRoaa•, 1976


@ ~Ttal>CUO cBaemu mRonat, 1988,
CBalRB88BJlllB
@ English translalion, N. Wadhwa, 1990
CONTENTS

Pntface t.o the Fifth RUlliaD Edition • • • • . • . • •

latroduction
t. The Role of Physica in Compreheadillg Material
World .....•.•• , .••••...• ,
~=.m:.tic:l 'V:=t~::l~~)'l~Cl0
••
2. : :

Elemeata of Diflerent.ial and Integral Calculus


Procedure for Solviog Problems . • • • • • • • •
: : : : :

12
15

t.t. . 11
26
20
y v.cyioi R~tiiiDeâr. Moüoi.. ·. ·. ·. : : 21
41
pas of ~~~ie' Mo\Min' ." ." .' .' ." ." : : : : '3
Ezerciaes ....... · · · · · • · · • • 54
Motion of a Point. Mau in a Circle. Rotational
Motion of a Rigid Body . . . • • . • • • • • • 55

t.2.
Exercises • . . . . . • • . . . . . . • . . •
Quest.iom for Revision • • . . . • , . . . • •
D~mica •.•••••••••..••.••
t=.!~C::!:,:~,~Bo'ty(P!f.Pz:.•;tca. ~ . ~ec:u:
...
83
83

Exerciaes •.•......••..••..• 83
Application of tbe La.ws of Dynamica to the Mo-
tion of a Body (Point Mau) in a Cirde . . • • . Mi
Ezercises • . . ............ . 91
:ri&~i:uº.ª .º'. ~·s.ic. ~~ª _ºr_ ~~·~1~-~ ~~~ 92
Ezerciaes ...........• , • • . . . 101
Momentum of a Body. Momentum Comervation
Law . . . • . . . 102
Exerciaes . . . . . • . . . • . . • . . • . . 107
QuestiODB for Revition . . . . . • . • • . . • 108
1.3. Work, Power, and Energy. Energy Comervation
Law . . . . . . . . . . . . • . . • . . • • 108
Ezercises . . . . . . • . . • . . . • . • . . 131
1.4.U:ri!!oDS•~·R·~i~O~:::::::::::: a~
EierCiees • • . . . . • . . • . . • • . • • 148
1.5. a;t~DSanlor ~V:.:~i! ::::::::::: ~::
E:r.ercisee • • • . . • • • • . • • • • • • • • 156
QuesUODB for Reviaion • . • • • • • • • • • • 157
Contenta

Chapter 2.::,~ and

2.t. ~:~ ~r-&~k' .~ .M~~~r .T~~


EurcillllS
: : :
, ...• , , , ••••......
i=
175
2.2. ~;.e;.i::
Eurcisea
':-, RG=~ : : : : : : : : : : : :
, . . . . . . . , .. , • , • • . .
1~
189
2.3. i:s~iy,u:n.:0by ~ªG~~H~i EaKiDeS : : : : : : 1~
Exercil88 . . . . . . . . . . • . • . . • . . 198
a.~:.~= ':~d ªü=':~ted. V.Po~r.' HÜmtditY 1:
2.4.
~=::. f~r-~leViaioD:: : : : : : : : : : '.
0

2.5. P'roperties of Solida and Liqulds . . • , , • , .


Tbermal Expansion of Soliils an.d Liquida • . .
=
207
207
Exerci.ses , • • . . . . . . . . . . . , . • . 213
Surface Tenaion. in Liquida. Capillary Pbeaomena 21'
Exercil88 • , . , • . . . . . . . . . . . . . 220
Deformation of Solida. Hooke'a Law . • . . . . 221
Exercbes • • . . . 224
QuestiODS for Reviaioa . . 225
Cbapter 3. Eleclrlelty
3.1. Electroat.atica . . . . . . • • • • . . . • . . 226
Coulomb'a Law. Electrost.atic Field Strength 228
Exerciaes • . . • . . • • • . . . . . . . . . 2'3
Electroatatic Fleld Poteat.ial. Work Don.e ia Mov-
iDg a Charge ia aa Electroatatic Field . . • . . 244
Exercbes • . . . • • • . . . . . . • . . . . 258
g:=::i~~J~fE~~~~ief:1.d~~r: 255
Eurciaes • . . . . . . . . . . . . . . • . . 267
Queationa for Reviaion. . . . . . • • • • • • • 268
3.2. Direct Curreat . . . . . . . . . . . . . . • 268
CUrreat ia Met.als , . • . . . . . . . . . • • 268
Exerci.ses • . . . . • . . . • . . . . . . • . 287
Work aad Power of Curreat. Tbermal Effect of
Curreat . . . . . . . . . . . • • . . • • . 289
Ezerei.ses • • . . • • . . • . . . . • • • • • 297
Curreat iD. Electrolytes an.d Gaaes . . • • . • • 298
Eurciaes . . . . . . . . . • • • . . • • • . 806
Questiona for Reviaion. . . . . • • • • , • • . 807
3.3. Electromagnetiam • . • . , . • • • . . • . . 807
Magaetlc Field of a Curreat. Forces Act~ in a
:T:t;.F;f!:ic ªc:f=.~rr:~, i!:p 307
Exercises . . • • . • • • . • • • • • • . • • 324
Coatonle

Work Done in Moving a Current..carrying Con-


t:~!:~io~~ Ja,!~fc°'~~:ld F:1n!~YE~~~~~tic 325
Ehl'Cisee . . . . . . . . . • . . . . . . . 339
Questiom for Reviaion . . . • . • . . . • . M1

Chapter '· Optlm


,,1. Baaic Quantities and Laws in Photometry . . • 342
ExerciSElll • . . . . • . . • . • • • • • • • • 351
8!.estiom for Revision . . . . . . . . . • . . 352
'· 2· ExercisM
aen!:~:'!1 L~t~:Mir~~ : : : : : : '. : : :
................ .
352
352
367
Refraction o( Light. Thin Le1198S. Optical In-
stnunenta . . . . . . . . . • . • . . . . . 368
Exercisfll ....... · · · · · · · · · · 394.
4.3. 3=~= J:p!~ie!:i:r i.iihi : : : : : : : : : ~J
EnrciSM . . . . . . . . . . . . . . . . 404.
Questiom for Revision . . . . . . . . 405

Chapter 5. OsclllaUoD& and Wavee


5.t. Mechanic:al Vibrations and Waves . . . . 406
ExercisM . . . . . . • . . . . . . . . 424
Questiom for Revision • . . . . • . . . 425
5.2. Electromagnetic OscillatiODS and Waves . 426
Exercisee . . . . . . . . . . . . . . . . '38
QuestiODS for Revilion . . '39
5.3. Wave Propertiee of Light . . . . . . . . . . . 440
Exerci888 . . , . . . . 450
Questiom for Revision . '5t
Chap&er 6. Struetuze oi Atoms anel Atomle Nuelei
6. t. Struct.ure of Atoms . . . . . , '52
Exerci888 . . . . . . . , . . . , . . . . . . 456
6.2. ~=-= :rA=::: NuC1ei : : : : : : : : : :
0
Enrciaee , . . . . • • . • • . . . , , • . .
~i
463
Questiom for Revision . . . . . • . . . • . 4.64
Appendlees . • . . • • . • . . • . . • • • • 465
AD9Wt1'8 to Problema • • • . • • , • • , • • , • • • • 473
PREFACE TO THE FIFTH llUSSIAN ElllTION

The maio purpose of the book is to help those preparing


for entrance examinations to engineering colleges in
revising the high-school physics course and in further
studies at the college.
The fourth edition of the book came out in 1981.
Amendments to the physics curriculum at the high-school
and polytechnic levei have been incorporated as well as
extra material on other hranches of the physics course.
The lifth edition was prepared by taking into account the
modified style of problems set at the entrance examina-
tions.
Each seetion begins with a brief description of the ba-
sic theory, physical laws, and formulas. This is followed
by worked problems and a few descriptive problems. Exer-
cises and questions for revision are given at the end of
each section. The problems are solved according to the
unified and optimal approach described in the introduc-
tion. By solving the problems, students will acquire
a firm theoretical background and knowledge which will
helpthemintheirworkin whichever sector of the econo-
my they will be empoyed. The appendices contain tables
required for solving problems, SI units of physical quan-
tities, and the rules for approximate calculations.
ln addition to the prohlems composed by the authors,
this hook also includes a selection of problems set for
the aptitude tests and entrance examinations in physics
at the N .E. Bauman Higher Technical School and other
techDical institutions in Moscow.
lntended for students of preparatory courses at engi-
neering colleges, this book can also he used hy high-
school students, students of intermediate colleges, and
those interested in self-education.
The author is indebted to Prof. A.N. Remizov and
Asst. Prof. N .V. Tygliyan for their enormous help in
preparing the manuscript for publication.
T.N. Osanova
IMTRODUCTIOM

t. The Role oi Physlcs


in Comprehendlng Material World
Phyaica ia one of lhe fundamental natural scienees. lt
studies tbe regularitiea of the most general forms of
motion of matter.
According to Lenin, matter is a philosophieal category
to denote tbe ohjeetive reality, whicb ia given to man in
bis sensations and wbich is reDected, pbotograpbed, and
depicted by our perception, existing independent of it.
Matter exists only in motion. The physieal forma of
motion of matter include mechanical, molecular, electro-
magnetic, and nuclear motion. Moving matter exists in
space and time. Hence apace and time are forma of exis-
tenee of matter.
The world of moving matter can he perceived. Physics
plays a major role in the process of investigation of moving
matter. The study of physics forma lhe basis of dialectic
and materialistic views and helps develop the produetive
ability of soeiety.
Physies is an experimental science. All studies of
pbysical laws start with an experiment and the laws are
eon&rm.ed (or disproved) by experiments. Tbe results of
new experiments re&ne tbe pbysieal laws or de&ne lhe
limits of their applicability.
Pbysics is an exaet science employiog mathematical
apparatus.
2. Mathemllllcal Apparatus
of Physlcs
ELEMENTS OF VECTOR ANALYSIS

Pbysical quaotities cao be either Kalars or veeton.


Sealar qunnUties (acatara) are cbaraeterized ooly by their
oumerical values. Examples of scalar quantities are time
10 Selected Problema ou Phyaics

t, mass m, temperature T, electric charge q, and potential


q>. Scalars can be positive or negative and are added
algebraically.
Enm.ple t. Determine the total charge of a system
consisting of q1 = 2 nC, q1 = -7 nC, and q1 = 3 nC.
The total charge is q = q1 + q1 + q1 = (+ 2 X
10_, - 7 X 10-• + 3 X 10-9) C = -2 nC.
Vector quantitifS (vectors) are characterized both by
a numerical value and a direction. Examples of vectors
are velocity v, acceleration a, force F, momentum mv,
electric fi.eld strength E, magnetic induction B, and
magnetic iield strength B. Vector quantities are added
geometrically.
Eu.mple 2. Com pose two forces F 1 = 3 N and F1 =
4 N if vectors F1 and F 1 form angles of 100 and 40°
respectively with the X-axis (Fig. t).
The compoaition of vectors is symbolically written as
F=F, +F,.
The result of composition of these forces is a force F
called the resultant. The vector F is directed along the
diagonal of the parallelogram formed by vectors F1 and F1
as its sidea, its magnitude being equal to the length of
this diagonal (see Fig. 1). The magnitude of vector F
can be determined from the coaine law:
F=l'F:+F:+2F1F2 cos(e1t-«1)
=V3'+4'+2x3x4cos(40' IOº)N
<><6.8 N.

V~•---, « ,,.,,"
F,
F .:tz_----
y
o «
:
IA
a

O a., X ª.z X
Fig. 1 Fig. 2
Introdueüon 11

The angle formed by vector F and the X-axis can be found


from tbe formula

a=arctan ~::::t~:~:~ ,
a=arctan ~~~:~t!~~:~ arctan0.51 ~0.47 rad.
The projections of vector a on the X- and Y -axes of a
rectangular coordinate system are a~ = a cos a and a11 =
a sina, wbere a is the angle formed by vector a and the
X-axis. Knowing tbe projections of a vector, we can
determine its magnitude and the angle witb the X-axis
(Fig. 2):
a=Va:+~. a=arctan(a/a~).
Multiplying a vector A by a positive scalar k, we ob-
tain a new vector kA whose direction coincides witb that
of vector A and whose numerical value differs from tbat
of A by a factor of k.
Example 3. Determine the momentum of a body of
mass 2 kg, moving at a velocity of 5 m/s.
The momentum of tbe body mv = 2 x 5 kg· m/s =
10 kg·m/s is directed along tbe velocity vector v
(Fig. 3).
Multiplying a vector A by a negative scalar k, we ob-
tain a new vector kA whose direction is opposite to
tbat of vector A and whose numerical value differs from
that of A by a factor of k.
Example 4. A charge q = -7 .5 nC is in an electric
fi.eld of strength E = 400 Vim. Determine the magnitude
and direction of the force acting on the charge.
By definition, the force F = qE. Since the charge is
negative, the force vector is directed against the vec-
tor E (Fig. 4). Tbe magnitude of the force F = 1 q IE =
7.5 X 10-• C X 400 Vim~ 3 X 10-• N ~ 3 ~N.

......---.
1
V

1
..---E
•'~~~'~~_.mu-q~Eo-~~~~-'
Fig. 3 Fig. 4
12 Seleeted Problema on Physics

The sealar product oi veetors A and B is a scalar equal


to the produet of the numerical values of vectors A and B,
multiplied by the cosine of the angle between them:
C = AB cosa.
ln symbolic form, a scalar product is written as
C=A·B.
Eu.mple 5. Determine the work done by a constant
force F = 20 N if the displacement of the body is 1 =
7.5 m and t.he angle a between t.he force and displaee-
ment is t20".
By deftnit.ion, the work done by a force is equal to
the scalar product of the force and displacement:
A = Fs = Fs eos a. = 20 X 7.5 cos 1200 J = -150 X
(1/2) J = -75 J.
The veetor product of veetors A and B is delined as
a vector e whose magnitude is equal to the product of
the numerical values of A and B, multiplied by the sine
of t.he angle between them:
C = ABsina.
The vector C is normal to the plane containing the vec-
tors A and B, its direction being connected with the
directions of vect.ors A and B through t.he right.-hand
screw rule (Fig. 5).
The symbolic form of a vect.or product is
C =A X B.

ELEMEl'\TS OF DIF'FERENTIA.L
AND INTEGRAL CALCULUS
Let. a function f (x) exist. in a certain domain x (Fig. 6).
We shall use the not.ation 11.x = x1 - x, l1.f (x) = f (x1) -
/ (x). The expression

l!~ A~~) =/'(x)=~


is called t.he fint derlvaUve of t.he function f (x) wit.h
respect t.o x.
Iatroduction 13

ct ;---7
l&_/
Fig. 5
A.
1=?t. O
1 1

.ro;
Fig. 6
X

Graphically, df (z)/dz = tan ci, where a. is the angle


of inclination of the tangent to the cwve at point A
(see Fig. 6).
The derivative of the first derivative of a function is
known as the seeoncl derlvative and is denoted by
d'/ (z)/lb;'.
Properties of Derivattves
1. lf f (z) = C<p (z), wbere C = eonst, then
dtt> =C dt>l
2. lf f (z) = <p, (z) + <p2 (z), then
~= dqi;:z) + d~z(:) .
3. lf f (z) = <p, (z) <p, (z). tben
d~~:i:) = d~~(z) 'i?z (z) + d'~:) 'Pt (z).

4. li f (z) = ::e~) , then

d/(:) - ~fP:t.(:a:)-~qi.(:)
di'"""- qil(z)
S. lf f (z) = f (IP (z)), then
d~~~) = d~z) d~>) •
Selected Problema on Phyaies

The values of derivativea of some elementary functions


are giveo io Table t of Appendices.
lf d~;i;) > O, lhe function f (z) increases with ::r;.
If d!:) <O, the function / (z) decreases with increas-
ing x.
If d~~) =O, tbe function f (x) has ao extreme (min-
imum or maximum) value.
Let a function f (z) exist in a certain range of z
(Fig. 7). We divide the range ab of variation of z into

"

Fig. 7

infinitesimal segments Ax,, Az1 , 1 •• ., âz,. ... , âxn and


compose the sum :f / (z
·-·.
lim ~ f(z 1) llz 1 =
1)
.
âx1• The expression

1.l(z) dz
....
"':!..:.º
is called a definlte Integral.
Ao expression of lhe type ~ f (z) dz is called an in-
definlte Integral.
Propertles o/ / ntegrals

i;
t. li f(z)= ,_, f,(z)dz, tben

1~ /,(z)dz= ~ l t (z)dz,
i-=l i-t
1
lntroduct.ioD.
••
2. If f (•) - aq> (z), then
1 aq> (<) d• - a 1 q> (<) dz.
The values of integrais of some elementary functions are
given in Table 2 of Appendices.

Procedure for Solvlng Problems


1.. Read the formulation of a problem. lndicate the
physical phenomena and processes involved.
2. Recollect the deiinitions of physical quantities char-
acterizing these phenomena as well as the properties
of bodiea involved in them.
3. Recall the physical laws describing the phenomena
encoun tered in the problem. >
4. Clarify the physical meaning of the quantities char-
acterizing the phenomena and processes mentioned in
the problem.
5. Write down the given quantities (in 'SI units) and
required quantities on the left-hand side.
6. Draw a diagram (plot, figure) of the problem accord-
ing to adopted roles, taking into account the conditions
of the problem.
7. Write the required physical Iaws and de6.nitions of
the physical quantities in analytic form, taking into
account the conditions of the problem.
8. Write analytically the relations expressing the
physical meaning of additional conditions specifying the
phenomena mentioned in the problem.
9. Solve the obtained system of equations in general
form to determine the required quantities.
10. Check the dimensions of the quantities in the ob-
tained formula.
11. Calculate the values of the required quantities,
using the rules for approximate calculations (see Appen-
dices).
Chapter 1
MECHANICS

1.1. Klnemllllcs
Kinematics stud.ies various forms of mechanical motion
of bodies without taking into account their causes.

UNIFORM RECTILINEAR HOTION


Ualform. recUlloear motion is a motion in which
a point mass (body) covers equal distances in equal intef"'
vais of time.
The dlsplaeement of a point mass (body) is a vector
connecting its initial and final positions.
Any motion obeys a law defining the position of a body
in space at a given instant. ln order to describe a motion,
we introduce a Cartesian system of coordinates XOY.
Then the position of the body will be determined by its
z- and y-coordinates. Consequently, the law of motion
must specify the dependence of x- and y-coordinates on
time t. lt can be determined by calculating the projec-
tions of the displacement vector on the coordinate axes.
For a uniform. motion along the X-axis, the projeetion
of the displacement of the body on the X-axis is

and the equation of motion has the form


X= Xo + V:ct, (1)
where Xo is the coordinate of the body at the initial in-
stant, V:c the projeetion of the velocity vector on the
X-axis, and t the time of motion of the body.
Ch. t. Mechanics 17

tenif:m~ ~~~fon&he~i~~:c;_~ =-:~~: (t) are e.


Figure 8 shows the displacement graph. lt can be
aeen that the velocity
11= s1/t1 = tan(Z.
The velocity graph is shown in Fig. 9. lt follows from
the graph that the displacement • 1 = 11it1 is numerically
equal to the area of the rect.angle OABC.

-~----Ai
o
..

Fig. 8
1
,,.
u Fig. 9
1
e

t. Two bodies start to move in the same direction si-


multaneously from two pointsA and B separated by 90 m.
The body starting from point A has a velocity of 5 m/s,
while the body starting from point B has a velocity of
2 mls. How long will it take the first body to catch up
with tbe second? What will be the displacement of each
body? Solve the problem analytically and graphically.

C X

Fig. to

2-0170
te Select.ed Problema on Physics

GWen: z-01=90 m, 111=5 m/s, V1=2 mfs.


•1 ?1ii-? ?'1
Solution. 1. We choose as the origin a point A of the
X-axis and direct the axis along the motion of the body
(Fig. tO). Then the equations of motion for the bodies
will be written as follows:
Z1 = V1t, Zo1 = º· Z1 = Zot + v.t, (t)
where z1 and z1 are the coordinates of the 6.rst and second
bodies. At point C where the 6.rst body catches up with the
second, zt = z 1 and t = t 1 . Using Eqs. (1), we can write
(2)

where t1 is the time of motion of the bodies to point e.


From this equation, we determine the time of motion of
the bodies:

t1 = 5 ~2 s-30 s.
Then t.he displacement.s of the bodies are given by
Si = zt - Zo1 = V1t11

S1 = 3:1 - Zo1 = V1f11


s1 = 5 x 30 m = 150 m,
• 1 =2x30m=60m.

2. We plot. the time t of motion on the abseissa axis and


the values of t.be a:-coordinate on the ordinate axis to
a cert.ain scale. Let. us write t.he equations of motion for
the bodies:

Then the time dependence of the coordinates can be rep-


resented by st.raight. lines 1 and 2 (Fig. tt). Let us deter-
mine t.he coordinates of the point C of their intersection:
Ch. t. MechHiCS
••
iake33i: :~~ b~d; âb =:!!ºc:;hc:;se:i:n!~· !:e;~~
The displacementl of the bodiea are •u: = Zi = 150 m
and 1111 - .z1 - :r11 = 60 m.
2. A htgh-speed lift in a skyscraper moves upwards
untformly at a velocity of 3 m/s. Plot tbe displacement
graph and use it to determine the time in which the lift
reaches a helght oi 90 m (26th Door).
GiVt'n: 11=3 m/1, N=9D m.
'
Solutlon. Let the Y-axis coincide with the direction of
motion of the lift, and tbe origin with the position of tbe

t,.
Fig. H

lift at the initial instant. Then 11o = O and 11 = vt.


Consequently, 1 = 11 - 11o = vt. ln order to plot the dis-
placement graph, we plot the time t tin the absciSBa axis
and the displacement s on the ordinate axis (Fig. 12).
The time dependence of the displacement will be presented
by the straigbt line OA whose alope is numerically equal
to the velocity v. Using the graph, we 6.nd that it will
take the lift 30 a to reach the height of 90 m.
3. Using tbe displacement graph (Fig. 13a), plot the
velocity graph and determine tbe nature of motion of the
body relative to the X-axis (Fig. 13b).
Answer. Figure 13a shows that tbe body moves uniform·
ly in the negative directlon of the X-axis since the
20 Selected Prohlem.1 OD Pbyaica

Q X
(6)

:L---,
30
Fig. 12
40 1, 9 1.· Fig. 13

z-projection
increaaes
•z of the displacement vector is negative and
in magnitude in direet proportion to time
(Fig. t3c).
EXERCISES
4.. A motor car moving uniformly at a velocity of
12 mls covers the same distance in 10 s as mother motor
car does in t5 s. What is the velocity of the second motor
car?
5. A hiker started from a point lying 2 km to the east
and t km to the north of a crosaing, took an hour to walk
5 km eastwards at an angle of 135°. Determine the 6.nal
position of the hiker.
6. Using the rectangularsystem of coordinates, plot the
displacement vector directed at 45º north-eutwards from
a point lying 1 km to the eut and 2 km to the north of
a fork in the road. Determine the coordinates of the tip
of the displacement vector whose magnitude i8 25 km.
Ch. 1. Mecbanics 21

'·"~ ··::i~
JO
zo

fO
O ZO 404• O to ZO~s
Fig. 14 Fig. 15

7. A body h88 moved from a point with coordinatea


x0 = f m and 1/o = 4 m to a point with coordinates
z1 = 5 m and 111 = f m. Determine the magnitude of the
diaplacement vector of the body and its projections on
th8 coordinate axes.
8. Uaing the graph in Fig. f4 1 determine the nature of
motion of a body.
9. Uaing the diaplacement graph in Fig. f5, plot the
veloeity graph. ..

UNIFORHLY VAB.YING RECTILINB.ul llOTION

Unlfomdy Yarylng rectlllnear moUon of a point mass


(hody) is a motion in which its velocity changes by the
same value during equal time intervala. This motion can
be either uniformly accelerated or uniformly decelerated.
ln order to desc.rihe a uniformly varying motion, we
direct the X- (or Y-)uis along the trajectory of the hody.
Then the equation of motion and the formula for the
velocity can be written 88 followa:
z = + z 0 + v0t + at'/2,
v = + v0 + at,
where v0 is the velocity of the hody at the initial instant
and a the acceleration. The plus or minus sign dependa
on the choice of the direction of the X-axis and of vectors
v 0 and a. For freely falling bodies, the acceleration muat
be taken 88 g = 9.8 mia".
The velocity vector of a nonuniform (in particular. of
a uniform.ly varying) motion can also he defined 88 the
22 Selected Problema aa Pby1ic1

6.rst derivative of the displacement vector with respect to


time:
d•
va::di",
Then the .z-projection of the velocity vector is Vz =
dzldt. The aceeleration vector of a varying motion
can be defined as the 6.rst derivative of the velocity
vector with respect to time:
dv
•=dT·
Knowin1 the projections Vz = Vz (t) and tlz = IZz (t)
of velocity and acceleration, we can determine the z..coor-
dinat.e and the .z-projection of the velocity vector as
follows:

:e=f vzdt, vz=raadt.


'1 ..
ln a varyin1 motion, the concept of averqe velocity ia
UB8d.
The average veloelty of a varyin1 motion of a body is
the ratio of ita displacement vector a to t.he time t during
whic.h this displaeement occura:
(•)=Ili•.
For a uniformly varying motion, the average velocity ia
defmed as t.he aritbmet.ic mean of the initial and fanai

...... Fi1. l1
Ch. t. Mechanica 23

velocities:
(v) - (v 0 + v)/2.
The velocity graph for such a motion is shown in Fig. 16.
Jt can be seen that
a= 111~11, =tan~.
and the displacement s is numerically equal to the area
Oabc. The displacement graph for v0 = O has the form
shown in Fig. 17. The velocity of the body at a given
instant t 1 is equal to the slope of the tangent to the graph
at a given point:
v = tan o:.
ln ali problems exeept 84, tOt, and f.f,6, the air resistanee ahould
be negleeted.

10. A motor car starts to move at zero velocity and


passes the first kilometre with an acceleration "1 and the
second with an acceleration a 2 • Its velocity increases by
10 m/s over the ftrst kilometre and by 5 m/s over the
second kilometre. Which acceleration is higher: a 1 or a2?
Determine these accelerations.
Given: s 1 =s2 =103 m, v0 =O, v 1 = 10 m/s, v2 = 15 m/s.
ªz~at?
Solution. Let the X-axis coincide with the direction of
motion of the motor car. We choose the origin at the point
from which the car starts. Let us write the equation of
motion and the formula for the velocity of the car:
x = x0 +
v0 t + at 2!2, v = v0 + at. (1)
For the final point of the first segment, Eqs. (1) have the
form
S1 = X1 - Xo1 = <Jit~/2, V1 = <Jit1•
Solving these equations together, we fi.nd that v: =
.2<Jis1 , whence
at= ~I 1

<Ji= 2 ~º:oa -i}=5X 10-z m/s2..


.. Selected Prohlem1 on PhJtice

For the final point of the second •gm.ent, Eq1. (t) assume
the form
11 = Zs - Z1t1 = Uit1 +
fltt:/2 (lince 1111 = V,,)1
v1 =Vi +a1 t 1 •
Solving the• equations together, we find that V: - V: =
2a.s1 , whence
111-11!
.. =-..;---·
a 1 = 1:~-;Jr -i}-=6.25x tO"'I m/s1 •
Con•quently, a 1 > a,,.
t t. Two cyclists move towarda each other. One of
them, whose velocity is t8 km/h, decelerates uniform.ly
at a rate of 20 cm/s1 , while the other, whOle valocity ia
,.::,. ..!l.
-~Vot:-
O A 8 X
Fig. 18

5.4 km/h, accelerates uniformly at a rate of 0.2 m/s1 •


How long will it take them to meet and what will be the
di.l!lplacements of the two cycliat.s when they meet if the
initia.l •paration between them is 130 m?
Gtwn: Vn = 18 km/h = 5 mls, a,, = 20 cm/s1 =
0.2mls1,
v19 = 5.4 km/h = 1.5 m/s, f1t = 0.2 m/s1 ,
z 01 = 130 m.
1i-?1,-?'f-?
Solutton. We direct the X-uis along the motion of the
first cyclist and choose the point O U which it is at the
moment t = O u the origin (Fig. 18). Then the equationa
of motion for tb.e cyclists will be
x1 = v01 t - a..tª/2 (z01 = O), z 1 = Z. 1 - v.,t - a1 t1 /2.
(1)
At the moment of meeting (t = 't) u point A,
.,=... (2)
Cb. t. Mechanic1 25

Substituting expreasions (t) into (2), wa obtain v01 'f -


4t,"C1/2 = z 01 - V09 "C - tJs"C1/2, whanee

"C= Po1~Pn 1

"C= 5 ~SC:.5 s=20 s,

lt..,.z1-zta=Vo1'f---y-1
•1-r•
1 1 =5X20-~ m=60 m,

'a= l Za-Zoo 1-•,.< + ..!f-,


1 1 =1.5x2o+ 0 • 2 ~3'.)I m=70 m.
12. A body moving with a uniform aeeelaration from
tha state of rest covers 90 cm during tha 6.fth second
following tha OD88t of motion. Determina tha displace-
ment of tha body during tha aavanth second.
Gtven: 1 1 =90 cm=0.9 m.
.. -1
Solutton. Wa direct tha X-azis along tha trajectory of
tha body and choo11 tha origin O at tha point from which
X

º1.: ... ....1 s, : 1

Fig. 19

tha body start.a to mova (Fig. 19). According to tha aqua-


tions of motion, we than hava z, = a1:12 and z1 = a1:12,
whare t41 = 4 s and '5 =- 5 s. Con11quanlly, tha displaca-
mant of the body during tha 6.fth second is '5 = z1 -
z, = a (t: - t:)/2, whenee
(1)
26 Selected Problems OD Pbyaica

Similarly, s, = z, - .z't = a(~ - t:)/2, where t7 ""' 7 s and


t, = 6 s. Taking Eq. (t) into accouot, we obtaio

1,= ..,~'!--;;I)= ,,~+~:~~'~:» 1

,, = º·~J~~>"i.r'.-~t m = 1.3 m.
130. The equatioo of motioo for a body haa the form
z = t5t + 0.4t1 . Determine the ioitial velocity and
aeceleratioo of the body, aod also its coordioate aod
velocity io 5 a.
Glvtn: t=5 s.
a-? v0 -?.z-? v-?
Solu.tton. Meth.od 1. Let us compare the giveo equatioo
of motioo for the body with the equatioo of motion io
general form:
+
z = z 0 + v0 t at1/2, z = 15t 0.4t1 • + (1)
Obviously, z 0 =O, aod the coeflieieots of t aod t1 are
u0 = 15 mls aod a/2 = 0.4 m/a1 , whenee a= 0.8 mla1 •
The coordioate of the body io 5 s cao be determioed from
Eq. (1):
~ = (15 X 5 + 0.4 X 51) m = 85 m.
Tbe velocity in 5 a can be determioed from the formula
v=v,+at,
v = (15 + 0.8 X 5) mia = 19 mia.
Method a. The coordinate z at t = 5 a can be found
from Eq. (1). By definition, the velocity is
·--*-f. (151+0.41') =15+0.St,
•=(15+0.8x 5) m/1=19 mia,
and the acceleration is
•=1f=f,.(15+0.81)=0,8 mia'.
14. A load is dropped from a helicopter Oyiog at an
altitude of 300 m. How long will it take the load to reach
Ch. t. Mecbanic1 27

the ground tf the heltcopter (1) is stationary, (2) descendi


at a velocity of 5 m/s, (3) ascends at a velocity of 5 m/s?
Givtn: 110 =300 m, v0 =5 m/s.
t-1
Solutton. We direct the Y-8.xi8 vertically downwards
and choose the origin O at an altitude 11o above the ground
(Fig. 20).
1. If the helicopter is stationary, the O
equation of motion for the load has the ~
form
V - fl'/2. (1)
When the load touches the ground (t = t 1 , ~y '
11=110 ), Eq. (1) assumes the form y0 =gt:12,
whence the time of fall of the load to the
ground ia
t,-JI~.
1/2X!ilÕ Fig. 20
'·=,, 9 r 8=7.8 8.
2. Since the load descended with the helicopter at
a velocity 1.10 before being dropped, the equation of mo-
tion for the load haa tbe form
V - v,t +
fl'/2. (2)
Wben the load touchea tbe ground (t = t 1 , 11 = y0 ),
Eq. (2) assumes the form. 110 = v0 t 1 + ff:/2, wbence
t: + 2vot1/1 - 2110 !1 - O.
Solving tbi8 equation for t 1 , we obtain
-1.1,±V~
t,

t, - 5±

V5':;xe.ax3íiô s!>!(-0.5±7.S)s.

Consequently, t1 !:!1:1: 7.3 8 (the negative root is rejected


aince t> O).
3. Since the load aacended with the helieopter at a ve-
locity v0 before being dropped, the equation of motion
28 Selected Problema on Pbyaica

for the load has the form.


y - -v,I + fl'/2. (3)
When the load touchea the ground (1 - 1., y - y 0), Eq. (3)
aaaumea the form 1/o = -v0 t1 +
rt:/2, whence
t:- 2";·-~·=o.
Solviag this equation for t1 , we obtain

t3 11o±YF°
t1 5 ±1'5•:-:.; 9.sxS()J s~(0.5±7.8)1.
Rejeeting the negative root, we obtain t 1 ~ 8.3 s.
15. A load is thrown verlically upwards from a bal-
loon ata velocity of t8 mia relative to the ground, de-
scending ata constant veloeity of 2 mia. Determine the di&-
tance between the balloon and the load at the m~

:!t~i;:w~o.!;t:f1ii~ ~ h/:i1H: f:d\:r,!:


by the balloonl
Giwn: v11 = 2 m/s, vn..,. t8 m/s.
•-? 'f-?
Salutlon, We direet. the Y-axi1 verlically upwards and
cho088 the origin O at the point where the balloon waa at
lhe moment of separation of the load from it (Fig. 2t).
Then the equaUons of moUon for the balloon and the
load will bo
Jll = -Vo1t, fia = V11t - gt•/2. (t)
The veloc.Hy of the load vuiea aceording to the law v =
v., - gl. At tbe higbeat point A oi ite IUIC8Dt, the
veloeity of the load is zero: O = v11 - ftmu:• anà hence
the time of ueent of the load is tmn: = v01/g. The coordi-
nate of the load at point A is

flu=Voitmi:r.-~=Voz "'; -~= t•


;
1TI
Ch. 1............

""!o
...
29

, '
Fig. 21 Fig. 22

During the same time tm1:ii:• the balloon ducends to point


B with the eoordinate 1'1a = -1Jo1tmH = -1Jo1Vot/g.
The 1eparation between points A and B is
•= IABl=I AOI +IOB 1=Ir... l+I p,. 1
=-t-+ ..~.. =-li-<•.. + 2v,,),
•= 2 ,!8u x(t8+2x2) m<><20 m.
At the moment 'I wben the load pa~s by the balloon, the
coordinates of the bodies will be equal: Ih = p1 • Taking
Eqs. (t) into aceount, we ohtain -v01 '1 = v09'1 - gt1/2,
whence
'I= 2(r1e1:"ti>'

'1=2(2ts18) s~4 s.

16. A body is thrown vertically upwards at an initial


velocity 110 , and another body falis freely from a height
z~:1d:ib:d~~ 1}1
:h: :~~d:cem:!.!8tos:r:i·~=u•~
neously.
Gtwn: 110, llo = h.
âp-f(l)-1
30 Selected Problema on Phyaiee

Solution. We direct t.he Y-axis vertically upwards and


chooae the origin O at the ground (Fig. 22). Since the
bodies start to move simultaneously, the time of t.heir
motion is the same, and the equations of motion have the
form. 111 = V01 t - gt8 /2 and 111 = 1/o - gt•/2, where 1/o is
the initial coordinate of t.he second body.
Before one of the bodies falis, the separation between
the bodies at any instant is expressed by fly = y1 - y1 =
1/o - gt2/2 - v01 t +
gt•/2 = Yo - v0 t, or
fly =k-v01t.
17. Two bodiea are thrown verti~ally upwards at the
same initial velocity v0 with a time interval 't'. Determine
the velocity of the second body relative to t.he first. What
law describes their separation?
Given: v0, 1'.
v~-"""'"1~l>l!--~1-
Solutlon. We direct the Y-axis vertically upwards and
choose the origin at the ground at. the point from which
the hodies are t.hrown. For the starting time we take the
moment when the ftrst body is thrown. The equations of
motion for the bodies have the form YJ. = v0 t -gt1 /2
and fls = v0 (t - '1) - g (t - '1)2/2. The separation be-
tween the bodies varies according to the law
liy =Ih - 112 = v 0 t - gt 2/2 - v 0 (t - '1) + g(t- '1)1 /2
= Vo'I + g'l2/2 - g"t.
ln the chosen coordinate system, the velocities of the
bodies vary according to the laws v1 = v0 - gt and v1 =
v0 - g (t - '1). Then the velocity of the second body
relative to the first is
v = v1 - "1 '""' v0 - g (t - '1) - v0 .+ gt = f'I = const.
Therefore, the relative motion of the bodies is uniform.
18. A bali is thrown vertically upwards. It wasobserved
ata height h twice with a time interval /it. Determine
the iniüal velocity of the bali.
Gtven: h, lit.
Vo-?
Ch. t. Mechanics
ª'
Solution. We direct the Y-axis vertically upwards and
choose the origin O at the ground (Fig. 23). The equations
of motion for the bali for instants t and t + lit are
,. - "•' - gt'/2, ,. - "• (! + à!) -g (1 + à!)'/2. (1)

Solving them togeth81', we obtain v0 t - gt1/2 """ v0t +


v0 ât - gt1 /2 - gt lit - g At1 /2, whenee
t - (2v0 - g àt)/(2g). (2)

Substituting expression (2) into (t), we get


h=vo( au.;:At )-f(~)'=4D3-a:•At•,

whenee

19. A body falis freely from a height of 490 m. Deter-


mine the displaeement of the body during the last second of
its descent.
Given: h = 490 m, lit = 1 s.
ày-1
Solutton. We direct the Y-axis vertically downwards
and choose the origin O at a height h above the ground
(Fig. 24.). Then the equation of motion for the body has

Fig. 23 Fig. 24 Fig. 25


.. Selecled. Problema OD Ph7alc1

the form
y = gl'/2. (1)
When the body touches the ground (t = t1 , y = y1 = h),
Eq. (1) becomes h = lfl!/2, whence 1, = V2illg. At lhe
moment (t1 - At), the body is at point A with the coor-
dinate 111 = g (t1 - At) 2/2. Therefore,
i1.u=u.-u.=f11:-c1,-11.t)'1= •:• 121s-11.1>
= ·:· (2 v~-11.t J.
9.Bxt ( 2 1/2x490 t)
...
u.11=-.- r e.s- m= 93
m

20". A ball thrown vertically upwarda falls to the
ground in 3 s. Determine the initial velocity of the bali
and the muimum height of its ueent.
Glven: t 1 = 3 s.
Vo-?Ymu-?
Solutton. We direct the Y-axis vertically upwards and
cho088 the origin O at the ground (Fig. 25). Then the equa-
tion of motion for the bali and the formula for its velocity
will be
11-v0 t-gt212. (1)
V=Vo-lto (2)
AI lhe l!fOUDd, 1 = 1,. y = O, and Eq. (1) becomee O =
v0 t1
- ,i:12, whenee

Vo=~1 1

"o= 9.82x8 "'i-=14.7m/s.


At the highest point A of the ball trajectory, t = tmu:•
S: !,m~• :..~.~ ~~.::t US write Eq. (2) for point A:
tma:r. = v,lg. (3)
The lime of ucent. can be found in a di&ermt way. Since tbe
1-eoordinale at. point A attalna IU muimum nlue, dimar'dr = O.
Ch. t. Mechanics 33

Using Eq. (t), we can write

-/r( v tmu:- ff~ax )=v -ftmu:=0,


··-
d'r,u: = 0 0

tmu:=Vo/f.
Substituting Eq. (3) into (1), we obtain

Ymu=Votmu:-~=vo..!!f- ~ =t,
Y'mu:= 2t;·;~8 m=11 m,
21. A body is thrown vertically upwards at a velocity
of 4.9 m/s. Another body is thrown vertically downwards
at the same initial velocity simultaneously

~rtA
from the maximum heigbt that can be at-
tained by the first body. Determine the
time in which the bodies meet.
Gtven: v0 -4.9 m/11.
<-1
Solution. We direct the Y-axis vertically
upwards and cbo088 the origin O at the
ground (Fig. 26). Then the equations of
motion for the first and second bodies can
be written as
:;l
Fig. 26
Y1=Vo1t -ft'/2, Yt = Yo-Vott-ft'/2,
where Voi = v11 = v0 • At point B wbere they meet
(t = t', y1 = y 2 ), we have
Vot' - 1t"'/2 = Yo - Vot' - ,-r2/2,
whence t' = yo/{2u0), wbere g0 is the ma:r.imum height
of ascent of tbe first body, y0 = Ymu: = V:1(2f) (988 Prob-
lem 20). Substituting this upression into the formula
for 1', we obtain

t'= Zr~. =-ii-·


1'= 4 ~·:.a s~0.13 11.
3-0870
Selected Problems on Physics

22. A motor car moves the first half of its path at


a "·elocity of 80 km/h and the second half ata velocity of
40 km/h. Determine the average velocity of the car.
Giwn: v1 = 80 km/h~ 22 m/s, v1 =40 km/h~ t t m/s.
(v)-1
Solution. Since for the uniform rectilinear motion the
path length is equal to the magnitude of the displace-
ment vector, we have
(v) = s/t. (1)
Here 8 is the path length of the car, t = t1 + t, = 81lv1 +
s2 1v2 is the total time of motion, where s1 and 8 2 are the
distances covered hy the car moving during a time t 1 at
a velocity v1 , and during a time t 2 at a velocity v2 res-
pectively. By hypothesis, 81 = s2 """' 8/2, and hence
1 = s/(2v,) + s/(2v,). (2)
Suhstituting expression (2) into (t), we obtain

{v) 1/2·111+1/2·11 2 11~~~ '

{v)= 2 ~~;•,m~t4.7 m/s.


23. The cahin of a lift ascends with a constant accelera-
tion for the fi.rst 4 s, attaining a speed of 4 m/s. The cahin
moves at this velocity for 8 s, after which it moves with
a constant deceleration for 3 s. Determine the displace-
ment of the lift cahin and plot the velocity, dieplacement,
and acceleration graphs.
Gtwn: At 1 =4 s, At1 =8 s, ât 3 =3 s, v2 =4 m/s.
k-1
Solutton. We direct the Y-axis vertically upwards and
choose the origin O at the initial position of the cahin.
Let us consider the motion of the cahin on three segments.
The displacement corresponding to the first segment is
given hy
(1)
where {v1 ) is the average velocity corresponding to this
segment. Since the motion is uniformly accelerated, we
Ch. 1. M'echaniee 35

have
(v,) = (v,. + •11.J/2 = (O + v,)/2 = v,12.
Using tbis expressioo, we can write Eq. (t) in tbe form
h.i = v. !it112.
The displacement.s eorrespooding to t.be second and t.bird
segments are given by
h 1 = V 1 !it1 , h 1 = (v1) !it1 •
Sioee (v1 ) = v1 /2, h1 = v1 !it,12. Coosequeot.ly 1 the to-
tal displaeemeot of t.he eabin is
h=h,+h,+h.,
or
h=~+v1 !it 1 + "'•~'• =-f-(!it1 +2lits+lit1),
h=i(4+2x8+3)m=46 m.
We sball plot t.he veloeity, displaeement, and aeeelerat.ion
grapbs for eaeb segment aeparat.ely. The complete veloeity
graph (Fig. 27a) is tbe broken line OBCD. The displaee-
meot graph (Fig. 27b) eoDSists of t.hree regiona: OB' is
a segmeot of parabola wit.b tbe vertei: at poiot O, B'C'
a line segmeot, aod C'D' a segment. of parabola witb the
vertei: at poiot D'. The aeceleratioo grapb (Fig. 27c) is
lhe broken line ABB'CC'D.
24. A train gaiDS speed from 36 to 54 km/h duriog tO s.
Duriog the next 0.3 min, it. moves at. a uniform. veloeity.
Determine tbe displaeemeot. and the average veloeity of
the train, and plot tbe veloeit.y aod displacement gnphs.
Gtven: v0 =36 kmih=10 m/s, v=54 km/h=l5m/s,
!it 1 =f0 s, !it1 =0.3 min=18 s.
(V)-1 r-1
Solutton. We direet tbe X-axis along t.he trajectory of
motion of tbe t.rain aod choose the poiot O at. whieb the
veloeity of the train is v0 as t.be origin. We shall eonsider
the motion of the traio on two segments. The displaee-
ment eorrespondiog to t.he &rst. segmeot is giveo by
11 = (V1) fit1.
3•
36 Selected Problems ou Ph11ics

v, m/s

o
:~ f81216t,s
s, m (a)

lJ,m,{c_•
20
fO A 8

o 8 16 2<- t, s
(a)
s. m

o 8 12 16 t, s
a, rri/s 1 fD)

2
A 6
e
o 12) 1flft.•
-1 L.J
e' D
-2
o 8 16 2f i, s
(C} (h)
Fig. 27 Fig. 28
Ch. t. Mechanics 37

Since t.he t.rain moves on this segment with a uniform


accelerat.ion. its average veloeity is (v1) = (v0 + v)/2.
Tberefore,
(1)
Since the mot.ion on t.he aecond segment is uniform, we
can write
&1 = V ât 1 • (2)
Conaequently, the t.ot.al displacement. of t.he train is&=
s, + s,. or (see Eqs. (1) and (2))
B= º1 ât1+vât1,
17 17

·-( 1 º1 15 10+1sx1s) m=395 m.


The average veloeity of t.he train is
(V)= s/t,
where t = dt1 + dt 1• Therefore,
(V)= At1 ~At,

(v) = 13~i's • ""14.I m/s.


We const.ruct the velocity (Fig. 28a) and displacement
(Fig. 28b) graphs, considering each aegment aeparat.ely.

F
º~e
.D

C7
Fig. 29
E I'

25. Charact.erize t.he motion of a mot.ortyclist. whose


1(

veloeity graph is shown in Fig. 29.


Auwer. The veloeity graph shows that. the mot.orcyçlist
starts t.o move from t.he state of rest. (point. O). 1ts motion
is uniformly accelerated on segment OA, is uniform on
38 Selecl.ed Problems on Phyaics

segment AB, and is uoiformly deeelerated oo segment BC,


the magnitude of acceleration on this segment being larg-
er than tbat on segment OA (tan a 1 < tan a 1 ). Segment
CD corresponds t.o a stop.
Segmenta DE, EF, and
FK of the graph corre-
spond to the motion of the
motorcyclist in the oppo-
t•
1
site direction: uniformly
accelerated (segment
DE), uniform (segment
1 EF), and uniformly de-
a 1 celerated (segment FK).
1 26. Using the velocity
1 grapb shown in Fig. 30a,
h 1 plot the aeceleration aod
1 displacement graphs ..
01--~l~t,~~.......,,~--+,~~~t Answer. On segment
Ot1 , the motion is uni-
: thJ á~e formly accelerated in the
positive direction of the
1 1 X-axis (Fig. 31, segment
r OA of the trajectory).
e, Tberefore, tbe accelera-
1 tion graph bu the form
1
of a straight Une paral-
1
1
lel to the t-axis (Fig. 30b,
r segment bc), and tbe dis-
ql""'~~--IC-z---71,-.,-t ~:~::t:!r~'ti~ :k~;fb
(CJ tbe vertex at point O
Fig. 30 (Fig. 30c). On segment
t 1t 1 , the motionisuniform
(Fig. 31, segment AB of
o A
Fig. 31
8 e L :::e. tr~~:toi2;e1Z!1t~~
grapb is a line segment
coinciding with the t-axis
(Fig. 30b, segment t 1 t 1 ), and the displacemeot graph is
a straight line c:1cfi touching the parabola at point c1
(Fig. 30c). On segment t 1 t 1 , the motion is uniform.ly
Ch. 1. Mechanics 39

decelerated in the same s


direction (Fig. 31, seg-
ment BC of the trajecto-
ry). Tberefore, the accel-
~!;~º:r ~·;~ai:~~ l~~= or---r,--+:-+.----l,~t, t
parallel to the t-axis 1
(Fig. 30b, segment de), :
and the displacement 1
graph is a segment of pa- 1
rabola with the vertex v 1
at point e1 , touching the
straight line c1d1 at point
d, (Fig. 30c).
27. Using the displace- O t--->l'o--+-'k--,--!-
ment graph consisting of
two segments of parabo-
las (Fig. 32a), eonstruct
the veloei ty and accelera-
tion grapbs.
Answer. On segment
Ot1 (Fig. 32a), the motion a 1

~~:~:~~ :;T~~t:h~ :d,' :


~;x~<::·::~js:::~f Of----.~--r-+1 ~-'--
Therefore, the velocity is
negativeand decreases in
1t,
i
1t2
1
1tJ
1
:t•
1
f;

;::~i~~e !~~~3~:~ ':f- ~'1


celeration is positive fcJ
(Fig. 32c, segment b1c1). Fig. 32
On segment t 1t 1 , the mo-
tion is uniformly acceler-
ated in the direction of A ___.. o -- B
tbe X-axis (Fig. 33, seg-
ment AO of the trajec- Fig. 33
tory). Hence the velocity
is positive and increases in magnitude (Fig. 32b, segment.
cd), while tbe acceleration remains the same in magnitude
and direction (Fig. 32c, segment c 1 d~). On segment t 1 t 8 , the
Selected Prohlems on Physics

motion is uniform.ly decelerated in the same direction


(Fig. 33, segment OB of the trajectory). Therefore, the ve-
locity is positive and decreases in magnitude (Fig. 32b,
.z; m segment de), while the accel-
eration is negative (Fig. 32c,
6 segment d;e1 ). On segment
6 t1 t4 , the motion is uniformly
accelerated and directed
+ 1 against the X-axis (Fig. 33,
z : segment BO of the trajectory).
T~erefore, the velocity is neg-
O 111 Jt+ SI+ li+ /, s ative and increases in magni-
tude (Fig. 32b, eegment e/),
v, m/s <ai while the acceleration remains
the same (Fig. 32c, segment

~~~"
eJ,).
28°. A body moves along the
X-axis according to the law
:e = 6 - 3t + 2t*. Determine
the average velocity of the
body and acceleration during
the time interval from t to
-z 4 s. Plot the displacement,
-j 16) velocity, and acceleration
a, m/s"' gr(ft~~: f1 = 1 s, f:1=4 s.
(v)-1 •-?
Solution. By de6nition,
~ v-{r~f,cs-31+21')
•t+ -li+ 4fl 711 (. ~ -3+41, (!)

Fig(~ (v)=~·
Subatituting the values of t 1 and t 2 into Eq. (1), we
(~t3i~ 4•x=4 :~~s ~ ~3 ~Js. ;~se; 1 m/s and v=
1

(v)= 1 +./3 .!i-=7 m/s,


a=i = f,-(-3+4t)=4 m/s2 =const.
Ch. t. Mechanics 41

The displacement graph is a parabola (Fig. 34a), where


z 0 = 6 m is the initial coordinate of the body. The veloci-
ty graph is a straight line (Fig. 34b), where 110 = -3 m/s
is the initial velocity of the body. The acceleration graph
is a straight line parallel to the abscissa axis (Fig. 34c).
29°. The velocity of a body is expressed by the formula
11 = 2.5 + 0.2t. Determine the displacement of the body
20 s after the beginning of motion.
Given: t 1 =O, t 1 = 20 s.
• 1

Solution. By de&nition,

20
1=
..r
11 dt. Then

20
•= ~ (2.s+o.21)dt=(2.s1+f xo.21•) 1
o o
=2.Sx 20 m+{ x 0.2(20)' m-90 m.

EXERCISES

30. A traio moving over a horizontal segment at a ve-


locity of 36 km/h aequires a uniform aeceleration over
a distance of 600 m so that ils velocity becomes 45 km/h at
the end of the segment. Determine the acceleration and
the time of the accelerated motion.
31. Two bodies start to move at the same time from the-
same point and in the same direction. The 6.rst body
movesuniformly ata velocity of 98 m/sand thesecond body
moves with a uniform acceleration of 980 cm/s1 , its ini-
tial velocity being zero. How long will it take the second
body to catch up with the first?
32. A motor car covers 1.2 m during the second second
after the beginning of motion. What is its acceleration?
Determine the displacement of the car during the tenth
second after the beginning of motion.
33. The equation of motion for a body is :r: = 5t + 0.8t1 .
Determine the acceleration and the initial velocity or
the body.
•• Selected Problems on Physics

34, A ball falls from a height of 20 m on a plane surfaee


and bounces to a height of 5 m. What is the velocity of the
ball at the moment it strikes the plane surface? What is
the time taken by the ball to reach the highest point from
the beginning of the fall? What is the velocity of the bali
at the moment when it leaves the plane surface?
35. A body falls from a height of 2000 m. How long will
it take the body to traverse the last 100 m?
36. A train start.s from a station with an acceleration
of 20 cm/s1 , Having attained a velocity of 37 kmfh, it
moves uniformly for 2 min and then brakes over a distance
of 100 m. Determine the average veloeity of the train
and plot the velocity graph. ·
37. A motor car moves at a velocity of 80 km/h for the
first half of the duration of it.s motion and at a velocity
-of 40 km/h for the second half. Determine the average
velocity of the car.
38. A point mass moves in a straight line. Ata distance
of t km from its initial poaition, it stops and then moves
in the oppoaite direction over 1.2 km before it stops.
What are the displacement and the total distance tra-
versed by the point?
39. A motor car moving with a uniform. acceleration
covers two tOO-m long adjacent segments of its path in
5 and 3.5 s respectively. Determine the acceleration and
the average velocity of the car on each segment and on
both segments taken together.
40. A body A is thrown vertically upwards at a veloci-
ty of 20 m/s. At what height will it collidewitha body B,
which wuthrown horizontally at the same time ata veloc-
ity of 4 m/s? The horizontal distance between the initial

Fig. 35 Fig. 38
Ch. t. Mechanics

positioos of the bodies is 4 m. Fiod the time of motioo of


the bodies before the collisioo aod the velocity of each
body at the momeot of collisioo.
41. Usiog the velocity graph showo iD Fig. 35, plot the
displacemeot aod acceleratioo graphs aod describe the
type of motion of the body oo various segmeots.
42. Usiog the displacemeot graph shown io Fig. 36,
plot the velocity and acceleration graphs aod describe
the type of motioo oo each segmeot.
43. A box with a metal bali at the centre, which does
not touch the walls of the box, falis from a certaio height.
Describe the motion of the ball relative to a wall of the
box duriog falling, oeglectiog air resistaoce.

TYPES OF COllPOSITE MOTION


Unlform recUllnear motion. Aoy uoiform motioo at
a coostaot velocity v aloog ao arbitrary straight lioe AB
(Fig. 37) cao be decomposed ioto two iodepeodeot uoi-
form rectilioear motions along the X- aod Y-nes at
ve~~c~t~"!r "io ~:~~L~ this motioo, we choose a Cartesiao
system of coordinates XOY. Theo the equations of mo-
tioo for the body can be writteo io the form
z=z0 +uzt,
Y = Yo + u,t,
where Vz = v coe a aod u, = u sio a.
y
8

J_ ~
si~
X 'l>z
O 8 X
Fig. 37 Fig. 38
Selected Problems on Physics

The velocity of the body at any point of the trajectory


is given by
v=Jlv:+v=
and is directed along the trajectory.
Motion of a body thrown horizontally from a eerlaio
height. This motion can be decomposed into two inde-
pendent motions; a uniform rectilinear motion in the
horizontal direction at a velocity Vz equal to the initial
velocity ve (vz = v0) anda free fall from the initial height
of the body characterized by acceleration I·
ln order to describe this motion, we choose a Cartesiao
system of coordinates XOY and direct the X-axis along
the horizontal and the Y-axis vertically upwards. Then
the equations of motion for the body along the X- and
Y-axes will be
X = Zo + Vot, y = Yo - gt1/2.
The velocity of the body at any point of the trajectory
is given by
i·=Vv~+v:
and is directed along the tangent to the trajectory at the
given point.
Motloo of a body thrown at ao aogle to tbe horizontal.
This motion can be decomposed into two independent
motions: a uniform rectilinear motion in the horizontal
direction at the initial velocity Voz = v0 cos cz and a free
fall at the initial velocity v 01 = v0 sin cz, where cz is the
angle between the directions of the velocity vector v 0
and the X-axis (Fig. 38).
ln order to describe this motion, we choose a Cartesian
system of coordinates XOY and direct the X-axis along
the horizontal and the Y-axis vertically upwards. Theo
the equations of motion will be
X = Zo + Vozt, Y = Yo + v 01 t - gt1/2.
The velocity of the body at each point of the trajectory is
Ch. 1. Mechanica 45

where L':i: = Vu, v, = L'or - gt, and is directed along the


t.angent to the trajectory at the given point.
The tlirection of the velocity vector at an arbitrary
point of the trajectory is determined by theangle qi formed
by the veloeity vector v with the X-axis:
tan qi = v,lv:i:.
Tbe valUl!9 of z 0 and lo and the signs of the terms ou the right-
hand sides of the above equations depend in all three cases on the
clioice of the origin and the directions of the X- and Y-axes.

4.4, A passenger on a train moving at a velocity of


40 km/h sees a 75-m long train passing from the opposite
direction in 3 s. What isthe velocity of the second train?
Given: v1 =40 km/h~11 m/s, t=3 s, 1=75 m.
"• l
Solution. Let us consider the motion of the second train
rela tive to the reference frame conneeted to the 6.rst train.
Then the velocity of the second train relative to the
chosen referente frame is v = v; + v 1 , where v; = -v1 is
the velocity of the ground relative to the 6.rst train and
v 1 the veloeity of the second train relative to the ground.
ln scalar form, v = v~ + v1 = v1 + v1 • Considering that
v = llt, we obtain Ut = v1 + v2 , whence
v2 = llt - vi.
v, = (75/3 - li) m/s = 14 m/s.
45. A boat ftoats across a river at right angles to the
banks ata velocity of 2 m/s. Determine the angle to the
chosen direction of the Y-axis and the velooi ty of the
boat relative to water if the velocity of the Bow is 5 km/h
(see Fig. 38).
Given: v=2 m/s, v1 =5 km/h~1.4 m/s.
a ? V2 ?
SoluUon. Let us eonsider the motion of the boat rela tive
to a bank. Then the velocity of the boat is v = v1 + v1 ,
where v 1 is the translational veloeity due to the river
Dow and v 2 the velocity of the boat relative to water.
•• Selected Problema on Physics

Since, by bypotbesis, v ..L v1 and v 1 = v - v1 (Fig. 39),


we have
v1 =V•:+v' =Yi.41 +2' m/s<><2.4 m/s.
We can write tan a. = v1/v, whence
cz = arctan (v1/v),
" = arctan (1.4/2) "' 0.84 rad.
46. What must. be the velocity and direction of motion
of an aeroplane to traverse 300 km nortbwards in 2 h if
a northwesterly wind blows during the Oight at an angle
of 30° to the meridian at a velocity of 27 km/h?
Glven: t=2 h=7.2x!O's, 1=300km=3xtll'm,
a=30" <><0.52 rad, "1=27 km/h= 7.5 m/s.
V1;-? cp-?
Solution. Let us consider the motion of the plane in the
reference frame fixed to the Earth. We direct the X-axis
eastwards and the Y-a:r.is nortbwards (Fig. 40). Tben the
velocity of the plane in the chosen reference frame is

or (see Fig. 40)


(1)
y

~ O
Fia. 39
'\ X
Fig. 4.0
X
Ch. t. Mechanics

From the cosine law, we calculate the velocity:


Vz= V V:+v2+2v1vcosa ,
where v = l/t. Therefore,
Vz""" VV:+ l2/t 2 +2v1lcosa/t,
_ 1/ 7 sz+( 3xt0' )'+2x7.5xõ.86x3xt0' I
~-r · ux~ ux~ ms
<><48.3 m/s.
Since the projection of the sum is equal t.o the sum of
projections, we determine the projections of Eq. (1) on
the X- and Y-axes:
v1 sincp=v1 sina., v2 coscp=v1 cosa.+v, (2)
whence tan cp= ,,1 ::.:n_::11, , or
cp=aretan ,,1 :!.8~º.;,,,
cp= aretan 1.sxo.86~35:1~~(1.2x tOI)
<><aretan0.078<><0.078 rad.
47, An aeroplane Dies along the horizontal at a veloc-
ity of 360 km/h at an alt.itude of 490 m. When it flies

~ A
Fig. 41
8

over point A (Fig. 41), a load is thrown from it. At what


distance from point A will the load fall t.o t.he ground?
Giwn: v0 =360km/h=100m/s, h=490 m.
•-?
.. Select.ed Problems on Physics

Solution. We direct the X-axis along the horizontal and


the Y-axis upwards alongthe vertical and choose point A
as the origin (see Fig. 41). Let us write the equations of
motion for the load along the X- and Y-axes:
z = v0 t, y = Yo - gt'/2, (1)
where y 0 = h. For point B where the load falls (t = t 1 ,
x = x B• and y = y B = O), Eqs. (1) have the form
x 11 = v0 t 1 , (2)
o= h - gt:12. (3)
The duration of motion of the load to point B can be de-
termined from Eq. (3)
1, = V2hlg. (4l
The required distance s = x11 can be calculated from
Eq. (2) taking Eq. (4) into account:

B-=vJ1=v0 V~,

s=100lf2:.:9° m=tOI m.
4.8. A jet of water is ejected from a hydraulic giant at
a velocity of 50 m/s at an angle of 35° to the horizontal.
Determine the horizontal range of the water jet and the
maximum height the jet can reach.
Gtven: v0 =50 m/s, a=35°::!=:0.61 rad.
h-1•-1
Solution. We choose the coordinate system with the
origin at point O from which water is ejected (Fig. 42) and
write the equations of motion for the water'jet:
X= V 0 xt, (1)
11 = v0 ,.t - gt1/2. (2)
The velocity of the jet varies along the Y-axis according
to the law
(3)
Ch. 1. Mechanics
••

Fig. 42

For point A (the vertex of the parabola), t = tma:r.i 11 = h,


and v, = O. Then Eq. (3) assumes the form O = v0 , -
gtmn.r whence ,
tmn. = vo,Jg. (4)
Using Eq. (2) for point A, we obtain h = v0,tmn. -
gtfu11/2,
or taking Eq. (4) into account,

h=v,,7- ';1. 11 =~.


Comidering that v0 , = v0 sin a., we obtain

h= "1 8!;'" 1

h= oo;~:::7' m~41.3 m.
Let us write Eq. (2) for point B at which the jet falls
to the ground (t = ta, 11 = O, and z = s):
O = v0 ,ta - gtJ,12.
Then the duration of motion of the jet to point B is
given by
ta = 2v0 ,Jg = (2v,lg) sin "· (5)
We write Eq. (1) for point B:
s = V 01:ta· (6)
i-0970
50 Selected Problem1 on Physica

Substituting Eq. (5) into (6) and considering that Vez =


v0 cos a, we obtain
l=Vocosa 2u,~ine1. = 11ls~n2c:i'

s= 5019~:· 94 m ~ 240 m.
49. At what angle to the horizontal must a body be
thrown for its ma:r.imum height of ascent to be equal to
its horizontal range?
Given: h, s.
a-1
Solutton. The height of ascent of the body (see Prob-
lem 68) is
h = V: sin1 a/(2g),
and the horizontal range is
s = (20:/f) sin a cosa.
By hypothesis, h = 1, and hence V: sin1 a/(2g) =
20: sina cos a/g, which gives
tana=4,
a = arctan 4 ~ t.3 rad.
50. A body is thrown at an initial velocity v0 at an
angle a to the horizontal. Determine the veloci ty of
the body at the point of maximum ascent and at the point
of its fali to the horizontal plane.
Given: v0 , a.
~V."--1.----va-1,-
Solutton. Let us construct the trajectory of the body
in the chosen coordinate system (see Fig. 42). At any
point of the trajectory, the total velocity of the body can
v:
be determined from the formula 11 = V + V:. where
v. = 110• = 110 cosa and 11., = 11011 - gt are the horizontal
and vertical velocity components at the given point. At
point A of ma:r.imum ascent, we have
11.=110.=VoCOSa, 11,=0.
Ch. t. Mee.b.anics 51

Consequently, the total veloeity of the body at point A is


vA =Vv:+v;= Vii!= Uz= vocosa.
The velocity vector at point A is obviously directed along
the horizontal. Similarly, for point B at which the body
falis to the ground, we have
Uz = Voz = Uo COS at, u11 = Uo 11 - gtB, (1)
where the duration of motion to point B can be calculated
by the formula (see Problem 48)
'• = (2v,Jg) sin "'· (2)
Substituting the second formula of (1) into (2) and con-
;~~::};f ~:t =º~=-~00 ~~ ~o::!:!i~:t~y~ vt~n t~t-;j
':'.
velocity at point B is
ua = Jf vz+v; =1'v:cos2.a+v!sin2a -v0 •
ln order to determine the direction of the velocity vector
at point B, we shell use Fig. 42 which shows that
tan at1 = v111vz = -v0 sin at/(u 0 cos at) = -tan at,
i.e. the veloeity vector at point B at which the body falis
forms with the horizontal ao angle ati equal to the angle
at at which the body was thrown.
51. A body falls freely from a height of 4 m. Ata height
of 2 m, it strikes elastically against a small surface
element .6.xed at ao angle of 30° to the horizontal. Deter-
mine the total time of motion of the body and its horizontal
range.
Giuen: H=4 m, h.=2 m, a=300~0.52 rad.
t-11-?
Solution. We choose the coordinate system with the
origin at point O (Fig. 43) and write the equation of mo-
tion for the body on the .6.rst segment AB of the trajec-
tory:
li="º -gt1/2, (1)
where llo = H. The velocity of the body on this segment is
V= -gt. (2)
Selected Problema on Phy1ic1

= h, and v = v8 ), Eqa. (1) and (2)


For point B (1 = 1., y
hecome h = H - rt:t2 and 1 118 1 = gt1, whence

•,=V2(H-h)lg, (3)
l••l=1V2(H-h)lg=V21(H-h). (4)
On the second segment. BD of the trajectory, the body
moves along a parabola, and the equations of motion
have the form
Z=Vs,/- 1 (5)
11=h+va11t-gt 2/2, (6)
where v8 • = 118 coa rt and 118 = v8 sin y. Conaidering
that. the impact ia elastic an"d that °' = 30", we have

D X
Fig• .f.3

<i = P, = y. Then v8 • = 118 coa°' and 1181 = v8 sin çc.,


or, taking Eq. (4) into account, we obtain

••.=Y2g(H-h) cosa, ••,=V2g(H-h) sina.


(7)
At point D where the body falis (t=t3 , 11=0, and
z=•), Eq. (6) becomu 0=h+va,t3 -gt:t2, whence
Ch. i. Mechanics 53

••11 ±l'•J11 +2i•


t1 = 1 , or, taking Eq. (7) into account,

11 V~oln+V'lfh+2i(H h) oin•a ( 8)

(the negative root is rejected). Then the total duration


of mot.ion of t.he body is
l=t,+t,= y2<H;h)
+~sina+~2í11+2í<H h)sin•a,

t=y •<::;2í
+ Y~xo.s+ J129~89.sx2+2x9.8(4 2)x0.25 8

""10.1 •.
The horizontal range can be determined from Eq. (5),
taking Eq. (8) into account:
s=}'2g(H-h) cosa
X V~sina+~2P+2f(H-h)ain•a

•=l'2 x 9.S(4-2) x 0.87


x V~xo.s+v"2~x,~o.-..s~x'"'2""'+""2-cx'"'•"".s~(4~2)x0.25m

::!!!5.6 m.
52. What will be the change in the time and horizon-
tal range of a body thrown horizontally from a certain
height if its initial velocity is doubled?
Amwer. The horizontal range is s = 11:ct, where the time
t of motion is determined only by the height of the body
above the ground. Therefore, if the height remains un-
changed and the initial velocity is doubled, the horizon-
tal range must increase by a factor of two.
53. A water jet escapes from a hose at an angle to the
horizontal. Why is the ascending branch of the jet coo-
.. Selected Problema ou Physica

tinuous, while the descending splits into fraetions?


Answer. Let us eonsider two adjaeent regions of the
ascending bra.nch. The velocity of water on any segment is
lower than that on the preceding segm.ent, wbieb leads to
a com paction of the jet. On the other ha.nd, in the de-
scending hrancb the diflerence in velocity on two neigh-
bouring segments leads to a splitting of the jet.

EXERCISES
M. An aeroplane Oies ata velocity of 800 km/h relative
to air. A westerly wind is hlowing ata velocity of t5 m/s.
At what velocity will the plane Oy relative to the Earth
in the southward direction and at what angle to the me-
ridian should it be routed?
55. The traces o! raindrops falling vertically on the win-
dow of a motor car moving at a veloeity of 45 km/h form
an angle of 30° with tbe vertical. Determine their velocity.
56. The range oi a body throwo aloog the horizontal at
a velocity oi tO m/s is equal to the height from which the
body is thrown. Determine the height.
57. What will be the chaoge io the time of Dight aod the
range of a body throwo along the horizontal if the height
from which it is thrown iocreases fourfold? The initial
velocity remains unchanged.
58. A shell is fired from aguo at an initial velocity of
tOOO m/s at an angle of 30" to the horizontal. De&ermioe
the horizontal range and the flight time of the shell if
the gun and the point where the shell falls to the ground
are on the same horizontal line.
59. A body is thrown at ao angle ci to the horizontal
ata velocity v 0 from a height h. What will be the velocity
at which the body falis to the ground?
60. A stone is thrown at an angle of 30° to the horizon-
tal at a velocity v0 = 5 m/s from the top of an inclioed
plane which forms an angle of 36° with the horizontal. At
what distance from the initial point will the stone fali?
61. At what angle to the horizontal sbould a body be
thrown so that the maximum height of its asceot is equal
to the horizontal range? Assume that a tail wiod imparts
a horizontal acceleratioo a to the body.
Ch. t. Mecbanic11

62. A bali is thrown at an angle of 30° to the horizon-


tal at an initial velocity of 14 m/s. At a distance of 11 m
from the initial point, the bali elastically bounces off
a vertical wall. At what distance from the wall will
the ball fall to the ground?
63. A carriage moves at a velocity of 72 km/h. Rain-
drops leave traces on the carriage window at an angle
of 60° to the vertical in the absence of wind. What is the
velocity of the raindrops?
64.. A motor boat passes the same distante downstream
and upstream in 4 and 6 h respectively. What time would
it take the boat to traverse this distance in still water?

MOTION OF A POINT MASS IN A CIRCLE.


ROTATIONAL MOTION OF A RIGID BODY
When a point mass moves in a circle, the direction of
its linear acceleration vector does not coincide with the
direction oi its linear velocity vector. At any point of

Fig. 44

the trajectory, the linear veloci ty v of the point mus is


directed along the tangent to the circle, while the accel-
eration vector a can be decomposed into two: ª" and ª"
(Fig. 44), where ª"" is the component of the vector a
along the tangent to the trajectory at this point, known as
the tangential aceelera.tion, and a 11 is the component of
lhe vector a along the normal (radius) to the trajectory at
this point, known as the normal (centrlpetal) aeceleration,
.. Selected Problems on Physic1

a,., = ri'/R. 1t follows from the figure that


a=Va:+a:.
For a uniform mot.ion in a circle (a,. = O), a = ª" and is
directed along the radius to the centre of the circle. ln
this case,
v = l/t = 2nRIT, ª" = luc1RIT',
where l is the length of the circumference, T the period
of revolution (the time of one revolut.ion), and R the radi-
us of the circle.
Rotational motton of a rigid body is characterized hy
the following quanti~ies: cp is the angular displacement
(the angle of rotation of an arbitrary radius from the
initial position), m the angular velocity, v the frequency
of rotation, and T the period of revolution. The quanti-
ties w, v, and Tare conneeted through the relation
o> - 2"1T - z.c,.
For a uniform rotation of a body, the equation of mo-
tion is
cp=wt.
For a uniformly varying rotation (uniformly accelerated
or uniformly decelerated) of a body, the equation of mo-
tion and the dependence of the angular velocity on time
have the form
cp = m0 t ± 8'2 /2, m = m0 ± et,
where m0 is the initial angular velocity ande the angular
acceleration.
ln these formulas, the plus and minu signs are taken
for a uniform.ly accelerated and uniformly deeelerated ro-
tation respectively. For a nonuniform (in particular,
uniformly varying) rotation, the angular velocity can be
expressed as the first derivative of the rotational angle
with respect to time:
m=%·
The angular acceleration for a uniformly varying rot.a-
tion can he defined as the first derivativa of the angular
Cb. i. Mechanies 5T

veloeity wit.h respect. to time:


du
8=Tt·

Knowing (1) = (1) (t) and e = e (t), we can determine


cp and (1):
ts ts
cp= i wdt,
f1
(I)= i edt.
t1

The angular quant.it.ies cp, (1), and e are connected with


the corresponding linear quantities l, v, and a t.brough
the following relations:

l = cpR, v = (l)R,. ª-r = eR, a11 = (1)1 R.

65. Determine tbecentripetal accelerat.ion of the point.s


on the equat.or, at. a latitude of 45°, and at. a pole due
to diurnal rot.ation of the Earth.
Given: T=24h=B.64.xtO's, cp1 =-0rad,
cpz=45ºe!!!0.79rad, cp3 :::!:::1.57 rad.
a.-1
Solution. All points on the Earth's surface participate
in t.he diurnal rotat.ion of t.he Earth at. an angular velocit.y
u=2'<1T. (1)

Consequently, their cent.ripet.al acceleration is

where r """" R cos cp is the radius of the circle in which


a point moves, cp the latitude of t.be point., and R the
Eart.h's radius (Fig. 45).
From Eqs. (1) and (2), we obtaina, = (4rt'/T') R cos q>.
Hence t.he acceleration of point.s on the equat.or, at.
Selected Problema on Physics

a latitude of 45°, and at a pole are respectively given by

ªc1= ~~· R,
ªc1 4 X~:.:~ 8;g?,~ 10' !:= 3.4 X tO"'I m/sl
(cosq>,-1),
ªci = ªct cos Cfi,
ªci !:= 3.4 x tQ-1X0.7 mJsl ~ 2.4 x to-a m/sl,
ac1 =0 since cosqi1 =cos:r./2=0.
66. A pulley 20 cm in diameter completes 300 revolu-
tions in 3 min. Determine the period of revolution, the

Fig. 45

angular and linear velocities on the rim of the pulley.


Given: D=20cm,,,,.0.2m, N=300,t=3min=180s.
T-1 .. -1 v-1
Solution. The uniformly rotating pulley completes N
revolutions in t seconds. Therefore, the time of one com-
plete revolution (period) is
T= t/N,
T = (180/300) = 0.6 s.
Ch. d.. Meohanics 59

The angular velocity is


0>=2"IT,
., = (2 X 3.14/0.6) rad/s = 10.5 rad/s.
The linear velocity is given by
v = 0>R = roD/2,
v = (10.5 X 0.2/2) m/s = 1.05 m/s.
67. Determine the radius of a rotating wheel if the
linear velocity of a point on the rim is known to be
2.5 times the linear velocity of a point lying 5 cm closer
to the wheel axle.
Given: V1 =2.5v1 , âR=-5cm=5X10-S m.
R-1
Solu.tion. Since the angular velocity of ali points of a
rigid body is the same, the linear velocities of the two
points are gi.ven by
"' = 0>R, v, = ., (R - àR). (1)
Substituting expressions (1) into the condition v1 =
2.5v1 , we obtain mR = 2.5m (R - âR), whence
R= 2.5'('_5AR'
R=-2.5xi.~to-• m~8.3x1Q-2 m.
68. A shaft starts to rotate, making 50 revolutions
during the first 10 s. A.ssuming that the rotation of the
shaft is uniformly accelerated, determine the angular
acceleration and the fi.nal angular velocity.
Gioen: t=10 s, N=-50.
m-? e-?
Solutlon. Since the initial angular velocity is zero,
the equation of motion and the formula for angular
velocity will_ be
qi=8t2 /2, m=et.
Since the angular displacement during one revolutioo
is 2n:, the total angular displacement of the shaft corre-
60 Select.ed. Problema on Phyaics

spoodiog to N revolutions is q:i = 2nN. Substituting this


expression into the equation of motion, we obtain 2nN =
ef'l2, whence
e=~' e= 4x3i!!x50 r::- =6.28 rad/s2.
Knowing e, we can calculate the final angular velocity:
a> = et, m = 6.28 X 10 rad/s = 62.8 rad/s.
69. Tbe jnitial rotational frequency of a wheel is 5 s- 1 •
After the brakes bave been applied, its frequency de-
creases to 3 s- 1 • Determine the angular acceleration of the
wbeel and the number of turns made by it during this
time.
Gtven: v0 = 5 s-•, 'V= 3 s-•.
e 1 N-1
Solution. The equations of motion for the wbeel bave
the form
'!'- .,,,_ et1/2, (!)
(l)=C0 0 -et. (2)
Since q:i -= 2:tN and m = 2n:v, we transform Eq. (2) as
follows: 2nv =2nv0 - et, whence
e= 2n(v:--v> •

2x 3.14~ (5-3) r;:- =0. 2 t rad/s2.

Taking Eqs. (1) and (2) into account, we determine the


total num.bar of turns:
N=vot-:!:,
N=5x 60- º4~~~ 1 =240.
70º. A wheel rotates according to the law q:r = 4 +
5t - t'. Determine the angular velocity of the wheel at
the end of the firat second of rotation, as well as the linear
veloeity and the total acceleration of tbe points on the
rim. The radius of the wheel is 2 cm.
Ch. t. Mechanica 81

Gioen: R=2 em=Cl.2 m, t=t 1.


m-? v-? a-?
Solutton. By defanition, the angular veloeity is
OI=~; =f.(4+51-1')=5-3t',
(!)
01=(5-3xl) rad/s-2rad/a.
The linear veloeity ia
V= mR,
v = 2 X 0.2 m/a = 0.4 mia.
By deftnition, the angular acceleration is e = ~·
Taking Eq. (t) into account, we obtain
•=f.-(5-31')= -61,
e= -6 rad/s2 •
The total linear acceleration ia
a= va.:+at.
where 4r = eR and an = (J)2 R. Then
a=RV•'+"'·
•= 0.2 v.,..<-~6>~'"'"+'""'<"'2>.,...' m1.•"' 1.44 mi•'·
71°. A body rotatea 10 that the time dependence of the
angular veloeity ia given by the equation m = 2 + 0.5t.
Determine the total number of turna completed by the
body during the ftrst 20 a.
Gtoen: t 1 =O, t2 = 20 s.
N-1

Solutton. By definition, q>=


.
i (J) dt. Then
,,
<p= r(2+0.51)d•=(2•+
,, º·~· ii:·
=2x20+~=140rad.
Selected Problema on Physics

The total number of turos made by the body is

N- i,.'
N= 2 ;~~ 14 ~22.
72. A point moves in a circle of radius 20 cm with
a constant tangential acceleration of 5 cm/s'. How much
will it take the normal acceleration of the point to be
twice as large as its tangential acceleration?
Giwn: R=20 cm=0.2 m, a"=5cm/s2.=5X1()-2 m/s2 •
411=2a'T.
1-1
Solution. By definition, v = a"t. Substituting this
expression into the formula for the normal acceleration,
we obtain a,.= v'IR = (a"t) 1/R = ~t 1 /R. By hypothesis,
a,. = 2a", or a~t 1 /R = 2a", whence

t=Jf!~.
t=V ~~:;. s=2.86 ª·
73. Are tbe distances traversed by the right and left
wheels of a motor car during a tum equal?
ADBWer. The right and left wheels cover different dis-
tances since their linear velocities are different: the
farther a wheel from the centre of curvature of the road.
the higher its linear velocity.
74. Why did the carrier rocket of the first artificial
satellite of the Earth get ahead of the satellite after its
separation?
Answer. After the separation, the satellite acquires
a velocity higher than that of it.s carrier roeket (see
Problem 150). For tbis reason, the semi-axis of the ellip-
tical orbit of the satellite is larger than that of the carrier
rocket, and they move in different orbits. According to
Kepler's third law, the period of revolution of the carrier
rocket is smaller than that of the satellite, and the carrier
roeket gets ahead of the satellite.
Ch. t. Mechaaics 83

EXERCISES

75. The minute hand of the clock on the Spasskaya


Tower of t.he Moscow Kremlin is 3.5 m long. What is the
displacement of it.s t.ip during 1 min?
76. Determine the average orbital velocity of a satellite
if the mean radius of its orbit from the Earth's surface
is 1200 km and the period of revolution is 105 min.
77. What. are the angular and linear velocities of the
points on the Earth's surface at a latitude of 45"?
78. The t.urbine of a hydroelectric power plant has
a working wheel dia meter of 9 m and completes 68.2 turns
during 1 min. Determine the velocity of the tips of the
turbine biades.
79. A Rywheel rotating at a frequency of 2 s- 1 is
stopped during 1.5 min. Assuming that. its motion is uni-
formly decelerated, determine the number of tums t.he Ry-
wheel makes before stopping and its angular acceleration.
80. A wheel rotating with a uniform acceleration at-
tains an angular velocity of 20 rad/s having made 10 turns
after the beginning of motion. Determine tbe angular
acceleration of tbe wheel.
81. A fan rotates ata frequency of 15 s-1 • After switch-
ing olf, the fan rotates with a uniform decelerat.ion and
completes 75 turns before it stops. How long will it take
the fan to stop after switching?
82. A wheel rotateswit.h a constant. angular acceleration
of 2 rad/s1 • The total acceleration of the wheel becomes
13.6 m/s1 0.5 s after t.he beginning of mot.ion. Determine
t.he radius of the wheel.
QUESTIONS FOR REVISION

t. List the maia brancbea of mechanica and cbaract.erize each of


them brieOy. 2. Wbat motion is uaifonn? uaiformly varying?
3. Name the trpes of uniformly varying motion . .fi. Writ.e the equa-
tion of a uniform motion and Plot the graph of lhe motion. 5. De-
line the average velocity, instant.uieoua velocity, and acceleration
:!uiaf:~~~°[!~ ~· :gie the equation of motioauai':!mit f~
celerat.ed motion (with an i without it).
7. Plot the velocity graphs for uaiformly varylng, uai-
formly accelerat.ed, and uniform y decelerat.ed motion. 8. What is
Selected Problema on Physics

t.2. Dynemlcs
Dynamks deals with the factors cawdng a change in tbe
state of motion of bodies.

APPLICATION OP TBE LAWS OP DYNAHICS


TO RECTILINEAR MOTION OF A BODY (POINT MASS)

ln mec.banics, we deal wit.h three types of force:


(1) the elastic force emerging upon deformation of
a support (N) or a string (T);
(2) the force of gravlty P = mg;
(3) the frlctional fonie F tr = p.N, where p. is t.be
coeflieient of fri etion and N t.he normal reaet.ion of t.he
ªº\Ih~:~ solving problems in dynamics, speeial attention
should be paid to eorrect. applieation of Newton's seeond
law.
The solution of a problem must begin wit.h a drawing in
whieh a reference frame must be indicated, all t.he forces
aet.ing on the given body must be depieted, and the diree~
tions of veloeit.y and aeeeleration veetors pointed out if
neeessary. Then Newton's second law should be written
in veet.or form.:
(1)
Ch. 1. Mechanics

where t F1 =F1 +F1 + ... +F,11 m is the mass of


·~· and a its acceleration.
the body,
Finally, the vector equation (t) should be projected

. .
onto the chosen directions of the X- ao.d Y-axes:

,~1 F1a-.-maa-., ~1 F 1,=ma, (2)


and the obtaiued system of equations (2) must be solved
for unknown quantities.
lf several bodies connected to one another participate
ia a motion, all the above procedures should be carried
out for each body separately, ao.d the obtained equations
should be solved simultaneously.

83. A carriage whose mass is 20 t moves with a uniform


deceleration of 0.3 m/s1 at an initial velocity of 54 km/h.

-" ~ mg
----..!
~
Fig. 46

Determine the braking force acting on the carriage, the


time of motion before it stops, aud tbe displacemeut.
Given: m=20t=2x1Cl'kg, •=0.3mls',
v0 =54km/h=15 m/s.
F,, ? t ? 1 ?
Solution. The carriage experiences the action of the
force of gravity mg, the friction Frr• ao.d the normal reacr
tion N (Fig. 46). Writing Newton's second law in vector
form, we obtaiu
mg + F 1, + N = ma. (1)
!l-0970
66 Selected Problema on Physics

We cboose the direction of motion of the carriage as the


positive direction of the X-axis. Since the motion of the
carriage is uniformly decelerated, the aceeleration vector
is opposite to the direction of motion. Projecting both
sidesof Eq. (1.) ont.o the X-axis, we obtain -Frr= -ma,
or
Frr=ma,
Fr, = 2 X 10' X 0.3 N = 6 kN.
The stopping time and tbe distance covered by the
carriage can be determined from the kinematic equations:
O = v0 - at and a = V:1(2a), whence
t = vJa,
t = 15/0.3 s = 50 s,
s = 15'
x (2 x 0.3) m = 375 m.
84. A body of mass 3 kg falis in air witb an accelera-
tion of 8 m/s1 • Determine the air drag.
Given: m=3 kg, a= 8 m/s2 •
F ?
Solution. The body falling in air is under the action
of the force of gravity mg and the air drag F (Fig. 47).
Since the motion is uniformly accelerated, the accelera-
tion vector is directed along the trajectory of motion.
Writing Newton's second law for the body in vector form,
we obtain
me +F=ma.
Projecting the forces and acceleration onto the Y-axis
direct.ed along the motion, we get
mg-F=ma,
whence
F = mg - ma = m (g - a),
F = 3 x (9.8-8)N = 5.4 N.
85. A load of mass 50 kg hoisted by a rope in the
vertical direction with a uniform. acceleration reaches
a beight of 10 m in 2 s. Determine tbe tension of the
rope.
Gfven: m=50 kg, t=2 s, h=tO m.
T-1
Cb. 1. Mechanice 87

Solutton. The load is acted upon by the force of gravity


mg and the tension T of the rope (Fig. 48). Since the load
moves upwards with a uniform acceleration, the vector a
is directed vertically upwards. Writing Newton's second
law for the load in vector form, we obtain
T + mg - ma. (1)
Directing the Y-axis along the motion of the load and
projecting the forces and accelerations onto this axis, we
get T - mg = ma, whence
T - mg + ma - m (g + a).
Since the motion is uniformly accelerated without an
initial velocity, h = att/2 and a = 2hJtt. Then
r-m(i+~).
T-50x(9.8+ 2 ; 1 )N-740N. º
Consequently, for a uniformly accelerated upward motion
of the load, the tension exceeds the force of gravity.
86. A man weighing 70 kg is in a lift moving vertically
upwards with a uniform deceleration of t m/st. Determine
the force of pressure exerted by the man on the cabin
Door.
Gtven: m= 70 kg, a= 1 m/sz.
F-1
Solutton. The man in the lift is under the action of the
force of gravity mg and the reaction N of the cabin Door

Fig. 47 Fig. 48 Fig. 49


,.
.. Selected Problema OD Physics

(Fig. 49). The acceleration is directed vertically down-


wards. Writing Newton's second law for the man in vector
form, we obtain
N+mg=ma. (1)
Directing tbe Y-axis along the motion of the lift and
projeeting tbe forces and accelerations onto tbis axis,
we get
N-mt= -ma, (2)
whence
N = mg - ma = m (g - a),
N - 70 X (9.8-t) N - 616 N.
According to Newton's third law, the force of pressure F
exerted by the man on the cabin Door is equal in magni-
tude to tbe reaction N of the cabin Door: F = N = 6t6 N.
87. The cage of an elevator of mass 5000 kg operates in
a 900-m pit. Wben the cage is at the bottom of tbe pit,
a driving force of 60 kN starts to set on it in the vertical
direction. At a distance of 150 m from the bottom, the
driving force changes so tbat the motion of the cage
becomes uniform over the next 600 m. Finally, tbe driv-
ing force changes once again so that tbe cage stops when
it reaches the surface. Assuming that friction is 5 kN,
consider tbe motion of the cage on tbese segments and
determine the time of ascent.
Gtven: m=5x1Qlkg, h=9x102m, F 1r=5kN=
5xt0' N, F,-60kN-6xtO' N,
h 1 =1.5xt02m, h1 =6xt02m.
t-?
Solution. Let us consider the motion of the cage on
eacb segment. On the ftrst segment of length h1, the cage
moves with a uniform acceleration at zero initial velocity;
on tbe second segment of lengtb h 1 , it moves uniformly,
wbile on tbe third segment of length
h, - h - (h, + h,), (!)
the cage moves witb a uniform decelêration and stops
at tbe end of tbe segment.
Ch. i. Mechanics 69

ln order to descrihe the motion of the cage. we direct

f
the Y-axis vertically upwards and choose the origin at
tbe bottom of the pit. The equation of motion for the
cage on the first segment is

Y1 = a 1t /2.
1
(2)
=:
!~~e; :h~ ~~: yh 1 h~;1:12~d w~~c~i) as- F,i
t1= V2h/a1.
Writing Newton's second law in projections
<3> a.t
:i~:.h~fiª:Js tf'..\1· 50) Fd - mg - Frr = F,r
•1 (F• - mg - F1,)lm.
= (4) rng
Substituting Eq. (4) into (3), we obtain Fig. 50
t,=V2h,m/(F•-mg F 1,). (5)
The velocity of the cage at the end of the ftrst segment is
v1 = a1 t1 , or, t.aking Eqs. (4) and (5) into account,

v, Fd-m.1-Frr
m.
v ..,m
Fd ""I Ftr
= y2.1a1 (Fd ::' Ftr) •

On t.he second segment, the cage moves uniformly at the


velocity acquired hy the end of the first segment:
Vi=Vi =Jf2AdFd ,.""' Frr). (6)

Choosing t.he origin of t.he Y-axis for the second segment at


the height h1 from tbe bottom of tbe pit, we write t.he
equation of motion for tbe cage:
y, = v,t. (7)
If t = t~111 then y1 = h 1 • and Eq. (7) becomes h 1 - v1 t 1 ,
whence t 1 = h 1 /v1 , or, taking Eq. (6) into account,
11 =hs fm/[2h,(F• mg F 1,)).
The time of motion on the third segment. can he found hy
using the concept ofaverage velocity: (vs) = (v1 + 0)/2=
70 Selected Problems on Phyaic1

v1 /2, or, taking Eq. (6) into account,


<v.>=Vlk,(F0 -mg-F,.)J1(2m).
Then the time of motion on the third segment is
t, = k,l(v,). (8)
Traa.forming Eq. (8) by applying Eqs. (1) and (7), we
obtain

ta=lh-(h,+hJJV lldFd 2:, Frr)'

The total time of the ascent of the cage is

t=t1+tz+ta=V,d 21aam,
m.f Ir

+hz Y
v-..-
21ai(Fd mm1 Frr)

+ (k-(k, +1isn 1-,,-_"':::,,_.-_-,-,,-)

=(2h-k:!.)y1' 2h1(Fd : , Frrl'


1=(2x9x10'-6x10')

x Vzxt.5xtosxc~-5)xtoa 8-
Sx101 ...., 69

88. A load of mass 45 kg is moved over a horizontal
plane by a force of 294 N direeted at an angle of 30" to tbe
horizontal. The coefficient of frietion between the load and
the plane is O.t. Determine tbe acc.eleration of the load.
Given: F=294 N, m=45 kg, ~=0.1, a=30" ,.0,52rad.
•-1
Solution. The load experiences the action of tbe forc;e
of gravity mg, the normal reaction N of the plane, the
driving force Fd, and the friction Frr· The vec.tor a is
direeted parallel to t.he plane t.o t.he right. (Fig. 5t).
Writing Newton's second law for t.he load in vect.or form,
we obt.ain
mg + N + F, + Fr, =me. (1)
Ch. 1. Mechanics 71

Choosing the direc;tions of the X- and Y-axesand project-


ing the forces and accelerations onto the axes, we get
Fd cosa - Frr = ma, (2)
N + F, sin"' - mg - O. (3)
From Eq. (3), we find that N = mg - Fd sina. Then
Frr = p.N = J.1. (mg - Fd sin a). Substituting Eq. (3)

~
into (2), we obtain
whence
Fd
Fig. 51

cosa - J.I. (mg - F4 sin cz) =ma,

F4 cosa-p(m1-Fdsina)

a--2Mx0.87-0.tx<_:x9.8-2Mx0.5) -i}~s. 9 m/sª.

89. A body slides uniform.ly down an inclined plane


forming an angle of 40" with tbe horizontal. Determine t.be
coefficient of friction of the body against the plane.
GiW!n: a=40° ~ 0.7 rad.
µ-1
Solutton. The body sliding down t.he plane is acted
~f~:eh:1!!. 1:::ie .
b~ fr~: ii?n F~;. 1.~: !0::~~,:·:!~nº~
.
aceeleration is zero (Fig. 52). Writing Newton's second
law for the body in vector form, we obt.ain
mg+N+F,.-0.
72 Selected Problema ou Pbysics

ProjectiPg the forces onto the X- and Y-a:r.es, we get


mgsinci-Frr =O, (!)
N - mgcosci =O. (2)
Equation (2) gives N = mg cos ci. Using the expression
for sliding friction Frr = µN = µmg cos ci, we write
Eq. (1) in the form mg sin a = µmg cosa = O, whence
p. = tana,
µ = tan 40" ~ 0.84.

y~

~ Fig. 52

90. A motor car of mass 1 t climbs a road with a slope


of 30" under the action of a driving force of 7 kN. Deter-
mine the acceleration of the car, assuming that the force
of resistance is independent of velocity and constitutes
0.1 of the normal reaction.
Given: m=I t=IO' kg, F=7 kN=7x!O' N,
a=30º<><0.52 rad, µ=0.1.
•-1
Solution. The car experiences the action of the force of
gravity mg, the normal reaction N of the road, the driving
force F 11.t and the friction Frr· Cboosing the direction of
vector a up the inclined plane (Fig. 53) and writing
Newton's second law for tbe car in vector form, we obtain
mg + F0 + N + F,, =ma.
Ch. t. Mechanic1 73

Projecting the forces and accelerations onto tbe X- and


Y-axu, we get
-mg sina + Fd - F1r = ma, (1)
-mgcos0< + N - O. (2)
Equation (2) gives N-= mg cosa. Considering that F rr =
p.N = .t'mgcosa, we write Eq. (1) in tbe form
-mg Bln a + Fd - p.mg cosa = ma, wbence
Fd-m1(sinm+µ.c01m)
a= m '
7xtoa-tCPx9.~~(0.s+o.tx0.87): t. 2 m/s2

Fig. 53

91. A block of mass 2 kg slides over a horizontal plane


under the action of a load of mass 0.5 kg fixed to tbe end
of an inextensible string passing over a stationary pul-
ley. Tbe coeffi.cient of friction between tbe block and the
surface of the plane is 0.1. Determine lhe acceleration of
the load and tbe tension of tbe string. Tbe masaes of the
pulley and tbe string, as well as the friction in the pulley,
sbould be neglected.
Gtven: m1 =2 kg, mz-=0.5 kg, p.=0.1.
a-1 T-1
Solution. Let us consider tbe motion of each body
separately. The block is under the action of the force of
gravity mil• the normal reaction N of tbe plane, tbe tension
Ti of tbe string, and the friction Frr (Fig. 54). Writing-
Newton's second law for the block in vector form, we-
Selected Problema on Physics

obtain
m1g + N + T1 + F 1 r = m 1a1• (t)
Project.ing Eq. (t) onto the ehosen directions of the X-
and Y-axes, we get
T1 - F,, = m1ª1• (2)
m1g-N =0. (3)
Equation (3) gives N = m1g, and henc.e Frr = JJ.N """
µm 1g. Then (see Eq. (2)):
r. - fU'l-11 = m•ª•· (4.)
The load is acted upon by the force of gravity m 1g and the

Fig. 54

=-~~: ro~dofi::.::~nf~:.n;:~b~:;:,.ton's second law

m 1g + T1 = m 11a 11 • (5)
Projecting Eq. (5) onto the Y-axis, we get.
m 1g - T 1 = m 1 a1 • (6)
Summing Eqs. (4) and (6) and considering that T1 = T1 =
T and a1 = a 1 = a, we obtain m1g - T + T - ,un.11 =
(m1 + m1 ) a, whence

a= "':.+'mil'= •<z:+rc·>.
9.8x(~·.~+~·tx2) -if !:!!t. 2 m/s2.
Ch. 1. Meehanics 75

The tension of the string can be determined from Eq. (6):


T = m1 g - m1 a = m1 (g - a),
T - 0.5 x (9.8-1.2) N "'4.3 N.

m:!· 21i~~d a:f i:!,~':e!si~,ec::~::t;!S:~;~~~~e: ~:.~~o:~


ary pulley attached to the top of an inclined plane slides
down the plane forming an angle of 36° with the horizon-

Fig. 55

tal. Determine the tension of the string and the accelera-


tion of the loads if the coefficient of friction between the
flrst load and the plane is 0.1. The masses of the string
and the pulley, as ·well as the friction in the pulley,
should be neglected..
Given: m1 =5kg, rni=2kg, p.=0.1,a=36°~0.63rad.
T-1 a-1
Solution. We consider the motion of each load separate-
ly. The first load is acted upon by the force of gravity
m,_g, the normal reaction N of the inclined plane, the
tension T1 of the string, and the friction Frr (Fig. 55).
By hypothesis, the acceleration vector a1 of the first load
is directed down the inclined. plane. Writing Newton's
second law for the first load in vector form, we obtain
m,g + N + T, + F,, - m,a,. (1)
Projecting Eq. (1) onto tbe X1- and Yi-axes, we get
m1gsina-T1 -Frr=m,,a1 , (2)
-m,gcosa + N -o. (3)
16 Selecled Problems on Phyaic11

Equation (3) gives N = m1 g cosa, and hence


Ftr = p.N = 11m11 cosa. (4)
From Eqs. (2) and (4), we obtain
m1g sina - T 1 - fU1'il cosa = m1a,,. (5)
The second load experiences the action of the force of
gravit.y m1 g and the tension T1 of the string. The accelera-
tion vector a 1 of the second load is directed vertically
upwards (see Fig. 55). Writ.ing Newton's second law for
the second load in vector form, we obtain
m 1g + T 1 = m 1a1 . (6)
Projecting Eq. (6) onto the Y ,-axis, we get
-m 1 g + T 1 = m 1 a1 . (7)
Summing Eqs. (5) and (7), we obtain m1g sina - T1 -
flll'il cos a + T 1 - m1 g """ m1 a,, + m1 a1 • Considering
that T1 = T1 = T and a,,= a,= a, we ohtain
m.11(sina-µ.cma)-"'4
m1+11tt
m.i{sina;.!f.c:m)-m.1 g,

a= 5x(0.59-so;xo.8t)-2 x9.8-i}!:!::::0.77 m/sª.


The tension of lhe string can he determined from Eq. (7):
T =m 1 g+m,a= m1 (g+ a),
T-2x(9.8+0.77)N,,,2t N.

cli~~d ~1::S~~~mf:!1:~~ie::f~ª!:3e,5re~it~ t~;ºho~~:


zontal. The loads A and B of mass 1 kg each are con-
nected by a string passing over the pulley. Determine the
acceleration of the loads and the tension of the string,
assuming that the string is weightless and inextensihle
and neglecting friction.
Given: a-30"<><0.52 rad, P-45°<><0.79 rad,
m1 =m:=1 kg.
T-? a-?
Ch. t. Mechanic11 77

Solutlon. From the assumption that the masses of lhe


string and lhe pulley are zero it follows that lhe tension of
the string is the same in each cross section, while lhe
condition of inextensibility of the string implies that the
accelerations of the loads are equal in magnitude:
T1 = T 1 = T, a 1 = a 1 =a. (1)
Let us consider the motion of each load separately. The
load A is acted upon by the force of gravity mig, the

Fig. 58

normal reaction Ni, and lhe tension Ti of lhe string


(Fig. 56). Assuming that the load A slides down the
inclined plane, we determine lhe direction of aceeleration
vector a1 • Writing Newton's second law for the load A in
vector form, we obtain
mig + Ni +Ti = mia1• (2)
Projecting Eq. (2) onto the Xi- and Yi-axes, we get
m1g sin P - T1 = m11Ji, (3)
-m1g cos ~ + N, - O. (4)
The load B experienc.es the action of the force of gravity
m1g, the normal reaction N1 of the inclined plane, and the
tension T1 of the string. Writing Newton's second law
for the load B in vector form, we obtain
m1g +N +T
1 1 = m 1 a1 • (5)
78 Selected Problems oo Physics

Projecting Eq. (5) onto the X 2 - and Y 1ra:r.es, we get


-m 1 t sin a +T 1 = m 1 a1 , (6)
-m1 g coso:+ N 1 =O. (7)
Equations (4) and (7) are not used in this problem since
we negleet frietion. Solving the system of equations (3)
and (6) by summing them, we obtain m1g sin ~ - T1 -
m1 t sina + +
T1 = m1a1 m1 a 1 • Taking Eqs. (1) into
aceount, we lind that
m111in1S-m111inci (m1 sinfS-1nt1inci)1
m1+Ma m1+m1
a= (tx0.7-/.;to.5)x9.8 -i}=0.98 m/s•.

The tension of the string can be determined from Eq. (3),


taking Eqs. (1) into aecount
T = m1g sin ~ - m1a = m1 (g sin ~ - a),
T ~ t X (9.8 X O. 7 - 0.98) N <>< 5.9 N.
94. Two loads of mass 100 g each are attached to the
ends of a weightless inextensible string passing over a
musless stationary pulley. Determine the force exerted
hy a small load of mass 10 g placed on one of the loads
and the force of pressure on the axle of the pulley.
Given: m,=m,= 100 g=O.t kg, m~to g~O.Ot kg.
F-? Fpr-?
Solution. The conditions of weightlessness and inexten-
sibility of the string imply tbat the tension of the string
is the sarne in ali segments and that ali the loads move
with the same acceleration:
T1 = T1 = T, a1 = a 2 =a. (1)
Let us consider the forces acting on each load.
The first load experiences the action of the force of
gravity m1g, the tension T1 of the string, and the force
of pressure F of the small load (Fig. 57). Writing Newton's
second law for the first load in projections on the Y -axis,
we obtain
(2)
Ch. 1. Mechauics

Tbe second load is acted upon by tbe force of gravity


m 1 g and tbe tension T1 of t.he string. Writing Newton's
second law for the second load Y.
in projections on the Y-axis, 2
we get
T 1 - m 1 g = m 1 a 1 • (3)
Tbe small load is under tbe
action of tbe force of gravity
mg and the normal reaction
N. Writ.ing Newton's second
law for tbis load in projections
on the Y -axis, we obtain m9
mg - N - ma,. (4)
Summing Eqs. (2)-(4) and con-
sidering that m 1g = mJ =
Mg and F = N,we getMg+
N - T +T - Mg + mg - Y,
N = Ma+Ma +ma, whence
Fig. 57
a - mg1(2M + m). (5)
Let us determine t.he force of pressure F exerted by t.he
small load. According t.o Newt.on's tbird law, this force
is numerically equal t.o tbe normal reaction N. Solving
Eqs. (4), (5), and (1) together, we obtalo
F=N=mg-ma= 2~!'m ,
F 2x2o~~-~~~.Ót9.8 N =9.3x 10-z N.
The force of pressure on tbe axle of tbe pulley is
F., - 2T. (6)
From Eq. (3), we determine the tension of tbe string:
T = M (a + g). Taking Eq. (5) into account, we obtain
T - 2Mg (m + M)/(2M + m). Theo Eq. (6) become
Fpr=4Mg ;,++~ ,
Fpr=4xO.tx9.8x 2 ~·~t:o~ot N~2.1 N.
80 Selecf.ed Problems on Physics

95. Three loads of mass t kg each are tied by strings


and move over a horizontal plane under the action of
:.l~~t~~!~n~ 1t:!r:~!:1e:!t~!nª!f~~:~:e: !:~ ~~~i::~=
sions in the strings if the coefficient of friction is 0.t.
Given: m1 =m2 =m3 =m=t kg, F=iO N,
a=30"~0.52 rad, fl=Ú.t.
a-? T11 -? Taa-?
Solution. Let us consider the forces acting on each load.
The first load is acted upon by the force of gravity mg,
y

,,,.,
o
mg mg
Fig. 58

the normal reaction N1 , the applied force F, and the fric-


tion F 1 r 1 (Fig. 58). Writing Newton's second law for the
fint load in projections on the X- and Y-axes, we obtain
Fcosa.-T11 -Frn=ma, N 1 +Fsina-mg=O. (1)
From the second equation in (1), we find that N 1 = mg -
F sina, and henc;e Frr1 = µN 1 = p. (mg- F sina.).
Consequently, the first equation in (1) assumes the form
F cosa - T11 - .._ (mg - F sina)= ma. (2)
Similarly, for the second load, we obtain
T 11 - Frr2 - T 13 =ma, N1- mg =O. (3)
After transforming these equations, we get
T 11 - iun1- T 23 =ma. (4)
Ch. t. Mechanic1 81

For the third load, we can write


T 81 -Ftn=ma, N 8 -mg=0, (5)
whence
T,. - µmg =ma. (6)
Summing Eqs. (2), (4), and (6) and considering that
T11 = T11 and T 18 = T 81 , we obtain F cosa -
y. (mg - F sin a) - 2,ung = 3ma, whence
a=F (cosci+e3~nci)-311-m1

a tOx(o.s1+0.tx~·~;-axo.txtx9.8:. ~ 2 .t m/s2.

From Eqs. (4) and (6), we find that


T 11 = Tu = 2m (a + JJ.g),
T11 = 2 X 1 X (2.1 + 0.1 X 9.8) N <:>< 6.2 N,
T 81 = T11 = m (a+ tiK),
T., = 1 X (2.1 + 0.1 X 9.8) N <:>< 3.1 N.

96. A body is weighed on a spring balance and on


a beam balance at a pole and on the equator. What are
the readings of the instrumenta?
Amwer. The maas of the body determined with the help
of the beam balance is compared with the mass of stan·
dard weights. Since the weight of the body and of the
standard weights varies identically as we move from one
point on the Earth's surface to another, the readings of
the beam balance at the pole and on the equator will be
the same. The weight of the body determined with the
help of the spring balance is compared with the elastic
force of the spring. Tbe spring constant does not depend on
the latitude, and hence the readings of the spring balance
at the pole will be larger than on the equator. Indeed,
using Newton's second law for the body suspended on the
spring at the pole and on the equator, we obtain
mg-T11 =-O, mg-T8q = mrJ'/R,
8-0170
82 Selected Problema ou Physics

;:i:rear?'l ::dti:ee~:::t!~ere~;~~~e~.t~eh::ri:~ !t !~:


97 ~Amfod ~v;,'~ieJ·:im!'; f;e ~~;face a
T of table with
the help of a string att.ached to a spriog balance indicat-
ing 30 N. Then the same load is set in motion with the
help of a string passing over a stat.ionary pulley with
a load of mass3 kg at the ot.her end. ln whichcase does the
load move faster?
Answer. ln the second case, a force of 30 N acts on the
system of bodies with a larger ma.ss than in the fi.rst..
Therefore, the load moves faster in the fi.rst case.
98. Why does t.he velocity of a train on a horizontal
segment not increase indefmitely although the driving
force of the engine acts constantly?
Answer. As t.he velocit.y of the body increases, the
resistance forces become stronger and ultimately balance
the driving force. Therefore, t.he motion of the train be-
comes uniform after a certain time.
99. Bodies fali due to the attraction of the Eart.h. ln
what. respect. is this stat.ement inaccurate?
Answer. There are no bodies in nature which only exert.
an action or are only subject.ed t.o an action. On the
contrary, bodies experience equal act.ion and react.ion
(Newt.on's t.hird law). Consequent.ly, the correct. way of
putting this is that. the body and t.he Earth at.tract. each
ot.her, but. since t.he mass of t.he Eart.h is huge in
comparison with t.he mass of t.he body, we do not. notice
its mot.ion t.owards t.he body.
100. Why does a running person fali in the direct.ion of
mot.ion when he stumbles and in t.he opposit.e direct.ion
when he slips?
Auwer. This phenomenon can be explained easily by
using Newton's flrst. law. ln the formercase, the feet. of t.he
person are slowed down, while t.he body retains its previ-
ous state of motion by inert.ia, and the person falis in t.he
direct.ion of mot.ion. ln the lat.t.er case, the body of t.he
fe:ºsii;~:i~:ei~~':;Jºd~r:t.~!8n ~t 1:º~!:h~r~~V!:i~:
and t.he person falis backwards.
Ch. t. Mechanica 83

EXERCISES

101. A body of mass 200 g falis vertically downwards


with an ac.eeleration of 920 cm/s1 • What is the air drag?
t02. What is the force of pressure exerted by a load
of mass 100 kg on the Ooor of the cage in a mine, u-
cending vertically with an acceleration of 24.5 cm/s1 ?
t03. At wbat distance from a crossroad must tbe driver
of a motor e.ar apply brakes at a red traffic ligbt if the car

Fig. 59

moves uphill alonga slope of 30" ata velocity of 60 km/h?


The eoefficient of frietion between the tyres and the
road is 0.1.
UN. An inclined board forming an angle of 60" with the
horizontal Jeans against a table (Fig. 59). Two equal loads
of mass t kg each and connected through a light string
passing over a massless stationary pulley can move over
the board and the table respectively. Determine the tension
of the string and the acceleration of the syst.em if the
eoeffieient of friction between the loads and the surfaces
of the board and the table is the same and equal to 0.3.
t05. A funicular railway forms an angle of 30" with the
horizonlal and has two eabins of mass 4600 kg eacb,
conneet.ed by a steel rope passing over a pulley at the
upper station. The descending cabin carries an additional
load of mass 600 kg. Determine the ac.eeleration of the
system and the distance traversed by eacb eabin if the
motion slarts from the state of rest, and the maximum
velocity attained by the cabins is 14.4 km/h. Determine
the t.ension of the rope, neglecting frietion and tbe maues
of tbe rope and the pulley.
a :=~g1:s~ ~:~s ::~~!:.i::d:!~~~~º~':~~i
.. Selected. Problema on Pbysics

the same load at its end is tied to lhe system of loads


and passes over a stationary pulley. Determine the accel-
eration of the system of loads and lhe tension of the string
connecting the loads on the surface, neglecting friction.
107. A load of mass 500 g is suspended on a string which
passes over a pulley and has a load of mau 300 g at it.s
other end. Determine the acceleration of the system, lhe
displacement and the velocity of each load 1.2 s after the
beginning of motion. Friction and the masses of the
string and lhe pulley should be neglected.
108. Two bodies connected by a string move over a
horizontal plane under lhe action of a horizontal force of
100 N. lf the force is applied to the right body (m1 = 7 kg),
the tension of the string is 30 N. Determine the tension of
the string if lhe force is applied to the left body (m 1 =
3 kg), assuming that in both cases the bodies move in the
direction of the applied force and neglecting friction.
109. A man pulls two sledges of mass 15 kg each and
connected by a rope, applying a force of 120 N at an angle
of 45º to the horizontal. Determine the acceleration of the
sledges and the tension of the rope connectinr the sledges
if the coefficient of friction between the runners and snow
is 0.02.
110. A wooden block is initially at rest on a table.
Two strings pass over pulleys fixed at two ends of the table
and are tied to the block. When loads of mass 0.85 and
0.2 kg are suspended on the free ends of the strings, the
block starts to move and in 3 s covers a distance of 0.81 m.
Determine the coefficient of friction and the tension of
the strings if the mass of the block is 2 kg.
111. Two weights of mass 2 and 1 kg are connected by
a string passing over a stationary pulley. Determine the
acceleration of the weights, the tension of the string, and
the force of pressure on the axle of the pulley.

APPLICATION OF TRE LAWS OF DYNAMICS TO TBE


MOTION OF A BODY (POINT MASS) IN A CIR.CLE
The solution of problema on dynamics of circular mo-
tion does not differ in principie from. the solution of
problema on dynamics of translational motion.
Ch. i. Mechanica 85

ln the case under consideration. the projection of


acceleration of a body on the Y-axis directed along the
radius to the centre of the circle is known to be the cen-
tripetal acceleration. Therefore, according to Newton's
second law, we have
mv2/R = F, or mw1R = F,
where F is the sum of the Y-projections of the forces acting
on the body.

112. A truck with aloadof mass5 t moves over a con-


vex bridge at a velocity of 21.6 km/h. What is the force
of pressure exerted by the truck at the midpoint of the
bridge if the radius of curvature of the bridge is 50 m?
Giuen: m=5t=5x1()3 kg, v=21.6 km/h=6 m/s,
R-50 m.
F-?
Solution. The truck is under the action of the force of
gravity mg and the normal reaction N (Fig. 60). Directing
the Y-axis vertically downwards along the bridge
radius and writing Newton's second law for the truck
in vector fonn, we obtain
mg+N-ma.
Projecting this equation onto the Y-axis, we get
mg-N =ma,,
where a,= ac = rJIR. Then mg - N = mv'1/R, whence
N = mg - mv'1/R = m (g - rJ/R). According to New-
ton's third law, the same force is exerted by the truck on
the bridge, i.e. F = N, or
F-m(g-~).
F-5 X 10' X (9.8-.;) N <><4.5X10' N.
113. A bucket with water is rotated in a vertical plane
on a 0.5-m long rope. What is the minimum veloeity of
rotation at wbich water does not Dow out of the bucket
at the upper point of the trajectory?
.. Given: l=0.5 m.
Select.ed Problema oa Physics

v-?
Solutlon. At the upper point of the trajectory, tbe water
:J~~eh:~~!ta:ªr!~!:to:~º~r bÍbeth:O!~:i: (~ir.ªGf )~ Ôi~
recting the Y-axis vert.ically downwards to the centre C of
the circle and writ.ing Newton's second law for the water
in the bucket in projections on this axis, we obtain
mg+N=ma,,
where a, = ªe = v2/l. Tbis gives
mg + N = mv'll.

~\~~eb1:~:~~~º1:3Õ~~~ 0b~!!1::,w~e~~~/t~~~ctº:~:
v=Vli,
v-y'9.Sx0.5 m/s<><2.2 mts.
114. At the lowest p0int of a wingover (Nesterov'!ll
loop), tbe pilot exerts a force of pressure of 7t00 N on
the seat of an aeroplane. The weigbt of the pilot is 80 kg
and the radius of the loop is 250 m. Determine the ve-
lo~:!:n~f J,1:f\ª~8 f03 N, m=80 kg, R=2.~ X 1()2 m.
v-?
Solution. The pilot experiences the action of the force
of gravi ty mg and the normal react.ion N of tbe seat

ci-~J',__ _,/
mg
Fig. 60 Fig. 8t Fig. 82
Ch. 1. Mechanics 87

(Fig. 62). Direc.ting tlte Y-axis to the centre C of the


circle and writing Newton's second law for the pilot in
projections on this a:r.is, we obtain
N - mg = n&ac1
or
N-mg - mv'/R,
whence v = Y (N - mg)Rfm. According to Newton's
third law, N = F, and hence

v=V~.
V= y (7.txtoa-80~9.8)x2.5x10• ~ ~ 140 m/s.

115. A bali of mass 200 g suspended on a string de-


ci~y1:8~:i::~~;nt:e h::f::i~~ª~f~':' b~tlla .C:dstiS:~;~~d
of its rotation if the length of the string is t m and its
angle with the vertical is 60°.
Given: m=200g=0.2kg, l=t m,m=60" ~ 1.05 rad.
v-?T-?
Solution. The bali experiences the action of the force
of gravity mg and the tension T of the string (Fig. 63).
Writing Newton's second Iaw for the bali in vector form,

mg
Fig. 63 Fig. 64
.. Selecled Problems on Physics

we obtain
mg+T=ma. (1)
Projecting Eq. (t) onto the chosen directions of t.he X-
and Y-axes, we get
T sina.= maz, -mg + Tcos a.= ma 11 • (2)
Assuming that a, = O (the bali does not move in the
vertical direction), az = u'IR, R = l sina., and
substituting the expressionsfora~:. a,, and R into Eqs. (2),
we obtain
T sin a: = mr?/(l sin a.), T cos a. = mg. (3)
Dividing the equations (3) termwise, we obtain

v=sinaJI'" c!'a.,
v = 0.87 V 9 ·:.~ 1 -i-- ~ 3.8 m/a.
The period of uniform motion of the ball in a eircle is
T = 2ir.R/u = (2:nl/u) sina,
T= 2xs.1;~81 x0.87 8 ~ t.4. s.
116. A bali of mass 500 g suapended on a t-m long
ine:xtensible string performs oscillations in a vertical
plane. Determine the tension of the string at the inst.ant.
when it forms an angle of 60" with the vertical if the
velocity of the ball at. this moment is t.5 m/s.
Given: m=500g=0.5kg, l=1 m,a=60"~1.05rad,
V=1.5 m/s.
T-?
Solutton. The bali is acted upon hy the foree of gravity
mg and the tension T of the string (Fig. 64). Writing
Newton's second law for the bali in nclor form, we get
mg+T ~ma. (1)
Directing lhe Y-a:xis aJong t.he radius and projecting
Eq. (1) onto this a:xis, we obt.ain
T- mgcosa. =ma,. (2)
Ch. t. Mechanics 89

Considering that ª•
= ae = vlll. we write Eq. (2) in th&
form T - mg eos et = mvsll, whence
T=m { ~+geosa),
T-0.5x( ';5' +9.8x0.5)N<><3.6 N.
tt7. A hemispherical cup of radius 20 em rotates about
baiir!C:\i~x;~:: i~.ªfi~::~~82 s- 1• It contains a small
the angle between the radius \l)w
drawn to the position of the ball y 1
an~i~!:v~rt!eil,. em, \l = 2 s-1. 1 R
a-1
Solu.tion. The bali experi-
ences the action of the force of X
gravity mg and the normal reae-
tion N of the inner surfaee of 1119 li
the eup (Fig. 65). Writing New-
ton's second law for the ball in 1
vector form, we get Fig. 85
mg + N - ma. (1)
Direeting the X-axis towards the centre of the circle of
radius r in which the bali moves and the Y-axis along the
vertical and projecting Eq. (1) onto these axes. we obtain
N sin et = maz, N cose - mg =O.
Sinee Oz = ac = v'l/r = w2r, w = 2ttv, and r = R sin e,
we get
N sin et = 4n"'v1 Rm sin e, N cos et = mg. (2)

~~~''> ~~·ª.<2~h::~:~:: ,m~1v~1R>t .!dn


a= arccos .f.n~•R
a=arooos 4 xa.t.f.::zsxo. 2 ~1.26 rad.
t 18. What must be the height of the outer rail relative
to the inner rail on a bend in a railway track of radius
90 Selected Problem! on Ph19ics

300 m if the width of the track is 1524 mm? Assume that


the normal velocity at. which the force of pressure on the
rails is perpendicular to them is 54 km/h.
Gtven: R = 300 m, l = 1524mm=1.524 m,
v=54 km/h=15 m/s.
k-?
Solution. ln order to impart a velocit.y v to the train
moving in a circle of radiusR', theforcesactingon it. must
e.reate a centripetal aeceleration in the horizontal direc-

o'
X

Fig. 66

tioo (by hypothesis) (Fig. 66). The track is banked in order


to improve the stability of the train and to reduce the
wear of the rails. Then the resultant of the force of gravi-
ty mg and the normal reaction N of the rails imparts the
required centripetal acceleration to the train. Writing
Newton's second law for the traio in projections on the X-
and Y-axes (see Fig. 66), we obtain
N sin ci = maz, N cos ci - mg = O,
where az = ac = u'1/R, whence tan ci = v'l(Rg) and
tan ci = 15'-/(9.8 X 300) ::::!!:! 7.7 X 10-1 • Since tan ci is
a small quantity, sin ci ~ t.an ci.
On the ot.her hand, from 6ABC we have sin ci = hll.
Consequently, we obtain

h=-ii-·
h-t.524x7.7x10-• m"'0.12 m.
Ch. t. Mechanics 91

119. Why is a p8S9eoger standing at a door to the right


(relative to the direction of motioo) of a moviog car of
a metro train pressed against the door as the traio turos to
the left round a bend?
Answer. As the train tums to the left round the bend,
the penoo retains the previous motion by inertia and is
pressed against the door.
120. Why do rivers of the northem hemisphere erode
their right banks?
Answer. The Earth rotates from west to east. lt is well
known that the linear velocity of rotation of points on
the Earth's surface decreases from the equator to the
poles. Therefore, water in a river Rowing to north will be
deRected eastwards by inertia, retaining its previous
velocity, and will erode the right bank.

EX EH CISES
121. Determine the velocity of a lorry of mass 2 t mov-
iog over a concave bridge of radius 100 m if it exerts
a force of pressure of 25 kN on the middle of the bridge.
122. A weightsuspended on a30-cm longstring del'Cribes
a circle of radius 15 cm in a horizontal plane. Deter-
mine the frequency of its rotation.
123. A ball is suspended on a string from the ceiling of
a tram moving at a velocity of 9 km/h round a bend of
radius 36.4 m. Through what angle will the string with
the ball be deDected?
124. A boy is swinging on a giant stride at a frequeo-
cy of 16 mln- 1• The leogth of the ropes is 5 m. Determine
the tension of the ropes if the weight of the boy is 45 kg.
125. Ao aeroplane Oying ata velocity of 720 km/h makes
a loop of radius 400 m in a vertical plane. Determine
the force of pressure exerted by the fuel on the bottom of
the tank of area 1 m9 , filled with fuel to 0.8 m, at the
lowest point of the loop.
126. A skater runs at a velocity of 12 m/s in a circle
of radius 50 m. At what angle to the horizontal must he
incline bis body to maiotain the equilibrium?
127. A diac rotates in a horizontal plane at a frequency
of 30 mio-1 • What must be the coeffi.cient of friction
92 Selected Problems on Physics

between the disc and a body lying on it for tbe body to


remain on tbe disc? Tbe distance from the axis of tbe
disc to the body is 20 cm.
128. A bali suspended on a 50-cm long string rotates
uniformly in a vertical plane. Determine the frequency
at which tbe string breaks if the ultimate strength of tbe
string is 9mg, wbere m is the mass of the ball.
APPLICATION OF BASIC LAWS OF DYNAMICS
TO SPACE FLIGHTS
Along with tbe laws of dynamics considered above, tbe
law of universal gravltallon will be used in this section:
F =Gm1m2 /r2 ,
where F is the force of interaction (gnvitatlonal force)
between two point masses m1 and m 1 separated by adis-
tance r, and G the gravitational eonslant.
A gravitational force acts on a body in a gravitational
field irrespective of whether or not there is a support. ln
a free fali, the body is not acted upon by tbe normal reac-
tion, and the state of weigbtlessneu (zero gravity) is
observed. For example, zero gravity sets in when a satellite
orbits round a planet.
Tbe orbital (or clreular) veloelly is tbe minimum ini-
tial velocity at which a body starts to move parallel to the
surface of a planet and becomes its artificial satellite.
This velocity is different at different altitudes and for
different celestial bodies. Near the Eartb's surface, the
orbital (circular) velocity v1 = 7 .91 km/s is given by
1=V10R.
•1

where lo is the free-fall aceeleratlon at tbe surface of


tbe planet and R the radius of tbe planet.
The minimum initial velocity tbat must be imparted to
a body starting to move near the surface of a planet to
overcome its attraction is known as the escape velocity:
v,-v2,,11.
Tbis velocity varies witb altitude and is diBerent for dilier-
ent celestial bodies. For the Eartb, v2 = tt .19 km/s ~
11.2 X 10' m/s.
Ch. t. Mechauic1 93

lf a body moves at the escape velocity at lhe initial


instant and does not e:r.perience the action of any force
other than lhe gravitational force, it will move relative to
the celestial body in a parabola.

129. Calculate lhe free-fall acceleration for a body at


a distance of 100 km above lhe Earth's surface.
Given: h=100 km=105 m.
g-?
Solution. The body at a distance h above the Earth's
surface is acted upon only by the gravitational force
directed towards tbe centre of the Earth and equal to
F = GMmlr', (!)
where Mis the mass of lhe Earth, m lhe mass of the body,
and rthedistancefrom lhe centreof lhe Earth to the body.
Under lhe action of this force, the body will move
vertically downwards with acceleration g. Writing New-
ton's second law for lhe body in projections on the Y-axis
directed vertically downwards, we obtain
F=mg,
or, taking into account formula (1), we get
GMmlr2 = mg,
whence
g=GM/r'.
Here r = R + h, where Ris lhe Earlh's radius and h the
altitude of the body. Then at lhe altitude h
g =G (R~h)' . (2)
At the Earth's surface h ~ O, and hence r = R and
g = GM/Rs. The value of this acceleration at the Eartb's
surface will be henceforlh denoted by g0 :
GM
fo=Rt• (a)
fo= 6.87x(~~~::ó~!xtou 7~ 9•8 m/s2.
.. Selected Problem. on Pby11ics

Dividing Eq. (2) hy (3) termwise, we obtain g/g0 =


R 1/(R + h)1 , whence g = g 0 R 1/(R + h) 2 • Since h/R <1,
we can use the approximate relation (1 + h/R)-" :::=:
1 - 2h/R. This gives
1
t= lo R'(t! h/R)t :=t: lo ( t-if-) •
g=9.8x(~~:::;}-i}~9.51 m/sz.
130. A spaceship of mass 1.()9 kg is launched vertically
upwards. The driving force of its engines is 2.94 X 107 N.
Determine the acceleration of the spaceship and the
weigh\ of a body in it if the force of gravity acting oo this
body on the Earth's surface is 5.88 x 1011 N.
Given: m1 =1(}CI kg, Fd=2.94x107 N, 7nilo=5.88x
1()1 N.
a-?P1 -?
Solution. The spaceship is acted upon by the driving
force F d of its engines and the gravitational force F
(Fig. 67). According to the law of uni-
versal gravitation,
F =GMm1/r 2 ,
where r = R + h (see Prohlem 129).
Since h<.R at the beginning of the Oight,
neglectingh in comparison withR, wecan
assume that r ~ R, and hence
F =GMm1/R 9 •
Writing Newton's second law for the
space.ship in projections on the Y-axis,
we obtain
Fd - F = m1a, or
F, - GMm,JR' - m,a,
whence
Fig. 67 a=~- G:
=~-lo
since GMIR1 =la· This gives
a - (2.94 X tO' X 10-.e - 9.8) mls' - 19.6 m/s1 - 2g,.
Ch. t. Mechanics ..
Let us determine the weight of the hody in the Dying
spaceahip. It should be recalled that the weight of a hody
is the force exerted by it on a constraint (the force with
which it stretches a spring or presses on a support). ln
the case under consideration... the weight of the hody in
~e lb:b~~~d ::~~~fpi.s 1t!~~:'t:f l::::nr:i:
fhi:fe:~e~
let us consider the forces acting on the body in the ship.
There are two such forces, viz. the normal reaction N of
the ship and the gravitational force F" = GMm2/r2 A:#
GMm 21R2 • Writing Newton's second law for the body
in projections on the Y-axis, we obtain
N-F' =m 1a,
whence
N = F' + m2 a = GMm 1/R 1 + m1 a = m2 (GM/lfl +a.).
Considering that the ship and the body move upwards
with acceleration a = 2g0, we ftnd that N = m1 (g0 +
2g0 ) = 3m 1 g0 • According to Newton's third law,
Fpr = N = 3m1lo1
F,, = 3 X 5.88 X 10' N <>< 1.76 kN.
131. Having approached an unknown planet, the
astronauts impart a horizontal velocity of 11 km/s to
their spacecraft. This velocity ensures the ftigh\ of the
spacecraft in a circular orbit of radius 91.00 km. What. is
the free-fall acceleration at the surface of the planet if
its radius is 8900 km?
Given: v=t1. km/s=1..1x11)l m/s, r=9100 km=
9.lx!OO m, R=8900 km=8.9x10' m.
fo-?
Solution. The spacecraft is under the action of only the
gravitational force exerted by the planet and directed to-
its centre. According to the law of universal gravitation,
this force is
F =GMmlr1 ,
where M is lhe mass of the planet, m the mass of the
spacecraft, and r lhe distance from the centre of the planet.
to the spacecraft (the radius of lhe orbit). Multiplying the
.. Selecled Problems on Physics

numerator and the denomillator by R 1 , we obtain


F=G ~: =G~m~=g.,m~. (1)
As for the Earth, here g0 = GMIR' is the free-fall accel-
eration at the surface of the planet. Writing Newton's
second law for the spacecraft in projections on the Y-axis
directed to the centre of the planet, we obtain
F""" ma,, (2)
where
a, = a, = v'IR. (3)
Substituting Eqs. (1) and (3) into (2), we obtain
fr/RR. 1/rl = mvtlr, whence
....
to=""Jli'"•
lo (Li ~J~>~ ~09~! x 10' :. ~ 14 m/si.
132. A body on the equator of a planet weighs half ih
weight at the pole. The density of matter of the planet is

$ 1
Fig. 68

3 g/cm•. Determine the period of rotation of the planet


.about its axis.
Gtven: Peq=Pp/2, p=3g/cm3 =3x103 kgtm•.
T-1
Solutton. The body on the surface of the planet is acted
upon by the gravitational force F exerted by the planet
.and the normal reaction N of the surface of the plane&
(Fig. 68). At the pole, the body is at rest relative to an
Ch. t. Mecbnica 97

inertial referente frame. ln this case,


F +
Np = O or F = N,.
Accordine to the law of universal gravitation,
F =GMmlR',
where m is the mass of the body, M tbe mass of the planet,
and R the radius of the planet. By the de6.nition of densi-
ty, M = pV, wbere V is the volume of the planet. ~
ê4i3::Ji.th:i:/1MP!:º('4~Â)nifa~~ri~~~nweth~·~:::::ti:n:.
force is
F=G 4A3 ",;:"° =(413)nGpmR, (1)
whence N p = (4/3)..CpmR.
On the equator, the body rotates together witb the plan-
et, i.e. bu an acceleration a:c = m'R = 4ic'R/T', where
T is the period oi rotation of the planet. According to
Newton's second law, F - Neq = mac, or
(4/3)nGpmR - N., = 4n'Rm/T'. (2)
Since accordine to Newton's tbird law tbe normal reac-
tion is equal to tbe weiaht of the body and, by bypotbe-
sis, Peq = P,12, Neq = N,12 = (2/3)nGpmR, we obtain
(4/3)..CpmR - (2/3)nGpmR = 4n1Rm/T'. (3)
Finally,

T=if~.
T= i/-,-_,-7x~~~;~~.~-~~,,-x-IO'- s !:!:! 9. 7 x 103 s.
133. A satellite moves in a circular orbit in the equa-
torial plane at an altitude equal to the Earth's radius.
At what velocity must a terrestrial observer. move for'
the satellite to appear above him every 5 s? Çonsider tbe
cases when the directions oi motion óf tJÍe' satellite and
the rotation of tbe Earth coincide and are opposite.
Given: k=R, T=24 h=8.64xl0' s, !=5 h=
t.Sx 10' s.
v-1
1-0970
. Selected Problema on Physlca

Solutton. The satellite moving in a circular orbit is


acted upon by the gravitational force F = GMmlrS
exerted by the Earth. Considering that r = 2R by hy-
pothesis, we transform the expression for the force:
F = GMm/(4R') = (GMIR') (m/4) = g,m/4. · (1)
Here lo = GMIR 2 is tbe free-fall acceleration at tbe
Earth's surface. Writing Newton's second law for the

satellite in projections on tbe Y-BJ(.is directed to the cen-


tre of the Earth, we obtafo - ·
F =ma,, (2)
wbere
a, = ac = v21r = 112/{2R). (3)
Substituting Eqs. (1) and (3) into (2), we obtain
g0 m/4 = m•'/(2R),
whence v =V toR/2. The augblar valooity of tbe satel-
lite is 61=•/r=•/(2Rl=Vg,1(8R)
We shall consider two cases.
1. Tbe satellite moves in tbe direction of rotatiOD of
the Earth (Fig. 69). During a time t, the satellite tra-
verses a distance ABCAB = s = 11t along its orbit and
reaches point B separated from point A by the distance
1AB 1 = 11t - 2:rcr = (J)rt - 2nr = r ((J)f - 2:rc)
= 2R ("'' - 2n). (4)
Ch. i. Meahanice ..
For the satelli\e to be above the observar, the latter must
:~'! Pr~~L it =eª ~~i:Ci~;13: ~e~.~~e=~~~.i;ed~::
locity of the observer relative to the Earth and v1 the
linear veloeity of the Earth. Consequently,
1 A 1B 1 1 = (v, + v,) t. (5)

The ligure shows that the angular displacements of the


satellite and the observer during the time t are the same,
i.e. 'l'J=~,. Since ~ 1 = !A 1 B1 \IR and ~.= \AB!i(2R),
taking into account formulas (4) and (5), we obtain
("1-;u.)t= 2R(~-2n).

Considering that v, = 2nR/T and w = yl.i(Mj, we &nd


that
(1.11+2;_RIT)t Vit-2:n:,
whence

V1=(Yi-~-~)R.
v.=(JÍsx&~i:xi0•-1~::1~ - a~il~!Í)
x 6.37 x t()ll m/s ~ 1.04 X toz m/s.
2. The satellite moves against the direction of rota-
tion of the Earth. We leave it to the reader to verify that
in this case

v.=(Jii-~+~)R~t.txtoa m/s.
134. The average altitude of a satellite above the
Earth's surface is 1700 km. Determine its orbital velocity
and the period of revolution.
GiW1n: h=1700km=t.7x1()11 m.
v-?T-?
,.
iOO Selected Problema on Physics

Solu.tlon. The motion in a eircular orbit oceurs only


under the action of the gravitational force exerted by the
"Earth:
F = GmMl(R + k)', (!)
where R is the Earth's radius. Writing Newton's seeond
law for the satellite in projections on the Y-uis direeted
to the e.entre of the Eartb, we obtain
F= ma,, (2)
where
a,= a,= v'l(R +k). (3)
Taking into account Eqs. ~t) and (3), we transform
Eq. (2): GmMl(R + h)' = m• l(R + k), wbence
•' = GMl(R + k). (4)
Multiplying tbe numerator and tbe denom.inator of the
rigbt-band si de of Eq. (4) by R 1 , we obtain
•' = (GMIR') (R'l(R + k)),
where GMIR1 = lo is the free-fall acceleration at the
Eartb's surfaee. Consequently,
•' = g,R'l(R + k), (5)
whence

V=RV R~I&.
v=6.37 x t()ll y'" 6_37 xto::1.?xtOº ~!:!::!. 7.01x103m/s.
The period of revolution of the satellite in the circular or-
bit of radius R + h is
T= 2n(~+1a) ,

T =2x3.t4x e~--~~ ~~o;+i.7 xtOO) 8 !:!::!. 7.24 x 103 s.

135. Can a satellite be launched so that it remaios all


the time above the same loeation on the Earth's surface?
Answer. This is possible if the period of revolution of
the satellite is equal to the period of rotation of the Earth
Ch. 1. Mechanics 10!

about its axis. and the plane of the orbit coincides with
the equatorial plane of the Earth.
136. Can a match struck in a spacecraft orbiting the
Eart.h bum?
Answer. All the objects on board the spacecraft are in
the zero-gravity state (as long as its engines are switehed
ofl). For this reason, there is no convection in the space-
craft, and the combustion products will accumulate near
the Dame and extinguish it.

EXERCISES
137. Determine the gravitational force exerted by the
Earth on a body of mass 1 kg on the Moon's surface. The
distance between the centres of the Earth and the Moon
should be taken as 384 000 km.
138. Determine the density of a planet whose day lasts
for T hours if the bodies on the equator of the planet are
known to be weigbtless.
139. A satellite completes 16 revolutions during one
turn of the Earth. Determine the period, the altitude,
and the velocity of the satellite, assuming that its orbit
is circular.
140. The velocity of a satellite decreases with altitude
from 7. 79 to 7 .36 kmls. Determine the change in the peri-
od of its revolution and in the separation from the Earth's
surface.
141. An artificial satellite orbits a planet A with a pe-
riod of revolution T1 • Wbat will be the change in the pe-
riod of revolution of the satellite if it orbits a planet B
having the same density as that of the planet A but a
twice as large radius? The satellite moves in both cases in
circular orbits close to the surfaces of the planets.
142. Determine the g-factor for an astronaut rotating
in a horizontal plane in a centrifuge of diameter 12 m at
an angular velocity of 4.04 rad/s.
143. The period of revolution of a satellite moving in
a circular orbit around the Earth is 240 min. Determine
the separation of the orbit from tbe Earth's surface.
ta.1'':~1~~:;:ri~~ t:r~1K:~~~ds:~~li-:1 :.~:n:0!r!11U:.dorfi:;
Selected Problems oa Physics

MOon ata distance of 200 km from its surface if the Moon's


mass is 7.3 X 1ou kg and its radius is 1.7 x 10' m.
sa~~i·~:r!a;.i.~~::i~c~:~º~a~ ~:º~~!:dedf:: :r~:::~
geneous sphere of density p.

MOMENTUM OF A BODY.
MOMENTUM CONSERVATION LA.W

Newton's second law can be written in the form


F!J.1 =l!.(mv),
where F dt is the impulse o( loree and mv = p is the mo~
meotum. of a body.
Let us write Newton's second law in differential form:
Fdt=d(mv), or F=~.
lf a system of interacting bodies is under the action of
externai forces, the following relation holds:

i
i-1
/!. (m 1v 1) = i; F !J.t,
,_,
1

where Am,v, is the change in the momentum of a body of


mass m1 of the system acted upon by an externai force
F 1, and F, l:J.t the impulse of this force.
ln the absence of externai forces (if the system is
closed), the momeotum consenation law is valid:

~ m1v 1 =const,
i•l

where m1 is the mass of an individual body in the system


ofinteractingbodies, v 1 the velocity of this body, and n
the number of bodies interacting in the system.
For a system consisting of two interacting bodies, the
momentum conservation law has the form
m1v1 + msv1 = m1u1 + m1u1
for elastic interaction and
m1v1 + m1v 1 = (m1 + m1)u
Ch. t. Mechaaica 103

for inelasticint.eraction. Here v1 and v 1 are the velocities


of the bodies before their int.eraction, and u 1 , u 1 , and u
the velocities of the bodies after their interaction.
While solving problems, we shall write tbe momentum
conservation law in vector form, choose the directions of
coordinat.e axes, and project botb parts of the vector
equation onto them.

14.6. A body of masa 0.2 kg falis from a height of 1 m


with an aeceleration of 8 m/s1 • Determine the change in
the momentum of the body.
Given: m=0.2 kg, h-=1 m, a=B m/s1 •
<l.(mv)-1
Solu.tton. Tbe body falis witb an acceleration a, and
its velocity cbanges. The cbange in momentum is
A (mv) = mv - mv 0 = m (v - v 0 ). (1)
Projecting both sides of Eq. (1) onto the Y-axis directed
vertically downwards, we obtain
<I. (mv) = m (u - u0). (2)
Since the body starts to fall from the state o( rest, its ini-
tial velocity v0 =O. Tbe final velocity v can be determined
from tbe equation v = V2iik. Using this expression
in Eq. (2). we obtain
<l.(mu)=mV2ah,
<l.(mu)=0.2V2x8x 1 kg·mls=0.8 kg-m/s.
147. A molecule of mass 5 x 10-111 kg Oying at a veloc-
i ty of 500 m/s elastically impinges on a wall at an angle
of 30" to the normal. Determine the impulse of the force
transferred to tbe wall during tbe impact.
Gtven: m=5x10-ze kg, v=500 m/s, m=30"~
0.52 rad.
F<l.t-?
Solutton. We direct the X-axis along the normal to tbe
wall and the Y-a:lis vertically upwards (Fig. 70). New-
... Selected ProhlelDll ou Phyaics

ton's third law implies tbat the impulse of the force


acting on the wall is numerically equal to the impulae of
, y

4u
Fig. 70

the force acting on the molecule. According to Newton's


aecond law, the impulae of the force act.ing on the mole--
cule is
F At = m (v 1 - v1),
or, in projections on the X- and Y-axes,
(F ât)a: = m (vH - vuJ,
(F ât), =- m (v111 - v111 ).
Since tbe impact is elastic, v1 = v1 = v and a 1 ,,,,. a 3 =
a. Therefore, Vu: = -v cosa, vH = 11 cosa and v111 =
-v siu a, 111 , = -v sin a, whence
(F <l.t), - 2mv coe a, (F <1.t), - O.
By de6nition, the impulse of a force is
F<l.t-V<Fàt)~ + (F.1.t);.
Taking into account the expressions for (F ât):ii: and
(F ât)11 , we obtain
F.i.t- V(2mvcosa)'-2mvcosa,
Fàt-2X5X10-Ux 500 X 0.87N·s-4.35X10-U N·s.
148. A shell of mass 100 kg fiying alonga railroad hori-
zontally ata velooity of 500 m/s bits a carriage with sand
of mau tO t and gets stuck in it. Determine the veloc.ity
Cb. t. Mechanica 105

of the earriage after the impaet if it moved towards the


shell at a veloeity of 36 km/h.
Gtven: mi=tOO kg, v1 =500 m/s, m.,,=10 t=tO'kg,
">=36 kmih=!O m/s.
•-?
Solution. Writing the momentum eonservation law for
the inelastic collision between the shell and the carriage
with sand, we obtain
miv1 + m1 v1 = (mi + m1) u. (t)
Cboosing the direction of the X-axis along the motion of
the shell and projecting both sides of Eq. (t) onto it, we-
get m1 lJi - m1 v1 = (m1 + m1) u, whence

U-= m~:+::- '


tOOx500- fO'xtO m
U= 100+t0' -5 m/s.
Consequently, the direetion of motion of the earriage bas
not changed.
t49. A grenade Oying at a veloeity of 15 m/s explodes
into two fragmenta of mass 6 and t4 kg. The velocity of
the larger fragment bas increased to 24 m/s in the direction
of motion. Determine the velocity and the direction of
the smaller fragment.
Given: v=15m/s, u,=24m/s, m1 =6kg, m.,,=14kg.
Ui-?
Solution. Directing the X-axis along the motion of the
grenade and writing the momentum conservation law in
projections on this axis, we obtain
(mi + m1) 11 = m1u,, + m1 u 1 , (t)
wbere mi + m1 is the mass of the grenade. From this re-

..
lation, we get

...
(m1+mJv-m.u1

u 1 =(8+t4)X~-Hx24. ~ -Gm/s.
106 Selected. Problema on Physics

The minus sign indicates that the smaller fragment will


Oy in the direction opposit.e to the direction of motion of
thj~~: ibird 8 stage of a rocket consista of a carrier
rocket of mass 500 kg connected througb a compressed
spring witb a head cone of mass to kg. During terrestri-
al teste, tbe spring imparts a velocity of 5.1 mls to the
cone relativa to the canier rock.et. Determine the veloc-
ities of the cone and tbe rocket if tbey are separated while
orhiting the Earth at a velocity of 8 kmls.
Given: m1 =5xt02 kg, m1 =10 kg, 110 =5.t m/s,
v=8 km/s=8xt01 m/s.
U 1 -?U:1-?
SoluUon. Writing tbe momentum conservation law for
the tbird stage of the roek.et in projections on the X-
axis coinciding with tbe direction of orbital motion of the
rocket, we obtain
(m 1 + m1 ) v = m1 u1 +
m1 u 1 , (1)
wbere m1 is tbe mass of the carrier rocket, m1 tbe mus of
the cone, u 1 tbe veloeity of the carrier rocket relative to
the Eart.h after its separation from tbe cone, and u 1
the veloeity of the cone relative to the Earth after its
aeparation from the rocket.
Since the velocity of the satellite relative to tbe car-
rier rocket after the separation is the same as under ter-
restrial conditions, u 1 - u 1 = 110 , or
U1 = U1 + Vo, "1 = U2 - 110• (2)
Substituting consecutively eacb equation of (2) into
Eq. (!), we obtain
U1=11- m1~"'s 110,

u1 =8x 103- s!ºt~•~\ 0 ~~sx t()I m/s,

Us=11+ mi~ms 110,

Uz=:-8 x toa+ ~~!~:~~i f ~soos m/s.


Cb. 1. Mechanics 107

t5t. Why is the blow of a hammer against an anvil on


the chest of a circus performer harmless, while the same
blow made directly against bis chest would be lethal?
Answer. Since the mass of the anvil is larger than the
mass of the hammer, the velocity acquired by it upon an
inelastic collision with the hammer is small, and hence
the force exerted. on the chest is not very strong. Besides,
the area of contact of the anvil with the performer's body
is larger than that of the hammer. Therefore, the addi-
tional pressure exerted by the anvil on the body during the
impact is lower than the pressure produced by the ham-
mer. For these reasons, the blow is harmless.

EXERCISES
152. A meteorite and a rocket move at right angles
relative to each other. The rocket bits the meteorite and
gets stuck in it. The mass of the meteorite is m, the mass
of therocketism/2, the velocity of the meteorite and the
rocket being 11 and 211 respectively. Determine the mo-
menta of the meteorite and the rocket after the collision.
153. A shell of mass 20 kg, flying horizontally at a ve-
locity of 500 mls, bits a Ratear with sand of mass to t
and gets stuck in it. Determine the velocity acquired by
the Oatcar as a result of the impact.
tM. What velocitywill be acquired by a stationary boat
having a mass of 200 kg with a load as a result of a shot
fired by a passenger in the horizontal direction? The mass
of the bullet is to g and its velocity is 800 m/s.
155. A shell of mass 50 kg, Oying along a railway track
at a velocity of 600 m/s, hits a Ratear with sand of mass
to t and gets stuck in it. The velocity vector of the shell
forms an angle of 45º with the horizontal at the moment
of impact. Determine the velocity of the Ratear after the
collision if the Datcar moves towards the shell ata veloci-
ty of tO m/s.
156. A rocket whose mass without propellaot is 400 g
rises to an altitude of 125 m as a result of combustion of
50 g of fuel in it. Determine the velocity of the gas eject-
ed from the rocket, assuming that the fuel burns instan-
taneously.
108 Selected Problema OD Physics

157. Two balis of mass 6 and 4 kg move along the same


straight line at a velocity of 8 and 3 m/s respectively.
What will their velocity be after a perfectly inelasticcol-
lision if (1) the first ball catches up with the second,
(2) the balls move towards each other?
158. A boat of mass 150 kg and length 2 m, wbich is at
rest on tbe surface of a lake, faces the shore at a distance
~~~·~:1iii! :1t!:n~'Wiliw::1J:o~t 7.?e~\fºes over from the
QUESTIONS FOR REVISlO~

1. Deli.ne force aud maa aud uame the1r SI UDitl of meaaurement.


2. What is density? 3. Formulate Newton's laws. 4. What is iner-
tia? 5. List the types of force in mechauics. 6. Define the momentum
of a body. 7. Formulate the momen&um conservation law. 8. For-
mulate the law of universal gravitation. What is the phyaical mean-
:.~1w1:1Ja ~v~'it~~ud!l :J:\:J~ ~':!Yi:e~~·~º!'!r~~"D:!
the maa of the body chanp in the proceu? to. Define the orbital
and escape veloeit.iea.

1.J. Work, Power, and Energy.


Energy Conservllllon Lllw
The work done by a constant force is defi.ned as
A= Fscoscz,
where F is the force acting on a body, s the displacement
of the body under the action of the force, and cz the angle
between the directions of the force and the displacement.
If cz < n.12, then A > O, if cz = n.12, then A = O, and
if cz >n./2, then A <0.
If a varying force acts on a body, then
A =(Fcoscz)s,
where (F cos cz) is the mean value of the projection of the
force on the displacement. A more rigorous definition of
the work done by a varying force is

A-i'.. F, ds,
Ch. 1. Mechaaics 100

where F. = F eos ci is the projeetion of lhe force on lhe


displacemenl.
Power is defi.ned. as
N=Alt,
where t is the time during which the work is done.
For a uniform motion,
N =1'11,
where 11 is the velocity of the motion.
For a nonuniform motion, the concepl!I of lnstantaneous
and average power are introduced:
N=4f.=Fv, (N)=F(v)=Alt,
where 11 is the instantaneous velocily of the motion and
(11) the average velocity of lhe varying motion.
The ef6.eieney of a mechanism is de6.ned as
fl = AulA4, fl = NulN4, '11 = WulW4,
where Au (N•• Wu) is the useful work (power, energy)
of the mechanism and A 4 (N4 , W4 ) is the work (power,
energy) done by lhe mechanism.
There are two types of mechanical energy: klnetle ener-
gy and potenUal energy.
A moving body has a kinetic energy
wk = m1.12/2,

where m is lhe mass of the moving body and 11 ili velocily.


A body localed above the ground has a potential energy
Wp =mgh,
where h is the height of lhe body above the ground.
A compressed or slretched spring has a potential ener-
gy

where k is the spring constant and z the compression (ex-


tension) of the spring.
The total mechanieal energy W of a body is the sum
of its kinetic and potential energies:
W= W,+ w,.
110

A system of bodies is eloaed if bodiea whicb do not con-


stitute the system do not act on it. For a cloaed system of
bodies (in whicb only conaervative forcea are acting),
the total mechanical energy remains constant (the law of
eonaervatlon of mechanlcal energy):

wbere W, is the total mechanical energy of an individual


body of the system and n the number of hodiea in the
system.
For a body moving at a velocity v at a heigbt h above
the ground, the law of conservation of mechanical energy
has the form
mv'/2 + mgh - co..t.
When externai forces are acting on a body (system of
bodies), the total mechanical energy of the body (system
of bodiea) changes. The following relation holds in this
case:
A-6W-W-W,,
where A is the work done hy the externai forces, d W
the change in the total mechanical energy of the body (sys-
tem of bodies), and W and W0 the final and initial values
of the mechanical energy of the body (ayatem of hodies).
The same relation is valid when nonconservative forces
(like friction) are acting in a closed ayatem of bodiea.
ln thia case, A is the work done by the nonconservative
forces.

159. A hullet ftying at a velocity of 400 m/s bits a bar-


rage and comea to a halt over a distance of 0.5 m. Deter-
mine the reaiaiance offered to the motion of the hullet if
ita mau ia 24 g.
Glwn: v0 =400m/s,s-0.5m,m=24g=2.4x 10-•kg.
F-1
Ch. t. Meahaniee IH

Fig. 7f Fig. 72

Solution. We direct the X-axis along the motion of the


bullet (Fig. 71). Since a. = :n:, cosa. = -1, and
A= -Fs. (!)
On the other hand,
A= W-W,. (2)
By hypothesb, W = O (the bullet has stopped), aod
w, = .W,12. (3)
Substituting Eqs. (1) and (3) into (2), we obtain -Fs =
-mVJ2, whence
F=.!!jf-,
F""'" 2.4 Xi~~.~ 4001 N ~ 3.8 kN.
160. A traio of mau 600 t acquires a velocity of
60 km/h over a distance of 2.5 km from the previous sta-
tion. What is the average power developed by the locomo-
tiva if the coef.6.cient of friction is 0.005?
Given: m=600t=6 x IO'kg, s-2.Skm= 2.S x !O'm,
v=60 km/h=l6.7 m/s, µ=Sx 10-'.
(N)-?
Solution. We direct the X-axis aloog the motion of th&
traio. The traio is acted upon by tbe force of gravity
mg, the normal reactioo N of tbe rails, the driving force
F, and the friction Frr (Fig. 72). Since the change in the
kinetic energy must be equal to the work done by exter-
nai forces, AW =A, or
W - W0 = A1 + A 1 , (!)
112 Selected Problema OD Physics

where W = mr1'/2, W0 = O, A1 = Fs is the work done


by the driving force, and A 1 = -FrrS the work done by
friction. The work done by the force of gravity and by
the normal reaction is zero since these forces are perpen-
dicular to the displacement. For a horizontal surface,
friction is defi.ned as Frr = .,._N = ..-.mg. Substituting the
expreasionsfor W, W0 , A1 , and A 1 into Eq. (1), we obtain
mv112 = Fs - p.mgs. Then the driving force is

F= m;.• +JURg=m(~+p.g). (2)

By delinition, the average velocity of a uniformly varying


motion is
(V) = (v + v )/2 =
0 v/2 (3)
since v0 = O. By definition, the average power is
(/V)= F(v). (4)
Substituting Eqs. (2) and (3) into (4), we obtain

{N)= ~v (~+P.t),
<N>= ext0'2xte.1 ( zx~~S:toa +sxto-•x9.8) w
<><0.52 MW.
161. A load of mass 2 kg falling from a height of 5 m
tJenetrat.es a soft soil to a depth of 5 cm. Determine the
nerage resistance of the soil.
Given: m=2 kg, h =5 m, h 1 ,,,,..5 cm=0.05 m.
(F)-1
Solution. We direct the Y-axis vertically upwards and
take the origin at a depth h1 from the surface (Fig. 73).
An externai force (resistance of the soil) is acting on seg-
ment CO, and hence
AW=A, or W-W 0 =A, (1)
where W 0 = mgh + mgh1 is the mechanir.al energy of the
load at point B and W the mechanical energy of the load
Ch. 1. Meehanict

at the depth h.1 from the surfaee. Since 11 = O at point O


and v = O, W = O as well.
The work done by externai forces on segment CO is
A= -(F) h.1 • Subsütuüng the expressions for W0 ,
W, and A into Eq. (1), we obtain O - mgh. - mthi =
-(F) h,,, whenee

(F)=m1(-i;--+ 1),
(F)=2x9.8x (ok + 1) N ""1.98 kN.
162. A block slides Jirst down an inclined plane of
len.gth 42 cm and height 7 cm and then over a horizontal
plane. Havin.g covered a distance of 142 em along the
horizontal, it stops. Determine the coeffteient of frietion,
assuming that it is the sam~ everywhere.
Given: l 1 =42em=0.42m, l1 =142em=1.42m,
h.=7 em=0.07 m.
µ-1
Solutlon. Let us eonsider the motion of the bloc.k on two
segm.ents: the inclined and horizontal planes (Fig. 74).
Durin.g the motion down the inelined plane, the block ex-
periences the acüon of the force of gravity mg, the nor-
mal reaction N1 of the inelined plane, and the frietion
y

_;o
Fig. 73 Fig. 7'
8-0170
... Select.ed Problema on Phy1lcs

F rri which ia de&ned 81!1


Frn = p.N1 = .,_mg cos ct. (!)
During the motion over the horizontal plane, the block
is acted upon by the force of gravity mg, the normal re-
action N1 of the horizontal plane, and the friction F Ir•
given by
Ft 11 = .,.H, = .,.mg. (2)
The change in the total mechanical energy of the block is
equal to the work done by the frict.ional forcea F rri and
Frn
6W-A, (3)
wbere
6W - W - W, - O - mgh - -mgh,
(4)
A = .A. 1 + A, = -Ftr1l1 - F 1r2l 2•
Suhltituting expre81!1ions (4) into (3), we obt.ain -mgh =
-F rr1 '1 - F rr1 11 • Taking Eqs. (1) and (2) into ac-
count, we obtain mgh = .,_mg (l,, coa ct + l1 ), whence
p. = h/(l1 coa ct + 11). The figure ahowa that cos ct =
V 1: - h'll1• Then
p.=

Vi:-11•+i.'
.,._ V0.421 ~:011 +1.c ,.., o.04.
163°. Determine the work done in order to compress by
20 cm a spring whose constant is 29.4 N/cm, assuming
that deformat.iona are elastic.
Gtven: z 1 =0, z 1 =20 cm=0.2 m, k=29.4 N/cm=
2.94 X 10' N/m,
A-?
Salutian. Methotl 1. The work done to compress the
spring is equal to the change in its potent.ial energy:
A=6W--!!jl,
A- 2.9'.XiO'X0.2• 1=58.8 J.
Cb. 1. Mechaaica li•

Method 2. By definition, the work donetocompreu the


spring is

A= r.Fcosadz

..1 =';' J::= ~ -58.8 J.


=
. (-kz)(-!)d•

164º. A sledge moving over the horizontal surface of


ice ata velocity of 5 m/s drives outona road and comes

Fig. 75

to a balt. Determine the distance covered by the sledge


on the road if the sledge is 1 m long and the coef6.cient of
friction between the runners and the road is 0.5 (Fig. 75).
The friction between the runners and the ice sbould he
neglec;ted.
Gtven: p.=0.5, v0 =5 m/s, l=i m.
L-? . .
SoluUon. We divide the distance covered hy the sledge
over the rough surface of the road into two segmenta. On
the 6.rst segment equal to the length l of tbe runnen, the
friction is varying since the coeflicient of friction in·
creases from zero to its value for the road. On the second
segment 11 , wben the runners leave the ice completely,
the friction is constant. Let us calculate the work done hy
the friction on these two segments: Âtr = Atn + Âtrt·
We suppose that the sledge has covered a part z of the
first segm.ent (see Fig. 75). Since the friction increases in
proportion to the covered distance, the friction acting on

..
the sledge can he written as F tr = p.mgzll. Tben the work
116 Selecled Problema on Phy1ic1

done by the frictioo on the first and second segmeot8 i8


1 1
Arn=~ F 1 rcosad.i:--~ '7' zdz
o o
= -.ffl--f I: = _ P~ri ,
A 11'2= -p.mgl1•
The total work done by the frictioo on these seg-
ments is
A 1r= -µmg({-+1 1 ) . (1)

eale~ied°: 1~:..!~s~1 Í~Í~~ ~ ::i!ri~O:rnto'°d1isc;f.r!!:>eat


i.e. F1r = p.mrzll, and ioereases from O to Fma:r. = pmr, the
nerage frietion is
(F) = (pm1 + 0)/2 = p1111/2,
whence the work clone by lhe lriction on this segmeat is
A1 = -(F) l = -pmrl/2.
On the other hand, the work done by friction is equal to
the change in the kinetic energy of the sledge: A rr =
.âW = W 1 - W1 • By hypothesis, W 1 =O, W1 = mV:/2,
and hence
A 1, = -W 1 = -mV:/2. (2)
Combining Eqs. (1) and (2), we get _,..., (1/2 + ~) =
-mV:/2, whence
1, = (~ - µgl)/(21'/1)·
The distance covered by the sledge is
L -l+l -1 ,_
- v:....:,.,, -
,- '--.;;;--- ,,.,
vl+,.,z .
L 51 t~·g.~ !·:.; 1 m ~ 3.1 m.
t65º. A load of mass 5 kg falis freely from a certain
height and reaches the ground in 2.5 s. Determine the work
done by the force of gravity.
Gtven: m = 5 kg, t = 2.5 s.
A-1
Ch. 1. Mechanic1 111

Solutlon. The total work done by the force of gravity is


o
A=\
Fcoaadh,
H
(1)

where k is the height from which the load falis, F =


mg Ui the force of gravity acting on the load, and e.os ci =
-t. We transform expreuion (t) as follows:
o
Â= -i mgdk= -mgkl~=mgH.
H

Since H = gt2/2, we have

A=mg~=~,
A sx9.s;x2.s• J~t. 5 kJ.

166. A load of mau 0.5 kg falls from a certain height


on a slab of mass t kg, &xed to a spring whose constant
is k = 9.8 X t()I N/m. Determine the maximum compres-
sion of the spring if the velocity of the load at the mo-
ment of impact Ui 5 m/s and the impact is inelastic.
Given: m,=0.5 kg, m,=I kg, k=9.8x 10' N/m,
11=5 m/s.

z-?
SaluUan. According to the energy conservation law,
the total mechanical energy of the load and the slab after
the impact is equal to the potential energy of the com-
pressed spring:
(rn, + m,)u'/2 + (m, + m,) gz = kz'/2, (1)
where m1 is the mass of the slab and u the velocity of the
load and the slab after the impact, which can be det.er-
mined from the momentum conservation law for an in-
elastic collision: m111 = (lni. + m1 ) u, whence
(2)
UB Select.ed Problema on Pbysics

Substituting Eq. (2) into (t), we obtain

"'1t"'• (m;!":i.,, +(m.+mz)gz=~'


or
k:r.2-2g(m1 +m.i):r.- ,,.~!':. """º•
whence
1(m1 +m.>+ 1'I' (m1 +m.l 1 +kmfv1/(m1 + m.)
•=

9.8(O.S+t>+1'9Jí(0.5 + i)•+9.8 X toa X 0,5• X 51/(Õ.s+t)
s.sxtos
<><8.2x to-< m.
(The negative root does not satisfy the conditions of the
problem.)
f67. The slope of a highway is 1 m per 20 m of lhe road.
A motor car with its engine eut out moves down the slope
at a constant velocity of 60 km/h. Determine the power
of a car movinr up the road at the same veloeity if the
mass of the car is t .5 t.
Gl,,.n: sina= h/l = 1/20 = 0.00, v = 60 km/h<>< 17 m/s,
m <>< 1.5t=l.5X10' kJ.
N-1
Solutton. Since the motion of the car movine upwards is
uniform,
N=Fv, (1)
where F is the driving force of the engine durinr the ascent
and v the veloc.ity of the car. The car moving upwards is
acted upon by the force of gravity mg, the normal reac-
tion N of the highway, the driving force F, and the fric-
tion Frr (Fir. 76a). Writing Newton's second law for the
:' ~nã'°th:!io;.s~°F:!'8-x~,an!ny~ª!'ô ((a !ig· !f:'J~
v = const), whence
F = Frr + mgsincz. (2)
Ch. t. Mechanics 119

Tbe car moving downwards experiences t.be action of the


force of gravity mg, the normal reaction N, and the fric-
tion Frr (Fig. 76b). ln analogy witb tbe previous case,
we can write mi sin a. - F rr = O, whence
Frr = mg sina.. (3)
Substituting Eq. (3) int.o (2), we obtain F = 2mg sina..
Then Eq. (1) becomee
N """2mg sin a.·11,
N = 2 X 1.5 X 10' X 9.8 X 0.05 X 17 W"' 25 kW.
168. Determine the work done for lifting a load on an
inclined plane, the average power, and the efliciency of
the lifting mechanism if the mau of the load is 100 kg,
tbe length of the inclined plane is 2 m, the slope of the
plane is 30°, the coeflicient of friction is0.1, and the accel-
eration during the ascent is 1 m/s". Tbe load was initial-
ly at rest at the foot of the inclined plane.
Giuen:m=100kg, 1=2m, a.=30",.0.52rad,
~=0.1, •=1 m/s'.
A-? (N)-? ~-?
Solution. The cbange in the total mechanical energy is
caused by the driving force and the friction acting on the
load:
A +A,,- Ll.W= W-W, = W. (1)
Here A is the work done by the driving force and A rr
the force done by friction (W0 = O by hypothesis). The

Fig. 78
12D Selected Problema OD Physics

final value of the total meehaoical energy is


W = mv'/2 + mgh = mal + mgl sin cz
since vS = 2al.
The force of friet.ion oo ao inclined plane is defined as
Frr = µN = IURfCOSci, and hence Arr = -11mglcosci

y P CA
T,,... -

8
mg mg
Fig. 77 Fig. 78

(Fig. 77). Substituting theae expressioos ioto Eq. (t),


we obtaio
A= ml (a+ gsio cz + p.gcosci),
A = 100 X 2 X (t + 9.8 X 0.5 + 0.1 X 9.8 X 0.87) l
= t.35 kl.
The average power of the liftiog meehanism is
(N) =Alt,
where t is the time of liftiog the load, which can be ob-
tained from the equatioo for a uniformly accelerated mo-
tion:
l = at1/2 (v, = O). (2)
From Eq. (2), t=VW.. Co...quently,

(N)=~·
(N)= ~ W=675 W.
Cb. i. Mechanic11 121

By definition, the efliciency of the lifting mechanism is


'I = AulAd•
where A,= A = 1.35 kJ.
The useful action of the lifting mechanism is the dis-
plaeement of the load to the height h = l sin ci. Therefore,.
Au = mgl sina;,
A, = 100 X 9.8 X 2 X 0.5 J = 980 I,
whenee
~ = 980/(1.35 X 10') <:>< 0.73.
169. A bali of mus m suspended on a string of length l
is deOected through an angle of 900 from the vertical and
released. Determine the ma:dmum tension of the string~
Giwn: m, l, a=900~1.57 rad.
Tmu-?
Solutton. The tension of the string attains its ma:dmum
when the bali passes through point B. At this point, th&
ball e:iperiences the action of the force of gravity mg
and the tension Tmn of the string (Fig. 78). WritingNew-
ton'ssecond law for the bali in projections on the Y-a:lis.
we obtain
Tmn - mg = ma11 , (1)
whAl,re ª'- = v1 /R and R = l. Using these expressions, we
can wnte Eq. (1) in the form T ma:r. - mg = mv'll,
whenee
Tma:r. = mg + mv'll. (2)
ln order to determine the velocity of the ball at point B,
we apply theenergyconservation law for points A and B-.:
WA = w.. (3)

:;,c8= !~12.into::~n~:dvs
sion for vi
~ª 2;1.~~:!Ut~Ü~g~lie1:;::
Eq. (2), we oht.ain
T mn: = mr + m·21Ul = 3mg.
122 Selecied Probleme on Phyeics

170. A satellite of mass 12 t moves in a circular orbit


around the Earth so that it.a kinette energy is 54 GJ.
What is the velocity of the satellite and its altitude?
Glven: m-12t-1.2x10' kg, W,-54 GJ-
5.4X10" J.
v-1h-1
Solution. The kinetic energy of the satellite is Wt =
mv212, whence
(1)
Tben
v=Jf 2!• •
v=Jf 2 ~~2~~J.°10 T=3xt03 m/s.
The altitude h of the satellite and its velocity v are con-
nected through the following relation:
v' - g,R'l(R + h) (2)
(see Problem 134, formula (5)). Equating the right-hand
sides oi Eqs. (1) and (2), we obtain 2W,lm-g0 R'l(R +h),
whence
h=mg 0 ;t -R=R( ~: -1),
h=6.37x 108x ( t.2xtf:s~:;tg;!7xtoo t) m
~3.8x107m.
171. Wbat is the altitude attained by a rocket launehed
vertically at a velocity of 9 km/s? Air resistance should
be neglected.
Given: v = 9 km/s = 9 X 1()3 m/s.
h-1
Solutton. A rocket of mau m is acted upon by a vary-
ing gravitational force
F = GmM/r', (1)
where M is the Earth' s mass and r the separation between
the rooket and the centre of the Eartb.
Ch. t. Mechanic9 123

...
As the rocket attains an altitude h, this force will do
the work

.
A= ~ Fcoscidr, (2)

where a = n rad is the angle between the directions of the


force and displacement ond R the Earth's radiwi. Sub-
stituting Eq. (1) into (2J, we obtain

•A= -
•+•
( GmM dr= GmM 1•+•
~ri r R

=GmM ( R~. -j-) = - R~O). (3)

On the other hand, the work is equal to the change in the


kinetic energy of the rocket: A = âW = W 1 - Wl'
At the altitude h, W1 =O, and hence
A = -W, = -mv'/2. (4)
Equating the right-hand sides of expressions (3) and (4),
we ohtain
GmMh/IR (R + h)I = mv'/2. (5)
Multiplying and dividing the left-hand side of Eq. (5) by
R, we ohtain
RhGM/l(R + h) R'I = v'/2. (6)
Since GM/R1 =lo is the free-fall acceleration at the
Earth's surface, Eq. (6) can be written in the form
Rhg0 (R + h) = v1/2, whence

h= 2R;:8_D' '
h 2 x 6~~:~~J:~::;r_x;~t<P)• m~t.t7xtar m.
172-'. Calculate the escape velocity which should he
imparted to a rocket at the Earth's surface.
Given: f 0 =9.8 m/s3 , R1 =6.37xtO'm, R1 =00.
v-1
12' Selected Problem11 on Phyaics

SolutWn. The rocket of mau m ia acted upon by the


varying gravitational force
F = GmM/r'. (1)

Tbe work done by thia force ia


..!
Â=
.. Fcosadr.

Conaidering tbat a = n rad and substitut.ing .F.4-


(2)

(1)
into (2), we obtain

On the other hand, A = AW = W, - W1 • Since


W1 = O, we bave
A = -W, = -nw'/2. (4)
Equating the right-hand sidea of Eqs. (3) and (4), we ob-
tain GmMIR 1 = rnv'-/2, or (alter transformat.ions similar
to thoae carried out in the previoua problem), toR1 =
v1 /2, wbence
V= J/2K.lf.,
V= ]/2 X 9.8 X 6.37 X 1()1m/s"'11.2 X !O' m/s.

~:d ~:~ll~~:l!?~d~~!.~h: ~°i!i::::1i:i~-~~~1~:


of the bullet is equal to 0.001 times the mau of the bali.
The distance between the point of suspension of the rod
and the centre of the ball is 1 m. Determine tbe velocity
of the bullet if the rod witb the bali is known to be de-
Dected through 10" as a reault of impact.
Given: mz=1000m1 , 1=1 m, a=100~0.t7 rad.
v-?
Solutton. Writing the moment.um conservation law for
the inelutic impact in projections on the X-axis
Ch. t. Mechanics 125

(Fig. 79), we obtain m1v = (m1 +m 1) u, whence


V=
m, "·
m1+ma (1)

Here v is the velocity of the bullet before tbe collision


and u the velocit.y of the bali and the bullet after t.he col-
lision. ln Eq. (1), t.here is one more unknown velocity
u in addition t.o v, which can be det.ermined from the

Fig. 79

energy conservation law. Let. the centre of mau of t.he


bali be raised to a height h as a result. of the collision
with the bullet.. Tben by the energy conservation law,
(m 1 + m1 ) u1/2 """ (m1 + m1 ) gh, whence
u1 = 2gh. (2)
1t can be seen from the figure t.hat h. = l - l cos ~ =
l (1 - cos ~). Substit.ut.ing this expression for h int.o
Eq. (2), we obtain u1 = 2gl (1 - cos ~). whence u =
V21l (1 - cosa). Tben Eq. (1) can be reduced to the
form
•- m,:,.., v211<1-co•a). <ª>
Using the trigonometric relat.ion sin (cz/2) =
V (1 - cos ~)12, we t.ransform expression (3) as follows:
v=2 mi~"'a sinf Jffi,

v=2 m1 +1000m1 0.09'V9.8xt.!!..~57om/s.


m, '
126 Selected Problema ou Physic1

174. A man standing on a trolley pushes another


trolley so that they are set in motion and stop after some
time doe to friction. Determine the ratio of the stopping
distancesof thetrolleysifthemauofthe first trolley with
the man is thrice the mau of the second trolley.
Given: m1 =3m1 •
•1'•2-?
Solution. Writing the momentum conservation law
for an elastic interaction between the trolleys in projec-
tions on the X-axis directed along the trajectory of the
first trolley, we obtain
(1)
where u1 and u 1 are the veloci ties of the fint and second
trolleys immediately after tbe interaction.
Equation (1) implies that
(2)
Transforming Eq. (2), we get by hypothesis 3m 1 u 1 =
m 1 u 1 , whence
(3)
The change in the kinetic energy of each trolley is equal
to the work done by friction: âW =A. Considering that
õ.W =O - mu1/2 and A= -Frr1 = -µmg8, we obtain
-mu1 /2 = -µmg8, or

ü•t2 ~I•. (4)
Then relation (4) for the first and second trolleys can be
written in the form
u:/2 = µgs1, u:12 = µg81• (5)
Dividing the first equation in (5) by tbe second termwise,
we obtain 8 1/s1 = u:tu: or, taking into account relation (3),
81/81 = u:t(eu:> = t/9.
175. The beetle head of a pile driver bas a mass of
400 kg and falis on apipe of mass 100 kg wbich is driven
into the ground. Determine the average resistance of the
Ch. t. Mecbanics 127

ground and the efficiency of the pile driver if the pile is


known to sink into the ground by 5 cm upon each blow,
the height from which the beetle bead falls is 1.5 m, and
tbe impact is inelastic.
Given: m,=400kg-4x102 kg, m,=100 kg=102 kg,
•=5 cm=5x t~ m, h=t.5 m.
(F)-1~-1
Solution. The work done by the resistance of the ground
is equal to the change in the total mechanical energy or
the system of bodies (tbe pile and the beetle head):
A = àW = W - W, = -W0 (1)
(W = O by hypothesis).
By definition, tbe work done by the resistaot~.e force is
A= -(F) r. (2)
The initial mecbanical energy of the system is equal to
the sum of the kinetic and potential energies of tbe pile
and the beetle head after ~heir collision, i.e. W0 =
W P + W k· Tbe kinetic energy can be preaented in the
form Wk = p 1/l2 (m1 + m1 )], where p is the momentum
of the pile and the beetle head after the collision. Since
W p = (m1 + m1 ) p, we have
W 0 = (m, + m,) gr + p'/(2 (m, + m,)J. (3)
According to the momentum conservation law for ao in-
elastic collision, we have p = p 1 , where p 1 is the momen-
tum of the beetle head"before it strikes against tbe pile.
The mecbanical energy conservation law for the beetle
head falling from a height h (Fig. 80) implies that
P:/(2m1) = m 1gh, and therefore
p 1 = p: = 2m:gh. ( 4)
Substituting relations (2], (3], and (4) into Eq. (1], we
obtain
(F)=(m1 +m1)g+ (m;!'!.l• ,

<F>=9.8x(4xt02+102)+ c•~~~~;~·~"."x'i~ N
<><9.9X10< N.
128 Selected Problema on Phy11ic11

Tbe efficiency of the mechanism is defined as


~ - "·'"•· (5)
The work done is A d = mgk = p:t(2m), while tbe \188-
ful work A u = W k• where W k is the kinetic energy of the

Fig. 80

pile and tbe beet)e head after the eollision, W k =


p 1/(2 (m1 + m 2JI. Sines p = p 1 , subatituting these ex-
pressions into Eq. (5), we obtain

'1=m1~"'s 1
~ - 4004: 100 -o.s.
176. A washer of maSl!I 10 g rests at the top of a smooth
hemisphere of radius 0.5 m. The washer start.s to slide over
the hemisphere under the action of a horizontal short im-
pulse of force of 2 x 10-2 N ·S. At wbat beight from the
base of tbe hemisphere will tbe washer be separated from
its surface?
Gtven: R-0.5 m, m-to g-to-• kg, Fl!.t-2x
10-0 N.s.
H-?

m::::~=~ ~:i:8u!~~rmtb::~i:~ S:f:~:i;::c;e t~; ;::~:;


mg and the normal reaction N of the spherieal surface.
Writing Newton's seeond law for the washer in projec-
tions on tbe Y-axis directed along the radius towards t.he
Ch. 1. Mechanica 129

centre of the circle (Fig. 81), we obtain


mgcosa.-N=mac,
where ac=vl/R and cos a.=HIR. At the moment of
N

Fig. 81

separation, the normal reaction vanisb.u. Therefore,


mgHIR=mv'lR, or mgH=mv'. (1)
According to the law of conservation of mechanical ener-
gy, we have
mv'/2 + mgH- (F àt)1/(2m) + mgR, (2)
where F ât is the impulse of the washer at the top of the
hemisphere. From Eq. (2), we obtain
mv' = (F àt)'lm + :ir,;, (R - H).
Then
mgH=(Fàt)'tm+2mgR-2mgH,
H=fR+( F,:t )2 ~ '
H=~-+- 2~;~· 3xt9.8 m~0.47 m.
177. A body of mass m slides down a hill of height h.
What work should be done to lift the body to the top of
the hill if the force is directed along the displacement?
Given: h, m.
A2 -1
t30 Selected Problems on Physics

Solutton. Since the body sliding down the hill comes to


a halt, its total mecha~ical energy has changed. The
change in the total mechanical energy of thebodyduring
sliding must be equal to the work done by friction:
!J.W1 = A1 . Since
!J.W1 = W1 - W01 =0 - mgh= -mgh, (t)
we obtain A 1 = -mgh.
The change in the total mechanical energy of the body
during the ascent is
AW, =A, +A,. (2)
Here !J.W1 = W 2 - W01 = mgh - Q,,,,, mgh, A 2 is the
work done by the driving force, and A 9 the work done by
friction during the ascent. Since the driving force is di-
rected along the displacement, the normal reaction of the
support (and hence the friction) will be the same as for
lhe body sliding down the hill. Therefore, A 9 = A1 , and
taking Eq. (t) into account, we obtain
A,= -mgh. (3)
Finally, from Eqs. (2) and (3) we get
A,= AW, - A,= mgh - (-mgh) = 2mgh.
178. A load is lifted to a height h and then uniformly
moved over a horizontal surface through a distance h.
ln which case will the work done be larger? The coefli-
cient of friction between the load and the surface is p.,
and the air resistance should be neglected.
Answer. The work done for lifting the load of mass m
to a height h is mgh. The work done during the uniform
displacement of the load over the horizontal surface
through a distance h is p.mgh. Since thecoeflicient of slid-
ing friction is known to be always smaller than unity,
the work done in the latter case will be smaller.
179. How should a ball be thrown to the ftoor from a
height h for it to be bounced to a height H larger than h?
Answer. The ball should be thrown at a nonzero initi-
al velocity. Then the initial mechanical energy of the ball
is mgh -+- mv'/2. If the impact of the ball against the ftoor
Ch. i. Meth•Die& 131

is elastie, it will jump to the height H whieh can he de-


termined from the energy comervation law mgh. +
mv"l2= mgH. This equation shows that H >h..

EllRCISES

180. A traio of mass 500 t movesup the hill withaslope


of 10 m per kilometre of the railroad at a velocity of
30 km/h. The eoefficient of fri&tion is 0.002. Determine
the power developed by lhe locomotive of the traio.

Fig. 82

181. A stone sliding over a horizontal surface of ic.e


stops after eovering a distance of 48 m. Determine the
initial velocity of the atone if the coefficient of friction is
0.06.
182. A hullet of mass 10 g, Dying at a velocity of
400 m/s, pierc.es a 5-cm thick hoard, after which its ve-
locity decreases to half the initial value. Determine the av-
erage resistance offered by the hoard to the motion of the
hullet.
m~! !p t:~ ;tr~:·~ ~º~~ to:n:J.ow:h~~ ~ ~
maximum velocity of the tank?
UM. A cyclist must make a loop of radius 8 m in a ver--
tical plane. From what minimum height must the cyclist
start to make the loop without falling (Fig. 82)? Fric-
tion should he negleeted.
185. The heetle head of a pile driver of mau 500 kg
falis on a pile of mass 100 kg ata velocity of 4 m/s. De-
termine the effic.ieney of the pile driver for elastic and
inelastic eollisions.
132 Selecled Problema on Physics

186. A chandelier of mass tOO kg is suspended from a


ceiling on a metal chain whose length is 5 m. What is the
maximum height to which the chandelier can be defiected
wilhout breaking the chain during swings if the rupture
of the chain is known to occur at a tension of 2 kN?
187. An ice hockey player whose weight is 70 kg
throws a puck of mass 0.3 kg in the horizontal direction
ata velocity of 10 m/s. By what distance will the player
move backwards if the coefftcient of friction between
the skates and ice is 0.02?
188. The spring of a toy pistol has a constant of
10 N/cm and a length of 15 cm. To what height will a
ball of mass 10 g shot from the pistol in the vertical di-
~i~ti~:si:~S:n~~ 1.::o!fdin:e h::g::ce~d~ompressed to 5 cm?
189. A box with sand wbose mass is 10 kg is held by a
spring constant of 30 N/cm. A bullet of masa 10 g moving
ata velocity of 500 m/s bits the box from below and gets
stuck in the sand. Determine the contraction of the spring.
190. Two loads whose masses are in the ratio 1 :4
are connected through a compressed spring and rest on the
horizontal surface of a table. When the spring is released,
the load of the smaller mass acquires a kinetic energy of
40 J. Determine the potential energy of the compressed
spring neglecting friction.
191. A bali falling from a height of 3 m is bounced to
2.5 m after the impact against the Door. How can this be
put in accord with the law of conservation of mechanical
energy?
QUESTIONS FOR REVISION
t. Define energy. 2. What is the kinetic eaergy of a body? 3. What
is the difference between com1ervative aad nonconservative fon:ee?
4. What is the tential ene
for ·
Ch. 1. Mechanics 133

t.4. Stdcs
Statles deals with the equilibrium conditions for a body
under the action of applied forces. Equilibrium is a st.ate
of rest. or of uniform rectilinear motion or rotation.
Equilibrium can be stable, unsllble or neutral. ln equi-
librium, the potent.ial energy has ao extremai value. Ana-
lytically this is written in the form
d:p =0.
The condition for stable equilibrium is that t.he poten-
tial energy must be minimum, which is equivalent. to t.he
following mathematical condit.ion:

~>º·
The necessary equilibrium condition for a point mass
is the equality to zero of t.he sum of ali the forces applied
to it:

...:t F 1 =0 .
lf we project. ali the forces act.ing on t.he point mass on-
to t.he X- and Y-axes, the equilibrium condition assumes
the form

:t F ,=0, :t F.,=O.
t-t
1
f=t

The equilibrium of a rigid body depends not only on the


magnitude and direction of forces acting on it but also
00
i:: .::!:.~~fff!ê~~t!~ ~~of:~~·axis is defined
as the product. of the force F by the arm l (i.e. t.he per-
pendicular dropped from the rotational axis on t.he line of
action of the force):

The moment of force tending to rot.ate a body counter-


clockwise about ao axis is assumed to be positive and the
one rotating a body clockwise negative.
... Selected Problem1 on Pby1ics

The following two conditions are nec8188ry for the equi-


librium of a body:
1. The vectorsum of all the forces applied to the body
must be zero:
t,F,=0,
wbere n is the number of forces.
2. The algebraic sum of the torques about any axis must
be zero:

i: M,~O,
,_,
where n is the number of torques.

192. Aloadof maS110 kg is balanced by two loads, the


mass of the second load being 18 kg. The string holding
the third load is direc:.ted from point A along the horizon-
tal (Fig. 83a). Determine the mass of the third load and
angle ci.
Giwn: m 1 =10 kg, mz= 18 kg.
m1 -? u-?
Solution. Point A is acted upon by the tensions T1 ,
T 1 , and T 1 of the strings (Fig. 83b). Since the system of

mzg m,9 m,g


(a} (ÍJJ
Fig. 83
Ch. 1. Mecbanies 135

bodies is stationary, we can write for each load


T~=m 1 g, T;=m 1g, T;=mJI. (1)
The equilibrium condit.ion for point. A has the form
T, + T, + T, =O, (2)
where T1 = T~. T1 = T;, ao.d T1 = T;. Choosing the
directions of the X- and Y-axes and projecting Eq. (2)
onto them, we obtain
-T, sin"' + T•=O, (3)
T, eos ex - T,=0. (4)
From Eqs. (4) and (1) we get. cos ti= T1/T 1 =m1/m1 ,
whence
ti = arceos (m1/m1),
"' = arceos (IOl18) "' 0.98 rad.
Equation (3) gives T1 = T1 sin ti, or maf = m.g sin "•
whence
m, = mssin "·
m, = 18 X 0.832 kg <>< 14.9 kg.
193. A wooden block lies on an inclined plane. What
force must prese the block again.st •he inclined plane for
it t.o remain at rest? The mase of t.he block is 2 kg, the
length of the inclined planeie 1 m and ite height is 60 cm.
The coeffi.cient of frict.ion between the block and the
plane is 0.4.
Glwn: m=2 kg, l=I m, h=60 em=0.6 m, ~=0.4.
F-1
SoluUon. The block e:r.periences the act.ion of the force
of gravi ty mg, the normal reaction N of the incline d
plane, the frict.ion F rr• and the force F pre.ssing the block
against the inclined plane (Fig. 84). Since the block can-
not. rotate, the ftrst equilibrium condition is sufftcient for
it. to remain at rest:
mg+N+F+Fr,=0. (1)
... Selected Problema on Phyaics

Choosing the directions of the X- and Y-axes and pro.-


jecting Eq. (t) onto them, we obtain
mgsina.-F 1 r=0, N-mgcosa.-F =º·
Considering tbat Frr= µN, sin a.=hll, cos a=
)l"l9 - h 2 /l and solving tbe obtained system of equa-
tions, we find that
F~ ~1 (~- \ll'-h'},
F= 2 X19·8 X ( ~1-\fl-(0.6)') N0<13.7N.

194. One end of a thin homogeneous beam AB of mass


too kg resta on the smooth horizontal Door and tbe other

Fig.84

Fig.85

end rests on a smooth plane inclined at an angle of 3C1'


to lhe horizontal. The end B is supported by a string
with a load passing over a pulley C (Fig. 85). Determine
the mass of the load and tbe normal reactions of the Ooor
Ch. t. Mechanics 137

an3,~::: i::l~fi/~:,ep =~~;~8.J~i~~:~·


mz-? N 1 -? N 2 -?
Solution. Let us consider the forces actiog on the beam:
m1g is the force of gravity, N1 and N1 are the normal re-
actions of the ftoor and the inclined plane, and T is the
tension of the string. Under the action- of thue forces, the
beam is in equilibrium. Writiog the first equilibrium con-
dition for the beam, we obtain
m1g+N,+N,+T=0. (1)
Directing the X- and Y-axes as shown io the figure and
projecting Eq. (1) onto them, we get
N,sioP-Tco•P=O, (2)
-m1g+N, +N,cosp+ TsinP=O. (3)
Writing the second equilibrium condition for the beam
about ao axis passing through point B, we obtain
M, - M,= O. (4)
Here M 1 =N1 l1 and M 1 =m1 gl1 are the moments of
forces N 1 and m1 g about the choseo axis, where Z. =
t.c':n: :.~.'·.~ <f~ j~B ª~ J!1ee ,;:sle~:.es!i::
k-
tutiog the expressioos for 1 and M 1 into Eq. (4), we
obtain N 1L cos m - m 1g (L/2) cos m = O, whence
Ni=m:'
Ni= t00~9.8 N=490 N.
The teosion can be determined from Eqs. (2) and (3):
T=+m,gsinp. (5)
Usiog the equilibrium condition for the load, we get
T' = m 1 g, wbere T' = T, wheoce
m.z=f= "; sinp,
1

mz=*2 kg=25 kg.


t38 Selected Problema on Phyaics

Solving Eqs. (2) and (5) together, we obtain

N,,= ~~d~= m211 eos~,


Na 100x9:x0.87 N= 426 N.

195. A thin homogeneous rod OB reais on two support.s


D and C 99parated by a distance a (Fig. 86). The coeffi-
cient of friction between the rod and the support is p.,

Fig.86

the slope of the rod is CI, and the length of region 1 CB /


is b. What must be the length L of the rod for it to be in
equilibrium?
Given: iOB,l=L, iDC 1=•, iCB l=b,~,a.
L-1
Solutton. The rod OB experiences the action of the force
-of gravity mg, the normal reactions N1 and N1 of the
supports D and C, and the frictional forces F 1 ri and
F 1, 1 • The frictional forces are directed upwards along the
rod since the rod has a tendency to slide down. Writing
the lirst equilibrium condition for the rod, we obtain
mg + N, + F1n + F,., + N,=0. (1)
Directing the X- and Y-axes as shown in the figure and
projecting Eq. (1) onto them, we get
F rn + F rn- mg sin)z= O, (2)
N1 -N1 - mgcoacr.=0. (3)
Ch. t. Mechanic1 139

Considering that Frr" l'N, we ean write Eqs. (2) and


(3) in the form
f'H, + ~N, - mg sin cz;;;. O, (4)
N 1 -N1 -mgeosci=0. (5)
Writing the second equilibrium condition for the rod
about an axis passing through point C, we obtain
M, -M, =0. (6)
Here M1 = mgl1 and M1 = N1 l1 are the moments of the
forces mg and N 1 about the chosen axis, where li =
(L/2 - b) cos ex and l1 =a are the arms of the forces
mg and N 1 • Substituting the expressions for M 1 and M 1
into Eq. (6), we get
mg (L/2 - b) cos cz - N,a= O. (7)
= mg coa cz (L/2 - b)la
From Eqs. (5) and (7), we &nd N,

=~:o!l, ;e~~:::1:u\~~ the t~311!ith!~!fr~!11~8q~~}~


L;;;> 2b + a (1 + tan cz/~).
196. A 4-m long ladder leans against a perfectly smooth
wall at an angle of 60° to the horizontal. The coeflicient
of friction between the ladder and the Boor is 0.33. To
what distance along the ladder can a man climb before
the ladder starts to slide down? The mass of the ladder
should be neglected.
GWen: l=4 m, a=SO"~t.05 rad, 1'=0.33.
S-1
Solution. The ladder is acted upon by the force of pres-
sure F exerted by the man, the normal reactions N1
and N1 of the wall and the Door, and the friction Frr
(Fig. 87). The sliding of the ladder can be treated as a
combination of two motions: rotation (about point O)
and translation (against the X-axis). Writing the lirst
equilibrium condition for the ladder, we get
F + N1 + N1 + F Ir =O. (1)
"º Selected Problema OD Phyaica

Projecting Eq. (1) onto the X- and Y-axes, we obtain


F,, - N 1 -0, N, - F=O. (2)
Writ.ing the second equilibrium condition for the ladder
about point O, we get
M, - M,=0. (3)
Here M1 =N111 and M 1 =Fl 1 are the moments of the
forces N 1 and F about the choaen axis, where l1 =

y

Fig.87

l sin a. and Z. = S cos a. are the arms of the forces N 1


and F. Using these relations, we can write Eq. (3) in t.he
form N 1 l sina. - FS cosa.= O, whence
S= ~1!::cz = N/ tanu.
Using Eqs. (2) and the expression F rr = 11N1 for fric-
tion, we obtain N 1 = 11F, and therefore
S=titl tana.,
S= 0.33 X 4 X Vã mo. 2.3 m.
197. A sphere of radius 5 cm reating on a vertical wall
is suspended on a 20-cm long string. The string touches
the sphere at point C. Determine the coef&cient of fric-
tion between the sphere and the wall.
Giutn: l=20 cm=0.2 m, R=5 cm.=0.05 m.
~-1
Ch. 1. Mechan.ica

Solution. Thesphere is acted upon by the force of grav-


ity mg, the normal reaction N of the wall, the teosioo
Ay

Fig.88

T of the string, aod the friction F rr (Fig. 88). Writiog the


first equilibrium cooditioo for the sphere, we obtain
mg+N+T+Fr.=O. (1)
Directing the X- and Y-axes as shown io the figure aod
projectiog Eq. (1) ooto them, we get
N - T sin a=O, -mg + Tcosa + Frr=O. (2)
Writiog the second equilibrium cooditioo for the sphere
about ao axis passiog through poiot O, we obtaio
-M, + M,=0. (3)
Here M 1 = F irl 1 aod M 1 = Tl 1 are the moments of the
forces F rr aod T about the choseo axis, where 1,,- l1 =
R are the arms of the forces. Takiog this ioto account,
we write Eq. (3) io the form
-Fr,R + TR=O, or Fr.= T. (4)
Solviog the system of equatioos (2) aod (4) and coosid-
eriog that Frr =s;;- p.N, we obtain
µ ;> t/sin e<. (5)
Selecled Problema oa Physics

Since AC and A B are tangents to the circle drawn from


the same point, LOAC = a.12 and tan a/2 = Rll.
Determining the sine of the angle from the formula
, 2tan(e&/2)
sina= t+tan•(a/2)
we finally get
i.i;ai: R;til' ,
µ;ai: 2º~2~;,º~~~2 ~ 2.13.
198. Two spheres of masa 3 and 5 kg are connected hy a
rod whose mass is 2 kg. Determine the position of tbe
common centre of masa if tbe radii of tbe first and the
second sphere are 5 and 7 cm respectively and tbe length
of the rod is 30 cm.
Given: m1 =3 kg, m:r.=5 kg, m3 =2 kg, R 1 =5 cm=
0.05 m, R,=7 cm=•0.07 m, 1=30 cm=0.3m.
z-?
Solutfon. Since the centre of mass of the system coin-
rideswitb itscentre of gravity, we can determine its posi-
tion from tbe equilihrium condition for the system in the
gravit.ational field. For this purpose, we fix tbe system at
an axis passing through the centre of gravit.y e (Fig. 89).
Then it is sufficient to consider only one equilibrium con-
dition for t.he system about this axis:
M, -M, + M,=0. (!)
Here M 1 = m 1gl 1, M 1 = m1 gl1 , and M 1 = ma1l8 are
1 '+ Rm.f
~~::n°1::i~~w~e!!1ez :Czi~ 11 1 ~.ªi d=mJf. ~ºi/~~~~
1 1
and l 8 = z are t.he arms of tbe forces m1g, ms1, and
ma11 x heing the dist.ance from tbe midpoint of the rod to
the centre of gravit.y (see Fig. 89). Taking this int.o ac-
count, we write Eq. (1) as follows: m1g (l/2 + R 1 + z) -
m1 g (R 1 +l/2 - z) + m8 gz= O, whence
Z= lll1R1-=:~·::~r:-.-m1)/2 '
z= sxo.01-a~~~.:20.axcs-a)12 m ~o.os m.
Cb. t. Mechanics
•••

Fig. 89

199. A plane bomogeneous plate bas the shape of a cir-


cle with a circular bole baving a radius equal to balf tbe
radius of tbe large circle and a
common tangent witb it
(Fig. 90a). Determine tbe position
of tbe centre of mass of tbe plate.
Given: R.
z-1
Solutton. Since the centre of mass
of tbe plate coincides witb its cen- (a)

.~
tre of gravity, we shall determine
tbe position of tbe centre of grav-
ity of the plate. If we lill tbe bole,
tbe force of gravity mg of tbe body
can be presented as tbe resultant of ...,gLg,,. .,
two forces (Fig. 90b): the force of
gravity m1g of tbe . bole portion·
and tbe force of gravity tn,:g of tbe
remaining portion (tbe circle witb
tbe bole). Tbe plate will be in equi- Fig. 90
librium about an axis passing
tbrougb point O. Writing the second equilibrium condition
for tbe system about the cbosen axis, we obtain
-M1 + M 1 =0. (1)
Here M1 =m1grandM1 =m1 gz are themomentsofthe
forces of gravity m1g and ms1 about point O, wbere r
and z are lhe arms of lhe forces of gravity m 1g and m1g.
Taking this into account, we can write Eq. (1) in the form
,.. Seleeted Problems on Physics

-m1gr + m gz =
1 O, wheoce
.:r:=m1rlm 1 • (2)
The messes of homogeneous plates of the same thickness
are given by
m= pSh= p:nR1h, m1 = pS1h= pnr1h,
m 1 = m - m 1 = pnh (R• - rt),
where p is the density of tbe material of t.he plate, S
the area of t.be entire plate, 8 1 the area of the bole, and
h the plate t.hickness. Hence, we can write Eq. (2) in t.he
form
:&= pn:~~rl) = R•r:_,1 •
Using the condition of t.he problem (r=R/2), we obtain
R' R
.:r:= 8(R1 -R•/4) =g·
Consequently, the centre of mass lies at a distance of
R/6 from the centre of the plate.
200". The dependence of the potential energy of a sys-
tem on the ~coordinate is given by W (z) = -Szl +
4x- 3. Determine the coordinate of a point corresponding
to the equilibrium position of the system and indicate
the type of equilibrium.
Giuen: W (z) ~ -5z'+ 4z-3.
z-1
Solution. By definition, the system is in equilibriuai if

~=º·
Coosequeotly, evaluatiog the first. derivative 'Íi (-Szl +
4x - 3) = -10z + 4 of t.he expressioo for the pot.eotial
energy wit.h respect to x and equatiog it. t.o zero, we obtaio
the equatioo -10.x +4 =O. Therefore, the coordinate
of the poiot. correspondiog to t.he equilibrium posit.ion
of the syst.em. is :e = 0.4 m. ln order to determine the type
of equilibrium, we t.est. t.he sign of t.he second deriva-
Ch. t. Mechauics 145

tive:
~=}.-(10•+4)-!0<0.
Consequently, the equilibrium of the system is unstable.
201°. The potential energy of a body of mass 0.5 kg
varia according to the law W (.z) = 6.zs + 4.z - 2.
Determine the acceleration of tbe body at the moment
when it passes through the equilibrium positioo.
Given: m=0.5 kg.
a-1
Solution. By defioit.ion, the work dA Ui equal t.o the
ehaoge dW io the eoergy of the body:
dA = dW. (1)
Oo the other band,
dA =Fdz. (2)
Equatiog the right-haod sidas of Eqs. (1) aod (2), we ob-
taio d W = F d.z, wheoce
F=~. (3)

Eq. (3) ioto accouot, a = *: .


Accordiog t.o Newton's secood law, a=- Fim, or, takiog
Diflereotiatiog the ex-
pressioo for the poteotial energy, we obt.aio
•=f-J.<&r+ú-2)=,:.(12.+4). (4)
Tbe coordinate of the body in tbe equilibrium position is
determined from the eondition ~~ = O, or I;-
(6.zS +
4.z - 2) = 12z + 4 = O, wbence .z = -1/3 m. Substi-
tuting .z aod m int.o Eq. (4), we get
•= 015 [12-(-i-)+4]=0.
202. Why is it easier to unserew a nut wit.h a long
spaoner than with a short spanner?
Answer. ln order to turn a nut with a spanner, we must
apply t.o it a force whose torque must be equal to or larger
t0-0970
... Selected Problema on Phyaica

than the moment of the friction acting hetween the nut


and tbe spanner. Since the torque is equal to the produet
of the force and its arm, the Jonger the spanner, thesmall-
er the force that should be applied to it.
203. One truck is carrying wood and another hay.
Which of the two will overturn more easily if their
masses are equal?
Answer. The truck with hay will overturn more euily
since its centre of m8S8 is higher than that of the truck
witb wood. The stability of a body is the higher, the low-
er its centre of mus.
EXERCJSES
204. A wall-mounted jib crane has a jibstay BC of
length 4 m and a tie-rod AC of Jength 3 m, the d is-
tance AB between tbem being 1.5 m. Determine the forces
acting on AC and BC if the mass of the load heinghoist-
ed is 2 t. What action is exerted by these forces on the
jibstay and the tie-rod (Fig. 91)?
205. An electric lamp of masa 2 kg is suspended from
the ceiling on a cord A B and is pulled towards the wall by
string BC. Determine the tensions of the cord and the
string if a. = 60" and p = 135° (Fig. 92).
206. A cylindrical tank of diameter 20 cm is 6.lled with
water and suspended from a hínge A on a weightless rod
rigidly 6.xed to the lank (Fig. 93). WaterOowsat a veloc-

H~F:?~
Fig. 9t Fig. 92 Fig. 93
Ch. f. MechaDicl

i ty of 4.43 m/s from a side bole of area 2 cm' drilled in


!~: ~~:c~i 1~Üiª~!~a~o~tb!°d~-:~~ T~:\:~1 b!;;::
is t m. The mass of the tank and the drop in the water lev-
ei due to leakage should be neglected and tbe angle of
deDection should be regarded as small.
207. A homogeneous rod AB is hinged to a vertical
wall at point A and is held at an angle of 60" to the verti-
cal with the help of a string BC forming an angle of 30°
with it (Fig. 94). Determine the normal reaction of the
hinge if the mass of the rod is 2 kg.
208. A 10-m long beam rests on tWo supports .. \ load
of mass 5 t is put at a distance of 2 m from tbe left end of
the beam. Determine the forces of pressure exerted by the
beam on the support.s if the beam mass is tO t.
209. The mass of a roller is 100 kg and its ndius is
0.5 m. What minimum force must be applied to the roller
to roll it over a beam of height 10 cm?
210. A rod is subjected to tbe action of two parallel and
opposite forces of 10 and 25 N. The points of application
of the forces are at 1.5 m from each other. Determine the
resultant of the forces and the point of its application.
211. Determine the position of the centre of mass of a
homogeneous pla\e whose size and shape are indicated in
Fig. 95.
212. A sphere of mass to kg rests on two identical in-
clined planes forming angles of 45ºwiththe horizontal. De-
termine tbe force of pressure exerted by the sphere on the
planes. ·

... Fig. 94 Fig. 95


... Selected Problema on Physics

QUESTIONS FOR REVISION


1. Define the equilibrium of a body. 2. Name ali the types of equi-
librium. 3. Formulate the eqtdiibrium condition for a point mau.
4. What ia torque? 5. How is the direçtion of the moment of force
vector determined? 6. Defme lhe arm of a force. 7. Formulate the
equilihrium condition for a body relative to a lixed rot.ational axis.
:;:~~Í::~t~~:~r~;::;i:,i~~~~~º.!; :r~Ji:a!1di:d11!~
:ri::! :rd:'bod;?iif. i~:W-~~h~ ;::tú;,~~r ':ª!e~t~f::
0
of a hody of an arhitrary .shape he determiJled?

t.5. Hydro· and Aerostatlcs


Pn>SSure is de6.ned as
p - FIS,
where F is the force acting on a surface of area S perpen-
dicular to this force.
The pressure exerted by a eolumn of a homogeneous liq-
uid at a depth h (hydrostatic pressure) is defined as
p - pgh,
where p is lhe density of the liquid.
A ftuid (t.e. liquid or gu) transmiti a prusure ezerted on
itl surface uni/ormly in all dtrections (Paseal's law).
Pascal's law irnplies that if a hornogeneous liquid is in
equilibriurn in cornrnunicating vessels, the pressure acting
on the surfaces of the sarne levei in these vessels will be
tbe sarne.
A body tmmersedin a ftutd is acted upon by a buoyanl force
equal to the weight of the fluid f.t displaces anti. applied at the
centre of gravity of the displaced volume (the Archimedean
principie). This principie leads to the ftoalation eonditioo:
if the buoyancy i8 equal to the force o/ gravity acttng on a body
~ tn a f/.uid, the body /Watl tn the f/.uid.

2t3. Equal masses of rnercury and Water are poured in


a cylindrical vessel. The total heighl of lhe liquid column
is 29.2 cm. Determine lhe pressure exerted by lhe column
Ch. 1. Mechanica •••
on the bottom of the vessel (Fig. 96).
Gtwn: k=29.2 cm=0.292 m, m1 =mz.
p-?
Solution. The total pressure exerted by tbe liquida on
the bottom of the vessel is
p = p, + p,. (1)
Here p 1 = Pitk1 and p 1 = p.gk 2 are tbe pressures exert-
ed by mercury and water on the bottom of the vessel,
wbere p1 and p1 are the densities of mercury and water.
Substituting these expressions into Eq. (1), we obtain
p = g (p,k, + p,h,), (2)
and
k=k,+h,. (3)
By hypothesis, the masaes of the liquid columns are
equal: m1 = m 2 , or p1"1.S = p,/& 28, wbence
p,h, = p,A,. (4)
where S is the area of the vessel bottom. Using Eqs. (3)
and (4), we obtain
h, = hp,/(p, + p,), k, = hp,/(p, + p,). (5)
Substltuting Eqs. (5) into (2), we get

P=t(~~+~t:,)=~~·
P 2x13.6~3~~xx11:.:1~axo.202 Pa~5.3 kPa.
214. The limbs of a U-tube are fi.lled with water and oil
separated by mercury (Fig. 97). The interfaces between

Fig. 96 Fig. 97
150 Selected Problems ou Physics

mercury and the liquids in the limbs are at the sarne


level. Determine the height of the water column if the
height of the oil column is 20 cm.
Given: hi=20 cm=0.2 m.
h1-?
Solution. According to Pascal's law, the pressure in
the limbs of the U-tube is the sarne at the levei AB (at
the interfaces between mercury and the liquids):
Pi= P2· (1)
Here p 1 = p1gh1 and Pz = pJh 2 are the pressures in lhe

u
left and right limbs respectively at lhe level AB, where

~· -
--
-
Fig. 98

p1 and Pz are the densities of water and oil. Substituting


these expressions into Eq. (1), we obtain p1gh1 = pJh 2 ,
whence
h1= PP~'
h1 0.9x:~x0.2 m=0.18 m.

215. A force of 196 N is applied to the smaller piston of


a hydraulic press. Under the action of the force, the piston
is lowered by 25 cm during one stroke, and, as a result,
the larger piston is raised by 5 cm (Fig. 98). What is the
force of pressure transmitted to the larger- piston?
Given: h 1 = 25 c111 = 0.25 m, h:. = 5 cm = 0.05 m,
F,=196N.
Fi-?
Ch. 1. Meebanica

SoluUon. According to Pascal'a law, p1 = p 1 , whera


p 1 = F 11S1 ia the p1'888ura exerted by the smaller piaton
of area 8 1 on the liquid and p 1 = FJS 1 the pressura exert-
ed by the liquid on the larger piston of area S 1 • Writing
Pascal'a Iaw in the form F1 /S1 """ PiS 1 , we obtain
(1)
Since the liquid ia incompressible, tho volume of the liq-
uid Dowing from t.he amaller to the larger cylinder ia the
aame: V1 = V1 , or S 1k1 = S 2k 2 , whence
s,1s, = h,th,. (2)
Substitut.ing rat.io (2) int.o exp1'888ion (t), we obtain

F1=F1~1
F,=196 ~:: N=980 N.

216. What must. be t.he height of a cylindrical vesael of


radiua 5 cm 6.lled with water for the force of p1'888ure ex-
erted by water on the bottom to be equal to the force of
preasura on the lateral surface?
Given: R=S cm=0.05 m.
h-1
Solutton. By deJinition, the force of preasure on the bot-
tom of the vessel ia
F =pS. (1)
Since the pressure on the bottom ia p = pgh, where p
is the denaity of water, h the vessel height, and S =
nR1 the area of the vessel bot.tom, expresaion (1) becomea
F = pglmR'.
Similarly, the force of pressura on the lateral aurface
of the vesael ia
F1at = (p)S1at1
where (p) = p/2 ia the mean prauura exerted by water
on the lateral surface of the vesael and S lat the ara a of the
lateral aurface. Conaidering that p = pgk and S 1., =
152 Selected Problems on Physics

2nRh, we obtain
F1at = pgnRh'.
Since F = or pghnR 2 = pgnRh 2 , we obtain
Fiat•
h ~ R, h ~ 0.05 m.
217. A spring balance with a piece of copper-silver al-
loy suspended to it indicates 2.41 N in air and 2.17 N
y

~~
(a)
9w Fig. 99
(D)

in water. Determine the masses of copper and silver in


the alloy, neglecting the buoyancy in air.
Given: T1 =2.41 N, T2 =2.17 N.
mc-?m9 -?
Solution. During weighing in air, the piece of alloy is
acted upon by the force of gravity mg and the tension
T 1 of the spring(Fig. 99a). Writing the equilibrium condi-
tion for the body in projections on the Y-axis, we obtain
-mg + T1 =O,
whence mg = T1 and m = T 1/g. During weighing in wa-
ter, the force of gravity mg, the tension T 2 of the spring,
and the buoyant force F act on the piece of alloy
(Fig. 99b). Writingtheequilibriumcondition for the body
in water, we get
mg + T 2 + F =O.
ln projections on the Y-axis, this equation has the form
-mg+T,+F-0. (1)
Ch. 1. Mechanics 153

Considering that mg = Ti and F = pgV, where V


is the volume of the body and p the density of water, we
transform Eq. (1) as follows:
T,-T,+pgV=O. (2)
ln other words, the mass of the body in air and itl vol-
ume are
·=~+~ ~
V= Vc +
V1 , or V= m~/Pc + m /p
1 1, (4)
where Vc and V1 are the volumes of copper and silver in
the pieee of alloy, and Pc and p1 the densitiuof copper and
silver. Solving Eqs. (2)-(4) together, we obtain
m,= (p.~Pc)1 ( r,-;ri +fc-)'
10.5X103x8.9X1CP
m,= (10.5X1CP-8.9X1CP)x9.8

X ( 2·171;;; 2·41 + S.;~ttó' ) kp,0,210 kg.


Considering t.hat. m = Tilg, we get from Eq. (3)
me = m - m, = Tilg - m..
m, = (2.41/9.8 - 0.210) kg = 0.0356 kg.
218. A hollow iron sphere is weighed in air and kero-
sene. Thereadingsoft.hedynamometer are 2.59 and 2.16 N
respectively. Det.ermine the volume of the cavity of the
sphere, negleeting the buoyancy of air.
Gtuen: T1 =2.59 N, T2 =2.16 N.
Ycu·-?
Solution. During weighing in air, t.he sphere is act.ed
upon by t.he force of gravity mg and the t.ension T1 of
the spring (see Fig. 99a). Writing the equilibrium condi-
tion for t.he sphere in project.ions on the Y-axis, we obtain
-mg+ Ti =0,
whence mg = Ti, or m = Tilg. During weighing in kero-
sene, thespbere experiencesthe action of the force of grav-
ity mg, the tension T 2 of the spring, and the huoyant.
IM Selected Problema on Phyaics

force F exerted by kerosene (see Fig. 99b). Writing the


equilihrium condition for the sphere in keroaene in projec-
tions on the Y-axi.s, we get
-mg + T, +
F =O. (1)
Considering that mg = T1 and F = PttV, where V
is the volume of the sphere and p1r; the deDJity of kerosene,
we transform Eq. (1) as follows:
T, - T, + Pt1V =O. (2)
The volume of the cavity is Ycav =V - V 11 whereV 1 =
m/p 1 is the volume occupied by iron. Consequently,
Yc.,. = V - m/p 1• Since m = T 1/g, we have
V,., = V - T,l(pi1). (3)
Solving Eqs. (2) and (3) together, we obtain
V _ r 1 -r. _.!L
cn- Prd Pll '

Ycav=( o.~·~;,2;:.8 1.sx2f:x9.8) m•


~ 2.1x10-s m'.
219. A homogeneous hody Doats on the surface of kero-
sene so that the volume of the submerged part constitutes
0.92 of the entire volume of the body. Determine the vol-
ume of the submerged part of the body Doat.ing on the
surface of water.
Gtven: V,ub=0.92V.
V~ub-?
Solution. We denote theentire volume of the body hy V,
the volume of the part suhmerged in keroaene hy V,ub•
and the volume of the part suhmerged in water hy
V~ub· The hody in kerosene experiences the action of the
force of gravity mg and the huoyant force F = -p1r; Y1 ubl
exerted hy kerosene (Fig. 100). The Doatation condition
implies that mg = F, or
mg = P•V,,,g = P•·0.92Vg, (1)
where Pt is the density of keroaene.
Ch. t. Mechuica ...
Similarly, we can write the Ooatation condition for
water:
mg = F,., mg = PwV.ubl• (2)
where Pw is the density of water.
Solving Eqs. (i) and (2) together, we obtain P1r. X
0.92Vg = p,.V;111J1, whence
V~ub= º·~k V,
v;..b o.02x1~sxtCJ1 v~o.74V.

220. One end of a 2Ckm long thin wooden stick is


hinged and the free end is immersed in water. What part
of the stick will he suhmerged in water in equilibrium?
Gtwn; L = 20 cm= 0.2 m.
1-1
Solutton. A stick suhmerged in water is acted upon by
lhe force of gravity mg, the buoya'nt force F of water, and
the normal reaction N of the hinge (Fig. 10f). Writing the
equilihrium condition for a body having a rotational
axis, we obtain for the stick
(1)

Here M 1 = Fl1 and M 1 = mgl 1 are the momenta of the


foroes F and mg about the axis passing through point O,

~
F
F

--
- -
--
----
---
.... 100 Fig. 101
156 Selected Problema ou Physics

l 1 = (L - l/2) coa a and 11 = (U2) coa a being the


arms of the forces F and mg.
Substituting the expressions for M 1 and M 2 into
Eq. (!), we get
F (L -112) cosa- mg (L/2) eos a= 0. (2)
Considering that F = PwtSl and mg = PwoodtSL, where
S is the cross-sectional area of the stick, we can write
Eq. (2) in the form
PwfS 1 (L - 112) - Pw 00 ogSL'l2 = 0,
whence
I' - 2LI + Pw 00 oL'/Pw = O. (3)
Solving Eq. (3), we find that
l=L± fL'-PwoodL'IPw=L(1 ± Y1-Pw00o/Pw)•
Sinee the length of the submerged part cannot exceed its
leogt.h, we obtaio
1= L (1- Vr.1~--P-w- -,~Pw~).
...
1=0.2(1-V1-0.810'/Hl') m<><0.11 m.
221. What is heavier in water: a brick ora piece of iron
of the same mass?
Answer. The piece of iron will be heavier.
222. Is the buoyant force acting on a body at di&erent
depths the same?
Answer. Since liquids are practically incompressible,
they have nearly the sarne density at diflerent depths.
Consequently, the buoyant forces will be the same at
differeot depths.
223. Where is the draught of a sbip larger: in river or
at sea?
Answer. The draught of the ship in sea water is smaller
than in river sinee the density of sea water is higher than
that of fresh water.

EXERCISES
224'. What force of pressure cao be developed by a hy-
draulic press if the force applied to the longer arm of the
lever transmittiog pressure to the smaller piston is
Ch. t. Mechanics 157

100 N, the ratio of the lever arms is 1:9, and tbe areas of
tbe pistons are 5 and 500 ems? Tbe prese ef.6.eieney is
0.8.
225. A barometer tube is inelined at an angle of 30°
to tbe horizontal. Wbat is tbe length of the mereury eol-
umn under tbe normal atmospberie prusure?
226. At what deptb is the pressure in fresh water thriee
higher than the atmospherie pressure equal to t .Ot 7 X
10' Pa?
227. Two eommunieating tubes with different eross-
seetional areas are filled with mereury. Tben 272 g of wa-
ter are poured in a wider tube of 8-ems eross-seetional
area. What will be the differenee in the mereury leveis
in 2i':~ At~~es~adeof gold and silver is weig~ed in air and
then in water. The readings of the spring balanee are 3
and 2.756 N respeetively. Determine the mass of the
gold and silver in the bar.
229. A pieee of wood floats in water so that three-fourths
of its volume are submerged. Wbat is the density of the
wood?
230. A hollow eopper spbere of volume 44.5 em• float.s
in water so that it is submerged to liaalf. Determine tbe
volume of the eavity.
23t. A homogeneous stiek is hinged at the upper end,
and its lower end is immersed in water. Tbe stiek is in
equilibrium when half of it is submerged. Determine the
density of the material of wbieh tbe stiek is made.
QUESTIONS FOR REVISION
t. Define pressure and the lon:e oi pre!Sure. What is the SI unit
of presaure? 2. Fonnulate Po.seal's law. 3. What is the pressure in
!dli~:idtb! boC:~t;:°.:dp:e"~ib":t.c~e!i ~~r:k!J::d?r:~J:F!
111ulate the laws of co111municating vessels for ho111ogeneous and
heterogeneous liquida. 6. Explain the principie oi operation of a
hydraulic preu. 7. What is the buoyant force? 8. Fonnulate lhe
Archillledean principie. 9. Formulate the Doatation conditions for
a body. 10. De6.ne the lifting force.
Chopter 2
MOLECULAR PHVSICS
AND THERMODVNAMICS

2. t. leslc Concepts
oi lhe Molecular Theory
HEAT ANO WOU.

Ali substances consist of atoms and molecules. The


amount of substance is measured in special units called
moles. A mole of any substance contains the sarne num-
ber of molecules N"' = 6.02 x 1023 mol-1 (Avogadro's
constant).
Ali molecules are in increasingrandom motion known as
thermal motlon.
The sum of the kinetic energy Wk of the random motion
of ali molecules of a body and the potential energy W P
of their interaction is called lhe Internai energy U o(
lhe body:
U = Wk+ WP.
For an ideal gas, lhe potential energy of molecules is
neglected, and hence its internai energy is completely de-
termined by its temperature:
U = cmT.
Here e is lhe specifi.c heat of the gas at constant volume,
m the mau of the gas, and T its thermodynamic tempera-
ture (in K): T = 273 + t, where t is the gas temperature
in degrees centigrade.
Gas molecules possess the mean kinetic energy of
translational motion, defined as
W • - (l/2)m, (v'>.
where m0 is the mass of a moleeule and Vr.JP, = Y (vi}
the root-mean-aquare velocUy of translational motion of
molecules.
Ch. 2. Molecular PhJBic1 and Thermodym.micl i59

According to the molecular tbeory, tbis energy is con-


nected witb thermodynamic temperatura tbrougb the
following relation:
W, = (3/2)kT,
wbere k = RIN A is the Boltzmann constant.
The internai energy of a body may cbange as a result of
two procU!es, viz. tbe beat excbange and the convenion
of mechanical energy inlo the intemal energy of the
body.
ln all procesaes occurring with a body, the energy eon-
servatlon law (lhe ftrst law oi thermodynamtcs) is ob-
served: the amount of heatsuppliedto thebody iB•pent on tn-
creasing the tnternal energy o/ the body and on doing work
by the body on the 1urroundtngs:
Q=IW+A. (!)
Here, it is assumed that
Q > O if the body receives ., certain amount of beat,
Q < O if the body gives away a cert.ain amount of beat,
A > O if tbe body does a work, and
A <O if the work is done on a body.
Let us consider two special cases of 'the cbange in inter-
nai energy.
t. The cbange in internai energy during bea.t excbange
(witbout doing a mecbanical work). ln this case, Eq. (1)
becomes
Q.=IW. (2)
The change AU in internai energy can be calculated
from tbe following relations:
IJ.U =cm IJ.T (3)
for heating and cooling,
IJ.U=Am (4)
for melting and crystallization,
tJ.U =rm (5)
for vaporization and condenaation, and
IJ.U=qm (6)
160 Selected Problema on Physics

for combustion of a substance. Here e is the specitic heat


of a body, 1 the latent heat of fwdon, r the latent heat of
vaporization, q the heat of combustion of the substance,
m the mllS8 of the body, and AT the change in tempera-
tura, AT=8 -T 1n, where 8 is the fi.nal and T 10 the
initial temperatura of the body. For crystallization and
.condensation, we must assume in Eqs. (4) and (5) that
AU <O. ln the general case, Eq. (2) has the form

Q=f 1J.u,. (7)


·-·
lf severa! bodies tak' part. in heat exchange, the algehraic
.sum of the amounts of beat given away by the bodi.es
whose internai energy decreases and the amounts of heat
received by the bodies whose internai energy increases is
zero for a closed system of bodies. Tbis statement is
known as the heat-balanee equatlon:

(8)

where Q•1v and Qrec are calculated by Eq. (7).


2. The change in internai energy as a result of doing
mechanical work by a body (without heat exchange with
the surroundings). ln t.his case, Eq. (1) becomes

O- IJ.U+A.
whence
IJ.U- -A. (9)
The mechanical work A can be calculated from \he
familiar rela\ions.
ln the presence of heat losaes, the distiDction should be made
between lhe amount of heat Qu spent t.o cbanp the intemal energy
r!~ ~r!!JnTfi'!~~~~ ~~e J°~~.:ei:w~ !~!;;' o?~r.lc~!=
1J is introduced:

•=-S'i-•00%.
Ch. 2. Molecular Phyaiea and Thermodynamies t6t

232. Determine the mass of a bydrogen molecule.


Gtv.n: M = 2 x 10-• kg/mol.
m,-1
Solu.tion. lf we take a mole of hydrogen whose molecu-
lar mass is M = 2 x to-1kg/mol, it contains, by defini-
tion, NA = 6.02 X 1011 moI- 1 molecules. Consequently 1
tbe mass of a hydrogen molecule is
M
mo= NA'

mo= 8~ !°;'ou kg~ 3.32 x to-17 kg.


233. How many water molecules are eontained in a
drop wbose mass is 0.2 g?
Gtven: m = 0.2 g = 0.2 x 10-' kg, M = 18 X 10-8 kg/mo!.
N-1
Solution. The number:of moles in tJi, drop of mass m is
"=m/M.
Since a mole contains NA molecules, " moles of water
contain vN A molecules. Consequently,

N=i-N"',
N= ~::.W: x6.02xtOH~6.7xtfJ2t,
234, Determine the root.-mean-square velocity of oxy-
gen molecules at 20 ºC. At what temperature is tbis ve-
locity equal to 500 m/s?
Giwn: T1 =293 K, "ar.m.,=500 m/s.
"ir.m.,-? Ti-?
Solu.tion. By definition, the mean kinetic energy of a
gas molecule is
(1)
lt-0970
Selected Problema on Physics
"'
where m0 is the mesa of a molecule and Vu.m 1 = V <V:>
the root-mean-square velocity. Alternately, aecordiog to
the molecular theory, the mean kinetic energy of transla-
tional motion of a gas molecule is given by
W., - (3/2)kT,. (2)
Equating the right-hand sides of Eqs. (1) and (2), we
obtain (l/2)m,(rJ,) - (3/2)kT,. whence
(3)
Since k = RIN A• expression (3) can be written in the
form Vir.m.1 = l'3RT1/(mo/{A) or, considering that
moN A. = M, we obtain
Vir .•.• =V3RT1IM,

Vir.m.1 = Jf 3 ~s;t':r-293 2;- ~ 480 m/s.


Similarly, Uzr.m.1 = "V3RT1/M, whence

T,=PLS~ 1

Tz soo·;~~tO""' K~320 K.
235. What amount of heat must be supplied to 2 kg of
ice taken at -10 ºC to melt it, to heat the obtained water
to 100 "C, and to vaporize it?
Giuen: m=2kg, T1 =263K, T1 =373K.
Q-1
SolutiOn. The chaoge in the internai energy of ice is
l!.U, ~ c.,,m (T m - T) (!)
for ib heatiog to the melting point,
l!.U, -1.m (2)
for melting the ice,
l!.U, - c,.m (T• - T m) (3)
Ch. 2. Molecular Pbyaics alld 'I'bermodynamics 183

for heating the obtained water to the boiling point, and


(4)
for vaporizing the obtained water.
Then the total amount of heat that must be supplied to
the body is
Q = IJ.U1 + IJ.U, + IJ.U, + IJ.U,. (5)
Substituting Eqs. (1)-(4) into (5), we obtain
Q = <1,,.m (T m - T,) + l.m + cwm (T • - T m) + rm
m lc1c:e (T m - T1) + Â. + Cw (Tb - T m) + r1,
=
Q = 2 X (2.1 X 10' X (273 - 263) + 3.35 X !01
+4.19 X 10' X (373 - 273) + 22.6 X !01) J
""61 MJ.
236. A metal body of mass 192 g heated to 100 ºC is
immersed in a brau calorimeter of mass 128 g containing
240 g of water at 8.4 ºC. The &nal temperature which sets
in the calorimeter is 21.5 °C. Determin~ the speci&c heat
oi the teotad body.
G111<1n: rn,=128 g=0.128 kg, m,=240 g=0.24 kg,
T1 =T,=281.4 K, m, = 192 g = 0.192 kg,
T,=373 K, 6=294.5 K.
e,-?
SolutiOn. Three bodies participate in heat exchange:
the calorimeter, the water in it, and the metal which
gives away heat, while water and the calOiimeter receive
heat. Let us write the heat-balance equation:
Q, + Q, + Q, = o, (1)
where Q1 ""'" AU1 = c1m1 (9 - T1 ) and Q1 = l!U1 =
c1 m1 (9 - T1 ) are the amounts of heat received by the
calorimeter and water respectively and Q1 = AU, =
c,m 1 (9 - T 1 ) the amount of heat given away by the
body.Substitutingtheserelationsinto Eq. (1), we obtain
c,m1 (6 - T,) + c,m, (6 - T,) + c,m, (6 - T,) =O,
ti•
••• Selected Problem.11 on Physics

whence
(c1m1+ct'Jll}(8-T1)
ca m1 (T1-8)

e,= (0.38 xi03 xo.1~M~~~J01 :~:~> (29t.5-281.4) ,f.r


""9.2 X 10' l/(kg · K). .
237. A brass calorimeter of mass 100 g contains 5 g of
ice at. -to ºC. Molten lead of mass 30 g at the melting
point is poured into the calorimeter. What subst.ances
will be in tbe calorimeter after the beat. exchange and
wbat will be the temperature after tbe stabilization?
Heat loSHS for vaporization should be neglected.
Gt11en: m1 =100 g=O.t kg, rnz=5X 10-3 kg,
T1 =263 K, m,=30g=3x10-' kg.
m-? 8-?
Solution. The heat.-balance equation cannot. be written
straightaway for problems of this type since the result of
the beat exchange is unknown. 1n order to determine the
result, we must analyze t.he process.
The beat excbange in the system involves three bodies:
the calorimeter and the ice receive while the lead gives
away the amounts of heat Q1 • Q9 , and Q1 respectively.
Naturally, the heat-balance equat.ion
Q, + Q, + Q, =o (1)
is v alid in this case. At the end of the process, a certain
temperature will be established in t.he syst.em. Its value
e lies in the temperat.ure interval T 1 < 9 < T ma• where
T 1 is the initial temperature of ice and the calorimeter
and T ma t.he init.ial temperature (melting point) of lead.
~:~.~~ ~:ie~~i:!~!~s~~i:: ~~:~ !~~fe!dt::yi~!~~ ~~
the process of heat exchange. For t.his reason, we shall
proceed as follows: we shall choose arbitrarily the value 9
of the final temperature and then calculate the values of
Q1 , Q9 , and Q8 for this temperature, after which t.he fulfil-
ment of the heat-balance equat.ion (1) should be verified.
If the equation does not. hold, we shall use its Ieft-hand
Cb.. 2. Molecular Physica and Thermodynamics 185

side to make final assumptions concerning the result of


the heat-exchange process. Thus, we assume that 8 = T mt•
where T ms is the melting point of ice. ln other words, we
aSBume that lhe heat exchange proceeds as follows: the
lead is crystallized and cooled, while the calorimeter and
the ice are heated to the temperature T ms = 273 K. We
also assume that the ice does not melt. We can now deter-
mine Q1 , Q1 , and Q1 :
Q1 =c1m.,, (Tm1 - T1),
Q, = 0.38 X 108 X 0.1 X (273-263) 1 = 3801,
Q 11 =c11 m11 (Tm11 -T 1),
Q0 - 2.1 X 10' X 5 X 10_, X (273-263) 1 = 105 1,
Q1 = -11m 1 + c1m 1 (T mi - T m1),
Q, = -3 X 10-• X [0.25 X 10' + 0.13 X 10'
X(600-273)J 1 o; -2025 1.
Let us determine the sum Q1 + Q1 + Q 8 = (380 +
105 - 2025) J = -1540 J. lt can beseen thatunderthe
assumptions made by us, the heat-balance equation does
not hold. Since its left-hand side is d.egative, we assume
that lhe final temperature 8 = T m:t as before, but
a part of the ice of maSB m has melted, i.e. the ice receives
the amount of heat Q1: + Am. Let us substitute the refined
value of the amount of heat received by the ice into
Eq. (1) and determine the mass of water (melted ice)
from it:
m= 1540/(3.35 X 10') kg= 4.6 X 10-• kg.
Since the mass of the obtained water does not exceed the
initial mass of the ice, our last assumptions concerning
the result of heat exchange are valid.
Thus, the established temperature is 273 K, and the
calorimeter contains ice of mass 4 X tO-" kg, water of
mass 4.6 x 10-1 kg, and solid lead of mass 3 X 10-1 kg.
238. Determine the mass of water converted into ateam
as a result of pouring 10 kg of molten lead at the melting
point into a vessel containing 1 kg of water at 20ºC.
The vessel is made of brass and has a mua of 0.5 kg.
Heat loSBeS should be neglected.
186 Selected Problema on Phyaica

Given: m1 =1 kg, T1 =Tz=293 K, m1 -10kg,


m,-0.5 kg.
m-?
Solution. Although the result of tbe heat-exchange
process is not known to us, we assume that the final temper-
ât:!.r;. b:f(s!:%e: s!iu~?::101~rt!b1!!il~n'7f.°~~!. º~:h~:~
excbange proceeds as follows: the water and the calorime-
ter, having received the amount of heat Q1 and Q1 respec-
tively, are heated, while the lead, having given away the
amount of heat Q1 , solidi6.es and cools to e = 373 K. We
also assume that a part of the water of massmis convert-
ed into steam. Under these assumptions, the Collowing
condition holds for the value of tbe mass of t.he steam:
o.;;m.;;m,. (1)
Writing the heat.-balance equation for such a result of the
heat-exchange process, we obtain
Q,+Q,+Q,-0, (2)
where
Q, - c1m1 (0 - T1) + r,m, (3)
Q, - c,m, (0 - T1), (4)
Q, - -l.1m1 + c,m, (0 - T mil· (5)
From the system of equations (2)-(5), we can determine
the mass of vaporized water:
m- ntsl1.+c,(Tm1-8)]-Cm1c1+m.c.)C8-T1)
'•
m ..,.10X (0.25 X 10't~e1! ~~01 X (600-373)) kg
(1x4.19x101+~.::·::.x101)(373-293) kg

<><0.086 kg.
Since the obtained value of the mass satis6.es condition (1),
our assumptions about the result of heat exchange are
correet.
Ch. 2. Molecular Phpics and Thermodynamics i.67

239. A v8Sl!llll whose heatcapacity is0.63 kl/K contains


0.5 lof water and 250 g of ice at OºC. What temperature
will set in after the addition of 90 g of steam attOOºC?
Given: C,=0.63kJIK=0.63x10' l/K, V1 =0.5 l=
SxtQ-i m•. m3 =250 g=0.25kg. m,=90g=
0.09 kg, T,..=273 K, r .. =373 K.
0-?
Solution. As in the previous problems (237 and 238), the
result o[ the heat-exchange process is unknown to us.
Therefore, we assume that ali the bodies will have as
a result of heat exchange a temperature 9 which is
higher than the melting point of ice but. lower than the
boiling point of water. ln this process, the calorimeter
is heated, having received the amount of heat
Q, = C, (0 - T mo), (!)

the water of mass m1 = p1 V1 is heated, having received


the amount of heat
Q1 = c1m 1 (9 - Tma) = c1 p1 V1 (9 - T ma>. (2}

the ice is melted and t.he obtained water is heated. having


received the amount. of heat
Q, =i.,m, +c,m, (0 - T..,), (3)

the steam is condensed and the formed water is cooled,


having given away the amount of heat
Q, = -r,m, + c,m, (0 - 7\,). (4)

Writ.ing the heat-balance equation, we obt.ain


Q, + Q, + Q, + Q, =o. (5)

Substituting the expressions for Q1 through Q., into


Eq. (5), we get
C1 (9 - Tm1) + c1 p1 V1 (9- Tm1 ) + Âams
+c 1m1 (9 - T ma} - r,m, + c1m, (9 - T1t1} =O,
168 Selected Problema on Phyaica

whence the steady-state temperature is


e m,r,+C1Tmac,'ltt~v~~!'::•.t!~V1+n1t) Tm1J
9 i~=~~~6_;',~t~t3~«:;~~~~o!~:.~~~:, K
+~~:a~s;:_~~--:~:;:c\::s5:s~'.to~:~~~~1 K
<><311 K.
Since 273 K < 3tt K < 373 K, our assumptions about
the result of heat exchange are correct.
24.0. A vesael from which air is rapidly pumped out
contains water atO ºC. As a result of intense evaporation,
tbe water gradually freezes. What fraction of the initial
amount of water can be frozen in this way?
Glu<n: T=273 K.
m,lm-?
Solutton. We denote by m the initial mass of water
and by mi and m1 the masses of water converted into ice
and ateam respec.tively. The amount of heat required for
evaporation can only be obtained at the expense of the
beat released during the freezing of water. ln this case,
the heat-balance equation has the form
Q, +Q.=0.
Here Q1 = AU1 = -m,,A is the amount of heat liberated
during the freezing of water and Q1 = AU1 = m 1 r the
amount of heat apent for the evaporation of water, where
A is the latent heat of melting for ice and r the fotent hco.t
of vaporization for water. Substituting these expressions
into the heat.-balance equation, we obtain m1A= m1 r,
whence m 1 = m,,Ur. Since the mass of water remains
unchanged during the heat exchange, we have m= m,, +
m2 , or m = m,, + m1Ur, whence
m,
m=l:"+'·
'
m, 3.35x~ª;:.ex10' º·87·
c.h. 2. Molecular Physica aud Thermodynamics 189

er!~: ~~rt;!=:a~:!ª~ft:~~! i~:~:sª r~::e:~ictoref2i1C


during 5 min, and the water is converted into ice in
t h 55 min. Determine the latent heat of fusion for ice.
Given: At1 =5min=0.3x 103 s, T1 =289 K,
T2 =285K, 't=t h 55 min=6.9x10•s.
A-1
Solution. The refrigerator receives heat from water dur-
ing its cooling and crystallization and transfen it to the
ambient with the help of a special apparatus. For this
reason, tbe temperature in the refrigerator chamber re-
mains constant. Therefore, we can assume that the rate of
change in the internai energy of water is also constant:
AUlt=k=CODSt. (1)
Since tbe internai energy of water has changed by AU =
cm (Tm - T1 ) - Mn during the time t = At1 + 'T, using
these expressions in Eq. (t), we obtain
m [e (Tm - T,) - Al/(At, + •)=k. (2)
Since the temperatura of water decreases during the tim&
At 1 , tbeinternal energybaschangedby AU1 = cm(T1 -T1 ).
Taking this into account, we obtain from Eq. (t)
cm(T,-T,)lllt,=k. (3)
Substituting Eq. (3) into (2), we finally get

A=c[Tm-T -(T -T Ait'tJ•


1 1 1)

A=4.19x !OS X [273-289-(285-289)

X(0.3X~~~~@X10')] J/kg
"'3.35 x !OS 1/kg.
242. A red-bot aluminium cube put on ice whose tem-
perature is-20ºC completely sinks in it. Determine the
initial temperatura of the cube, neglecting the change
in the cube volume as a result of cooling.
170 Selected Problema on Physiea

Giuen: T1 =253 K.
~
Solution. For the aluminium cube to be completely
sunk in ice, the ice in the volume of the cube should be
melted. Therefore, we shall assume that the cube and ice
in the same volume t.ake part in heat exchange. Writ.ing
the heat-balaoce equation, we obtain
Q, + Q, + Q,-0, (1)
where Q1 is the amount of heat received by ice duriog
its heating to the melting point, Q1 the amount of heat
received by ice during its melting, and Q1 t.he amount
:~ehe:eri\;~n ~~~:t byofth~c~~bCo~!~:~ni: ~:!iº~. ~
c1m1 (Tm - T1), Q1 = Am1 , and Q1 = c1 m1 (T m - T 1 ),
we transform Eq. (t) as follows:
c1 m1 (Tm - T1) + 1m1 = c1 m1 (T1 - T ml· (2)
Since m1 = p1 V and m1 = p1 V, where V is the cube
volume, Eq. (2) becomes
c1 p1V (Tm -Ti)+ 1p1 V=C1 p1 V (T1 - Tm),
whence
Tz=c1P1(Tm~1>+"P1+Tm,

Ta 2.t xtCl'x0.9xi:.,:~~~;~>:1~axtOlx0.9xtOIK
+ 273 K o.414 K.
243. A lead bullet lired vertically upwards reaches
a height of 1200 m. Falling to the ground. it is heated as
a result of impact. Assuming that 50% of the mechanical
energy of the bullet are spent on it.s heating, determine the
increase in it.s temperature. Air resistance should be
neglected.
Gtven: h = 1.2 x 10' m, ~ = 50%, or ~ = 0.5.
1!.T-1
Solution. Using the relation
1!.U- -')A,
Ch. 2. Molecular Ph)'llica and Thermodynamic1 t71

where dU =cm dT ia the change in the internai energy


of the bullet as a result of heating by dT and A = W -
W 0 =-mgh the work done by forces during the ineiastic
impact agaiJllt the ground. we obtain cm dT = 'lmgh,
whence
6T=~,
dT 0.5x:.~~~-~X10*K~45K.
24.4. At what velocity does a iead bullet striking a wall
melt? The temperature of the bullet before the impact
is tOOºC, and60% of its mechanical energy are convert-
ed into the internai energy.
Gtven: T=373 K, ~=60%, or ~=0.6.
u-1
Solu.tion. We shall use the relation
~<lll=-~A. (1)
Here ~ dU = mÃ. + cm (T m - T) is the change in the
intemal energy of the bullet during its melt.ing and
heat.ing to the melting point and A = W - W 0 =
-rrw'/2 the work done by forces during the inelast.ic
collision of the bullet with the wall, where T m ia the
melting point of lead. Taking these relations into ac-
count, we write Eq. (1) in the form cm (T m - T) + ml. =
flrrwl/2, whence
v=V2(c(Tm'I T)+l.] 0

v=JÍ2X(0.13X10'X(~;-ª13)+0.25X10t): ~426 m/s.

24.5. A sledge of mass 6 kg slides down a hill with


a alope of 30". Having traversed a distance of 50 m aiong
the hill, the sledge attains a velocity of 4.5 m/s. Deter-
mine the amount of heat liberated as a reault of friction
of the runners against snow (Fig. 102). '
Gii.'fln: m=6 kg, a=30º~0.52 rad, l=50 m,
V=4.5 m/s.
Q-1
172 Seleeted Problema on Phyaica

Solution. Let us consider a closed system of bodies


consisting of the runners and the ice surface of the hill.
We shall use the relation
1!.U =-A, (1)
where A = W - W 0 is the work done by lhe force of
friction between the runners and ice, W = mv1/2 + mgk

Fig. 102

lhe total mechanical energy of the sledge at the end of


motion, and Wo= mgH the total mechanical energy of
the sledge at the beginning of motion. Taking into ac-
count lhe expressionsfor A, W, and W0 , we write Eq. (1)
in lhe form
1!.U = -(mv'/2 + mgh - mgH)
= mg (H - h) - mv1/2 = m (gl sina - v"'/2).
Then
1!.U= 6 X (9.7 X 50 X 0.5 - 4.5') I <>< 1.4 kl.
Since the cbange in the internai energy of the interacting
bodies is measured by the amount of heat, the amount of
heat Iiberated as a result of friction of the runners against
snow must he equal to the change in lhe internai energy
oi these bodies: Q = l!.U "" 1.4 kl.
246. While beating 300 g of water from tS to 68 ºC
on a spirit lamp, 7 g of spirit were burnt. Determine
the efficiency of the spirit Iamp.
Given: m1 =300 g=0.3 kg, T1 =291 K, T2 =341 K,
m,=7 g=7X 1()-• kg.
~-?
Ch. 2. Molecular Phyaiea and Thermodynamics i73

Solution. By defi.nition,
~ = * 100%. (t)
Here Ou = ~ 4U1 = c1m1 (T 1 - T 1) is the amount of
heat spent to change the intemal energy of water and
Ql'=- 4U1 = qm 1 the amount of heat liberated as are-
sult of combustion of spirit, where c1 is the specifi.c heat
of water and q the latent heat of combustion of spiri t.
Substituting the expressions for Qu and Q1 into Eq. (t),
we obtain
11 = e1m1~~-T1) 100%,

11 4.19 ;~~ ~i·~~ ~b=i 29t) 100% ~ 31%.

247. The effi.ciency of a locomotive is 30%. Determine


the consumption of fuel in the locomotive per hour if
its power is 7.36 x 101 W.
Given: ~-30%, N =7.36x IO'W, t-1h=3.6 x IO's.
m-1
Solutton. The effi.ciency of the locomotive is

~=*100%. (t)
Here Q.,,.. 4U1 =A = Nt is the amount of heat spent
to change the internai energy of the fuel gas, as a result
of which a meehanical work is done, and Q1 = 4U1 = qm
the amount of heat liberated during the combustion of
the fuel, where q is the latent heat of combustion of the
fuel. Substituting the expressions for Q. and Q1 into
Eq. (1), we obtain

whence
m=Ne·::i",
7.36x:x~;::::xtookg~O.t9 kg.
Seleeted Problems on Pby1ics

248. An engine consumes 25 kg of petrol per'hour and


is cooled with water. The temperature difference of water
at the inlet and outlet of the radiator is t5 K. Determine
the water Dow rate per second if 30% of the energy liber-
ated as a result of combustion of petrol are spent for
heating water.
Giwn: m=25 kg, •=1 h=3.6xt()I s, âT=15 K,
~=30%, or ~=0.3.

m,-1
Solution. By hypotbesis,
~=Q,IQ,.
where Qu is the amount of heat required for heating
water and Q1 the amount of heat liberated as a result
of comhustion of the fuel. Considering that Qu =cm,, AT
=
and Q1 qrn, we ohtain
~=cm,. âT!(qm),
whence the mass of water is
m 1 = qm~ !(e âT).
Consequently, the water consumption per second is

m.=~=e":i.r'
m.-1.1:·!11>;,.1:1~~3>j~31oa ~ ~ 1.53 kg/s.
249. Under what conditions can ice be a heater?
Answer. Ice can he a heater in contact with the hodies
whose temperatura is lower than the temperatura of ice.
Since the temperatura of crystallization of water under
normal conditions is 273 K, the temperature of the hodies
for which ice is a heater must be Iower than this tem-
perature.
250. Why don't damp match sticks bum?
Answer. A match stick hurns when its temperature
attains the value at which the suhstance of its head is
ignited. When a damp match stick is ruhhed against a
matchhox, the match stick receives an energy the major
partofwhichisspentto evaporate the moisture contained
Ch. 2. Molecular Phyaics and Thermodynamics 175

in it. Therefore, the temperature of thehead caonot attain


the value at which it is ignited.
251. Why is salt sprinkled over pavements io winter
to remove ice from them?
Answer. The mixture of ice and salt has a lower meltiog
point than that of pure ice. Since the temperature of
atmospheric air is higher than this melting point, the
mixture rapidly melts.
252. Two bodies (copper and iron) of the same mass fali
from the same height. Which of the bodies will be heated
to a higher temperature as a result of the impact?
Auswer. We shall assume that the initial temperatures
of the bodies are equal and that the same fraction of their
mechanical energy is converted into the internai energy as
a result of the ioelastic impact. Since the messes of the
bodies, the initial heights, and the final velocities at
the momeot of impact are equal, we have
llU1 =1lU1 ,
or
c1m (T1 - T)=c,m (T, - T), (1)
where T1 and T1 are the temperatures of the copper and
iron bodies at the moment of the impact and Tis the ini-
tial temperature of the bodies. Since c1 < c1 , it follows
from Eq. (1) that (T1 - T) > (T, - T), i.e. aa a result
of the impact, the copper body will be heated to a higher
temperature than the iron body.

EXBRCJSES

253. Calculate the muses of ao ozone 0 8 , a carboo


dioxide C0 1 , and a methaoe CH., molecule.
254. How many molecules are cootained in t kg of
(a) hydrogen H 1 aod (b) oxygen 0 1 under normal condi-
tioos?
255. The mean kinetic energy of traoslational motion
of ga.s molecules is 1. 7 x 10-11 J at 5000 ºC. What are
the values of this energy at -273 and 1000 ºC?
256. A mixture coosists of 24 l of water at 12 ºC and
40 l of water at 80 ºC. Determine the steady-state tem-
perature if the thermal lomes during mixing are 420 kJ.
176 Selected Prohlems on Physics

257. A piece of aluminium having a mass of'WO g and


heated to 100ºC is placed into a calorimeter containing
270 g of water at 1.2 cc. The temperature at thermal
equilibrium is 23 ºC. Determine the specific heat of alu-
minium if the heat capacity of the calorimeter is 42 J/K.
258. A piece of ice of 20 kg at -20ºCis immersed in
20 l of water at 70 ºC. Will the entire mass of ice be
melted?
259. A lead bullet Dies at a velocity of 200 m/s. What
will be the change in the bullet temperature if its entire
energy is spent for its heating?
260. A steam hammer of mass 10 t falis freely from
a height of 2.5 m on an iron ingot of mass 200 kg so that
30% of the amount of heat Jiberated during the impact are
.spent for heating the ingot. How many strokes has the
hammer made if the temperature of the ingot has in-
creased by 20 ºC?
261. A locomotive having a mass of 21.3.5 t and a ve-
locity of 72 km/h comes to a halt. What amount of heat is
liberated during braking?
262. What power is developed by a bicycle engine if
the petrol consumption over 1.00 km of the path is 1..7 l
at a velocity o[ 25 km/h? The efficiency of the engine
is 20%.
263. What will happen with the levei of water in a glass
with a piece of ice after its melting?

QUESTIONS FOR REVISION


Ch. 2. Molecular Phy11ic11 aad Thermodynamies 177

of a substance. 13. What is tba chan_ge in the internai mergy of a


su6stance durinR 118 heaüag or cooling if the substance rem.Jins
in the aame atate of aggregation? Ui. De&ne speclfle heat, latent
heat of fuaion, and latent heat of v:i:rization. 15. What is the
~r.e~wtt i:'ü:~=~:: o~ ~m=o:~li~l/811c:'':,~'ü;
the heat-balance equation.

2.2. Propertles oi Gmes


The state of any gas can be charact.erized by its mau m,
the volume V occupiedbyit, the pressure p exerted by the
gas on the vessel walls, tbe temperature T, and the molar
mau M. Equations relat.ing these quantities are known
as the equations of state for the gas.
Boyle's law. For m = const and T = const, we bave
pV=const,
or for two states of a gas,
P1V1=P1.V2·
This law describes llothermal proce&11e1.
Charles' law. For m = const and p = const, we have
VIT=const,
or for two stat.es of a gas,
V1tT1 = V 11T,,.
This law describes iaobarle processes.
Gay-Lussae's law. For m= const. and V= const, we
have
plT= const,
or for t.wo st.ates of a gas,
P1IT1=P·iT,,.
This law describes lsoehorie processes.
Ideal gas law. For m= const,
p VIT = const,
or for t.wo st.ates of a gas,
P1V1IT1= P2V,/T2.
12-0970
178 Selected Problems on Physics

Consequently, aoy state of a gas can be compared with its


state under normal conditions: p 0 = 1.01 X t()I Pa, T 0 =
273 K (O cC), and V0 = 22.4 x 10--a m• (the volume of
a mole of any gas under normal conditions).
Clapeyron's equation of state (for an ideal gas):
pV - (m/M) RT,
where m is the mass of the gas, M its molar mass, and
R =8.3 J/(mol·K) a constant which is the same for ali
gases and is known as the molar gu eomtant.
Dalton's law. /n a ve111el contalntng a mizture of several
gases which do not enter into chemical reactWn1 with one
another, the pressure ts equal to the swn of parttal prusures
(viz. the pre8GUT'es produced by each gas in the ab&tnce of
other gases tn the veBBel):

P=Pi+Pa+···+Pn·
Any problem on gas laws can be solved by using Clapeyron's
equation of state or the ideal ~ law (depending on the conditions
of the problem). Boyle's, Charles', and Gay-Lu.ssac's laws can he
treated as special C891!S of these equations.

264. The volume of an air bubble increases threefold


as it rises from the bottom of a lake to the surface. What
is the depth of the lake?
Gtven: V2 =3V1 •
h-1
Solution. We assume that the temperature of water in
the lake is the same at any depth. Then the temperature
of air in the bubble is constant, and according to Boyle's
law,
P1V1 =Pzv••
where p1 and p 2 are the pressures of air in the bubble at the
bottom and at the surface of the lake respectively, and V1
and Vz the volumes of the bubble at the bottom and at
the surface.
Cb. 2. Molecular Physics and Thermodynamics t79

Tbe air pressure in the bubble at tbe surface is obviously


equal to the at.mospheric pressure p 0 , i.e. p 1 = p 0 • Then
p1V1=3p 0 V1, wbence p1 =3Po·
Consequently, the increase in pressure at the bottom is
Ap=p1 -Po=3p0 -p0 =2p0 • lt isknownfrom hy-
drostatics that Ap = pgh, wbere p is the density of the
wat.er and h the depth of the lake. Equating the right-
band sides of the last equations, we obtain 2p0 = pgh,
wbence
h=~,
h= 2 ~~~;'81 <Pm~20.6 m.
265. A vessel with air wbose pressure is 97 kPa is con-
nected with a pist.on pump. After five st.rokes of the
pist.on, the air pres.sure in the vessel becomes 29 kPa.
Determine the ratio of the volumes of the vessel and the
cylinder of the pump.
Given: Pe = 97 kPa = 9. 7 x 10' Pa,
p 5 =29kPa=2.9X10' Pa, n=5.
V1/V2 -?
Solutfon. Let V1 and V 1 be thevolumesofthevesseland
the cylinder of tbe pump. After the first conneetion of the
cylinder with the vessel, we bave, according t.o Boyle's
law, p 0 V1=p1 (V1 + V1), wbence p1=p0 V1/(V1+V1) 0

!~i:!ce th;1 =~:~1(V1c+n;::.io~ Ít1~1p:-'2i(~1: J:)~'.


Similarly, after the fl.fth connection, Ps= p 0 ~/(V1 + V1 )".
Let us transform the obtained equation:
~=( V1;i:V1)5, log-:= 5 log V1~V1,
wbence
log V1~V1=jlog~,

log v.~v. =jlog ::;~:: ~jlog3.35

=-}0.5250=0.1050 .
...
... Seleet.ed Problems <1n Physics

From the table of antilogarithms, we lind that


(V1 + V,J/V1 ~ 1.274, whence V1/V2 ~ 3.65.
266. A closed vessel whose volume is 11 contains 12 kg
of oxygen. Determine the pressure of the oxygen at 15 ºC.
Given: V1 =11=10--3 ms, m=12 kg, T1 =288 K.
p,-?
Solution. Let us compare tbe state of tbe given mass of
oxygen (p1 , V1 , T1 ) with its state under normal conditions
(p0 , V0 , T 0 ). According to the ideal gas law,
P1Y1IT1 =PoVolTo.
Since V0 = mlp0 , where p0 is the density of oxygen under
normal conditions, p 1 V1 /T1 = p 0ml(T 0 p0 ), whence
p,mT1
P1= V1ToPo'

P1 t.~~~~:a~~:3Pa~8.94x10' Pa.

py~!. ac:~1:::eoft~el ~~e:::i;!r:t;~1:f atx> 1.~s occu-


Given: T1 =300 K, V1 =11=10-3 m 1•
Pi-?
SoluUon. Wecompare thestateof agiven mass of the gas
(p 1 , V1 , T1 ) with its state under normal conditions
(p0 , V0 , T 0 ). According to the ideal gas law, we have

whence

P1= Pv:;.~• •
Pi= 1.01X1'1'1-:,..-i;-~.r.;;31()-1X300 Pa ~ 2.49 MPa.

268. How many molecules of air are contained in


a roam of volume 240 m8 at a temperature of 15 ºC under
a pressure of 10' Pa?
Giuen: V1 =240 m 3 , T1 =288 K, p 1 =10' Pa.
N-1
Ch. 2. Molecular Physlcs and. Thermodynamies 181

Solutton. ln order to use tbe ideal gas law, we shall com-


pare tbe states of a given mass of tbe air under given
(p1, V1 , T1 ) and normal (p 0 , V 0 , T 0 ) conditions. Tbis gives
p 1 V 1 /T 1 =p 0 V 0 /T 0 • (1)
Since tbe volume of tbe air under normal conditions is
V0 = m/p 0, wbere Po is tbe density of tbe air under
normal conditions, Eq. (1) assumes tbe form p 1 V1 /T 1 =
Pomf(T oP0), whence
m= PJ.V1ToPol<PoT1) 0

Let us calculate tbe number v of moles contained in tbe


given mass of tbe air:
V= m/M =P,.V1ToPol<PoT1M).
Since tbe number of molecules in a mole of any gas (in-
cluding air) is equal to Avogadro's constant NA, we bave
N=NA.v=NA P~l.:
N=6.02x 1023x 1 .o!~~~fu~~91~2:0- 1 ~6x 1011.
269. Tbe volume of air in a room is 100 m•. Wbat is
tbe mass of tbe air leaving the room as a result of an in-
::::r~n ::~b2rª~P:'? from 10 to 25 ºC if tbe atmospberic
Given: V1 =100 m1 , T1 =283 K, T1 =298 K,
p 1 -102 kPa = 1.02 X 10' Pa.
6m-?
Solutton. Let us compare tbe states of tbe maS8 m1 of
tbe air in tbe room under tbe given (p1 , V1 , T1 ) and nor-
mal (p0 , V~, T0 ) conditions:
P1V1IT1=PoV"JTo=Pom1/(ToPo),
wbere v; = milPo· Tben
m1 = P1 V1T0Pol(T1Po).
As tbe temperature increases, tbe volume and preS8ure
will not increase, but tbe mass of tbe air in tbe room will
182 Selected Problems on Phys.ics

be difterent (m 1). Similarly, we shall compare the 11tates


of the mass m1 of air at the given (p1 , V1 , T1 ) and normal
(p 0 , V;, T0) conditions. This gives p 1 V1 /T1 = p 0 Y;IT•=
Pom.tf(T oP 0 ), where v;; = m 21Po· Then
m1 = P1 V1T oPol(T1Po)·
Thus, the mass of the escaped air is
âm=mi-mi= Pi:::;"- P1~1:.oPe
= P1V;:oPo T;.~~1
âm 1.02x1~~t~1;21axt.29 :~:kg!::!!G.aa kg.
270. A cylinder with the area of the base of 100 cm2
contains air. A piston is at 50 cm from the cylinder bot-
tom. When loaded by a weight of 50 kg, the piston is
lowered by 10 cm. Determine the temperature of air
after the loading if i ts pressure before the loading was
101 kPa and the temperature was 12 ºC.
Giuen: 8=100 cmª=10-ª mª, k1 =50 cm=0.5 m,
m=50 kg, !=10 cm-0.1 m, p 0 =101 kPa=
1.01x10' Pa, T1 =285 K.
Ta-?
Solution. Let us consider two states of air under the
piston: before the loading and after it. The state of air
before the loading is characterized by the parameters p 1 ,
~:~s~l~ltp~: ;!~e~:i: ~ººª~º:·~':!; !~,s~d !d w;::
Sh 1 • or sinceh 1 =h 1 - l, v 2 ,,,,. S (h 1 - l). Let us apply
the ideal gas law to these two states: p 1 V11T1 =- p 1 V1/T1 ,
whence
(1)
Substitutingtheexpressionsforp1 , V1 , p 1 , and V1 into
Eq. (1), we obtain
r2 (P1+m1/!~!"1-l)T 1 (p0 +m1~~A1-l)r 1 •

T,,= (1.ot x10'+~.~t9:~1~b~:·5-0.t)x285K ~ 339 K.


Ch. 2. Molecular Phyaica and Thermodynam1cs 183

271. Determine the density of hydrogen ata tempera-


ture of t5 ºC under a pressure of 98 kPa.
Given: T = 288 K, p = 98 kPa = 9.8 x !Cl' Pa.
p-?
Solution. Using Clapeyron's equation of state for an
ideal gas, we obtain pV = mRTIM. Since V= m/p, the
above equation becomes pmlp= ml(MRT), whence
pM
p= RT'

p s.s~~~~~to-• :~ ~0.082 kg/mª.


272. Ten grams of oxygen are under a pressure of
0.303 MPa ata temperature of tO ºC. After heating at con-
stant pressure, the oxygen occupies a volume of 10 l.
Determine the initial volume and the final temperature
of the gas.
Giuen: m=IO g=ICJ-' kg, p=0.303 MPa=
3.03x 105 Pa, Ti=283 K, V2 =10 l=to-a m'.
Vi-? Ta-?
Solution. Using Clapeyron's equation of state pV1 =
mRT1/M, we can determine the volume of oxygen before
heating:
Vi=~!1,
Vi a 2 ~~~ª:.óà~O' mª~2.4x 10-a mª.
By hypothesis, the oxygen is heated at constant pressure,
and hence its temperature after heating can be determined
from Charles' law V1/T1 """V 11T 1 , whence
Ta= Vy~•'
T2 = ~:~~ K ~ 1.18X108 K.
273. Hydrogen leaks from a cylinder of volume 10 l
because of a faulty valve. At 7 ºC, the manometer indi-
cates 5 MPa. At t7 ºC, the reading of the manometer
18' Selected Problema on Phyaics

remains unchanged. Determine the mass of the gas


that has leaked from the cylinder.
Given: V=10 1=10-0 m•, T,=280 K, p=S MPa=
5x10'Pa, T1 =290K.
Am-?
Solutton. Using Clapeyron's equation of state pV =
m1 RT1 /M, we ean determine the initial mass of hydrogen:
m,=pVMl(RT.).
Similarly, we can tind the mass m1 of hydrogen after the
leakage:
m1 =pVMl(RT1 ).
Consequently, the mass of the leaked gas is

Am=m1-ma= ':r~ -"::: = p~M r;.~~1'


Am 5xtO•x~:sJ~ii~2iJ,2110-280) kg~t.5x t0-'kg.
274. Determine the densit.y of a mixture consisting of
!r ~ ~/; ~~:~~':1P::u!2 o1 ;~ :II.~ª at a temper_ature
Given: m1 =4g=4X :I0-1 kg, mz=32g=32x11)-3kg,
p=93 kPa~9.3x10• Pa, T=280 K.
p-1
Salutton. According to Dalton's law, the pressure of
the mixture is
P=P1 + P1• (1)
where p 1 and p 1 are the partial pressures of hydrogen and
oxygen under the given conditions. Writing Clapeyron's
equation of state for each gas separately, we ohtain
P1Vl= m1RT1IM1 , p 1 V1 = m1RT1 IM1.
By hypothesis, V1 = V1 = V aad T1 = T1 = T. Then
for hydrogen we have p 1 V=m 1RTIM1 , whence
p,=m,RTl(M,V). (2)
Ch. 2. Molecular Physics and Thermodynamics i85

Similarly, for oxygen we get


p,-m,RTl(M,V). (3)
Substituting expressions (2) and (3) into (1), we obtain

P= : : i:: + ;: ': = ~T (-i!-+;:)'


whenee
V= ~Te::+;:). (4)
By defioition, the density of the gas mixture is
p-m/V, (5)
where m - mi +
m11 is the mass of the gas mixture.
Substituting Eq. (4) into (5), we get

p= mi/::t:. :r '
1
/M1

P=-=4x1o=t1c2~~~11a~~\~;(32xlo-t) s~:2~1;, ~
<><0.48 kg/m8 •
275. The volumes of two vessels with a gas are 3 and 4 l
respectively. The gas is under apressure of 202 kPa io the
first vessel and under 101 kPa in the second vessel. Wbat.
will be the gas pressure after the connection of the vessels?
Assume that the temperature in the vessels is the same
and maiotaioed eonstaot.
Gtven: V1 =31=3x10-3 mª, V1 =41=4x1Q-J mª?
p 1 -202kPa-2.02x1()1 Pa, p1 -IO! kPa-
1.01x1()1 Pa.
p-1
Solution. Accordiog to Dalton's law,
P=Pa +p,. (1)
Since the process is isothermal, the partia! pressure oí
the gas ln eacb vessel can be determined from Boyle's law:
p1V1 =p 3 V and p1 V1 =p,V, where V=V1 +V1 .Tbeo
tbe partia! pressure of tbe gas in each vessel after th&
186 Selected Problems on Physica

conneetion is
p,=p,V,IV, p,=p1V1 /V. (2)
Substituting expressions (2) into (1), we obtain
p= P~1+ 1'1;1 = P1V1tP1V1,

p=2.02x 1()1 x:~!r-:+:·::O-!OI x4xi0-•Pa ~ 144 kPa.


276. Three grams of water are introduced into a t0-1
vessel containing dry air under normal conditions and
heated to 100 ºC. Determine the pressure of damp air
in the vessel at this temperature. The pressure dependenee
-0f the boiling point for the water should be negleeted.
Gtven: m=3g=3X10-3 kg, V=!O 1=10-Z m•,
T=373 K.
p-1
Solutton. Aceording to Dalton'slaw,p = p 1 + p 1 , where
p 1 and p 2 are the pressures of dry air and water vapour
respectively. Since the heating occurs at constant volume,
the pressure of dry air at 373 K can be determined from
Gay-Lussac's law: p 0 /T 0 =p1 /T, whence p 1 =p0 TIT 0 •
Let us determine the pressure of water vapour, treating
it as an ideal gas. The number v of moles of water vapour
in the vessel is
v=mJM,
)Vhere M is the molar mass o( the vapour. Then the vol-
ume occupied by vapour under normal conditions is
V~=V 0 v=V0 m/M. (t)
From the ideal gas law p 0V;IT 0 = p 1 VIT, we can find the
E~~º(1>.P~:S!re;trv7i7~~6<J,)1:c~~~~:~~1;~~hª:C;~e~
.sure of damp air is
p= ~: + p;~;: = ':;: (t +~i-).
P 1.01x2;x373 ( 1 + 22.:;/:·1•:x31~0"- 3 ) Pa
""190 kPa.
Ch. 2. Molecular Physics and Thermodynamics i.87

277. How many air molecules are contained in a cubic


centimetre of a vessel at a temperature of tO ºC if the air
in the vessel is rarefied to a pressure of 1.33 µPa?
Giuen: V=t cmª=1(}-I mª, T=283 K,
p-1.33 ~Pa -1.33 X 10.. Pa.
N-1
Solution. The number of molecules in a cubic cenüme-
tre ofair can be calculated from Avogadro's constant NA
and the number v of moles contained in a cubic centime-
tre of air by the formula
N=vNA.
The number v of moles contained in a cubic centimetre
of air can be defined as the ratio mJM, where m is the
mass of air contained in a cubic centimetre and M the
molar mass of air. The ratio m!M can be determined from
Clapeyron's equation of state:
m!M- pVl(RT).
Thus, the required number of molecules is
N= ~~NA,

N t. 339~~~·;;0-• x 6.02x 102ª=3.4x 108.


278. A sphere of mass 40 g filled with air isinequilib-
rium in a lake at a depth of tOO m and at a temperature
of 8 ºC. Determine the mass of the air in the sphere if
the atmospheric pressure is 99. 7 kPa. The walls of the
sphere are made of thin rubber.
GiV<n: h-tOOm, T-281K, m-40g-4xtC>-'kg,
Patm=99.7kPa=9.97X10' Pa.
m1-?
Solution. The sphere immened in water is acted upon
by the force of gravity mg and the buoyant force F = pVg
exerted by the water, where p is the density of water
(Fig. 103). By hypothesis, mg= F, or mg = pVg, whence
V = m/p. The mass of air in the sphere can be determined
Seleeted Problema on Phya.iea

from Clapeyron 'a equation of ata te:


m1 ~ MpVl(RT). (!)
Here p = Patm + Plu where Pb = phg is the hydrostatic
pressure of water at a depth h. Substituting tbe exprea~
sions for p and V into Eq. (1), we obtatn
m. M(Patm:rphf)m/p,

ml = 29 X to-• X (9.97 X tO't1: ~:X 9.8) X" X t.0-S X rn-• kg

""5.36 X 10"' kg.

~ Fig. tos
279. Why does the body of a deep-water fi.sh swell when
it is brought t.o the surface?
Answer. At a large depth, the pressure instde the fi.sh
is balanced by t.be externai pressure which is much higher
than the atmospheric pressure. When the fi.sh is brought.
to the surface, the externai pressure becomes equal to the
atmospheric pressure, while the pressure inside the fi.sh re-
mains unchanged. As a result., the volume of the fi.sh
sharply increases.
280. Elect.ric bulha are filled with kryp\on at a low
pressure. Why?
Answer. During the operation of a bulb, the gas in tt is
strongly heated, which considerably increases the gas
pressure. If the initial pressure were not low, it would
lead to an explosion of the bulb.
Ch. 2. Molecular PhysiCB and Thermodynamiea 189

EXERCISES
281. A narrow cylindrical tuhe sealed at one end is
in the horizontal position. Air in the tube (whose volume
is 240 mm8) is separated from the atmospheric air by
a mercury column of length 15 cm. Ir the tube is turned
to the vertical position with the open end facing up, the
air in the tuhe will occupy a volume of 200 mm8 • Deter-
mine the atmospheric pressure.
282. A 2-1 cylinder contains a gas under a pressure of
0.33 x 1()& Pa, while a 6-1 cylinder contains the same gas
under a pressure of 0.66 X 101 Pa. The cylinders are con-
nected through a tube with a valve. What pressure will be
estahlished in the cylinders after the opening of the
valve?
283. A cylinder contains a gas under a pressure of
131.3 x 1()& Pa at a temperature of 30 ºC. As a result
of leakage, the pressure in the cylinder drops to 2.02 X
101 Pa and the temperature to -25 ºC. What fraction
of the gas remains in the cylinder?
284.. A narrow cylindrical tube of length 100 cm open at
two ends is immersed in mercury to half its length. The
upper end is closed and the tube is taken out of mercury.
Determine the height of the mercury column that remains
in the tube, assuming that the atmospheric pressure is
normal.
285. Determine the difference in the masses of air
filling a room of volume 50 m8 in summer and winter if
lhe temperature in summer reaches 40 ºC and in winter
drops to zero. The atmospheric pressure should be regard-
ed as normal.
286. Determine the mass of a mole of a mixture con-
sisting of 25 g of oxygen and 75 g of nitrogen.
287, To what temperature should a flask containing air
at 20 °C be heated for its density to decrease by a factor
oi 1.51
288. Determine the mass of oxygen containing in
a cylinder having a volume of 1 l under a pressure of
0.93 x 1os Pa ata temperature of 17 ºC.
289. What is the volume occupied hy 3 g of carhon di-
oxide under apressure of 133 kPa ata temperature of 27 °C?
t!JO Selected Problema on Pbyaics

290. Determine the density of oxygen contained in


a cylinder under apressure of 3 X 10' Pa ata temperature
oi 17ºC.
291. Plot the p-V graphs for isothermal, isobaric,
and isochoric processes.

QUESTIONS FOR REVISION

2.J. Work Done by • Gas.


HHIEnglnes
An elementary work done by a gas contained in a cylin-
der under a piston during expansion is
d.A = F eos a.dl.

For a. = O, we obtain
dA = F dl = pS dl = p dV, (1)
where p = FIS is the gas pressure, S the area of the piston,
and d V """ S dl the ehange in the gas volume.
The total expanslon work of a gas from volume V1 to V2
is determined by integration:

A=l'pdV.
v,
A heat engtne is a deviee in whieh the internai energy
of a fuel is eonverted. into meehanical work.
Cb. 2. Molecular Physics and Tbermodyn.amics 191

Any heat engine includes a heater, a working substance,


and a cooler (Fig. 104). The heater supplies an amount of
heat Q1 to the working sub.stance. The cooler receives ao

Fig. 1<K

amount of heat Q1 from the working sub.stance. The


mechanieal work done by the working sub.stance is
A= Q, -Q,.
The operation of a heat engine is characterized by the
efficiency defined by the formula
~= QQ.Q' 100%.
The efficiency of ao ideal e.amo heat engine is
'1 = T1;::T1 100%,

where T1 and T1 are the temperatures oftheheaterand


the cooler respectively. The efficiency of a heat engine is al-
ways less than unity (the seeond law of thermodynamfes).
Ao ideal heat engine is reversible (refrigerator).

292º. Ten grams of carbon dioxide areheated from 20 to


30 °C at constant pressure. Determine the expansion work
of the gas and the change in its internai energy.
Giv•n: m=IO g=tO-Z kg, T,=293 K, T,=303 K.
A-? MI-?
,.. Selected Problema on Physies

Solution. The change in the internai energy of earbon


dioxide during ita heating from T 1 to T 1 ean be determ.ined
by the formula
l!U = cvm (T9 -T1).
wbere cv is tbe specific heat of carbon dioxide at con-
stant volume. Consequently,
1

/!.U = 0.83 X 10' X !O-• X (303-293) J = 83 J.


The expansion work of a gas is

A= 1·v, pdV. (!)

By hypothesis, the presSure p of carbon dioxide is con-


.stant and henee can be taken outside the integral sign.
Expression (1) then becomes
v,
i
A=p dV=p(V,-V,).
v,
(2)

From Clapeyron 's equation of state


pV=mRTIM
we determine tbe volume of carbon dioxide at a temper-
atura T: V= mRTl(Mp). Consequently, at the tem-
peratures T 1 and T 1 carbon dioxide will occupy the
volumes
V1 = mRT1/(Mp), V,= mRT,l(Mp). (3)
Substituting expressions (3) into (2), we obtain
A=p( m;:1 - "'~!' )= "': (T2 -T1),
A= ·~~~~ 2 x(303-293)1<><18.9J.
293º. Si:r. grams of oxygen (0 1 ) taken at a temper8.ture
of 30 ºC e:r.pand at. eonstant preMure so tbat tbe volume
1 Thia formula ean be applied sia.ce the internai energy of an
!:'!!rr!tdi::::;:!Y~~teul~:!i:J:').negleet the potent1al
Ch. 2. Molecular Phys.ica aad Thermodynamic1 193

inereasea twofold due to a heat supplied from outside.


Determine the expansion work of the gas, the change in
the intemal energy of oxygen, and the amount of heat
supplied to it.
Given: m=6 g=6xt0-3 kg, T1 =303 K, V1 =2V1 •
A-11!.U-1 Q-1
Solution. The cbange in the internai energy of oxygen
as a result of its heating by âT is
l!.U = cvm l!.T, (1)
where ev is the specifi.c heat of oxygen at constant volume
(see footnote 1 to Problem 292º) and AT = T1 - T1 the
change in its temperatura, Ti and T 1 being the initial
and final temperatures of oxygen.
Since oxygen expands at a constant pressure p, we can
UBe Cbarlea' law V1/T1 = V 1 /T 2 to determine the fi.nal
temperature of oxygen:
T. = v.T1IY1 = 2V1T11Vi = 2Tl" (2)

Substituting expression (2) into (t), we get


AU = cvm (T1 - T1) = cvm (2T1 - Ti)= evmT 1,
f!.U = 0.92 X 10' X 6 X 10-1 X 303 J <>! 1.67 kJ,
Let U9 determine the expansion work of the gas:

or, considering that p = const,


v,
A=p 1
v,
dV=p(V,-V,)=p(2V,-V,)=pV,. (3)

Using Clapeyron's equation of statepVi = mRTilM, we


can determine the volume of oxygen at the temperature T1 :
V1 = mRT1/(Mp). (4)
U-0970
... Selected Probleme on Phyaics

Substituting exprusioo (4) into (3), we get


A=p "';!1 = m;r1,
A 6xti;~~;!xaoa 1 ~ 473 J,
The amouot of heat supplied to the gas can be deter-
mioed from the first law of tbermodynamics:
Q = IJ.U +A,
Q = (1670 +
473) 1 "'2.14 kl.
294º. Ten grama of nitrogen (N 1 ) expand isothermally
at a temperature of -20 ºC so that the gas pressure drops
from 202 to 101 kPa. Determine the expansion work of
the gas, the change in the internai energy of nitrogen,
and the amount of heat supplied to it.
Given: m = 10g=10-' kg, T = 253 K, p, = 202 kPa =
2.02 X 10' Pa, p, = 101 kPa = 1.01 X 10' Pe.
A-? IJ.U-? Q-1
Solution. Since the change in the internai energy of
a gas is
âU =cm AT
and the gas temperature remains constant during expan-
sion, the change in temperature AT =O, and hence AU=
O. Let us determine the expansion work of the gas:

A=í'pdV. (1)
v,
Using Clapeyron 's equation of state, we can determine
the gas pressure:
p = mRTl(MV). (2)
Sub.stituting expression (2) into the integrand of (t), we
obtain
Ch. 2. Molecular Physlct and Thermodynamies 195

Having determined the volume ratio V~i/V1 = p1 /p 2 by


Boyle's law and substituting it into Eq. (3), we obtain
A= ni;r lnfs.
A 10-•,; !·~~ 258 ln ~:: ~ !~ ~ 521 J.
The amount of heat supplied to the gas can be determined
from the 6.rst law of thermodynamics:
Q - IJ.U +A,
or, since l!U =O,
Q-A <><521 J.
295°. Air of mass 0.2 g is eontained in a cylinder under
:i::'~}12ô !tto~e:r:!~::~h~re!,~l ~5o:1~!:i:gt~mJ::
uniform ascent of the piston from a height of 10 to
20 cm. The atmospheric pressure is nonnal.
Gtven: S = 15 cm1. = 1.5 x 10-3 m1., m = 0.2 g=
2x w-• kg, T-293 K, h,-to cm-0.t m,
h1 =20 cm=0.2 m.
A-1
Solution. During the uniform ascent of the piston, it is
acted upon by the externai force F e:rt• the
force of pressure F exerted by the air
::!eF atti:oft~!º:t'm'!~~~ç ':i':tF~. fã;}~
Let us determine the work done by the
Fot
F
externai force to lift the piston to the
height h1 :
..
A=
..i Feztdh.

ln order to determine the externai force,


(1)

Fig. t05
we write the equilibrium condition for
the piston in projections on the Y-u:is: F nt+F-Fatm =
O, whence

... Fe:rt = Fatm - F.


196 Selected Problems on Physics

By de6.nit.ion. F 11m = p 0S and F = pS. Here p 0 is the


atmospheric pressure and p = mRTl(MV) the pressure of
the air under the piston, where V = Sk is tbe volume of
the air under tbe piston when the latter is at an arbitrary
height k. Then the expression for the foree of presaure
exerted by t.he air under t.he piston has the form. F =
mRTl(Mk). Substituting the ex.pressions for Faim and F
into Eq. (1) and integrating, we obtain

A=h~ (F,,m-F)dh=I F"mdh-i'Fdh


A1 h1 111
A1 11.1 11.1 11 1

= i p,Sdh-1 -:: dh=p,S i dh- "!' i ~


A1 h1 11.1 111

=p,S(h,-h,)-"!' ln~,
A=[ 1.01X100X1.5X1()-•x (0.2-0.1)
21o-;::;~x 293 ln H J1 !::::: 3.5 J.
Sinee the piston moves slowly, the temperature T can be a.s-
aumed to be conetant and taken outaide the integral sigo.
296. An ideal heat engine received 3360 J of heat dur-
ing a cycle from a heater whose temperature is 500 K.
Determine the amount of heat given away during a cycle
to a cooler whose temperature is 400 K. Calculate the
work done by the engine dUJ"ing a cycle.
Giuen: T1 =500 K, Q,-3360 J, T,=400 K.
Q,-?A-1
Solutton. By de6.nition, the ef6.ciency of an ideal heat
engine is ~ = (Q 1 - Q,)IQ, = (T 1 - T ,)/T1, whence
Q,= Q;~·.
Q,= -~'°° J=2688 J.
The work done by the engine during a eycle is
A =Q,-Q,,
A = (3360-2688) J = 672 J.
Ch. 2. Molecular Phyaict and Thermodynamics 197

297. An ideal Carnot heat engine with a reverse cycle


(relrigerator) used water at O°C as a cooler and water at
100 ºC as a heater. What amount ol water should be
Crozen in the cooler to vaporize 500 g or the water in the
heater (boiler)?
Gtven: t1 = 100ºC, T 1 = 373 K, t2 = OºC, T2'""' 273K,
m, - 500 g - 0.5 kg.
m,-1
Solution. The amount of heat Iiberated during the
cooling or the water of mass m2 is
Q,-Am,, (1)
where A is the latent heat of fusion for ice.
The amount or heat required to vaporize the water of
mass m1 is
(2)
where r is the latent heat of vaporization for the water.
Using lhe expression ~- (Q, - Q2 )/Q 1 - (T 1 - To)/T,
for the ef6.ciency of the ideal Carnot heat engine, we ftnd
that
(3)
Substituting e:r.pressions (1) and (2) into (3), we obtain
>.m:i -= T2m1r/T i. whence
T.,n,r
ms=~·

"'s 27~0~5l3~~81~l0' kg~ 2.47 kg.


298. Determine the work done by a heat engine during
a cycle shown in Fig. 106.
Gtven: p., p2, V., V2•
A 1
Solutton. Tbe e:r.pansioo work of a gas is numerically
equal to the area of the 6.gure bounded by the graph p -
p (V), the abscissa axis, and the ordinates of the initial
and the final point of the graph. Tbe work done on seg-
menta 1-2 and 8-4 is zero since the gas volume does not
198 Seleeted Problema on Physies

Fig. 106 Fig. 107

change on these segment.s. Tbe work done by the engine


during a cycle is numerically equal to the difference in
the areas of rectangles V123V1 and V114V1 , i.e. to the
area of reetangle 1234. Using the figure, we can determine
this area:
A -p, (V, - V1)-p1 (V,- V1)-(p, - p 1) (V,- V1).

EXERCISES
299. Two hundred grams of nitrogen (N 1 ) are heated by
100 K first isobarically and then isochorically. What
amounts of heat are required in these cases?
300. A vessel contains 20 g of nitrogen (N1) and 32 g of
oxygen (0 1). Determine the change in the internai energy
of the gas mixture during ita cooling by 28 ºC.
301. Nitrogen (N 1 ) under an initial pressure of t.01 X
101 Pa having a volume of 10 l expanda isothermally so
that its volume is doubled. Determine the work done by
the gas.
302. The pressure of nitrogen in a vessel of volume 31
incresses by 2.2 MPa as a result of heating. Determine the
amount of heat supplied to the gas.
303. During the expansion of a monatomie gas from 0.2
to 0.5 m1 , its pressure inereases from 404 to 808 kPa.
Determine the work done by the gas, the amount of heat
supplied to it, and the ehange in its internai energy.
304. The amount of heat reeeived by a working sub-
stanee from the heater in an ideal heat engine is 6.3 kJ. De-
termine the effieieney of the engine and the work done by
Ch. 2. Molecular Physies and Tbermodynamics 199

it during a cycle if 80% of the heat are transfened to the


cooler.
305. Determine the work done by the heat engine shown
in Fig. 107 during a cycle.
306. Two kilograms of air are contained in a ve.ssel at
a temperature of 16 "C. What work will be done by the
gas during its isobaric heating to 100 ºC?
307. A cylinder contains nitrogen at a tempera tore of
20 "C. A light piston under a load of 100 kg i.s at a height
of 50 cm from the bottom of the cylinder. Determine the
work done by the gas during its isobaric heating to 60 ºC.
The area of the base of the cylinder is 100 cm 2 • The atmo-
spheric pressure is normal.

QUESTIONS FOR REVISION

2.4. Sllfurllfed and Unsllfurllled Vapour.


Humldlty
The surface of a liquid is always under its vapour due
to evaporation. A vapour can besaturatedorunsaturated.
Saturated vapour is in a dynamic equilibrium with its
liquid (the number of molecules escaping from the surface
of the liquid per second is equal to the number of mole-
cules returning to the liquid).
The density and pressure of unsaturated vapour are
lower than the corresponding values for saturated vapour
at the sarne temperature.
ln its properties, unsaturated vapour does not differ
.....
from an ordinary gas. Therefore, Clapeyron's equation of
pV = mRTIM
is valid for it.
200 Selected Problems on Phys1cs

Air containing water vapour is referred to as humld


(molst) alr, while air containing no water \'&pour is
caJled dry.
Humid air is characterized by absolute and relative
humidity.
The ab8olute humldlty p is defined as the mass m. of
water vapour contained in a cubic metre of air ata given
temperature. ln other words, it is equal to the density
of water vapour ata given temperature.
The relatlve humidity B is the ratio of the density p of
water vapour at a given temperature to the density p181
of saturated vapour at lhe same temperature:

....
B~__LICJ0% .

Relative humidity can be determined with lhe help of


lhe psychrometric table.
Unsaturated vapour can be converted into saturated
vapour as a result of isochoric cooling.
The temperature at which a vapour becomes saturated
as a result of isochoric cooling is known as the dew polot
T dew· Vapour cooled below the dew point is partially
condensed to liquid.

308. Oetennine the absolute humidity of air if the


partial pressure of vapour in it is 14 kPa and the tem-
perature is 60 "C.
Given: p=14kPa=l.~Ylil'Pa, t=60ºC. T=333K.
p l
Solutton. Using Clapeyron's equation of state pV =
mRTIM, we obtain
m/V = Mpl(RT).
Consequently, the absolute humidity of air is
Mp
P=""iif"•
P- tsx~~=~xtO' ~ ~ 9 .t x 10-•kg/m3.
Ch. 2. Molecular Physics and Thermodynamics 20t

309. Determine the absolute humidity of air if its tem-


perature is 15 "C and the relative humidity is 80%.
Given: t=15ºC, T=288K, 8=80%=0.8.
p ?
Solutlon. The relative humidity is defi.ned as
B = PIP1at•
whence p = BPut· The saturated vapour density at
288 K can be determined from Table 5: Put = 12.8 X
10-a kg/m 3 • Therefore,
p = 0.8 X 12.8 X 10-• kg/m 1 "" 1.02 X 10-• kg/m'.
310. A litre of moist air at 50 ºC has a mau of t.04 g
under normal atmospheric pressure. Determine the abso-
lute humidity of air.
Given: V=l l=to-• m', t=50•C. 7"=323 K,
m-l.04g=l.04x 10-'kg, p=l.01 x IO'Pa.
p 1
Solutlon. The absolute humidity is defined as
p = m.,IV, (!)
where mv is the maSl!I of water vapour in a given volume of
air. By hypothesis,
m =mair +mv. (2)
P = Pa1r + Pv• (3)
where m11 r is the masa of dry air in the given volume,
and Pair and p., are the partial pressures of dry air and
water vapour. The partial pressures p 11 r and p., can be
determined from Clapeyron 's equation of state, written
for dry air and water vapour
Pa1r = m11rRT/(M 11 rV), (4)
p, = m.,RTl(M,V), (5)
where M11r and M., are the molar masses of dry air and
water vapour respectively.
Substituting expressions (4) and (5) into (3), we obtain
p= ( : : +z::;) ~r • or, taking into account rela~
202 Seleeted Problema on Physies

tion (2), p = ( ; : + m;.:• ) ~7' , whence

mv= Ma1~:_M., ( pV;;ir -m). (6)

Substituting expression (6) into (t), we obtain


p= M.i::_M., ( p!;lr -T) 1

1sx1o-a
P = 2ext0-• 1sx1o-a
x ( 1.0tx8~~;::~xt0""'
<>< 0.082 kg/m•.
311. A vessel contains air whose relative humidity at
10 ºC is 60%. Determine the relative humidity of air
after the reduction of its volume to one-third of the initi-
al value and heating to too ºC.
Given: t 1 =10ºC, T1 =283K, 8 1 =60%=0.6, n=3,
•,= too·c. T,=373K.
B,-1
Solution. The absolute humidity of air before its
compression and heating is
P1 = B1P11t1•
where P.. u is the saturated vapour densily at T1 • After
the reduction of the air volume by a factor of n, its den-
sity also increases by a fact.or of n, i.e.
P1 = RP1 = nB1Paat1· (t)
It is well known that the saturated vapour pressure at
lhe boiling point of a liquid is equal to the atmospheric
pressure p 0 • Consequently, when the moist air is heated
to 373 K, the pressure of saturated vapour contained in
the air becomes equal to the normal atmospheric pressure
p 0 • The saturated vapour density Paats under these con-
ditions can be determined from Clapeyron's equation of
.state:
(2)
Ch. 2. Molecular Physics aud Thermodyaamics 203

Then the relative humidity of air after its comprusion


and heating is
B 2 =JL 100%. (3)
P1ats
Substituting expresaions (1) and (2) into (3), we 6.nd that

.8i=nB1ri:~:T• 100%,
Bs= 3X0f~~~;_~~~~o~!·~X373 t00%~ 2 . 9 %.
312. Determine the absolute and relative humidities
of moist air if its temperature is 18 ºC and the dew point
is 8 •e.
Given: t=IBºC, T=29!K, t,-=BºC, T,-=28!K.
p-1 B-1
Solutton. Knowing the dew point, we can determine, by
using Table 17, the mass of saturated vapour contained
in a cubic metre of air. This quantity will determine the
absolute humidity p of air at any temperature equal to or
higher than the dew point. Consequently, p = 8.3 X
10-• kg/m•. Let us determine the relative humidity of air:

B= .!'.. 100%,
where Put is the saturated vapour density ,at 291 K.
Therefore,
B= :s~4~11º;. 100%~54%.
313. The relative humidity of air in a room is 63% and
the temperature is 18 °C. What must be the temperature
drop outdoors for the window-pane in the room to be-
come misty?
Given: B=63%=0.63, t=IB •e, T=29! K.
IJ,T-1
Solution. The absolute humidity of air is de&ned as
P = BP1at1
204 Selected Problems (ID Phy1lc1

where P.. t is the saturated vapour density at 291 K.


Theo
p = 0.63 X 15.4 X 10-' kg/m' = 9.7 X 10-• kg/m' •
.A3 the air temperatUJ'8 outdoon drops to tbe dew poiot.
vapour near the window-pane becomes saturated and
starts to condense-theglass becomes misty. The mll88 of
vapour contained io a cubic metre does oot change up to
the dew point T dew = 283.5 K. Consequeotly, the win-
dow-pane becomes misty when the outdoor temperature
drops by
AT = (291 - 283.5) K = 7.5 K.
3t4.. Two vessels of volumes 5 and 3 m1 contain air at
temperatures of 15 and 28 ºC and relativa humidities of 22
and 46% respectively. Determine the relativa humidity of
air after the conoection of these vessels.
Given: V1 =5m3 , B1 =22%=0.22, t 1 =15"C,
T1 =288K. V,=3m1 , B,=46%=0.46,
t,=28 •e, T,=301 K.
B-1
Solu.tton. The m888 of water vapour io each vesael bafore
the coooection is
m, = P1V1, ma= P1V1, (1)
where p1 and p1 are the densities of water vapour in the
vessels. The relative humidity of air in eaeh vessel before
the eonnection is
B1 = P1IP11tJ.• B1 = Pr'P11t1•
whence p1 = B1p1au and p1 = B1Paats· Then (see Eqs. (1))
m1 = B1P11uY1, m1 = BsP1auY1. (2)
The absolute humidity of air after tbe connection of
the vessels becomes p = (m1 + m1 )/(V1 + V 1), or, taking
Eqs. (2) into aeeount,
B1P1atiY1+BtPaa11~·.
v1 +v.
Ch. 2. Molecular Phy1ics and Thermodynamics 005

Tbe relative humidity of air after the connection of the


vessels is
B=~ 100%= 1:1 1 ~::\v<V~!~j"v• 100%. (3)
Neglecting the heat capacity of the vessels and the mass of
water vapour in view of the smallness of the vessels, we
obtain from the heat-balance equatioos
m-,1c, (9 - Ti) - m-,,c, (T, - 9), (4)
where 9 is the temperature of air in the vessels after
their connection. Since mv 1 = p,.V1 and m"" = p,.V1 ,
Eq. (4) can be written in the form p,.V1c,. (9 - T1) =
p,.VtCv (T1 - 9), whence
9 _ Y1T1+ v,r,
-~·
9 - 5X2~~~x301 K-293K.

~2~8w~1o~•·k~i~:~;.~~ ~º21~2·~17J!a ;,::.~•!:IP::~ ~


17 .3 X 10-a kg/m1 • Substituting these values into
Eq. (3), we get
B o.22x12.sx1 ~4:a'xx1 ~-t'~:~:11.2xto-1x3 100%= 37%.

315. Explain why mist appears in low-lying areas at


night after a hot summer day.
Answer. At night, after a hot summer day the air tem-
perature drops by 10-15 ºC. AB a result, the water vapour
contained in air becomes saturated and is partially
condensed in the form of mist.
316. Why is mist seen in the exhaled air in winter and
is not seen in summer?
Answer. The temperature of the exhaled air is about
36 ºC. The water vapour contained in the air is cooled in
winter below the dew point, becomes saturated, and
partially condenses in the form of mist. On the otber hand,
lhe temperature of the ambient air in summer is above
the dew point, and hence no mist is formed.
206 SeJected Prohlcms oo Pbysics

EXEllCJSES
317. Tbe absolute humidity of air at 60 ºC is 5 x
to-s kg/m1 . Determine the absolute humidity of the air
after its temperature drops to 20 ºC.
318. Determine the absoluta humidity of air in a room
if its relative humidity is 80% aod the temperatura
is 15 "C.
319. Determine the absoluta humidity of air if the par-
tia! pressure of vapour eontained in it is t.4 x 10' Pa
and the temperature is 60 "C.
320. Tbe pressure of air at a temperature of 26 ºC and
a relative humidity oi 70% is 1.017 x 101 Pa. Determine
the air pressure alter the temperature drops to -5 ºC
and the relative humidity becomea 80%, other conditions
heiog the sarne.
321. The relative humidity of air in the evening is 60%
at 16 ºC. At night, tbe air temperatura drops to 4 "C and
dew is precipitated. What amount of water vapour is
condensed from a cubie metre of the air?
322. What was the relative humidity of air at 20 ºC if
under a pressure of 6 MPa the dew point is 100 ºC?
323. How many water molecules are contained in a
room of volume 100 m3 under normal conditions and at
a relative humidity of 20%?
324. 3.5 g of water and 2.9 g of water vapour are con-
tained in a eylinder under a piston at 40 ºC. The gas in the
cylinder expands isothermally. At what volume will the
water in the cylinder be evaporated completely?
325. 20 000 m3 of air at 18 ºC and a relative humidity
of 50% are supplied to a room. The air is pumped from
outdoors where tbe temperature is 10 ºC and the relative
humidity is 60%. What amount of water must be evap-
orated for the relative humidity oi the air in the room
to remain unchanged?

QUESTIONS FOR REVISION


1. What is evaporation? 2. What is the dynamic equilihrium he-
~un--~~i;::d8'!:i!,~/i\Vi::t 3· ~=:~ ~~r;i:1s vJ:~Ji!~~:;
of such a vapour? 5. What is the 'JiBerence hetween moist and dry
Cb. 2. Molecular Phyeica and Thennodynamica '111

air? 6. What ia meant by abaolute humidity? 7. What ia meant by


relatlve humidity? 8. Dellne the dew point. 9. How can unaatu-
rated vapour be converted into saturated vapour? tO. Wbat inatru.-
mentll are uaed to meaaure air humidity?

2.5. Propertles oi Sollds


and Llqulds
THERMA.L EXPANSION OF SOUDS AND LIQUIDS

When a solid is heated, its linear dimensione l vary


according to the law
l= 10 (1 +aâr). (1)
where 10 is the linear dimensions of the body at 273 K,
AT = T - T 0 the ehange in the temperature of the body
during its heating from T 0 = 273 K to T, and a the co-
ef&cient of linear expansion.
The volume of solide and liquids changes as a result
of heating according to the law
V= V, (1 + p âT). (2)
where V0 is the volume of a body at 273 K and p the-
coef&cient of volume expansion, connected with the coef&-
cient a of linear expansion through the approximate rela-
tion p ~ 3a. The coef&cients ~ of volume expansion
for liquida are given in Table 16.
A change in the volume of solide and liquids as a result
of heating leads to a change in their density according to-
the law
p = p,1(1 + p âT). (3)
where p0 is the density of a body at 273 K.
Since a6.T< 1 and ~AT< t, the followingappr0%imate fo11-
mulaa ehould be ueed in calculationa:
1/(1 + z) i:t1t - z, (1 + z) (t + r) • 1 + z + r.
where z < 1 and r < t.

326. An iron ruler bas a length of t m at t5 ºC. What


will be the change in its length as a result of its cooling
to -3ó"C?
208 Selected Problema on Physic1

Given: t 1 =f5°C, T1 =288K, 11 =1 m, t 1 = -35ºC,


T2 =238K.
61-1
Solution. According to the law of linear expansion,
1, = 10 (1 + "(T, - T0)], (1)
12 =1,(!+ci(T,-T,)l, (2)
where 11 is the length of the ruler after its cooling to T 1
and a. the coeffi.cient of linear expansion for iron. Then
the change in the ruler length ia
61 = 1, - 12 = !, li + "(T, - T0)1
- 10 (1 + "(T, - T0 )1 = l 0a (T, - T,). (3)
Expressing 10 from Eq. (1) and substituting it into (3),
we gel
cz(T1 -fs)l1
61 t+cz(T T.J. (4)
1

Considering that a. (T1 - T0)< 1, expression (4) can


be approximately written as
61 ""ai, (T 1 - T,) 11 - "(T1 - T,)l.., l,a (T1 - T,),
61 "" 1.2 X 10 .. X 1 X (288 - 238) m "" 6 X 10-• m.
327. At any temperature, the difference in the lengths
of aluminium and copper rods is 15 cm. What are their
lengths at O "C?
Given: âl= 15 cm=0.15 m, t0 =0ºC, T0 =273 K.
lai-?lc-?
Solution. According to the law of linear expansion, we
have
la1 =loa1U +a.a1AT), (1)
l, = 100 (1 + "o 6T), (2)
where a.11 and cr.ç are the coeffi.cients of linear upansion
for aluminium and copper. By hypothesis,
lc - l 11 = Al, loc - l 0 •I = âl. (3)
Ch. 2. Molecular Physica aPd Thermodynamics 209

Subtraet.ing Eq. (2) from (t) termwise, we obtain


Z.1-lc= 10 ai (t +a.1AT)-l011 (t+«aAT)
=l0 111-l011 +loa1Ua1AT-Zo 11a 11 AT. (4)
We transform. expression (4), taking into aeeount formu-
las (3): -Al = -Al+lo alaa 1 AT-l011 ac AT, wheoce
lo a1aa1 - lo cªc = O. (5)
Haviog expresaed 10 ai from the seeond of Eqs. (3) and
substituting it ioto (5), we obtaio lo 11a.1 - AI - ªª'
'ºcaa = º·whenee
10 0 = Alaa1f(aa1 - ao).
Similarly, we 6.nd that
l 0 ai = Alaa/(a 8 1 - a 11).
Then
loc 2.40~t::.,2.4 t~7t~-;o-• m ~ 0.51 m,
10 ai= 2.11.º~1 :~1}._ ~71 ~:0-• m ~ 0.36 m. l

328. The amount of heat. spent. in heat.ing ao iron bar is


1.68 MJ. What is the ebange in the volume of the bar?
Gi.,.n: Q=l.68M1=1.68x 1001.
li.V--?
Solution. Aeeording to the law of volume expansion,
V= V, (1 + p ll.T), whence
li.V= V - V 0 = V 0 p 11.T "'3aV0 11.T.
Here p = 3a is the eoeffi.eient. of volume expansion for
iron and a the eoeffi.eieot of linear expansioo for this
substaoee.
The amount. of heat. required to heat the bar by AT is
Q =cmAT,
whenee AT = Q/(cm). Sinee the mass of the bar is m =
pV0 , we ean write
11.T = Q/(cpV0 ).
U-09711
210 Selected Problema on Physic1

The ehange in the volume of the bar is


âV- V,lk<Q
cpV0
-""Q
ep '
3 x ~:~;:~~~~;;;oe m'~ 1.69x 10-1 m•.
!:t.V 0
329. Water and kerosene have the same volume of 4 1
at O ºC. Determinethe differencein their volumes at SOºC.
Given: T0 =213K, V0 .. =Vok"""4 l,,,..4xt0- 1 m3 ,
T-323K.
âV-1
Solutton. According to the law of volume expansion, we
have
Vw - V 0 w(1 + Pw âT), V• - V,, (1 + P• âT),
where !:t.T = T - T 0 , and Pw and Pk are the coefficients
of volume expansion for water and kerosene. Then the
differenee in the volumes of water and kero!ene is
âV - V, - v. - V,, (1 + P• âT)
- V,. (1 + Pw âT) - V, (T- To) (Pt - Pw),
where V0 = V0 11: = V, w· Consequently,
âV - 4 X 10-• X (323 -273)
X (10 X 10-• - 1.8 X 10-') m 1
~ 1.64 X 10-" m1 •
330. An iron tank has a eapaeity of 50 1 of kerosene
at O °C. What amount of kerosene will Dow out of the full
tank if it is brought in a room where the temperature is
20 ºC?
Given: V0 =501=5x10-Zm3 , ta=OºC, T0 =273K,
t-20 ºC, T-293 K.
âm-1
Solution. The increHe in the volume of kerosene as a re-
~h~t ~!1~8.!!1 :f ~6e~a:~:v!~d~ 1:~rºir::eit~;:,r:!ª!fü
Oow out. We denote by m0 the mau of keroiene in the
Ch. 2. Molecular Phy11ic11 and Thermodynamies 2t.t.

tank at 273 K and by m lhe mass of kerosene at room


temperature. Then lhe mass of kerosene Oowing out of
the tank is y
ó.m=m.0 -m, or
(1)
ó.m = PoVo - pV,
where p0 is tbe density of kerosene
at 273 K, and p and V are the den-
sity of kerosene and the volume of
the tank at room temperatura.
The volume of the iron tank af-
ter its heating is
V= V, (1 + ~ •.!.T), (2)
where p1 ~ 3ci 1• The density of
kerosene after tbe heating of the nrg
tank is Fig. 108
p = p0/(1 + ~• .!.T). (3)
Substituting expressions (2) and (3) into (t), we obtain

Ó.m=p0 V0 - t+~:Ar Y0 (t+3a 1 ó.T)


p.V,:~jl.:;.i (~Jr~ "'p0 V 0 (jl,-3a~ (T-T,),
.!.m=0.8x10'x5x10""'x (10x w-•-3x 1.2x 1()-')
X (293-273) kg°" 0.8 kg.
331. A steel bali of mass 100 g is tied to a string and
immeraed in kerosene. Wbat will be the change in the
tension of the string if the entire system is heated from
20 to 50 "C (Fig. 108)?
Given: m=t00g=O.tkg, t 1 =20°C, T1 =293K,
t2 =50 ºC 1 T2 ""'323 K •
.!.F,-1
Solution. At temperatura T 1 , tbe ball immeraed in
kerosene experiences the action of tbe force of gravity rng,
lhe tension Fu of lhe string, and lhe buoyant force F bt·

...
We write the equilibrium condition for the ball in pro-
212 Selected Problema on Phyaic1

jections on the Y ~axis:


Ft1 + Fb1 - m.f =O,
whence
Fu = mg - Fba· (1)
Here Fb1 = PtVaf, wbere Pt is the densit.y of keroaene.
Since p, = p,.1(1 + p, 6Ti), V, = V,. (1 + p, 6T1) =
V0 1 (1 + 3a1 âT1 ), V0 1 = mfp 0 ,, and âT1 = T1 - T 0 ,
the buoyant force is
P1tm(t+3r.r.1(T1-T1lll (2)
ft+jSíl (T1 fo)I POI '
wbere a, is the coefficient of linear expaDBion for steel.
Taking Eq. (2) into auount, we can write Eq. (t) as

Fu =mg P• 1~~:~::c:~~:iJ!º:''
'"'"1-'":,',' 11-(p,-Sa,)(T,-T.JIK·
Similarly, the tension of the string at temperature T 1 is

Fts=mg P•j~iP~c~'~.)1~! 1 '


'"mg-~11-(ll,-3a,)(T1 -To)IK·
Therefore
âFt=Ft 1 -Ft 1 ~ m::: 11 (l'it-3cJ.)(T3 -T 1),
<l.F,-0.1X9.8 x0.8 X ICPx (10X10""'-3X1.2 X 10-')

x( 7~;1~) N~3mN.
332. The mass of aleohol taken at O ºC in a volume of
500 cm• is 400 g. Determine the density of alcohol
at 15 'C.
Glven: t0 =0ºC, T0 =273K, V0 =500cm1 =5xtO-•m1 ,
t-15ºC, T=288K, m=400g=0.4kg.
p-1
Cb. 2. Molecular Physica aad Thermodynamica 213

Solutlon. The density of alcohol is

P- 1+~AT • (1)

.
Since p0 =m/V0 , we have (see Eq. (1))

P- V 0 (1+~AT) º
Using the formula for appro:ximate calculation, we &nd
that
p ... -;,-c1-p .1.n--;,-11-p(T-T,)],

P"" .:i"rr. x(1-11xt<r•x(288-273)J~


""0. 79 X 10' kg/m1 •
333, Heating changes not only the volume of a cube cut
out of a single crystal but also its shape. Why?
Answer. Due to ani90tropy 1 tbe coef&cient of linear
expansion in a single crystal is ditlerent in different direc-
tions, and for this reason the shape of the cube changes.
334. Why are the most sensitive measuring instrumenta
made of a special alloy like invar?
Answer. lnvar has a very small coef&cient of linear ex-
pansion. For this reason, random temperature nuctua-
tions do not affect the accuracy of a measuring instrument
made of invar.
l!XERCISES

335. The length of an aluminium ruler is 79.5 cm and


that of an iron ruler is 80 cm at O ºC. At what temperature
will the lengths of the rulers beco me equal?
336. The length of a copper and ao iron wire is 500 m
a\ O ºC. Determine the difference in their lengths at 30 ºC.
337. A lead bali has a volume of 1.8 dm8 at 20 ºC.
Determine the increase in its volume as a result of heating
to 100ºC.
338. At 20 ºC, kerosene and sulphuric acid have the
same volume of 500 cm8 • What will be the difference ln
tbe volumes of the liquids at O"C?
214 Selecf.ed Probltlms on Physics

339. The temperature of one arm of a U-tube is tO ºC


and that of the other arm is 80 ºC. The levei of kerosene
in one arm is 280 mm and that in the other arm is 300 mm.
Determine the coefficient of volume expansion for kero-
sene.
340. The volume of a brass vessel increases as a result
of heating by 0.6%. By how many degrees was the vessel
heat.ed?

m::!·in6?c= :~ :~!:;:,!~u0; :n,rc~W~!~ ~~1~b~et~~·~:~


ume of the indentation after the cooling of water?
342. Petroleum is contained in an iron cylindrical tank
wbose beight is 6 m and the diameter of the base is 5 m.
AtO ºC, the levei of petroleum is 20 cm below the brim. At
what temperature will petroleum start to overflow from
the tank? The thermal expansion of the tank should be
also taken into account.

SURFACE TENSION IN IJQUIDS.


CAPILLARY PHENOMENA

The surface layer of a liquid is in a stressed state and


possesses a potential energy. Surfaee tenslon is defined as
a = F 1 .tll or a = W/S,
where F, t is the force of surface tension, l the length
of the contour bounding the surface of the liquid, W the
potential energy of the surface layer, and S the area of
the surface layer. ·
A curved surface layer exerts an excess pressure on
: hislu~:,;:c~l~:e~~i~~o~~~x t::rf:ce:1:~~,º~o8~~~~!!:~~
lower layers of a liquid, while a concave layer stretches
them.
The u:eess pressure Pes. exerted on a liquid by a spher-
ical surface of radius R is
Pu. =±2a/R,
where the plus sign corresponds to the convex meniscua
and the minus sign to the concave meniscus.
Ch. 2. Molecular Physica and Therm.odynamica 2t5

ln narrow tubes (capillaries), the curvature of the


surface of a liquid (meniscos) becomes considerable due to
wetting or nonwetting of the tube walls for the liquid.
~e 1~~~~ssof1::!i~q~~r° a noticeable rise or fall in

ca~~ar~ei~~li!ºri=ici~ ~h~q~!:nr:::eci~ly wetting the


h = 2alpgR,
where p is the density of the liquid, g the free-fall accel-
eration, and R the radius of the ca.pillary.

343. A thin aluminium ring of radius 7.8 cm is in con-


tact with a soap solution. What force must be applied to
separate the ring from the surface of the solution? Assume
that the solution is at room temperature and that the mass
of the ring is 7 g.
Given: R=7.8cm=7.8x 10-1 m, m=7g=7x1.0-3 kg.
F-1
Solution. The ring is acted upon by the force of gravity
mg, the force of surface tension F, t• and ao externai force F
(Fig. 109). Since both the externai and intemal surfacesof
the ring touch the solution, tbe force of surface tension

nrg
Fig. 109 Fig. HO
216 Selected Problema ou Physics

ia
F 1 ., = 2al,
where l = 2nR. The condition of separation of the ring
from tbe solution in projections on the Y-axis has the
form F = mg + F,.,, or
F = mg + 2al = mg + 4"aR.
Then
F = 7 X 10-a X 9.8 + 4 X 3.14 X 4 X !O-•
X 7.8 X 10-• N ""O.li N.
344. A wooden stick of length 4 cm Doats on the surfaee
of water. A soap solution is carefully poured atone side of
the st.ick. With what acceleration will the stick st.art to
move if its masa is 1 g? Tbe water resistance sbould be
disregarded.
Given: l=4cm=4xto-am, m=1g=10-ªkg.
•-1
Solution. The stick in the horizontal plane is acted
upon by the forcu of surface tension exerted by water
(F1 u) and by the soap solution (F1 .t1) (Fig. 110). Writing
Newton's second law for the stick in projections on the
Y -axis, we obtain
F,u-F1.u=ma,
whence
a= (F1 u - F 1•12)/m.
Since F 1 u = a1 l and F, tt = o1 l, wbere a 1 and a 1 are the
surface tensiona for water and the soap solution, we have
a= l(a~-a.>,

a 4.xiO-•x(7:xto-•-4xtO-I) = 1.aSm/sl.
345. What energy ia liberated as a reault of merging of
amall water drops of radiua 2 X 10-1 mm into a aingle
drop of radius 2 mm?
Given: r=2x 10-1mm=2x101m, R=2mm=
2x 10-•m.
W-1
Ch, 2. Molecular Ph11ice and Thermodynamica 2t7

Solutlon. The ehange in the potential energy of the sur-


faee layer of the drops as a result of a deerease in the
surfaee area by /J.S during their mergenee ia
.i.w = a .i.s = a (S, - S,), (1)
where 8 1 is the surfaee area of ali the small drops, 8 1
the surface area of the large drop, and a the surfaee ten-
sion for water.
Obviously, 8 1 :::s. 4nr'n and 8 1 = 4nR1 • The mau of
water remains conslant, and hence
nm=M, (2)
where n is the number of small drops, m the mus of
a small drop, and M the mass of the large drop. Sinee m =
pV, = (4/3) pn,. and M = pV1 = (4/3) pnR1 , we obta!n
lrom Eq. (2)
n - 11'1,..

~~:i:~~"!: 4':Jrk. S:ba~t:~:n~8tb:°~!p:.:


for 8 1 and 8 1 int.o Eq. (t), we find that.
âW=a ( ~· -4"11')=4nR•a(~-i),
âW=4x 3.14x (2x 1<r')•x 7.4x1()-S
x[(~~!~ )-1]J-3.7m1.
3'6. What. is the air pressure in a bubble of radius
5 X to-1 mm under the water surfaee?
Given: R=5 X t0-1 mm=5 X tO--m.
p-1
Solution. The air pressure in the bubble is
P =Po +Pn•
where p 0 is the atmoapherie pressure and Pn. the exeeu
preSBure. Sinee Pex = 2a/R, we have p = p 0 + 2a!R,
where a is the surfaee tension for wat.er. Then
p-1.01x1()1 Pa+·•x;;,~o-• Pa= 130kPa.
218 Selected Problema on Pbysics

347. Two soap hubbles of radius 1.0 aod 5 cm are hlowo


at differeot ends of the same tube. Determine the pressure
difference io the bubbles. Will their size change if they
are left to themselves?
Given: R1 =10cm=0.1m, R1 =5cm=0.05m.
lip-1
Solution. The pressure p in a soap bubble is
P = Po + Pe::s.• (1)
where p 0 is the atmospheric pressure and Pex the excess
pressure produced hy the curved surface layer of the
soap solutioo. Obviously, Pex = 2·2a/R, where a is the
surface teosion for the soap solut.ion. The factor 2 is due
to the fact that the soap film has two surfaces: outer and
inoer. Then Eq. (1) assumes the form
P = Po + 4a/R. (2)
Equation (2) can be writteo for the first and the secood
bubhle io the form p 1 = p 0 + 4a/R1 and p 1 = Po +
4a/R 1 • Then the pressure diflerence in the bubbles is
Ap=p1-P1= 4.a(~:;.R,),

Ap 4x4xttt':Ji:-0.05) Pa=1.6 Pa.


Calculations show that the pressure in the smaller huhhle
is higher tbao that in the larger hubhle. Therefore, air
ftows from the smaller hubhle to the larger bubhle. The
volume of the smaller huhhle will decrease and that of
the larger bubhle will iocrease until the pressures become
equal.
3'8. A long capillary of radius 1 mm open at hoth ends
is filled with water aod turned to the vertical position.
Determine the height of the water column remah;iing io
tbe capillary, neglecting the tbickoess·of its wall.
Given: R = 1mm=10-' m.
h-1
Solution. The water column in the capillary is acted
upon by tbe force of gravity mg and the forces of surface
Ch. 2. Molecular Physics and Thermodynamics 2t9

tension Ff t in the upper and lower meniscuses (Fig. 111).


Writing the equilibrium condition for the water column
in projections of forces on the Y-axis, we obtain
2F8 t - mg =O.
Considering that F 8 t = u2nR and mg = pgV = pgnR 2h,
where u is the surface tension and p the density of water,
we obtain 2u·2nR - pgnR 2h =O, whence

h= p~aR'
h t~~~.~~~i;: m~3x 1&-2m.
349. The difference in the leveis of a wetting liquid
in the arms of a U-tube is 23 mm (Fig. 112). The diameters
of the channels in the arms of the tube are 2 and 0.4 mm.
The density of the liquid is 0.8 g/cm9 • Determine the sur-
face tension for the liquid.
Given: h=23mm=2.3x1(t-2m, D 1 =2mm=2x 1ü- 3 m,
D2 =0.4. mm=0.4 x 1ü- 3 m, p=•Ü.8g/cm 3 =
0.8X1Q3 kg/m3.
u-?
Solution. The equilibrium condition for the liquid in
the U-tube is PA = p 8 , where PA and p 8 are the pressures

Fig. ttt Fig. 112


220 Selected Problema on Pbyeics

in the Ieft and right arms on the levei AB. Considering


tbat PA = Po - Pen and PB = Po - Pu1 + PA• where
p 0 is tbe atmospheric preSBU1'9, Pen = 2a/R1 =
4.a/D1• Pu• = 2a/R 1 = 4a/D 1 , and PA = pgh, the equi-
librium condition for the liquid assumes tbe form p 0 -
4.a/D 1 = p 0 - 4.a/D 1 + pgk, whence

a=,r~~~l~'
o.sx 1Cl'x9.8x2.3x 10-•x2x 10-1 x0.4x 1lr3 N
0 = 4x(2xió'"1 óA.xt0-1)
<>< 2.25 X rn-• N/m.
350. Wby do the hair of a brush stiek together when the
brush is taken out of water?
Answer. If tbe brush is taken out of water, its hair is
covered by a water lilm and sticks together under the
aetion of surfaee tension. -
351. Wbat liquids ean be poured above the brim in
a glass?
Answer. Nonwett.ing liquids, since tbe forces of inter-
action between the liquid molecules are slronger than the
forces of interaction between the liquid and glass mole-
eules. The resultant of ali these forces is directed into the
liquid and keeps the liquid molecules which are above the
brim.
EXEB.CISES
352. Determine the surface tension for oil whose densi-
ty is 0.91 g/em8 if 304 drops are formed by passing 4 cm•
of the oil through a pipette. The diameter of the pipette
nozzle is 1.2 mm.
353. What is the mass of a drop of water Oowing out of
a glass tube of dia meter 1 mm? Assume that the diameter
of a drop is equal to the diameter of tbe tube.
354. An air bubble of diameter 0.002 mm is in water
near tbe surface. Determine the density of air in tbe
bubble.
355. What is the excess of pressure in a soa p bubble
over the atmospheric pressure if the diameter of tbe
bubble is 5 mm?
Ch. 2. Molecular Phyaiea and ThermodJD8.miea 22t

356. What is the ratio of the height.s of water and kero-


sene columns in capillaries, other conditions being equal?
357. The difference in the mercury leveis in communi-
cating capillaries of radius 0.5 and 2 mm is 10.5 mm.
Determine the surface tension for mercury.
358. A liquid rises in a capillary to 80 cm. Determine
the height of the liquid column which will be in the capil-
lary if it is lilled with the liquid in the horizontal poaition
and then turned to the vertical position.
359. A water drop of mau 0.2 g is between two glus
plates separated by a distance of 0.1 mm. Determine the
force of attraction between lhe plates.

DEFORMATION OF SOIJDS. ROOKE'S LAW


DeformaUon is a change in the shape and size of bodies
under the action of applied forces. Deformations can be
elutic and inelutic.
A deformation is elastle if it disappears aftertheforces
ceue to act on a body. An inelasUc deformation is parti-
ally retained after tbe forces are removed.
Elutic deformations obey Hoolr.e's law:
li.!= (1/E)a!,,
wbere âl = l - 10 is the absolute eloogatlon (deforma-
tion) of a body, l the length of the deformed body, 10 its
ioitial length, E the modulus oi eloogatioo (Young's mo-
dulus), a = FI S the stress, F the force acting on the body,
aod S the cross-sectional are a of the body.
The stress 0 0 at which a body begins to be ruptured is
called the ultimate strength.

360. Determine the elongation of a brass rod of length


4 m and cross-sectional area 0.4 cm1 under the action of
a force of 1 kN.
Gil:en: l0 =4m, S=0.4cm2=4x10-5 mª, F=1kN=
IO'N.
111-1
222 Selected Problems on Physics

Solution. According to Hooke's law, the absolute


elongation is
61 = (!/E) ai, = Fl 0 /ES,
where E and a are Young's modulus and the stress for
brass. This gives

Al O.Dxi1g;1 ~~xto-•m~1.1xto-am.
361. At what limiting load will a steel rope of diame-
ter 1 cm rupture if the ultimate strength for steel is 1 GPa?
Given: D=icm=tO-sm, ou.=1GPa=109 Pa.
Fum-?
Solution. The ultimate strength is
O'u = F11mlS,
where S = nD 9/4 is the cross-sectional area of the rope.
Consequently, the limiting load, i.e. the force acting on
the rope, is
Fum=O'u.8:::2 au.~D•'
F11.1D = 10' X S.t: X (10-1)' N ~ 78.5 kN.

362. A steel bar tightly fits the gap between two sta-
tionary walls at O ºC. Determine the stress in the bar
material at 20 ºC.
Given: •,=OºC, T,=273K, t=20•c, T=293K.
a-1
SoluUon. lf the free bar were heated by t;.T, its length
would be l = l0 (1 + ex AT), whence
61=1-10 = cd, {l.T, (1)
where ex is the coefficient of linear expansion for steel
and l 0 the length of the bar at T 0 •
The distance between the walls does not change, and
hence Al is the absolute compression of the bar emerging
during its heating. It follows from Hooke's law that the
stress of the bar material at 293 K is
a= Elll/l0 ,
Cb.. 2. Molecular Physics •nd ThermoclyD•mics 223

where E is Young's modulus for steel, or, taking into


account expression (t),
a= E~l 0 llT/l 0 =E~ (T - T0 ),
o = 2.2 X 10" X t.t X rn-• X (293 - 273) Pa
= 48.4 MPa.
363. A steel wire of length t m is fi.xed at one end so
that it can oscillate in the vertical plane. A load of mass
50 kg is tied to the free end of the wire, which is deOected
to the horizontal position and then released. Determine
the absolute elongation of the wire at the lowest point of
the trajectory of the load. The cross-sectional area of the
wire is 0.8 mm', and its mass can be neglected.
Given: 10 =tm, m=50kg, S=0.8mmZ=8x10-7m:t.
ll.!-1
Solution. According to Hooke's Iaw, tbe absolut~elon­
gation of the wire is
ll.! = Fl,l(ES), (1)
where F ia the tensile force acting on the wire at point .A
and E Young's modulua for steel. Let us consider the

p
forces acting on the load at the momeot it passes through
point .A: the foroe of gravity mg,
and the tension T of the wire Y
(Fig. tt3). WritingNewton's aec-
ond law for the load suspended
on the wire io projections on the I
I
Y-axis, we obtain
T-mg =ma,.
Here a, = ac - v21l, where l is
the length of the stretched wire
at poiot .A. Hence mg
T-mg + mv'll = m (g + v'/!).
Fig. tt3
(2)
ln order to fi.nd the velocity v oi the load at point .A, we
apply the energy conservation law for two positions of the
load at points .A and B: W A - W.8 , or mv1 /2 = mgl,
Selecl.ed Problema ou Phyaics

whence v2 = 2gl. Suhstituting this ei:pression into Eq. (2),


we ohtain
T=m(g+2glll)=3mg.
According to Newton's third law, the force F st.retching
tbe wire is equal in magnitude to the tensile force T:
F = T = 3mg. Using this expression, we write Eq. (1)
in t.he form
lll=3i's'°.

lll=' 2 .:: 1~~:::1~1 m~8.4x ~0-3m.


364. Why are cutting tools not made of glass whose
hardness is tbe same as that of instrument steel?
Answer. Glass has a low tensile (and Dexural) strength
at room temperature in comparison with steel.

EXERCISES

365. What force must be applied to a steel wire of


length 2 m to stretch i t hy 1 mm? The cross-sectional
area of the wire is 0.5 mm2 •
366. A copper wire of diameter 1 mm hreaks under
a load of 188.4 N. Determine the ultima te tensile strength
for copper.
367. What is the minimum length of a lead wire at
which it is ruptured under the force of gravity if the
rupture takes place near the point of suspension?
368. An iron wire is stretched in the horizontal posi-
tion between stationary supports at 30ºC. What force will
he exerted hy the wire at the points where it is fixed
upon a temperature drop to -10 '"C? The cross-sectional
area of the wire is 2 mm2 •
369. A copper rod of length 1 m uniformly rotates ahout
a vertical axis passing through its end. At what angu-
lar velocity will the rod hreak? The ultimate strength
for copper is O"u = 235 MPa.
370. Determine the work done in stretching a steel wire
of length 1 m and radius 1 mm, from which a load of
mass 100 kg is suspended.
CJa. 2. Molecular Ph}'llica and Thermodynamica 22,;

QUESTIONS roa REVISION


l. What ia the coefftcieot of linear e:m:pllD!lioD? 2. What ia the coef-
of volume expaUion? 3. What ia the D betweeo the
· J.!~:!f.:~li':foerx~~J? ~. ia ~Ji':.{!ii:!
the exceu preaaure under tlie cu f allquid.
oolumn in.
a eolid 8. Name the
d? ti. FormuJ!~mu::~e: ~:;
;o':l:::!1::~~::.oung'1

u-0110
Chapter 3
ELECTRICITY

3. t. Electrostatlcs
COULOMB'S LAW. ELECTROSTATIC nELD STRENGTB

Aecording to Coulomb'a law, the force of interaetion


between two stationary point electrie charges is
F = q1q1/(4:n:e 0 er1).

where q1 and q1 are the magnitudes of the charges, r is the


distance between them, e the relaUvo permiWvlty of lhe
:::::i: i~o:!~e:t, th~ºc:_·~~n9rex fQ:)ie;in!'~ 8~~5t~
10-11 Fim.
1

The following concepts are introduced for calculating


the interaction between nonpoint charges.
t. Linear charge denslty for a uniformly charged ôla-
ment
dq
't'=ar·
where dq and dl are lhe charge of the filament and its
element of length.
2. Surface ebarge demity for a uniformly charged
surface
dq
a=dS·
where dq and dS are the charge of tbe element and its
surface area.
Below we give the basic formulas for ealculating the
strength of an electrostatie fi.eld.
The 6.eld atrength is defi.ned as
E-Flq.
Ch. 8. Electrlcity 227

Hence the force aeting on a charge q in a magnetic .6.eld


Eis
F= qE.
The strength of the .6.eld produced by a point charge or a
charged sphere is
E= Q/(4n•0 u'),
where Q is the charge producing the field and r the dis-
tance from the charge (or the centre of the charged sphere)
to a given point of the field.
The field strength produced by an infinite uniformly
charged plane (unlform field) is
E= a/(2• 0 •),
where a is the surface charge density and 8 the permit-
tivity of the medium. The vector E is normal to the
plane.
The field strength produced by two unlikely charged
parallel planes (tbe fi.eld oi • parallel-plate capaeltor)
is given by
E= a/(• 0 •).
lf an electric field is produced by severa! point charges
q,, the electric field strength vector E at a given poiot is

Fig. 114

equal to the vector sum of the field strengths E 1 produced


at this point by each charge separately (the prlnelple of
superposltlon of electrle &elU):

E=i E,.
·-·
Two unlike point charges of thesame magnitude, sepa-
rated by a certain distance l, form an electrle dtpole

...
(Fig. 114). Tbe points at which the charges are located
Selected Problema oo Physics

are known as the poles of the dlpole. Tbe arm 1 of the


dipole is a vector directed from the negative to the posi-
tive pole, whose length is equal to the separation between
the poles. The quantity Pe = ql is known as the electrlc
moment of the dlpole (or electrlc dipole momeat).

371. Two identical small balls of mass 0.1 g each are


suspended on strings of length 25 cm. Alter identical
charges have been imparted to tbe balls, they diverge to
5 cm. Determine tbe charges of tbe balls.
Given: m1 = m2 = m""" 0.1 g ...... 10-4 kg, l1 = l 2 = l =
25cm.=25x10-Sm, r=5cm.=5xt0-2 m.
q-1
Solution. Each ball experiences the action of the force
of gravity mg, the tension T of tbe string, and the electric
force F of interaction between
the balis (Fig. 115). Writing
the equilibrium condition for
a ball under the action of the
applied forces, we obtain
"'ti + T + F = O. (1)
ln projections on the X- and
Y-ues,Eq. (1)canbewrittenas
-Tsina +F =O,
T coo" - mg = O. (2)
X ConsiderinJ that F~
qt/(4ne 0 er1 ), we write Eqs. (2)
"'9 in the form
Fig. 115 T sin a = q2 /(4a:e 0 u 2 ),
Tcos11 = mg. <3>
Dividing the ftnt equation from (3) by the second term-
wise, we ftnd that tan a = q1/(4a:e 0 u 2mg). Since the an-
gle a is small, we can write tan a ~ sina= r/(2l). Then
Ch. a Electrlclty 229

r/(21) = q'/(4ns 0 u'rng), wbsnce


q=ryr~rmg'
.• e
- 5 10-a ... /2~x~.~.•~.~x~8~8S~x~1~(f~b~x'"'1"'x~5~x~l~O-~•~xT.11fi=x~.~
q- X J' 25xt0-•
~5.2nC.
372. Two positively charged bodies bearing charges of
1.67 and 3.33 nC are sit.uated ata diatance of 20 cm from
each other. At what point on the line connectingt.he bodies
should t.he third body bearing a charge of -0.67 nC be
placed for it \o be in equilibrium? The masses of the
bodies should be neglect.ed.
Given: q,=l.61 nC= t.67xto-"C, q1 =3.33nC=
3.33xt0-'C, r=20 cm=0.2m, !q1 !=0.67nC=
0.67 X 10-' C.
•-?
Solutton. The body C wit.h t.he charge q1 is acted upon
by the forces F1 and F 1 , viz. the forcea of interact.ion wit.h

Fig. 118

the charges q1 and q1 reapect.ively (Fig. 116). Writing the


equilibrium condit.ion for the body C in project.ions on
the X-axia, we obtain
F 1 - F 9 =0,
whence F 1 = F,. Considering that

F1= tJ:s'c!.1 :r)•, F1 =?~~,


where z is the diatance between the charges q1 and q1 ,
we get
IJ'1l1J1I -.!!..!..!1..l
~-.fine.a''
230 Seleçt.ed Problema ou Pby1ic1

whence
z- ____t:ii__ r
- Vi;+fi. ,
Y3.3ãi'1iFi 0.2m~O.t2m.
V1.61 X 10-1 + va.33 X 10-1
373. Aeeording to the hypothesis put forth hy N. Bohr,
the electron in a hydrogen atom moves in a circular orbit.
Caleulate the velocity of the electron if the radius of its
orbit is 0.5 x to-• cm (Fig. tt7).
Given: R=0.5 X t0-•em=5 X to-um.
v-1
Solution. The electron moving in a circular orbit is
under the action of the electrostatie force of its inter-
action with the nueleus:
F - 1e 1q/(4ne,oll 1),
where 1e 1 is the magnitude of the electron charge, q the
nuclear charge of the hydrogen atom, and R the radius of
the eleetron orbit. Neglecting the force of gravitational
interaction between theelectron and the nueleus, we write
Newton's second law in projections on the Y-axis (see
Fig. tt7):
F =ma.,
where m is the electron m888 and a11 = 4c = ul/R. This
gives

whenee
v=Jf~,
v=Ji 4x3.t4xs.J·!~C:.~1;~~l~!~;Hxsx10-11 m
<>< 2.25 X 1()1 m/s.
374. Two point charges of 6.7 and -13.3 nC are situ-
ated ata distanee of 5 em from eaeh other. Determine the
Ch. 3. Eleetricity

electri.c 6.eld strength ata point located ata distaoce of


3 cm from the positive charge and 4 cm from the negative
charge.
Giuen: q1 =6.7nC=6.7xto-•c, q2 =-13.3nC=
-t3.3xt0-1 C, r=5tm=5xto-zm,
r 1 =3cm.=3x10-Zm, r 1 =4cm=4xt0-Zm.
F-1
Solution. According to the superpoaition principle, the
6.eld strength at point A is E = E 1 + E 1 , where E 1 is the
electric 6.eld strength at point A due to the charge q1
and E 1 is that due to the charge q1 (Fig. 118).
Since q1 and q1 are point charges, we have E 1 =
:~~~=~º:~:~~e ~~;;r:·~~'::~'i.· !~~~~:~~i:·o.!~:
streogth cao be determ.ined from the Pythagorean theorem:
E = V~+ E:, or, uaing the expressiona for E 1 and E 1 ,

E= V (4n~!rVª + (4n~rl>ª
=,n~ Y"*-+t.
E 4x3.t4x~.85xto-u
X V~3 ~\~~!· +(~:·! ~;~;· ~
7
o.tOtkV/m.
375°. A thin rod of length 20 cm is uniformly charged
with a linear denaity of 1 nC/cm. Determine the electric


E,

y ~ 'i
,,, r ,,,
Fig. H7 Fig. HS
232 Selected Problema on Phyaica

6eld strength produced by the rod at point A (Fig. 119)


lying on the continuation of its axi.s at 10 cm from the

Fig. H9

nearest end and the force of interaction between the rod


and a charge of to-• e placed at point .A.
Givsn: l=20 cm=0.2 m, 'f=i nC/em.=10-' C/m,
r=iOcm=O.tm, q0 =10-1 C.
E-?F-1
Solutlon. We divide the rod into small elements dz of
length and assume that the charge dq of ao element d:z: is
a point charge. Let us determine the field strength dE
produced at point .A by the element d:z: of the rod sepa-
rated from A by a di.stance :z::

dE=~,.
Since "= dq/d:z:, we bave dq= 't d:z: and

dE=~: ..
The electric field strength vectors due to other elements of
length of the rod have the same direction as the vector dE.
The electric field strength at point A is

E=rl+'.....!!=....=-'-'l+r~
r 4meuª 4as.s r za

=
' ( 1) 1·+•
4n&r8 7
'
r = 4ny
1
r(r+IJ '

E= 4x3.i4xt~5xto-H o.txc~::+o.2) ! = 6 kV/m.


Cb. 3. Electricity

Then the force of interaction hetween the rod and th&


charge is
F = Eq0 ,
F ~ 6 X 10' X 10-o N ~ 6 X 10-• N.
376. Three identical point charges of 5 nC each are at
the vertices of a square with a side of 40 cm. Determine
the electric field strength at the fourth vertex of the
square (Fig. 120).
Given: a=40 cm=0.4 m, q=5nC=5xHr•c.
E-1
Solution. According to the superposition principie, th&
electric field at point A is
E ~E, +E, +E,. (1)
where E 1 , E 2 , and E 1 are the field strengths produced by
the charges 91 , 9 2 , and q1 respectively at point A.
Writing Eq. (1) in projections on the chosen directions of
the X- and Y-axes, we obtain
E:.: = E 1 sina + E 2 + E 1 cosa,
(2)
Ev = -E1 cosa+ E 1 sina.
Consideringthat E 1 = E 1 = 9/(4ne 0 ea1), E 2 = q/(4ne 0 er1).
r 2 = 2a1 , sin a = cosa = v212 and substituting thes&

q,
y

X
X
E,
Fig. 120 Fig. 121
Selected. Prolllema on Phyaics

expressions into Eqs. (2), we find that


E,=0,

E.-E= z.i::.~ + 4..:...' + 1!!~


~ t.9 4,!a.1.
8 = 1·9 4.x3.Hx8.~~1~11 x1xõ.4. 1 ! ~s90V/m.
m. Determine the eleetric 6.eld strengt.h produced by
a dipole ata point lying on the perpendicular to the dipole
arm at a distance of 50 cm from its midpoint if the
dipole charges are 10-• and -to-• C and the dipole arm
is 5 cm.
Giwn: d=50 cm.=0.5 m, 91 =10-•C, 91 =-10-•C,
l=5cm=5xt0-1 m.
E-1
Solution. According to the superposition principie, the
6.eld strength produced by the dipole at point O is equal
to the sum of the 6.eld strengths E 1 and E 1 produced by the
charges q1 and q1 respeetively (Fig. 121):
E=E1 +E 1 ,
or in projections on the X-u.is,
E = E 1 cos cz +E 1 COI a.,
where a. is the angle between the vectors E1 and E 1 d.d
the X-axis. Since the dipole is formed by point charges,
we can write
E1 = E1 = q/(4ne 0u 1), E = 2E1 cosa..
Analyzing the figure, we 6.nd tbat r = d' + 11/4 and
V
<:os <Z - 1/(2 V tP- + l'f4). Subatituting the upressions
for E 1 , r, and cos a. into the expreaion for the electric
6.eld strength of the dipole, we obtain

E 4...,. (::+1•/4) 2 y ••~1•/4 - ""' (~~1~•1• ' (I)


Ch. 8. El8CVicity

Since in the denominator of Hpl'88Sion (1) 4dl > l 1, W8


can write

E~ ny~g.,,,, =~,
(2)

E= 4x3.ti~~~:.~:xo.51 ! 36V/m.
Equation (2) shows tbat the electric 6.eld strength pro-
duced by the dipole decreases in inversa proportion to d',
i.e. faster than lhe electric lield strength of a point
charge which decreases in inversa proportion to d1 •
378. Two metal concent.ric spheres of radius 5 and
10 cm bear charges of 2 x 10-1 and -10-1 C. Determine
the electric 6.eld st.rength produced by the spheres at
point.s separated from the centre of the spheres by 3, 8,
and 14 cm. Plot the graph of the dependence of the elec-
tric field streng\h on the distance from the centre of
the spheres.
Gtven: R 1 =5cm=5x10-•m, R1 =10cm-O.tm, 91 =
2x10-•c. 91 =-10-•c, r 1 =3cm=3x10-1 m,
r 2 =8cm=8 x tlr"m, r1 =14cm- t4xto-:r.m.
E 1 -? E,-? E,-? E=E(r)-?
Solution. According to the superposition principie, the
electric 6.eld strength at any point is
E=E 1 +E 1 ,
where E 1 is the electric field strength at any point pro-
duced by the sphere with the charge 91 , and E, the elec-
tric 6.eld st.rength at the same point produced by the
spbere with the charge q1 • Using the superposition prin-
cipie, we 6.nd the 6.eld strengths at poinb A. B, and C in
projections on the r-axis.
For point A (Fig. 122), we have
EA = E1A + E1A·
~c~s 8::ro}. 8A~ : ºo <~:i::1 ~ stl:s,..!i~°e8it:e 8s=!=).
236 Selected Problema on Physics

For point B, we have


Ea = E 1s - EtB,
or, considering that E 18 = q1/(4ne 0 er:) and E 28 = O,
we gel

Ea= .v:;erl •
Ea= 4X3.14x8.~~!~:'X(8X10-')' ! ~28kV/m.
For point C, we have
Ec = E1c - Eic•
or, considering that E 1 c = q1 /(4ne 0 er:) and E,c =
1 q 2 J/(4ne 0 er:), we get

Ec = 4~rl - 4~:~rl = 94~e~!j 1 '


Ec= 4x3.14x:.:S~~:~~14x10-')' ~ .::::4.6kV/m.

ln order to plot the graph, we shall use tbe obtained


values of EA, E 8 , and Ec and calculate the electric fi.eld
strengths at points M and N lying on tbe surfaces of the
spheres. For ali the points inside the smaller sphere, we

E, f0 4 V/m

;~
' 1

:~~+-.L
1 -:.._-_--1-~.=-b-.
D 2 4~6 8 Jre12 Mt;T0" 2 m

Fig. t22 Fig. 123


Cb. 3. Electricity 237

have
E:= EA =0.
The litild st.rength E; for a point. on the outer surface of
t.he sm.aller sphere is
E;= 4 n!~,
E;- 4XS.14X8.e:~:g::1 X(5Xi0""1)1 ! 72 kV/m.
The lield st.rength for a point. on the inner surface of the
larger sphere is
E;= 4n::Sffl'
E;= 4x3.t4x:.~1::0-1•xo.1• !
t8kV/m.
The field st.rength for a point on t.he outer surface of t.he
larger sphere is
E.='~;~1,
E;= 4xS.14~~~~~-;~'xo.t• V 9 kV/m.
Using the obt.ained values, we plot t.he E vs r graph
(Fig. 123). The figure shows t.hat t.he E vs r dependence
is quite complicated. For r <Ri, the curve coincides
with t.he abscissa axis (E =O), for r =Ri and r = R 1 ,
t.he curve sufters discont.inuit.ies.
379º. Two identical positive charges are separated by
a distance of 20 cm. Determine a point on t.he straight
line perpendicular t.o t.he line connect.ing the charges and
passing through its midpoint at which t.he electric field
strength has the maximum value.
Given: q1 =q2 , l=20 cm=0.2 m.
z-1
Solutlon. We take an arbitrary point A on t.he X-axis,
separat.ed from point. O by a distance z (Fig. 124). Aceord-
ing to t.he superposit.ion principie, the eleetric field
strength at this point is
E =E1 +E,. (1)
238 Selected Problema ou Physlcs

whereE1 and E 1 are the field strengths produced at point A


by the charges g1 and g1 respect.i vely.
Projecting Eq. (t) onto the X-axis, we obtain
E = E 1 coa cz +
E, COEI a, (2)
where cz is the angle between the vectors E 1 and E 1 •

E,
-- E X

Fig. 124

Since E 1 = E 1 , expression (2) can be written in the f'olom


E = 2E1 cos a. (3)

From .60AC, we find that cosa=yzs:, 114 =y4z~+zs·


Considering that E 1 = gl( 4:neoe:r2), where r =V r-+ l 2/4 =
V 4z'+ l'l2, we write Eq. (3) as
E 2<·2"·4

...
4ny(ú'+1•>Yú1+1•

= 21:8cJB(4z•+lí)1/I º
(4)

lf the Jield .strength at a given point has the maxim'um


value,

*"'"º·
Substituting expression (4) into (5) and diBerentiat.ing,
we obtain
(5)

.... *
-/;-[ neaect!:+fl)''']
=....!!....
(ú-•+11)11•-z iz (ú•+P)l/9
O,
((ú*+1*)1/l]i
Ch. a Eleckic.itJ 239

whence
*(4z'+l')"'-zi-(4z'+l')'"=0,

(4z'+l')ll•-zf(4z'+l')'"4·2z=0,

(4z'+l')'l1 ((4z'+ l')-12.z'( =0, 4z'+ 12 -12.z'=O,

z=± VPtB= ± uc2V2>.


z = ± 0.2/(2 V2) m"" ± 0.071 m.

380. A field is produced by an infinite vertical plane


with a surface charge density of 4 nC/cm1 • A bali of mass
t g bearing a charge of t nC is suspended on a string in tbe
field. Determine tbe angle between the string and the
plane.
Gtven: a=4nC/cmª=4xt0-5 C/mª, m=1g=10-'kg~
q=I nC=ICr'C.
a-1
Solutkm. The charged ball suspended on the string in
the electric field is acted upon by the force of gravity mg,
the electric force F witb which the field of tbe plane acts
on the charged bali, and the tension
T of the string (Fig. 125). Writing
the equilibrium conditions for the
ball in projections on tbe X- and
Y-axes, we obtain
F-T sin o:.=0, or F= T sina,

-:rm: ~ c;c:s :. O,
Dividing Eqs. (t) termwise, we get
<1>
Fig. 125
tan a. = Fl(mg). (2)
Considering that F = qE, E = a/(2ec,e) and substituting
these expressions into Eq. (2), we obtain tan a =
Selected Problema on Physics

qcr/(2e 0 emg), whence

a""" arctan ~=.,. ,


a= aretan 2 X 8_ir,°;;1~~'{':~;.. x B.8 "' aretan 0.23
<><0.225 rad.
381. An electric fi.eld is produced by two infi.nite,
parallel planes with surface charge densities of 2 and
-d
E1 E1

-----·
- E E----+
'---
I ][ ]/[
(U)

~ (b}
Fig. 126
1

-4 nC/m' (Fig. 126a). Determina the &eld strength be-


tween the planes and beyond tbem. Plot tbe graphs of the
electric fi.eld strengths for regions 1-111.
Given: a 1 =2nC/m2 =2x1o-'C/mª, a 2 =-4nC/m1 =
-4x 10-•ctm2 •
E 1 -? E 11 -? E 111 -?
Solution. Let us consider tbe regions /,li, and Ili of
the &eld (aos Fig. 126a).
Ch. 3. Elecll'icity ...
Region /. According to the superpoaition principie, the
field strength in this region is
E, = E1 + E,, (1)
or in projections on the X-axis,
E1 rr=E 1 -E1,
where E 1 = 1 a 1 1/(2e0 e) and E1 =a1/(2eue). Using these
expressions, we write Eq. (1) in the form

Er=W- ~ = 1 ª~ª•'
E1 = :::;;,•;12:; 11 ~;: r.J113V/m.
Similarly, we determine the values of the 6.eld strength
for the other regiona.
Region //:
E11=E1+E1=W+~= lo~a•'
E11=-.:~:~~12;1:~; ! ~339V/m.
Region ///:

E111=E1-Ez-=~- ~~I =ª•~11'


E111 :~:'.;~W1!~~ ! "V 113V/m.
Using the obtained values, we plot the graph of change
of the 6.eld strength along the X-axis (Fig. t26b).
382. AB a result of a collision between an air molecule
anda coamic particle, an electron is formed in the vicin-
it.y of the negative plate of a parallel-plate capacitar.
At what velocity will the electron reach the positive plate
if t.he plate charge is t nC, the plate area is 60 cm•, and
the separation between the plat.es is 5 mm?
Given: g=1 nC=10-1 C, 8=60 cm•rr=6xt0-ª mª,
d=5 mm=5 x 10-S m.
v-1
11-0970
Selected Problema on Physics

Solution. Tbe eleetron formed as a nsult of the eolli-


sion starts to move in the uniform. field of the parallel-
plate capaeitor with a constant acceleration at zero ini-
tial veloeity. From the equation d = at112 of a uniform.ly
aeeelerated motion, we determine the time of motion of
the eleetron from one plate to another: t = JÍ 2d/a.
Subatituting this expression foto the formula v = at for
veloeity, we obtain '
•~a v2a1. ~ J(2ad. (t)
Aeeording to Newton's seeond law, the aeeeleration of the
electron i.s
a=Flm,
where m is the electron mass, F = lef E the electric force
exerted on the eleetron by the capacitar 6.eld, e the
eleetron charge, E = a/(808) the eleetric 6.eld strength
within the capacitar, and a = q/S the surface charge
density on the capacitar plates. Using these expressions
in Eq. (t), we obtain

v=Jf~.
v=Jf 6x:t.~68~8~~1:t1!~11~59~ 1:.10--u m
~ 5. 76 X 10' m/s.
383. Explain why a light pith ball is 6.rst attracted to
an electrostatieally charged rod and is then repelled
from it?
Answer. When tbe charged rod is brought to the ball,
charges of opposite signs are indueed on the bali. A charge
opposite to that on the rod is induced on the side facing
the rod, while the charge induced on the opposite side of
the ball is of the same sign as that on the rod.. The force of
attraetion between unlike charges is stronger than the
force of repulsion of like charges sinee the unlike charges
on the bali and the rod are eloser than the like charges.
When the ball and the rod come in eontaet, the charge on
the bali opposite to that on the rod is neutralized. The
ball acquires a charge of the same sigo as that on the rod
and is repelled from it.
Ch. 3. Electrici&y ...
EXBllCISES

384. What charge must be impai'ted to two balls of mus


1 g each for the force of mutual repulsion of the charges
to balance the gravitational force of mutual attraction of
the balis? The balis are in air.
385. Two identically charged balis of maS8 0.5 g each
aod suspended on strings of length 1 m are deDected 4 cm
apart. Determine the charge of each ball.
386. A steel bali of radius 0.5 cm is immened in kero-
sene and is in a uniform electriCDintlif strength 35 kV /cm,
which is directed vertically upwards. Determine the
charge on the hall if it is in equilibrium.
ri:'~ ~h~~~r:rly.:;ax.d ~!::~~~:f ::~i:e!~: &!:d
strength on the axis of the ring at points separated from
its centre by O, 5, 8, 10, and 15 cm. Plot the graph of
dependence of the field strength on the distance from
the centre of the ring.
388. A~small ball of mus 100 mg and charge 16.7 nC is
suspended on a string. To what distance must an identical
charge be brought to it from below for the tensile force
to be reduced hy half?
399; What is the ratio of the gravitational force acting
hetween two protons to the force of their Coulomb repul-
sion?
390. Three negative charges of 9 nC each are arranged
at the vertices of an equilateral triangle. What charge
must be placed at the centre of the triangle for the system
to be in equilibrium?
391. A bali having a maS8 of 0.4 g and a charge of
4.9 nC is suspended on a string in the fi.eld of a parallel-
plate air capacitar with a charge of 4.43 nC and a plate
area of 50 cm2 • By what angle will the string with the
ball be deftected from the vertical?
~ Two like-charged balls suspended on identical
strings deDect through a certain angle. What is the densi-
ty of the material of which the balis are made if the
angle of deDection of the strings remains unchanged after
their immersion in kerosene?
18•
Selected Problem1 on Phy1ics

ELECTROSTATIC PIELD POTENTIAL. WORK DONE 1N


MOVING A CHARGE IN AN ELECTROSTATIC FIELD

The eleetrostatie fteld pot.ential is defioed as


cp = -Alq,
where A is the work dooe by the lield forces io moviog
a charge from inlinity to a given poiot in the lield.
The potentlal of the fi.eld produeed hy a polnt charge
or a eharged sphere is
cp = Q/(4ne0 er).
If a lield is produeed by several poiot charges, the p~
tential of the lield at a given poiot is equal to the al-
gebraie sum of the potentials of the lields produeed at
this point hy eaeh charge separately:

cp-~
·-· '1'1·

The relatioo hetweeo the streogth E of ao eleetrostatie


field and the potentlal dilerence U aeross the plates of
a parallel-plate eapaeitor (uniform lield) is as follows:
E = (cp1 - cp,)/d = Uld,
where d is the separation hetweeo the plates of the paral-
lel-plate eapaeitor.
The work done hy ao eJeetric foree ln movlng a charge
from one polnt of an eJeetrostatle field to another is
A =q(cp1 -cp,) =qU.

393. A dust partiele of mass 10-• g is suspended hetweeo


the plates of a parallel-plate air eapaeitor to whieh a volt-
age of 5 kV is applied. The separation hetweeo the plates
is 5 em. What is the charge of the partiele?
Giuen: m=I()-• g=IO-" kg, U=5 kV-5 x 10' V,
d=5xto-> m.
q-1
Ch. 3. Electriclty

Fig. 127 Fig. 128

Solution. ln the electrostatic field, the dust particle


experiences the action of the force of gravity mg and the
electric force F exerted by the field (Fig. t27). Writing
the equilibrium condition for the particle in projeetiQll.S
on the Y-axis, we obtain
F-mg=O,
whence F = mg.
Coosidering that. F = qE and E = U/d, we get qU/d =
mg, whence
q=mf
q= 10- 11
x:::;x 10-S C=9.8x 10-"C.

396. An electron fties into a parallel-plate air capacitar


at a velocity of 6 X 10' m/s parallel to the plates. The
separation between the plates is t cm and the potent.ial
diBerence is 600 V. Determine the deftection of the electron
caused by the field of the capacitor if the length of its
plate is 5 cm.
Gi11en: v=6x10 7 m/s, d=1cm.=10-Zm, U=600V,
1=5cm=5x10""' m.
h-1
Solutton. The force e:r.erted by the eleetric field on the
electron is
F - l• IE,
wh&1e E= U/d.
... Select.ed Problema OD Phyalca

Since the electric field strength is directed upwards


(Fig. 128), the force acting on the electron is directed
downwards. We can represent the motion of the electron
as a superposition of two independent motions in the hori-
zontal and vertical directions. ln the horizontal direetion,
the electron, as before, will move uniformly since no force
is acting on it in this direction. At the same time, it
will be displaced with a uniform downward acceleration
due to the electric force. The t.rajectory of it.s motion will
be a parabola. The motion of the electron in the capacitar
is similar to the motion of the body thrown along the
horizontal. During the time of motion, the eleetron will
traverse along the horizontal a distance
l - vi, (1)
and will be displaced along the vertical by a distance
h - al'/2, (2)
where a is the electron acceleration.
Solving Eqs. (1) and (2) togather, we find that
h - al'/(2v'). (3)
ln order to calculate the acceleration, we apply Newton's
second law. Since only one force F act.s on the electron in
the vertical direction (we neglect the force of gravity
acting on theelectron), we haveF =ma, whence a= Fim, or

• - I e iU/(dm). (4)
Substituting expression (4) into (3), we obtain
h=is~u
(5 X 10-11)1X1.8 X 10-111 X 800
2x(dx10ry•x1ó'"ix9.1x1PI m
~ 3.65 X 10-s m.
395. Two balis bearingcharges of 6.7and13.3 nC are at
a distance of 40 cm from eac'i other. Wb.at work should be
done to bring the sepoll'ation duwn to 2.5 cm?
Ch. 3. Electricity 2'7

Giv<n: q,=6.7nC=6.7x 1()-'C, q2 =13.3nC= 13.3x


tcr'C, r 1 =40cm=0.4m, r 1 =2Scm=0.25m.
Âe:r.t-?
Solution. ln thia type of problema, it is convenient to
assume that one bali is sl.ationary and lhe other bali
moves in the field produced by lhe first bali. Let the ball
bearing the charge q1 produce a &.eld, while the bali
bearing lhe charge q1 move in the field from the point at
a distance r 1 from the charge q1 to the point ata distance r 1
from it. The work done by an externai force is
A,., = -A = q, (q>, - q>,), (1)
where q> 1 anel q> 1 are the potentials of the initial and
final points ol the field. Since the field is produced by
a point charge, we can write

qi, = 4n!;u1 ' fila= 4n~1 . (2)


Substituting expressions (2) into (1), we obtain

Âe:r.t = q2 ( 4n::.r1 - tn!:sr1 } = 91 !!!:;;.-,:.> •


Âe:r.t :~;.:r;:.;:~~~?;~~~t~:·r~ J ~ t.2µJ.
396. A point charge of 10-• C is at a distance of 50 cm
from the surface of a spbere of radius 9 cm, charged to
a potential of 25 kV. What work must be done to decrease
the separation between the sphere and th" charge to
20 cm?
Gtven: l 1 =50cm=0.5 m, R=9 cm=0.09 m, q>,ph=
25kV=2Sx10SV, q-l<r•C, la=20cm=0.2m.
A-?
SoluUon. Let tbe chargeq move from pointB topoint C
in lhe eleetrostatic field produced by the charged sphere
(Fig. 129). The work done in moving the charge in the
eleetrostatic field is
A., = q ('Pa - q>c). (1)
Selected Problem.11 OD Ph)'llica

Consideriog that the potentials of the field produced by


!~:b1Mh~1R +~jj .:: :cº!!~.•~l~~e e~ ~.>f.ª,:
write Eq. (1) in the form

A11=9 (4n~(V+11) - 4ny9 (l+1.) J


(2)

Tbe potent.ial of the field on the surface of tbe sphere is


'Jl.pb= J:::R •
whence the charge oD the spbere is
91pb = 4P1pb·4ne0eR. (3)
Suhlltituting Eq. (3) iuto (2), we find that

Aei= ~~=(~~:~+:::
=qRqi.,b cR+1~ci+I.) ·
A,1=10-1 X 9 X 10-S X 25 X t()I X (Õ.ó9+~~)(g:~+Õ.2)
"'-39.5p.J.
Tbe minus sign indicates that the electri.c force prevent.s
tbe charge from moving, i.e. is directed against the mo-
tion. ln order t.o displace the charge, an externai force
must be applied t.o it. in t.he direction of motion. The work

Fig. 128 Fig. 130


done by thia force will be A = -..4 81• Consequently,
A =39.ó J'l.
397. Determine the work done by electrie fOl'C88 in mov-
ing a charge of 1 nC from point A to B and from point.
C to D if r = 6 cm, a.= 8 cm, q1 = 3.33 nC, and q1 =
-3.33 nC (Fig. 130).
Gtwn: q=1DC=10-•c, r=6cm=6x10-•m, a=
8cm=8X 10-Sm, q,-3.33nC=3.33 X Ur'C.
q,= -3.33nC= -3.33x10-•c.
A-1
Solution. 1. The work done by an electric force in mov-
ing the charge q from point A to B ia
AAB = q (~A - ~.). (1)
Here IPA = 'PA, + 'PA, = '11/(4n8oer1) + '11/(4n8oer,)
and cpa = cpa 1 + cpa 1 = q1/(4n8 0 er 1)+ q1/(4n8 0 8r1) are
the potentials of the field produced by the charges q1 and q,
at points A and B, where r 1 = r and r 1 = Vr 1 +a•.
Subatituting the expressions for IPA• cp., r 1 , and r, into
Eq. (1), we obtain

AAa=q [(~+ 4ne,s Vr1+11•)


- ( 41Ulol ;rr+.' + 421~ )]
9 C91-9.l cYrs+Gi-rl
4a!oU Jl",.S+11
1

AAa= 4!~~1~~3~x1~J!·~:a1:;~.
x <Ycexto-1)•+(8xto-l)•-sxtO-I) 1
Ycsx to-1)•+(8x 10-•)1
<><8X1()-71.
2. The work done hy an electric force in moving the
charge q fr om point C to D ia
Aco = q (~e - ~.). (2)
Here cpc = cpc, + cpc, = q1/(4ne 0 8r.) + q1/(4n8 0 er.)
and ~.=~D.+ ~D.= q1/(4ne 0 er.J + q,f(4n•o••.l
Selected Problema on Physica

are the po&entials of the field produced by the charges q1 and


q1 at points C and D, where r1 = a.12 and r., = y;:q:ãil4.
Substituting tbe upressions for 4Pc• 4PD• r,, and r., into
Eq. (2), we obtain
AcD=q[( 4~2+ 4n~2}
-(4nllo8;~+a1/4+ 4ftto1;~}]
2q (IJ'1+t.> <Jl1i+iiiii-a/2)
= 4ne,u V r1+a•10t:
Sinee q1 = -q1 , we 'bave (q1 + q.) = O and A CD = O.
398. What is the velocity of an electron tbat has eroased
an aeeelerating potential differenee of 200 V (Fig. 131)?
Giwn: U=200V.
u-1
Solution. ln the electric field, the electron uperienees
the ael.ion of the force F = 1 e 1E whieh makes il. Dy wil.h

+--€>
--F-,,---.-~-

Fig. t3t
a uniform aeeeleral.ion against the electrie field vector.
The electrie force does thereby a work on the electron,
whieh is equal to tbe ebange in its kinetie energy:
A-AWi.;= Wi.;-Wi.;,·
Considering that A = 1e 1u, Wi.; = mv1/2, and Wi.;, =O,
we obtain 1e 1U = mvl/2, wbenee

v=JI'~,
V= y2xt9~.Xxt'[;.1x3>D ~ ~8.4x 10'm/s.
Ch. 3. Electricity 251

399. A bali of mass t g moves from point A wbOl8 po--


tential is 600 V to point B wbOl8 potential is zero.
Determine tbe velocity of the ball at point A if its veloeity
at point B is 20 cm/s. Tbe charge of the bali is 10 nC.
Giwn: m=t g=10-3 kg, cp 1 =600 V, CJlz""'0 1 v""'
20 cmis=0.2 mls, q=10 nC-1()-8 C.
Vo-?
Solution. The ball moves under the action of the electric
fOl'Ce exerted by the Jield. Tbe change in the kinetic energy
of the bali is equal to the work done by the elect.ric
force:
ll.W,=A. (1)
Since ll.W, = mv'/2 - .W,12 and A= q(<p, - <p,), Eq. (1)
can be written in the form mv1/2 - mV:/2 = q (cp1 - cp1),
whence

V0 =Vv' 2q(<p, '1'1)/m,

Vo= V0.2 1 -2X1()-1X600X1()3m/s~0.17 m/s.

400. A ball of mass 40 mg, having a charge of 1 nC,


moves from in&nity at a velocity of 10 cm/s. To what
minimum distance can tbe bali approach a point charge
equal to 1.33 nC?
Giwn: m""'40mg""'4xt0-5 kg, q1 =1nC=10-1 C, V=
10 cm/s=0.1 mls, q1 =1.33nC=t.33x10-'C.
r-1
Solution. The charged ball experiences the action of the
force exerted by the electric 6.eld. The work done by the
force is equal to the change in the kinetic energy of the
bali:
ll.W,=A.
Since AWk ""' Wk - Wk, ""'O - mv'/2 = -mv'/2,
A = q, (<p. - <p,) = q1 (0 - q,/(4"o 0 sr)) =
-q1q.f(4.tte: 0 Er), we have -mv'/2 = -1JJq1/(4ne:.u),
Selecled Problema on Phyeica

whODCe
r=~,

' 2x3.t4:t~x:t.i:.~~O:xttrtxo.t• m~6 X 10-Zm.


401°. An electron moves in the field produced by a
charged sphere of radias 10 em along the radius eonnecting
the points separated by 12 and 15 cm from the centre of
the sphere. The velocity of the electl'on ehanges thereby
from 2 x 11)1 to 2 x to• mls. Determine the surface
charge density of the sphere.
Given: R=10cm=0.tm, r1 =12cm=0.t2m, r1 =
15 cm=0.t5 m, v1 =2xtO'm/s, v1 =2xtO'm/s.
a-1
Solution. During the motion of the electron, the elec-
tric lield does a work equal to the change in the electron
kinetic energy:
A=âW=W1 -W1 = na;'- m;f
(!)

On the other hand, the force e:r.erted hy the electric


field on the moving electl'on ia
F- l• IE,
where E= q/(4ne 0 er1 ) ia the electric field strength pfo-
duced by the charged sphere. Hence it is clear that the elee-
tric force depends on the diatance r and varies during the
motion of the electl'on io the field. Therefore, the work
done by the forçe ia

A= rFcosmdr,
"
or, taking into account the expre88ions for F and E and
conaidering that coe m = t and the charge on the sphere
Ch. 3. Elect.ricity 253

is q = a4nRt, we have

A _f"a.f.nR'l1ldr
-i~·
'•
Taking the constant quantities out.side the integral sign
and integrating, we obtain

Â= aR'l;I f' ~= aR' l•I (-.!.J'1•


e.a }. ,... e1e r "•
aRtl11(r1 -r1)
(2)
Se1r1rt
Equating the right-hand sidea of Eqs. (t) and (2), we
find that

whence
Se1r1r.m(1o1l-ul)
a 2Ril,l(r1 -ri)'
a=
8.85 X to-11 xO.t2 xO.t5x 9.t X to-ai X ((2 X UJl)'-(2Xt()l)t] e
2 X Ó.ti X t.6 X t0-11 X (Ó.t5-0.t2) Jiii'
~ 5.96 nC/m"'.
402. ldentical charges are imparted to two metal
spheres of different radii. Will the charges flow from one
sphere to another after their connection with a conductor?
Answer. Since the potentials of the spheres are diflerent
(<p = Q/(4n~ 0 ~R)), after the connection the charge will
flow from the sphere with a higher potential (smaller
radim) to the sphere with a lower potential (largar radim)
until the potentials of the spheres assume the same value.

EXERCISES
403. Two parallel plane plates separated by tO cm
are charged to a potential difference of t kV. What force
will act on a charge of tQ-fi C placed between the plates?
Selected Problema ou Phyaics

4.04.. Two charges of 1 C and of -6.67 nC are at a dis-


tance of 10 cm from each other. What work should be
done to transfer the second charge to a point separated
from the first charge by 1 m?
4.05. A point mass bearing a charge of 0.67 nC and mov-
ing in an accelerating electric field acquires a kinetic
energy of 10' eV. Determine the potential diflerence be-
tween the initial and final points of the trajectory of the
particle in the field if its initial kinetic energy is zero.
406. As a result of a radioactive decay of a polonium
atom, an alpha-particle is emitted at a velocity of 1.6 X
101 m/s. What potential diflerence should be applied to im-
part;the same veloeity to the alpha-particle?
40'7. Determine the force of mutual repulsion between
two balis in air if each bali is charged to a potential of
600 V. The dia meter of each bali is 1 cm and the distance
between their centrei is 20 cm.
408. A field is produced by a thin rod which is bent to
~~:it; ~( l~ri:~,:i~d1sp~~~~°:haligech ~~~ ;g~sa ~::::J
0
at the centre of the half-ring. Determine the work that
must be done to remove the charge from the centre of the
half-ring to infinity.
409. A small bali bearing a charge of 6.67 nC is at the
centre of a hollow metal sphere of radius 1 m carrying
a charge of 3.34 nC. Determine the potentials of the
field at points separated from the centre of the sphere
by 0.5, 1, and 10 m. ,
410. Point charges oi 1.33, -0.66, 0.99, and
-1 .32 nC are arranged at the vertices of a square. Deter-
mine the potential of the field at the centre of the square
if its diagonal is 20 cm.
411. Determine the potential of the point of a field
produced by a metal sphere with a surface charge density
of 10-11 C/cm1 and having a radius of 1 cm if the distance
from this point to the surface of the sphere is 9 cm.
Ch. 3. Elec&ricit.y

Capacit•nce. The Energy


of • Charged Conductor, CapacHor,
and oi an Ell!cfrostallc Fleld
The eapaeltanee of a eonduetor is
e= qt<J/.
The eapaeltanee of a sphere is
C = 4nee,R,
where R is the radius of the sphere.
The eapaeilanee of a parallel-plate eapaeltor is
C = &&08/d,
where S is the area of a capacitor plate and d the separa-
tion between the plates.
The eapaeltanee of a eapaeltor bank consisting of
parallel-connected capacitors is

C=i:
M
C1
and that of series-eonnected capacitors is

i-=~*· f=I
where C1 is the capacitance of a capacitor and n the num-
ber of capacitors in a bank.
The electrle energy of a ebarged eonduetor is
W = (1/2)q<J/ = (1/2)C<1/'.
The electrle energy of a ~harged eapaeitor is
w = (1/2) qU = (1/2) cu•.
The energy of a (unlform) electroslatle Jield is
W = (1/2) e,eE'V.
where V is the volume occupied by the lield.
The volume energy denslty of a Jield is
ID=(1/2)e,eE2.
Selected Problems on Physicll

4t2. A metal sphere of radius 5 cm is charged to a per


tential of 150 V. Determine the potential and field
atrength at point A separated from the surface of the
sphere by to cm.
Glven: R=5 cm=5x11r1 m, CJ>.pb=150 V,
d= 10 cm=0.1 m.
cp-? E-?
Solutlon. The potential of the field produced by the
charged sphere at point A (Fig, 132) is
cp = q/(4"BoBT), (1)
where q is the charge on the sphere and r = R + d the
distance from the centre O of the sphere to point A.

Fig. 132

The capacitance of the sphere is


Capb = 4neoeR. (2)
On the other hand, Capb = q/cp,Pb• whence q = Capb~•Pb•
or, taking into account Eq. (2),
q = 4nBoBllcp,... (3)
Substituting e:r.pression (3) into (1), we find that
cp= 4~pb =~=~.

cp= ~~~~~~~ V=SOV.


The strength of the field produced by the charged sphere
at point A is
E= q/(4"Bou'),
Ch. 3. Electricity 257

or, taking into account Eq. (3),

E= 'n:~ph =~= (~~p~1 •

E= ci: 11~.;~.~1 ! ~3.3x 10ZV/m.


413. Two spheres of radius 5 and 8 cm are charged to
a potential of 120 and 50 V respectively and connected
by a wire. Determine lhe potentials of lhe spheres after
they have been connected, and lhe charge lhat has passed
from one sphere to lhe olher.
Given: R1 =5cm"""5x10-Zm, R2 =8cm=8x10-Zm,
cp,-12ov. cr.-sov.
cp-11!.q-1
Solution. Knowing the radii of the spheres, we can write
lhe expressions for lheir capacitances: C1 =-Ú:8o8R1
and C2 = -Ú:8oBR 2. Then the charge on each sphere before
lhe connection is
g1 """C1'P1=-Ú:1BoBR1tp1• g2=C2CF2=-Ú:BoBR1f1>2 (1)
respectively. The total charge on the spheres before the
connection is
91 + g1 = -Ú:80Bll1Cfl + -ú:1BoBR1CF1
- 4n•o• (R 1cp, + R,cp,). (2)
After the connection, the charges on the spheres will be
redistributed: a part of lhe charge from the sphere wilh
a higher potential will pass to the sphere with a lower
potential. As a result, the pot.entials of the spheres equal-
ize and become equal to q>. Consequently, expressions (1)
and (2) after the connection become
g~ =
4nB0BR1q>, q~ = 4nB01BR1CF•
q; = 4 nBoBR1ip + 4n80BR 1q>
+ g~
- 4n•o• (R1 +
R,) cp. (3)
However, lhe total charge on the spheres does not change
as a result of lhe connection. Consequently, g1 +
q2 =
17-0170
258 Select.ed Problema on Physics

b; + q;,
or, taking Eqs. (2) and (3) into account,
4ne0 e (R1 cp1 + R 1 cp 1) = 4:n:e0 e (R1 R 1) cp, +
whence
cp= 'f~·::;· '
(f t20x:~:~:!:°x~~ttr' V!!:!.77V.
Knowing cp, we can determine the charge tbat has passed
from one sphere to tbe other:
4q = q1 - q; = 4nEoe:R14Pt - Ú:eoeR1cp
= 4ne0 eR1 (cp1 - q:i),
fi.q - 4 x3.14 xB.85xtO-"x1 xsx10-•x(120-77) e
= 2.4 X 10-1 0 C.
414. Three charged water drops of radius 1 mm each
merge ioto ooe large drop. Determine the poteotial of the
large drop if each small drop carries a charge oI to-to C.
Given: n=3, r=1mm=10-ªm, q=to-ioc.
cp-?
Solution. Tbe potentia] of the large drop is
<p-QIC, (1)
where Q=nq and C=4:n:Eoe:R.
The radius R of the large drop can be determined from
lhe mass conservation law:
M=nm, (2)
where m = pV= (4/3) prtr3 is the mass of a small
drop aod M=pV1 =(4/3)pnR8 the mUl!I of the large
drop. Using these expressions, we reduce Eq. (2) to the
form R'= n,.S, whence R= r Vii". Substituting tbe

q:i= 4nyr
..
expressions for Q, C, and R into Eq. (1), we obtain

~~,;'

'P= 4x3.t4xs.:::~:•xtxto-a.yã V~t.B7 kV.


Ch. 3. Electricity 259

415. Tbe area of a plate of a parallel-plate air capaci-


tor is 60 cm', tbe capacitor charge is t nC, and the poten-
tial diflerence across its plates·is 90 V. Determine the
distance between the capacitor plates.
Given: S= 60 cmª = 6 x tO-a m2 , 9 = t nC = 10-1 C,
U-90V.
d-?
Solutian. Tbe capacitance of a parallel-plate capacitor
is
c-.,eS/d, or C-q/U.
Equating the right-hand sides of tbese expre88i.ons,
we obtain e0 eS/d= 9/U, wbence

d=U~&tB'

d= 9Dx8x1o-a~s~5x10- 11 x1 m :::::- 4.8 x to-3 m.


416. The plates of a parallel-plate capacitor are insu-
lated from eacb olher by an insulator layer. The capacitor
is cbarged to a potential dillerence of 1 kV and disconnect-
ed from the voltage source. Determine lhe permittivity
of tbe insulator if the potential dillerence across lhe capac-
itor plates increases to 3 kV after its removal.
GllHln: U,-lkV-IO'V, U2 -3kV-3x!01 V.
•-?
Solution. Since lhe capacitor is disconnected from the
voltage source, the charge on its plates will be the same
in both cases: 91 = 91 • Considering lhat 91 = U1 C1 and
91 = U1 C1 , we obtain U1 C1 = U,C1 , whence
(!)

Using the expressions for the capacitance of a parallel-


plate capacitor C1 = e1 e0 S/d and C1 = e1 e0 S/d, we get
c,1c, - .,1<,. (2J
.,.
280 Selected Problems on Physies

Comparing Eqs. (t) and (2), we find that U9 /U1 = e,,le1 ,


whence

e1=.g; e,;,
e 3x!:x =ª·
1= t

417. A parallel-plate air capacitor with a plate separa-


tion of 5 cm is charged to 200 V and disconnected from
a voltage source. What will be the voltage aeross the capa-
citor after its plat.es have been moved apart to 10 cm?
Given: d1 =5cm=5x10-zm, U1 =200V,
d,-10 cm-0.tm.
Uz-?
Solution. The capacitanee of the capacitor and the
voltage across it are
C,-e,eS/d,, U,-QIC,-Qd,l(e,eS) (t)
before moving the plates apart and
e, - .,.std,, u, - QIC, - Qd,t(e,.S) (2)
after that.
We assume that the charge on the capacitor does not
change as a result of moviog the plates apart since the
capacitor is disconnected from the voltage source. Divid-
iog expressions (1) by (2) termwise, we obtain U1 /U 2 =
d1/d 9 , whence ·
U2=d~1'
U2 = ~~~~ V=400V.
418. The distance between the plates of a parallel-
plate air capaeitor conoected to a voltage source of emf
180 V is 5 mm and the area of the plates is 175 cm1 •
Determine the work done in moving the plates apart to
12 mm if (1) the capacitor is disconoected from the voltage
souree; (2) the capacitor is conoected to the voltage
source all the time.
Ch. 3. Electric1ty 261

Gtuen: 1=180V, d1 =5mm=5xt0-ªm,d1 =12mm=


t2xt0-ªm, 8=175cm1 =t.75xtO-•m•.
A1 -?A1 -?
Solution. 1. The work done in this case is measured by
the change in the energy of the capacitor, i.e.
(t)
where W1 = q1 /(2C1) and C1 = e0 eSld1 are the energy
and the capacitance of the capacitor before moving the
plates apart, and W1 =q1 /(2C1) and C2 = eoeS/d 1 the
energy and the capacitance after moving the plates
apart. lf the capacitor is disconnected from the source,
the charge q on the plates remains unchanged when they
are placed apart,
q= C1 U1 = e0 eSU1ldi..
or, since U1 = i before moving the plates apart,
q= o,eS/Bld,.
Using the expressions for W1, W 1 , C1 , C1 , and q, we write
Eq. (1) as lollows:
Ai= •l;::sslds _ a:~1
=~(d,-d,),
t.75x ~;: :!; 1x
A, s.ssx10-11x1 x 2 1 (t2xto-1 -sx t0-1 ) J

-705nl.
2. lf the capacitor is connected to the source, the po-
tential difference across its plates remains unchanged.
The total work done in moving the plates apart is
A=A, -A., (2)
where A 1 is the work done by an extemal force and
A1 = Bllq= 1 (q1 - q1 ) is the work done by the source
~ha1::ev~fo~e;;~:~: ~ w~::sq1a;a~'a!d ~e:!ªl::i::
charge after moving the pf.tes apart. The work done by
262 Selected Problema on Phyaiea

the source A, is taken with the minus sign since the source
does a negative work when a charge is moved from the
positive to the negative plate.
On the other hand, the total work is equal to the
change in the energy of the capacitor:
A = âW = W, - W 1• (3)
Equating expressions (2) and (3), we 6.nd that A. 1 - A, =
W 1 - W1 , whence
A,=W,-W,+A,. (4)
Here W1 = C11'1/2and W1 = C..1/2is the energy of the
:~i~~~!i;t=~:~~~!io8~~:o!°1:.i~.~1:'n~1::.•i:foariq~{lt
we obtain
A, = c,r•12 - c,r•12 + r cc,r - c,r)
C,) l'/2,
- (C, -
or, comidering that C1"""' BosSldi and e.= aoeSld.,
Ai= f ( e-;: - e-;: )= e.eSl~~1-cf1) '
Ai= s.ssx10- 1 x1xti1!~!~; 1xx1:xxt~~x10- 1 -sx10- 1 ) J
=293n1.
419. Three capacitors of capacitance t, 2, and 3 µ.F are
connected in series and to a voltage source with a polen-
tial di8'erence of 220 V. What are the charge and
voltage of each capacitor?
Gtuen: C,=t~F=l()-•F, C,=2~=2xtcr•F,
C,=3~F=3xtcr•F, U=220V.
qi-? 112 -? q,-? U1 -1 U2 -? U 1 -?
Solutton. We denote by cp1 the potential of plate 1, cp1
the potentials of plates 2 and 3 (the potentials are equal
sinee the plates are connected), cp1 the potentials of
~\~i:ª / re8:~:~ !ºct~~elr~~.!t!~~º!!'~~- 6T~!g~~;3~
- q is induced on pia te 2 and the charge + q on plate 9.
Ch. 3. Electrieity

Similarly, the charge -q is indueed on plate 4 and the


charge + q on plate 5, and -q on plate 6. Therefore, the
e, e, e,

L:;J Fig. t33

charges on series-connected capacitors are equal. We


denote these charges by Ili= q1 = q8 = q. By definition,
IJl1 - IJla= q/C1 = U1,
q>1 - «Pa= q/C1 = U1,
IJla - q>,=qlC 8 = U 8.
Summing up these equations lermwise, we obtain
~. - 'P• = q (llC, + llC, + 1/C0).
Considering that 'Pi - cp, = U is the voltage acrou the
source, we find that
u
11c1 +11c1 +11c1 •

q= 1110~+11(2x:i'l+l/(3xte>-•) C=12x l!r'C


Then the voltages across the capacitors are

Ui=t· U1 = t 2 ~Jr' V=120V,

U1 =t, Uz= ~2 ::::: V=60V,


Ua=f., U3 = ~2: 1W: V=40V.
Thus, in series connection, the capacitors of diBerent
capacitances are under di&erent voltages. The lower the
Selected Problema on Physica
capacitance, the higher the voltage across the capacitor
plates.
420. Seven capaciton of capacitance 2 and t l'F are
connected between terminais A and B as shown in
Fig. t34a. Calculate the capacitance of the system.
Giv.n: C,-2µF-2 x !o-<F, c,-1 µF-!o-<F.
C-1
Solutfon. Let us consider the subcircuit between points
D and E (the right-hand side of Fig. t34a) separately.

,J 1:-y
D

I~~
A" Ad

e) e, e,
81' I 1iL0 ª" I 1;::J
E
E
(a} (b}
Fig. t34

1t eonsists of two parallel branches one of which includes


a capacitar of capacitance C, and the other includes tbree
capacitors of capacitance C1 , connected in series. The
capacitance of this subcircuit is Cnz =e, + C', where
C' is determined from tbe condition t!C' = t/C1 +
t/C 1 + t/C1 = 3/C1 , whence C' = C1/3. Then
e•• - e, + c,13- (3c, + c,)t3. (!)

We replace the subcircuit between points D and E by an


equivalentcapacitanceCnz(Fig.134b). Therequiredcapac-
itance C is now the sum of the capacitances of two paral-
lel branches one of which includes a capacitar of capaci-
tance e, and the other consists of three series-connected
capacitors of capacitance C1 , C0 z, and C1 • Therefore,
C = C, + C", wbere C" can be determioed from the
condition
11c-11c, + 11c•• + 11c, - 21c, + 11c.. (2)
Ch. 3. Eleotricit.y

We transform expression (2) by using Eq. (t):

-#=i+ sc,~c1 = t~ât-~!"J,> •


whence C" = c~11,'t:~ 1 >
Consequently, the total capacitance of the system is
C=C2+ c~~a::.:.~1)'
C=t~+ 2xt~~~1ix'2t~~!o-t) F=t.62µF.
42i. A Leyden jar of capacitance 3.3 nF is charged to
a potential difference of 20 kV. Assuming that t0% of the
jar energy is dissipated during a discharge in the form of
acoustic and electromagnetic waves, determine the
amount of heat liberated during the discharge.
Given: C=3.3nF=3.3xllr'F, U=20kV=2xtO'V,
W'=O.IW.
Q-1
Solution. The electric energy of the charged Leyden jar
is
W=Cll'/2. (1)
The energy W'"""'" 0.t W is lost by radiation. Consequent-
ly, the amount of liberated heat is
Q=W-0.IW =0.9W,
or, taking into account Eq. (t),
Q=0.9 e~·,
Q 0.9x3.3xt~•x(2xt0') 1 J ~ O.G J.
422. A capacitor of capacitance t mF at a voltage of
1200 V is used for a shot butt welding of copper wire.
Determine the mean useful power of a discharge if its du-
ration is to-• s. The efficiency of the set.-up is 4%.
Given: C=lmF=t()-•F, U=1.2x!O>V, t=llr's,
~=0.04.
(N.)-1
Select.ed Prohlems on Physies

Solution. The mean power of the discharge is


(N)=Alt,
where A= .àW = W1a - Wr 1n = CU'/2 - O =CU'/2.
Consequently,
(N) = CU'/(21).
The mean useful power is
(N,)=~(N)-~ ci• '
(N11)= O.Mxto;x~~2xtCJl)I W=28.8 MW.

423. A metal sphere of radius 3 cm bears a charge of


2 X 10- 1 C. The sphere is immersed in kerosene so that it
does not touch the vessel walls. Determine the volume
energy density of the field at points lying at 2 an~ 4 cm
from the centre of the sphere.
Given: R=3cm=-3x10-2m, q=2x10-1 C,
r 1 =2X10-z m, r 2 =4X 1()-2 m, e-2.
W1-? wt-?
Solution. The volume energy density is defined as
W1=2aÚ::/2.
A point lying at a distance r1 from the centre of the
charged sphere is inside the sphere where the electric
lield strength E 1 is zero. Therefore, Wi = O at this point.
For a point separated by r11 from the centre of the sphere,
the volume energy density is
w,= e,eE:i'2,
where E 1 is the electric lield strength at this point. Since
E 11 = q/(4n8oer:), we have

Wz= 'f { 4n~ )2 = 32n::.1rf '


(2xtO-•J•
w11 32x3.M•xs.ssxtõ""11 x2x(4xtO-SJ' m•
~ 0.056 J/m3 •
Ch. 3. Electricity 267

424. The plates of a parallel-plate air capacitor are


at.tracted t.o each ot.her with a force F. Will t.his force
chaDge if BD insulator plat.e is iDt.roduced iDto t.he air gap
bet.weeD t.he capacitor plates?
Answer. IDt.roducing tbe plate made of insulator, we
reduce t.he elect.ric iield st.reDgth iD t.be space occupied by
t.he insulator but. do not. change the 6.eld st.rengt.h in the
gaps betweeD t.he insulator aDd t.he capacitor plates.
Tberefore, the attractive force F betweeD t.he plat.es
remains unchanged.

EXERCISES

425. A capacitor consist.s of t.bree tin foils of area


to cm" each, separated by mica layers of t.hickness 0.5 mm.
Tbe ext.reme foils are connected to each other. Determine
t.he capacitance of t.he capacitor.
426. AD air capacitor consista of two circular plates of
radius tO cm. The separat.ion between the plates is t cm
and the poteDtial diBereDce is 120 V. Determine t.he
charge on t.he capa.citor.
427. A capacitor charged to a volt.age of 100 V is con-
~::!~di~p:r:!~~: !tc2&ª~~~~a~h::fi~:ec:,i:i1c~~:ec:;
across the plates?
428. Three capacitors of capacitance 2, 4, and 6 pF
are coDnect.ed in parallel and to a source of volt.age of
t kV. Determine the charges OD the capacitors.
429. A bank consist.ing of t.wo series-connected Leyden
jars of capacitance 300 and 500 pF is charged to a voltage
of 12 kV. Determine the volt.age and the charge on the
plates of the 6.rst. aDd secoDd jars.
430. A sphere of radius 25 cm is charged t.o a poteDtial
of 600 V. Wbat. amount. of heat. will be liberated in
a conductor coDnectiDg the sphere to the ground?
431. Det.ermine t.he volume energy densit.y of t.he elec-
t.rostatic iield at. a point located at 2 cm from the surface of
a charged sphere of radius t cm. The surface charge deD-
sity of t.he spbere is 16.5 p.C'Jm1 and the permittivit.y
of t.he medium is 2.
... Selected Prohlem1 on Physics

QUESTIONS FOR REVISION

...
between t.hectír streDRth an potentiál dilference for a uniform
1electrostatic field? to. Define capacitance. tt. Write formulas for
calculatiDg the capacitance of a sphere and a parallel-plate capac-
ilor. t2. Calculate the capacitance or a capacitor bank for series
and parallel connection of c:C:citon in it. t3. Write formulas for
::c!:ul~i~t.h:ofum~ ~~ d::!i;ª~fi':: 1:iec~:!:taªt~cexr;:.
15. Name t.he uni ta of charge, electric &eld strength, and potent.ial.

U. Dlrect Cunent
CURRENT IN METALS
Current in metal!!! is due to tbe motion of free electrona
in a conductor. The amount ofelectricity (charge) q pass--
ing tbrough the cross-sectional area of the conductor per
seeond is called the eurrent:
l=qlt.
The eurl'f'D.t denslly is defined by
i=llS,
where S is the cross-aectional area of a conductor, normal
to the direction of the current.
Obm'a law for a eonductor has the form
l=UIR,
where U is the voltage acrOl!ll!I the conductor and R the
re&istanee of the eooduetor.
For a bomogeoeoua cooductor,
R=pllS,
where p is the resfstlvfty of tbe eondueting material, l
the cooductor length, and S the croas-sectional area of the
cooductor.
Ch. a Electricity 269

The temperature dependence of the resistivity of a con-


ducting material is given by
P= p, (! + czb.T),
where p0 is the resistivity of the material at 273 K, <i the
temperature resistance coefficient, and AT= T - T0
is the change in the temperature of the conductor.
For series-connected conductors, the total resistance is
equal to the sum of the resistances of individual conduc-
tors:

R= i: R
t=I
1•

For parallel-connected conductors, the reciprocai of the


total resistance is equal to the sum of the reci procals of
the resistances of individual conductors:

-Jr=i,_, *·
where R 1 is the resistance of ao individual conductor and
n the number of conductors in a given subcircuit.
Ohm's law for a closed clreult is
l=ti(R + r),
where 1 is the electromotive force (emf) of the current
source, R the resistance of the externai subcircuit, and r
the resistance of the internai subcircuit.
The voltage U acrosa a subcircuit containing ao emf
source is
U=l±IR1,
where Rt is the total resistance of the subcircuit.. The
minus sign is taken when the current in the source is
directed from t.he negative to the positive pole (Fig. 135a),
while the plus sign is taken when the current in the source
is directed from the positive to the negative pole
(Fig. 135b).
When several sources are connected in series, their total
emf and the total internai resistance can be determined
270 Selectad Problema ou Physlcs

from tbe following relations:

I= ~
lo=zt
1 1, r= ~
l-t
r1•
When tbe sourcu are connected in parallel, these
quantities can be found from the relation

f=h~.
·-·
where 1 1 is the emf of an individual source and r 1 is the
internai resistance of tbe source.
For calculating branched circuita, Kirchhofl's laws
sbould be used..

,, t,,r -
~u
-~
,j
(b)
Fig. 135
Ktrcbboff's Hrst law. The algebrak sum o/ the currents
convertlnt at a junctton ls uro:

i; I 1 =0,

·-·
where n ts tha nu.mber o/ currentl.
The directions of correnta are chosen arbitrarily so tbat
the algebraic sum contains both positive and negative
currents. The current arriving at a junction is usually
assumed to be positive and tbat Bowing away from the
junction is assumed to be negative.
lf a circuit diagram contains N junctions, using Kirch-
hoff's first law, we can write N -1 independent equations.
Kirebhofl's second law. /n any arbttrartly chosen clmed
contour o/ a branched ctrcuit, the algebraic mm o/ the voltage
drops ai:ross tndtvldual ekments o/ the contour ts equal to
tha algebraic sumo/ tha emf's encountered durtng tha eircum-
Ch. 3. Elecll'lclt.7 271

vention o/ tke contour:


i;
'""."t
11R 1 =iJ 18
t=t
1•

The direction of circumvention of tbe contour is chosen


J!Jl1k~i::a:~~d°i'!~~:~;!~eif 1tbet~i~e~~:~ ~; t1eº~1:~
rent / 1 coincides witb tbe chosen direction of circumven-
tion. An emf is assumed to be positive if it incresses tbe
potential in the direction of circumvention of tbe contour.
According to Kirchhoff's second law, we can write n -
(N - 1) equations, where n is the number of unknown
currents and N tbe number of junctions in the circuit
diagram.

432. Determine the resistance between points A and D


(Fig. 136a) if each of the three resistances is 1 '2 (the
resistance of the leads should be neglected).
Given: R=i '2, n=3.
RAD-1
SolutWn. Since points A and C1 as well as points B
and D, are connected by conductors whose resistance is

~ili
A_____!______C ~
(a) (b)
Fig. 136

disregarded, diagram (a) can be replaced by an equivalent


diagram (b). It can be seen that the resistance between
points A and D can be calculated from the formula for
parallel connection of conductors:
llRAD =!IR, + llR, + llR, = n/R,
whence
RAD =Rln,
RAD = 1/3 Q "" 0.33 Q.
272 Selected Problema on Physics

433. Determine the total resistance of the electric cir-


cuit (Fig. 137) if the internai resistance of the source is
1 g and the resistances of other resistors are 4, 3, 12, and
6 g respectively.
Gtven: r=10, R1 =4Q, R 11 ~3Q, R11 =120,
R,=6!l.
R-1
Solutton. ln the circuit diagram (Fig. t37a), the resis-
tors R1 and R 1 are connected ín parallel, and hence the
diagram can be drawn in a different form by making
points A and B coincide {Fig. 137b). lt is now easier to
calculate lhe total resistance of the circ.uit. Denoting tbe
total resistance of lhe resistors R 1 and R 11 by R', we can
write
llR' =!IR,+ llR,,
wbence
R' = R,R,l(R, + R,).
The resistors R' and R, are connected in series. Denoting
their total resistance by R •, we can write
R"=R'+ R,=R,R,t(R, +R,) + R,. (!)
We can now determine the total resistance R"' of lhe
parallel subcircuits with resistances /(' and R,.: 1/R"' =

R,
B

(a} lb}
Fig. 137
Ch. 3. Electric.ity 273

l/R"+llR,, or, taking into account Eq. (!), ;. =

R1RJ(R1~R1)+R. + !, ' whence


Ir R
R, ~·A:R:+~:~:+~:t.~R.R1
1 • (2)
Finally, the total resistance of the entire cin;uit is R =
r + R "', or, taking into account Eq. (2),

R=r+ R1R,~·A:~:+~:~:+°j,:R:tR1R1
R=t+ 4xe!~J~~~t;~~~~~~~xu '2=4 n.
434. Determine the current obtained from a battery of
emf 6 V if the resistances of different resistors are 2, 6, 3,

Fig. 138

and 1.5 g respectively. The internai resistance of the


battery should be neglected (Fig. 138).
Given: 1-6 V, R1 =2 '2, R1 =6 '2, R3 =3 '2,
R,-1.S!l.
1-1
Solution. The resistorsR1 and R 1 are connected in paral-
lel. Denoting their total resistance by R, we obtain
llR = llR1 + llR,,
whence
R = R,R,l(R, + R,). (1)
l~-0970
Select.ad Problema on Phyeics

The subcircuit ABC is connected in serits with t.he resistor


R,. Consequently, the total resistance of the subcircuit
is R' = R + R,, or, taking into account Eq. (1),

R' = R~':ti. + R, = R1R1~:+<:: +R.> • (2)


The subcircuit ABCD is connected in parallel with the
resistor R 1 • Denoting their total resistance by R •, we
determine it from the condition 1tR• = 1/R' + 1/R1 , or,
taking into account Eq. (2). ·~ 81 R.:R~::+R.>+k •
whence
(3)

The current consumed from the battery can be determined


from Ohm's Iaw for a closed circuit:
l=WIR",
or, taking into account Eq. (3),
l{R1Ra+R1R,+R 1 R1+R,R1+R,RJ
R1{R1R1+R1R,+R,R,)
6X{2X33:c:~:t:~:t~:~~t)t.5x6) A=4 A.
Thie problem can be aolved by ueing KirohhoB'e lawa. We
leave it t.o the reader t.o verify ih.is.
435º. What must be a direct current for whieh a charge
of 50 C passes through the cross section of a conductor
during the time interval from 5 to 10 s after the moment
of switching OD tbe current? What charge wm pass
through the cross section of the conductor during the
same time if the current in the conductor changes with
time according to the law I = 6 + 3t?
Gtven: âq=50 C, t 1 =5 s, t1 =10 s.

1-1 q,-1
Solution. For a direct current, we can write
l=IJ.q,/41,
Ch. 3. Eleciric.ity

where At = t 1 - t1 • Then
1= 10~ 5 A=IOA.
If the current ehanges witb time, the charge passing
through the cross section of a conductor during this time
interval is

Qz= r r
t1
1 dt-
••
(6+3t)dt= r
t1
6dt+ ~· 3tdt
t1

= (6t+-y- "'li'' 11= 6 t.+2t·-


3 • 6t1-2t1'
3 •

q,=( 6x 10+-} x rn•-6 xs-f x s•) C=142.5 e.


436. A eurrent passes through a conduetor having
a cross-sectional area of 50 mm1 • The mean drift veloeity

Fig. f39

of free electrons is 0.282 mm/s and the electron number


density is 7.9 X 1()17 m-1 • Determine the current and the
current density in the conductor.
Glven: S = 50 mm1 = 5 x 10-1 m•, (ri)= 0.282 mm/s-
2.82 x 10-' m/s, n = 7 ,9 x 1027 m-a.
1-1 J-1
Solution. We isolate in the conductor a region of length
.àl and of cross-seetional area S (Fig. 139). The charge
passing through the cross section of the conductor during
the time interval At is
6q= .. 6V,
where n is lhe number density of free electrons, e lhe elec--

...
tron charge, .à V= S âl the volume of the isolated regf.on
276 Seleeted Problema on Physics

of the conductor. By definition, 1 = l!ql .ó.t, or, using the


e:r.pressions for !J.q and .ó. V, we have
1- neS l!Ul!.t. (1)
Noting that, by de6.nition, !J.l/!J.t = (v) is the mean
drift veloeity of free electrons, we can write Eq. (t) in
the form
l=neS(v),
/-7.9xl0"Xl.6XIO-"x5x~x2.82XIO""'A"'l7.8A.
Then the current density is

1-f,
1= 5 !71~• ! 1 =3.156x tO'A/mª.
437. A circuit having a resistance of tOO O is fed by
a d.e. source. An ammeter having an internai resistance of
t g and conneeted to the circuit indicated 5 A. What was
the current in the circuit before the connection of the
ammeter?
Giwn: R-100 !l, R,-1 !l, /-5 A.
lo-?
Solutton. Before the connection of the ammeter, the CUl'-
rent in a suhcircuit was, according to Ohm's law,
10 -UIR. (1)
After the connection of the ammeter, the current hecomes
1- Ul(R + R 0). (2)
Solving Eqs. (t) and (2) together, we obtain
lo= R1,Ro 1,

1,- (100i.i1X5 A-5.05 A.


438. A cell having an emf of 2.t V .and an internai
resistmce of 0.2 g is connected to a potentiometer.
Ch. 3. Elec\rieity 277

Dete!'mine the current in the circuit and the resistance


of the potentiometer if the voltage 8Cl'OSS the cell te!'mi-
nals is 2 V. What must be the length of the iron wire

ô Fig. 140
u.sed for preparing the potentiometer if its crosa-sectional
area is 0.75 mm1 ?
Giwn: ll=2.! V, r=0.2!l, U=2V, S=0.75mm'=
7.5 X f(r 7 mZ.
1-1 R-1 l-1
SoluUon. According to Ohm's law for a cloaed circuit,
the current is
l=lli(R+r). (!)
On the other band, according to Ohm's law for a conduc-
tor, viz. potentiometer (Fig. t40), the same current is
given by
l=UIR. (2)
Solving Eqs. (!) and (2) together, we obtain
R= ,.U:...u
R= 2 ~.~.2 !l-4!l,

I=*·
l=fA=0.5 A.
Since R= pUS, the length of the iron wire is

!=a:'
l= 4 ~.~-!~~' m=25 m.
278 Selecl.ed Problema on Ph)'lica

439. A galvanic cell of an emf 1.5 V and an internai


resistance of 1 Q is eonnected to an externai resistanee of
4 Q. Determine the current in the eireuit, the voltage drop
in the interna! nbcircuit, and the voltage across tbe
terminais of the cell.
Given: lf=t.5 V, r=1 !l, R=4 !l.
1-1 u,-1 u,-1
Solution. Tbe current can be determined from Obm's
law for a closed circuit: ·

I= R"+.r'
1= :.:, A=0.3 A.
The voltage drop in the internai subcircuit is
U1 =1r,
U,=0.3X1 V =0.3V.
The voltage acrosa the terminais of the cell is smaller
tban the emf by the voltage drop in the internai subcir-
cuit. Therefore,
U 1 =M-/r,
U1 = (1.5 - 0.3) V= t.2 V.
44.0. Tbe internai reaistance r of a cell is smaller by
a factor of k than tbe externai resistance R by wbieh the
cell of emf 1 is closed. Determine the ratio of tbe volt-
age U across the terminais of tbe cell to the emf M.
Giuen: Rlr=k, li.
Ullf-1
Solutlon. Aceording to Ohm's law for a closed circui.t,
1 = rl(R + r). (1)
Since tbe voltage across the terminais of the cell is smaller
than the emf by the magnitude of tbe voltage drop in the
internai subcircuit, U = 1 - Ir, or, taking into
COUD• Eq. (1),

U=l- a";., =li R~' = ,.:,IR . (2)


Ch. 3. Elect.rici\y 279

By hypothesis, Rir= k, or rlR = llk. Theo Eq. (2) can


be reduced to the form U = 111(1 +
l/k) = klll(k 1), +
whence
Uill = k/(k + 1).

witb unity, and the value of


the voltage approachea the
value of the emf.
.Ut. A potentiometer made
Q
Equation (2) shows that if t.he externai resistance is large
in com parison wi\h the internai resistance, the ratio ri R
becomes small in comparison 1

+1
-
of an iron wire, a milliam-
meter, and an emf source are inA
~::0 ::!.i:c:er~;sih~t p~~~: Fig. i4t
tiometer is 200 Q and the re-
sistance of tbe milliammeter is 20 Q. The milliammeter
indicatea 30 mA. What will be tbe reading of the milli-
ammeter when the potentiometer is heated to 50 ºC? The
intemal resistance of the source should be neglected
(Fig. 141).
GhJen: T0 -273 K, R0 =200 Q, R.=20 Q,
/ 0 =30mA=3X10'<' A, T=323 K.
1-1
Solutton. According to Ohm's law for a closed circuit,
before the heating of the potentiometer, tbe current waa
I 0 - lll(R 0 + R,). (1)
After the heating, the current becomes
I = l/(R + R,). (2)
Here R = R 0 (1 + ci AT), AT,,,. T- T0 , wbere ci is the
temperature resistance coefficient of the iron. The change
in the resistance of the milliammeter is neglected. Solving
Eqs. (1) and (2) together, we obtain

I R1 (t~!~;.+~~J+Ra 1

/= 200x11!~~~~~~':"..t~~?a)l+20 Ao. 2.36X10'<' A.


280 Selecled Problema ou Physic1

442. Calculate the emf and the internai resistance of


a batt.ery consisting of tbree emf sources (Fig. 142) if the
emf's of the sources are 10, 20, and 30 V and tbeir internai
reaistances are 1 Q each.
Given: 1 1 =10 V, 1 1 =20 V, 1 1 =30 V, r 1 - r2 =
r3=r=1 n.
r-1 r-1
Solution. The emf sources are connected in tbe c.ireuit
in series and in parallel. If we assume tbat tbe emf's 1 1
ll,.r

A~~1~D
e,. r
Fig. 1.f.2

and 1 1 are positive, t.he emf 1 1 will be negative. Let. u.s


determine tbe total resistance r' and the total emf for
tbe subcircuit BC consisting of parallel-connected
sources 1 1 and 1 1 :
t/r' = 1/r1 + 1/r1 = 2/r,
r'lr'='l1/r1 + l 1/r1 = (W, + W,)lr,
whence
.'
r=2•
,. = ,.. cs~+•.> = •1t•· .
Let. u.s now calculate the total emf 1 and the total resist.-
ance r of the ent.ire bat.tery connected bet.ween points A
andD:
l=i'+la= • 1 t•• +1 1,

l= -l~+20 +30 V=35 V,

r=r' -l-r1 =f + r1 =f +r=-} r,


r=l.5 !l.
Ch. 3. Electricity 281

443. Determine tbe emf and tbe intemai resistanee-


of a current source shunted by a resistance of 6 g if th&
emf of tbe souree without a shunt is t2 V and its interna!
resistance is 3 e.
Gtven: R=6 ti, 11'=12 V, r=3 !l.
W'-1 r'-1
Solutton. Considering tbe shunt as a source whose in-
ternai resistance is R and the emf is zero, we obtain
a battery consisting of two parallei-connected sources.
Tben the internai resistance F and the emf I' of sucb
a battery can he found from tbe relations
1/r' = llr + l/R, 11'/r'= 11/r + 0/R,
wbence F = rRl(r + R) and B' = Fl/r. This gives
r'=w ll=2 !l, 16'= 2 ~ 12 V=8 V.
4.4.4. A voltmeter with an internai resistance of 2500 Q
indicates a voitage of 125 V in the circuit. What is the se-
ries resistance to be connected to tbe voitmeter so that it

à
indicates 100 V (Fig. 143)1
s
Fill'. 143

Gtven: R=2.Sx10' ti, U,=125 V, U,=100 V.


R,-?
Solution. Tbe voltage across the series resistance is
U1 - U,. Using Obm's law for an element of the circuit,
we can write
R,= (U, - U,)/l. (1~
... Select.ed Problema on Phyaica

With a series connection, t.he current in dilerent. elements


of a circuit is the same. Therefore, knowing the internai
resistance R of the volt.meter and the voltage drop U2
across it, we have from Ohm's law for t.his element. of the
circuit.
1- U,IR. (2)
Subat.it.uting expression (2) into (1), we 6.nd that .
R _ (U,-UJR '
.- u. •
R,- (125-tooi~2.5XtOI '2=G2S !J.

4.45. A milliammeter is intended for measuring cur-


renta below 10 mA. What should he done to use this

Fig. 144

i119trument for measuring current. up to t A if its internai


resistance is 9.9 Q?
Gtu•n: 1,-1omA-10"'A, R,-9.9!l, /-1 A.
R,.-1
Solution. The milliammeter is connected in series to
a circuit. lf tbe current 1 in it is st.ronger t.han the mui-
mum rated value 10 of tbe inst.rument, a resistor R,h
known as a sbunt should be connected in panllel to it
(Fig. 14.4). It is chosen so that the current through tbe
instrument is below 1 0 •
Applying Kirchhol's 6.rst law to the junction A, we
obtain
1-10 -1,b -O. (1)
Choosing the direction of circumvent.ion of t.he closed
.cont.our ABCDA clockwise and applying Kirchhofl's second
Ch. 3. ElecVloUy 283

law to the contour, we get


I ,11 0 - I, 0R, 0 = O. (2)
Solving Eqs. (1) and (2) together, we obtain

R1h= f~i, ,
R,h= 1 i..:.~:~9 '1=0.t '1.
446. Four resistors of t kQ each and two current
sources of emf t .5 and t .8 V are conneeted to form an

Fig. 145
electric circuit. Determine the current in all the resistors,
neglecting the internai resistances of the sources
(Fig. 145).
Giuen: R=I k!l=IO' l:l, i'1 =1.5 V, i'1 =1.8 V.
111 1.,,, 1,, 1,-?
Solution. It can be aeen from the figure that the resistor
R, is sho~ircuited by the conduetor .ABC wbose resist.-
anee is neglected. CoDSequently, the current does not
Bow through R,, i.e. 1, =O. The remaining part of the
circuit eontains two junctions at. points O and B and t.bree
closed contoun. Consequently, we ean write one equation
by using Kirchhoff's first law and two equations by using
Kirchhoff's second Iaw. Applying Kirehhoff's first law to
Selected Problema on Physica

the junction B, we 6.nd that


l, + l, - l, - o.
Applying Kirchhol's second law to the contour AOBA
(circumvented counterclockwise), we obtain
l,R + 11R - lf1•
Similarly, applying Kirchhol's second law to the con-
tour OCBO (circumvented counterclockwise), we get
l,R +1,11-11,.
Sol ving the obtained system of equations, we determine
the required correnta:
la= '11:-a.•• '
11 = 1 ã5x~~~ A= 1.1 mA,
lz= 1,-~+l.R '
lz L8-L5t!·:;to-axtCI' A=0.7 mA,
lt=l,-1,,
11 -(I.! X llr'-0.7 X !lr')A-0.4 mA.
447. Two cells of emf 1.9 and 1.1 V having internai
resistances of 0.8 and 0.1 gare connected in parallel to an
externai resistance of 10 Q (Fig. 146). Determine the cur-
rent in the externai circuit.
Gtvtn: •1=1.9 V, 1'2=1.1 V, r.=o.s Q, Ta=0.1 e.
R-IO!l.
1-1
Solution. The electric circuit contains two junctions
at poinbl K and C and three unknown currenbl. Con&&-
quently, we can writ.e one equation by using KirchhoJl's
first law and two equations by using Kirchhoff's second
law. Applying Kirchhoft's 6.rst law to the junction K,
we obtain
l,+1,-1-0.
Cb. 3. Elect.ricity

Applying Kirchhoff's second law to the contour ABDKA


(circumvented counterclockwise), we find that
/ 1r 1 +IR = i 1•

Similarly, applying Kirchbofl's second law to the contour


ABCKA (circumvented counterclockwise), we ohtain
/ 1r 1 - Ira= 1 1 - 1 1•
Solving the ohtained system of equations, we get
l1r1+Wt"1
R(r1+r.>+r1r1'

to~-~o~s~~i~~0.~:0.1 A~ 0 · 12 A.
448. The resistances of elements A B, BC, and AD of
a circuit are 1000, 500, and 200 Q respectively. Agalvan-
ic cell whose poles are connected to points A and C
has an emf of 1.8 V. A galvanometer detects a current of
0.5 mA in the direction indicated hy the arrow. Determine
the emf of the other galvanic cell, neglecting the internai
resistances of the cells and of tbe galvanometer (Fig. 147).
Given: R1 =10SQ, R1 =5x1029, R3 -2x1020,
lf,=1.8 V, I,=0.5mA=5x1()-' A.
16,-1
Solution. The electric circuit has two junctions at
pointsA and B and three unknown currents. Consequently,
we can write one equation by using Kirchhofl's first law
E,

:1
I)
{i f R
Fig. 146
D
Fig. 147
288 Selected. Problema on Phy1lc1

and two equat.iom by using Kirchhofl'ssecond law. Apply-


ing Kirchhoff's fi.nt law to the junction B, ~ obtain
l1-l1-l1=0.
Applying KirchhoB's second law to the contour ABCB1A
(cireumvented counterclockwise), we fi.nd that
/ 1R 1 + / 8R 8 = 1 1 •
Similarly, applying Kirehhofl'ssecond law to the con\our
ABB.DA (circumvented cloekwiae), we obtain
/ 1R 1 +/ R
8 1 = 1 1•
Solving the obtained system of equations, we get
12.= W1R1+I,(RR~+~:1R 1 +R1 R,) ,
I,=
t.s x to•+5 x to-t x ctoaxsx1o•+tO'x2xtos+sx1o•x2xto•)
t0'+5xfos
=1.47 V.
469. A wire and a sliding contact are connected to
point A of a homogeneous wire ring and to a diametrieally
opposite point B respectively. What will be the change
in the readings of a voltmeter during the motion of the
sliding contact (Fig. t48)?
re:;:d;sT~;:::Ji!i:~!!!~7:!ºof~:~:nd:c~~ft~:
same resistance. Whe• the.sliding eontact moves upwards
or downwards, the resistanee of one of these eonduetors

Fig. t48 Fie. t49


Ch. 3. Electricity 287

will decrease. This leads to a decrease in the resistance of


the subcircuit AB since the resistance R of a subcireuit
consisting of two parallel-connected conductors of resist-
ances R 1 and R 1 is always sm.aller than the resistance Ri
if Ri< R,,. Therefore, the voltage drop acroas AB, and
hence the readings of the voltmeter, will decrease.
450. Why is the voltage across the terminais of a cur-
rent souree close to zero during short-circuiting, although
the current in the circuit has the maximum value?
Answer. During short-circuiting, the resistance of the
externai subcircuit is small in comparison with the
internai resistance of t.he source. Therefore, although t.he
current in the circuit is large, the voltage drop in the ex-
ternai subcircuit, which is equal to the voltage across the
terminais of the source, is sm.all.

EXERC!SES
451. Calculate the total resi111tance of a subcircuit if the
resistance of each side and a diagonal of a square is 8 Q.
The resistance of the leads should be neglected (Fig. 149).
452. Eight conductors of resistance 20 Q each are con-
nected pairwise into four parallel branches. Determine
the total resistance of the circuit.
453. A copper and an iron wires of the same length are
connected in parallel to form a circuit. The diameter of
the iron wire is twice that of the copper wire. The current
in the copper wire is 60 mA. What is the current in the
iron wire?
454. The resistance of the filament of a vacuum tube is
40 Q, the resistance of the engaged part of a potentiom-
eter is 20 O, and the current in the circuit is 0.2 A.
When the same cell is connected to a resistance of 10 O,
the current is 0.1 A. Determine the emf of the cell and
its internai resistance.
455. When a cell is connected to a resistance of 4.5 Q,
the current in the circuit is 0.2 A. When the same cell is
connected to a resistance of 10 Q, the current is 0.1 A.
Determine the emf of the cell and its internai resistance.
456. A cell is fi.rst connected to an externai resistance
of 5 Q and produces a current of 0.25 A. Then it is connect-
288 Selected Problema on Physics

!dc:re~~ :;}Lliª~~:::::iTn:f~ ~~rr~~~c~hr:~::u~e:


cell during short-eircuiting.
457. Two cells of emf's 1.8 and 2 V and internai resist-
ances 0.3 and 0.2 g are connected in a battery so that
a current of 4 A flows in an externai circuit of resistance
-0.2 '1. How are the celb connected?
458. Six cells having an emf of 1.5 V and an internai
resistance of 0.4 '1 each are connected in a hattery so
that a current of 6 A flows in an externai circuitofresist-
ance 0.2 '1. How are the cells connected?
459. The resistance of a voltmeter is 200 '1. A conductor
of resistance 1 kC is connected in series to it. By what
factor is tbe value of the division of the instrument
changed?
460. Determine the resistance of a shunt of a galvanom-
eter rated for 1 A if the internai resistance of the galva-
nometer is 20 '1 and the full scale corresponcb to a current
of 5 mA.
461. Determine the change in the temperature of a cop-
per conductor whose resistance has increased twofold.
462. The externai resistance of a circuit is 1.4 C, the
emf's of the sources are 2 V each, and their internai resist-
ances are t and t .5 C. Determine the current.s in each
source and in the entire circuit (Fig. 150).
463. Determine the current indicated by an ammeter
if the voltage across the terminais of a source is 2.t V
and the resistances are 5.6 and 3 C. The resistance of the
ammeter and the internai resistance of the source should
be neglected (Fig. 151).
464. Determine the current through a resistor R 1 and
the voltage drop in it if the resistances of the circuit

Fig. 150 Fig. 151


Cb. 3. Electrlcity

Fig. 152

elements are R 1 = R 1 = 40 í.l:, R 1 = 80 O, and R" =


34 O, and the emf of the generator is 100 V. The internai
resistance of the generator should be neglected (Fig. 152).

WORK AND POWER OF CURRENT.


THEllMAL EFFECT OF CURRENT
The work done by an electric field in moving a charge
q over a subcircuit is
A - qU - IUt,
where I is the current in the subcircuit, U the voltage
across the subcircuit, and t the time during which the
current Oows in it.
According to the energy conservation law,
A -6W,
where AW is the change in the energy of a conductor and
surrounding bodies.
The power developed by a current passing through
a conductor of resistance R is given by
N - 6Wlt - Alt - IU - J'R.
The amount of heat liberated upon the passage of a cur-
rent through a conductor is determined by J'oule's law:
Q-l'Rt.
The efficieney is the quantity defined as
11 = !: 100%, 1) = :: 100%, 1) = z~ t00%.
19-0970
Selected Problema on Physics

where A u (W u• Nu) is the usefui work (energy, powerjand


Ad (W,, N1) the work done (spent energy, liberated
power).

465. Determine the emf and tbe internai resistance of


an accumuiator suppiying a power of 9.5 W to ao externai
surface ata current of 5 A and 14.4 W at 8 A.
Given: 11 =5A, N 1 =9.5W, lz=BA, Nz=14.4W.
r-? r-?
Solution. According to Ohm 's law for a ciosed circuit,
/ 1 = ll(R, + r), I, = l/(R, + r). (!)
The resistances R 1 and R 1 can be determined from the
reiations N 1 = 1:R 1 and N 1 = I:R 1 , wbence
R 1 = N 111:. R 1 = N,11:. (2)
Substituting expressions (2) into (t), we obtain
I,=Bi(N,ll:+r), 12 =1'/(N,IP,+r). (3)
Soiving Eqs. (3) together, we find that

1= ~}~~J.~~:l , r= ~f.·,-;.~~:l .
Then
# o.ss~~~~"~s~s·v~2.1v,
9 ·:~:~;:·'~ 5 0!:!::0.03 o.
466. ln an electric circuit, the same power is developed
at externai resistances of 2 and 0.1 Q. Determine the
internai resistance of the source.
Gi1.1en: R 1 =2 Q, R2 =0.t O, N 1 =Nz.
r-1
Solution. According to Ohm's law for a closed circuit,
for two values of tbe externai resistance we have
/ 1 = f:l(R 1 + r), I, = l/(R, + r),
(!)
lt = / 1 (R, + r), lt = I, (R, + r).
Ch. 8. Electrieif;J

The current is connected to power through the relation


N = PR, whence 1 = ViiTR. Subatituting / 1 = JfNili;
and / 1 = 'VNIR 1 into Eqs. (1), we obtain
l=]INT.if,(R,+r), 1-VN!R,(R,+r). (2)
Equating the right-handsides of expressions (2) and trans-
form.ing the obtained equality, we can determine r:
]INT.if, (R,+r) = VNiR,(R1 +r),
VR; (R, + r) = VR, (R, + r),
VR;R,+ VR;r=VJl,R,+ VR,r,
r(VR,-VR.l=Vif.R,-Vif,R,
=VR,R,(Vif,-VR,),
1'=~.
r=V2x0.I ll<><0.45 ll.
4.67. A battery is formed by a few parallel-eonnected
cells. When the current in an externai circuit is 2 A, tbe
useful power is 7 W. Determine the number of cells in
the battery if the em! oi a cell is 5.5 V and tha internei
resistance is 5 Q.
Gi1J1m: 1=2 A, N-7 W, 1=5.5 V, r=5 Q.
•-1
Solutlon. The useful power developed in the resiBt-
ance R i8
N=l'R. {!)
According to Ohm 's law for a closed circuit, we can
write for the current J = lb/(R + rb), whence R =
(lb-/rb)//, where lb = 1 is the emf of the battery
consisting of n identical panllel-eonnected cells, and
rb = rln is the internai resistance of the battery. Substi-
tuting the expressions for rb, 1 b• and R into Eq. (1), we
ob\aiD. N = J (1 - Jrln), whence

'"
n= Sl-N '
n= 5.~IXX25_7 =5 •
...
Select.ed Prohlem.1 on Physice

468. A cell having an internai resistance of 4 Q and an


emf of 12 V is closed by a conductor of resistance 8 Q.
What amount of heat will be liberated in the u:ternal
circuit per second?
Gtven: r=4!l, l=l2V, R=B!l, 1=1 s.
Qlt-?
Solutlon. The amount of heat liberated in the externai
circuit per second is
Qlt =l'IR. (1)

According to Obm's law for a closed circuit, we have


l=Sl(R+r). (2)
Substituting expression (2) into (1), we obtain
Q \)IR
T= <R+r)• 1

f= (~':;.~)! f = 8 J/s.
469. A kettle filled with 1 1 of water at 16 "C is put on
a bot plate of power 0.5 kW. The water in the kettle boils
in 20 min after switching on the hot plate. What amount
of beat is lost for heating the kettle and for radiation?
Gtven: N=5xtOZW, V=t l=t0-3m3, T1 =289K,
t 1 =20min=1.2X1()8 s.
W'-?
Solution. The total energy spent for heating water and
the kettle and for radiation is
W,=Nt.
Tbe useful energy required to heat water is
W 11 = !J.U =cm (T 1 - T1) 0

Here !J.U is the change in the internai energy of water as


a result of heating, e the specifi.c heat for water, and
m = pV is the mass of water, where p is its density.
Then the energy losses for heating the kettle, for radiation,
Ch. 3. Eleclrieit.y 293

etc. are
W' - W,- W, -Nt-cpV(T, -T,),
W' - [5 X 10' X 1.2 X 10' - 4.19 X 10'
X 101 X 10-1 X (373-289)] 1 <>< 250 k1.
470. A homogeneous iron conductor of lengtb too m
is connected to a d.e. source of too V for tos. What will be
the cbange in the conductor temperature? The change in
the conductor resistance upon heating sbould be neglected.
Given: l=10Z m, U=t0z V, t=10 s.
IJ.T-1
Solution. The amount of heat required for beating the
iron conductor is
Q1 - IJ.U - cm IJ.T. (1)
Here m = DSl is tbe conductor mus, S the cross-section-
al area, e tbe specific heat for iron, and D the density
of iron.
According to Joule's law, the total amount of beat
liberated in the conductor is
Q, - U't!R. (2)
Here R = pl/S, where p is the resistivity of iron. Neglect-
ing heat losses, we can assume that Q1 = Q2 , or, taking
into account expressions (t) and (2), cDSl AT =
U'Stl(pl), wbence
flT= cu:i~,
AT = 0.46 X 108 X 7 .8 ~o;~~ ~~OI)• X 1.2 X 10-'. K ~ 23 •3 K.
471. Determine the resistance of the leads to a source of
voltage 120 V if fuses made of a lead wire, having a cross-
sectional area of t mm.2 and a lengtb of 2 cm, melt during
short-circuiting in 0.03 s. The initial temperature of
a fuse is 27 "C.
Gtven: U=120 V, S:st mmt=to-8 mz, l=2 cm=
2xl0-' m, t-3xlo-> s, T-300 K.
R-1
294 Selected Problema ou Phyaica

Solution. The amount of heat required for heating the


lead to lhe melting point and it.s fusion at this t.em.pera-
ture is
Q, = l!.U, + l!.U,.
Since AU1 =cm AT, AU 2 = Am, m = DlS, and AT =
Tm - T, we have
Q, = DlS lc (Tm - T) +AI, (1)
where D is lhe density of lead, e the speeifi:c heat for lead,
and T m its melting point.
During short-circuiting, the resistance of the circuit
is R + Rr. where Rr = pl/S is the resistance of the fuse.
The short-circuit current is
I = Ul(R +R,J.
According to Joule's law, the amount of heat liberated in
the fuse during a time t is Q1 = 11R 1t. Therefore,

Q, = 1a.:'X~1·. · <2>
Assuming that the entire amount of heat liberated in lhe
fuse is spent for its heatiog and melting, we can write
Q1 = Q 2 , or, taking into account Eqs. (1) and (2),
D!S(c(Tm-TJ+Al= (R+°:.)•S ,
whence

R = +{u V o1<1r!'-•H•> pt} •


R = ~[120

x ( tt.3xtêPx10.~31xx1W;~~~~+o.25xtatJ
-2.1X10""' X 2X 10"')"']C!0<0.34 Q.
472. Air contained in a closed vessel of volume 1 l
under normal eondit.ions is hea\ed by an electrie heater
rat.ed for a current of 0.2 A anda voltage of tO V. ln what
time will the pressure in the vessel rise to t MPa? The
efficieocy of the heat.er is 50%.
Ch. a Electricity
GW.n: V=l 1=10-' m•, /=0.2 A, U=IO V,
p 1 =1 MPa=IO' Pa, ~=50%=0.5.
t-1
Solutton. The amount of heat liberated by the current
during a time t can be determ.ined from Joule's law:
Q, = I'Rt = IUt. (!)
The amount of heat required to heat the air from T1 =
273 K to a temperature T 1 is
Q, =cm (T, - T,), (2)
where e is the specific heat for air and m its mass. By
hypothesis, Q1 = 0.5Q1 , or, taking into account expres-
sions (t) and (2). cm (T1 - T1 ) = 0.5/UT, whence
t =cm (T, - T,)1(0.5/U). (3)
Here m = pV, where p is the density of air.
Since the volume V occupied. by the gas does not change
upon heating, by applying Gay-Lussac's law p 1/T1 =
p 11T1 , we can determine the final temperature of the gas:
T 1 = T1p 1/p 1 • Substituting the expressions for m and T z
into Eq. (3), we obtain
tpV(T1pJP1-T1) cpVTi(Pa/Pi-0
0.5IU O.SIU
t-- i.005XiOIXi.29Xio;:~~~~3X(10'/(1.0txt0')-1) 8

~3.18xtOS s.
473. Determine the ef&ciency of a current source with
an internai resistance of 0.1 D: loaded. by a resistanée
oi 1.5 !l.
Giuen: r-0.1 !l, R-1.5 !l.
1)-1
Solutton. By de&nition, the efficieocy is

tt= ~~ 100%,
2116 Selected Problema on Pby1ics

wbere Nu is the useful power liberated in t.he externai


subcircuit and N 1 is t.he power liberated io the entire
closed circuit.. The powers Nu and N 1 can be determined
from tbe relations Nu = / 1R and N 1 = / 1 (R + r).
This gives
11= ~:~~t!~ = R~r 100%,
~= •. :~~•.• 100%<><94%.
474°. The useful power liberated in an externai subcir-
cuit attains ita maximum value of 5 W ata current. of 5 A.
Determine the internai resistance and the emf of t.be cur-
rent source.
Given; Nmu=5 W, lmu=5 A.
r-1 B-1
Solution. Accordiog to J oule's law, the useful power
liberated in the externai subcircuit. of reaistanee R is
N = l'R. (!)
Aecording to Obm's law for a closed circuit.,
l = tl(R + r).
Therefore (see Eq. (1)), we can write
N = l'R!(R + r)'. (2)
Tbe power liberated in t.he externai subcircuit has the
maximum value (N = Nmu) when the following condi-
tion is satisfi.ed:
(3)

Di&erentiating Eq. (3) wit.h respect to R, we cao find the


externai resistanee Rma:i: at. whieh the power has the
maximum value:
-ik-[ (R·~~)I ]= l•(R+r>t;:;;·2(R+r) O,
(R+ r)'-2R(R+r)=0, (R +r) (R+r-2R)=0,
R+r-2R=0, R=Rma:r.=r.
Ch. 3. meotricity

Subatituting Rmu into Eqs. (1) and (2), we obtain


Nma• = 11..aP• whence
r = N mu.111u,..,,
r - (5/5') !l - 0.2 !l.
Therefore, N mn = 17/(4r1 ) = 1'1/(4r), whenee

M=Jf4rNmaz=J/ ~::z =2.!f:;- •


1-2+ v-2v.
475. Is the work done by a eurrent source in the inter-
na! subeireuit a eonstant quantity for a given souree?
Answer. No, it is not.
476. Why is the filament of a bulb strongly beated,
while the leads remain cold?
Answer. The amount of heat liberated in condueton-
conneeted in series is directly proportional to their resist-
ance (Joule's law). Tbe resistance of the filament is very
high, while the resistance of the leads is very low, and
hence the filament is heated strongly, wbile the leads
are heated only slightly.

EXERCISES
4.77. Three conductors of resist.anee 3, 6, and 8 O r~
speetively are connected in parallel. The amount of heat
liberated in the first conduetor is 21 kl. Determine th&
amount of beat liberated in the seeond and third eon-
ductors during the same time.
478. Two eonductors of resistanee 10 and 6 gare con-
neeted first in series and then in parallel between two
points with a potential difference of 20 V. Determine the
amount of heat liberated in each conductor per seeond.
479. A current Oows through a copper eonductor of length
2m anda cross-sectional area of0.4mm1 • The amount
of beat liberated per seeond is 0.35 J. How many eleetrons
pus through the cross section of the conduetor per seeond?
480. Wbat is the length of a Nichrome eonductor of
diameter 0.5 mm used for making an eleetric heater oper-
2118 Select.ed Problema on Phy1ic1

ating at. a voltage of 120 V and producing 1 MJ of heat.


per hour?
481. Two conductors of the same resistance R are con-
nected to a circuit at a volt.age U 6.rst in parallel and
then in series. ln wbieh case is tbe power consumed from
t.he circuit higher?
482. A pot containing 1 1 of water and 0.5 kg of ice at.
-O ºC is put on a hot pia te of power 600 W. ln what time
will tbe temperature of the water in the pot rise to 60 ºC
if the efticiency of the hot pia te is 80%?
483. A potential dillerence of 10 V is applied at the
ends of a lead wire of length 1 m. What time will elapse
from the beginning of the passage of current to the mo-
ment. at whieh t.he lead start.s melting? The init.ial t.em-
perature of the lead is 27 ºC.
484. Two identical refraetory vessels containing tin and
lead respeetively are heated on a hot plate. The masses of
t.he tin and the lead are equal, and their initial tempera-
ture is 20 ºC. What will be the ratio of the durations of
melting of the metais?
485. Determine the eurrent in a eireuit of a lead aeeu-
mulator if itsemf is 2.2 V, the externai resistanee is 0.5 O,
and the effieieney is 65 %•

CURRENT IN ELEcrR.OLYTES AND GASES


Solutions eondueting eleet.rie eurrent are known as
electrolytes. The eurrent in them is due to the motion of
positive and negative ions. The passage of the eurrent
through an eleetrolyte is aeeompanied by the deposition
of the eonstituents of the dissolved material (electrolysis).
The mass of a substanee deposited during an eleetroly-
sis ean be ealeulated by using Faraday's generallzed law
of eleetrolysls:
m ~ Altl(Fn),
where m is the mass of the deposited substanee, A its mo-
lar mass, I the eurrent through the eleet.rolyte, t the time
during whieh the eurrent. Dows, F the Faraday conatant,
and n the valenee.
Gases beeome eondueton when they are ionized, i.e.
Ch. 3. Elecli'iclty 299

contain free electrons and positive and negative ions.


Extrlnsic conducUon is due to tbe ionizat.ion of a gas
caused by an externai effect (Dame, ultraviolet or X-radi-
ation, etc.).
lntrlnslc conducUon is due to cbarged particles accel-
erated by an electric field and ionizing neutral molecules
as a result of collisions.
Tbe minimum potential difference required to ionize
an atom or a moleeule of a substance by an electron
accelerated by tbe potential is known as the fonfzatlon
potential U 1 for tbe given atom or molecule.
Tbe energy acquired by an electron baving passe d througb
this potential difference is called tbe lonlzatlon energy:
W 1 =eU 1,
where e is the electron charge.
Tbe glow dlscharge, tbe are discharge, the spark dls-
ebarge, and the eorona dlscharge are different types of
self-sustained gas discbarge.

486. During nickel plating of a plate, its surface is


covered by a nickel layer of thickness 0.05 mm. Determine
the mean current density if the nickel plating lasts for
2.5 h.
Given: h=0.05mm=5x10-3m, t=2.5h=9xf03s.
i-1
Solution. The current density is defined as
/ = l/S,
wbere S is the crou-seetional area of the conducting part
of the electrolyte, equal to the area of the plate.
ln order to determine the current, we sball apply
Faraday's generalized law
m = Alt/(Fn). (1)
Here
m = pV - phS, (2)
wbere p is the density of nickel.
Selected Problema on Phyaica

Suhstituting Eq. (2) into (1), we obtain tbe current:


I = phSFn/(At).
Then tbe current density is

i= ~:,n = ~n'
a.axtoax5xtO'""'x9.65xtoax2 A
160 A/m•.
58.7xt0"'4x9xtoa m1
487. During an electrolysis of silver nitrate solution
lasting for an hour, 9.4 g of silver are deposited at the
cathode. Determine the emf of polarization if the voltage
across the bath terminais is 4.2 V and the resistance
of the solution is 1.5 e.
Given: t=I h=3.6xlO's, m=9.4g=9.4xto-•kg,
U=4.2 V, R= 1.5 C.
rs,-1
Solution. During the electrolysis of silver nitrate solu-
tion, the symmetry of the electrodes made of the same
material is violated, and electrodes are polarized. ln this
case, the emf of polarization appears, and Ohm 's law for
a suhcircuit containing an emf of polarization beeomes
U = IR + ~P' whence
li,= U-IR. (1)
Since I = qlt, and according to Faraday's law, q =
mFn/A, Eq. (1) can be transformed as follows:
lp=U- m~;R '
Mp=4.2 9 · 4 ~~~~:::;~~!;t.s v~o.1 v.
488. How many atoms of a hivalent metal will be de-
posited on a square centimetre of the electrode surface in
5 min ata current density of 0.1 A/dm1 ?
Given: 8=1 cm2 =1<r' m2 , t=5 min=300 s,
1=0.1 A/dm'=IO A/m', n=2.
n,-1
Cb. 3. Electricitf SOi

SoluUon. According to Faraday's law, we have


m = Allf(Fn),
or, considering that I = JS and m. = A/Stl(Fn), we ob-
tain
m/A = jStl(Fn). (!)
Here m.IA is the number of moles. Multiplying this quan-
tity by Avogadro's constant, we obtain the number n 1 of
atoms deposited on the electrode surface:
n 1 = m.NAIA.
Using Eq. (!), we get
ni= 1s;~A

ni 1ox1o;.~~~~~2x1ou 9.4 X 1011.


489. A copper plate containing 12% of impurities is the
anode in a copper sulphate solution. During an electroly-
sis, copper is dissol ved and deposited in pure form at the
cathode. What is the cost of the relining of 1 kg of such
a copper if the voltage across a bath is maintained at 6 V,
and the cost of 1 kWh of energy is 4 kopecks?
Given: M=I kg, U=6 V.
•-1
Solu.Uon. The mass of the pure copper deposited at
the cathode is
m = M - 0.!2M = O.B8M.
According to Faraday's law,
m = Altl(Fn) = Aql(Fn),
whence
q = mnFIA = 0.88MnFIA. (!)
The energy spent during the electrolysis is W = qU, or,
taking into account expression (1),
W= O.~nl"U

W= 0.88x1~~:.;!5xtO'x8 1 = 4.5 kWh.


Selected Problema oa Phy11ic11

Consequently, the cost of the refining of t kg of such


a copper is z = 18 kop.
490º. Determine the mass of copper deposited at the
cathode as a result of the passage of a current which
uniformly increases from O to 4 A through a copper sul-
phate solution for tO s.
Given: t 1 =0, t2 =10 s, 11 =0, 1 2 =4 A.
m-1
Solution. According to Faraday's law, the mau of the
substance deposited at the cathode is
m = Aql(Fn), (1)

where the charge passing through the copper sulphate


solution during the time t 2 is

q= r
o
l dt. (2)
By hypothesis,
l = kt, (3)
where k is the proport.ionality fact.or. For the instant. of
time t 2 , l 2 = kt 2 , whence k = l 2lt 2 • This gives
l = I,tlt,. (4)
Substituting Eq. (4) into (2) and integrating, we obt.ain

q=
~ .
lf.--tdt=-{;-~tdt=f.--f=~. (5)
o o
Using expression (5) in (1), we get
m= A:.t,.•
m ~:9 ~~·:1~x~~ kg=6.65 x tO'"' kg.
491. Determine the mass of oxygen evolved as a result
of the passage of a charge of 16 e through an aqueous
solution of sulphuric acid. The mass of an o:1ygen atom
is 2.6 X tO-H kg.
Ch. 3. Electricity

Gimm: q=l6 C, nt=2.6x IQ-1' kg.


M-?
Solution. The molecules of sulphuric acid HtBO, disao-
ciate in water into positive (H+) and negative (SO;-)
ions. This reaction can be written in the form
a,so,-2H+ + so;-.
The group SO;- deposited at the anode enters into the
reaction with water:
so, + H,O = H,so, +o.
As a result, gaseous oxygen is evolved at the anode. The
last equation shows that as a group so;- bearing a charge
of 2e passes through the solution, an oxygen atom is
evolved at the anode. Consequently, when a charge q
passes through lhe solution, q/(2e) atoms of oxygen will
be evolved at the anode. Denoting by m the mass of ao
oxygen a tom, we find that themass of the evolved oxygen is
M=~' •
M ~·!~.~~·:::,6 kg= 1.3 X fQ-8 kg.
492. Undiluted sulphuric acid is stored in iron contain-
ers, while diluted acid is stored in glasa vesaels. Why?
Answer. Undiluted sulphuric acid is not ao electrolyte,
while diluted acid is. If diluted acid were stored in an iron
container, an electrolysis would take place.
493. The direction of current in modero galvanic baths
is being altered. Why?
Answer. A metal is deposited at ao article during
a passage of the forward current. During a passage of
a short-time reverse current, the deposited metal is partial-
ly diSl!lolved (mainly at microscopic projections of the
cathode). This improves the quality of plating.
494. Determine the ionization energy for a helium
atom if its ionization potential is 24.5 V.
Giuen: U 1 = 24.5 V.
~
Selected Problems on Phy1ic1

Solution. By definition, the ionization energy is


W 1=eU1,
where e is tbe electron charge. Tberefore,
W,= 1.6 X 1Q-IO X 24.5 1=39.2 X 1Q-19 J.
495. What must be the minimum velocity required for
ao electron to ionize bydrogen atoms? The ionization po-
tential for a hydrogen a tom is 13.5 V.
Gtven: U1,=13.5 V.
v-1
Solution. ln order to ionize a bydrogen atom, tbe elec-
tron must bave a kinetic energy equal to the iooizatlon
energy:
Wt = W1, or muS/2 = eU1,
wbere e and m are the electron charge and mass. Then

v=V~'.
v=J/Zxt:.:!~;.~t 3 .s ~ =2.2x101 m/s.
496. Determine lhe saturation curreot density in
a gas-discharge tube with an interelectrode distance of
10 cm if 10 pairs of monovalent ions are formed in a cubic
centimetre of the tube per second under the actioo of
cosmic radiation.
Given: l = 10 cm= O. t m, np1 =10 cm-3 • s-1 =
10' m-3_.9-1,
f,-1
Solution. The saturation CWTent density is
J, =l,IS, (1)
where / 1 is the saturation current and S the cross-sec-
tional area of the tube.
Considering that / 1 = q/t, q = enV, aod V= lS, where
q is the charge passing through the tube during a time
t, e the charge of a monovalent ion, V tbe volume of lhe
Ch. 3. Electrie.ity 305

tube, and n = 2np is the ion concentration (t;. tbe num-


ber of ion paira), and using these expresaions in Eq. (t),
we obtain
J, = 'l.en,lSl(tS) = 'l.en,llt. (2)
Denoting by n,lt = Apt the num.bel' of ion paira formed in
a cubic metre of the tube per .second, we can write expres-
sion (2) in tbe form
1. = 'l.en,,l,
j, = 2 X 1.6 X t0-19 X 107 X 0.t A/m1
= 3.2 x to-11 A/mª.
497. Determine the mean velocity of the directional
motion of monovalent ions in an ionization chamber if
their concentration is 108 cm-a and the saturation current
density is 10-11 A/m1 •
Gtven: n=1()11 cm-•=to• m-•, /,=to-1:1 A/mª.
(v)-1
Solutlon. By de.linition,
M=llt, (1)
where l is the length of the cbamber.
Using the solution of Problem 496, we can write
;, = 2en,1l = en 1l, (2)
where n 1 is the number of ions formed in a cubic metre per
second. It follows from expression (2) that l = jJ(en 1),
OI' since n 1 = nlt,
l = ;,t/(en). (3)
Substituting expression (3) into (1), we obtain

(v)= ::: =-!}'

{v)""" 1. 6 X!~:: X 10, ~ 6.2 x 16-1 m/s.


498. Why does a charged electroscope always get
discharged under the room conditions?
Answer. The electric field produced between the
charged leaves of the electroscope always contains a
Z0-0970
306 Selected Problema on Physic!I

certain number of ions formed as a result of action of


externai ionizers (like cosmic rays). These ions move to
the electro5cope leaves and neutralize their charge.
4.99, Why are high-tension transmission linés always
supplied with two addiUonal wires which are not in-
sulated from lhe steel supports and are arranged above
the maio wires?
Answer. These additional wires are intended for
protecting high-tension transmission lines from lightning
discharges.
EXERCISES
500. The amount of copper deposited at the cathode
per hour during an electrolysis of copper sulphate is
0.5kg. The surface area of lhe electrodes immersed in the
electrolyte is 7.5 m2 • Determine the current density.
501. What amount of electric energy should he supplied
for depositing 500 mg of silver during ao electrolysis
of a silver nitrate solution? The potential difference he-
tween the electrodes is 4 V.
502. The amount of a hivalent metal deposited at the
cathode in ao electrolytic hath during 10 min at a cur-
rent of 5 A is 1.017 g. Determine the atomic mass of the
0

metal.
503. Determine the thickness of the copper layer depos-
ited at the cathode during a 5-h electrolysis of copper sul-
pbate if the current density is 0.8 A/dm1 •
504. What amount of copper will he dep0sited at the
cathode during ao electrolysis if the amount of supplied
electric energy is 5 kWh? The voltage across the hath ter-
minais is 10 V and the efficiency of the set-up is 75%.
505. A power of 37 kW is supplied during ao electroly-
sis of a sulphuric acid solution. Determine the resistance
of the electrolyte if 0.3 g of hydrogen are evolved at the
cathode during 50 mio.
506. Determine the mass of chlorine evolved as a result
of the passage of a charge of 16 C through a hydrochloric
acid solution.
507. Ao electron Dying at a velocity of 2.2 X 10' m/s
ionizes a gas. Determine the ionization potential for the
gas.
Ch. a Elec&ricity 307

508. Determine the sat.uratioo eurnot io an iooizatioo


ebamber with a surfaee area of the eleetrodes of too em1
aod ao ioterelec::trode separatioo of 6.2 em. Ao iooizer ge-
nerates 10' monovaleot ions of eaeh sigo io a eubic::
centimetre of the ehamber per sec::ood.

QUESTIONS FOR REVISION


1. What ia eleetric current? 2. Deliue current and current denaity,
name thelr units. 3. What ia the resiatance of a conductor? How
does it depend on the 8!QtDetrlcal size, material, and temrerature
~~ ~i:euc:f:t:m º.!~º~? e~'r:1!J:::O~C~~~ far:ºfu;
:~~er,;~teS::p~=f::eu:re~:;!:!°~ :1~f~i?>!i~
taining aeries- and parallel~nneeted conducton. 7. Wbat are the
: :of a shunt anil a seri~ resiatance? What are the 'Yj~!!
lawa. 9
a cur
elect
plain p ~ical meanins of elech'ochemical and chemical equiv-
i~~~:·ia ~~~J:1181t1 N~=~º~ri::.F:r;;.r dc:ha~~
17. Define ionizaUon potential.
J.J. Electromagnetlsm
MAGNETIC PIELD OP A CUBRENT. FORCES ACTING
IN A MAGNETIC PIEI.D ON A CURRENT-CABBYING
CONDUCTOR, A MOVING ELECTRIC CllABGE,
AND A CUBRENT LOOP

Magnetle lndueUon is a vector physieal quantit.y whose


magnitude is given by
B- µµ 0 //(2xr)
for ao infioitely long straight conductor,
B- µµ 0 //(2R)
at the cent.re of a circular loop, and
B- µµ,ln
on the axis of a soleooid (ioaide it.), where ris the shortesl
dist.ance from the conductor to a poinl where the induction
20•
Selected Problema on Physica

is determined, R the radius of the loop, and n the number


of turns per unit length of the solenoid. ln ali cases, the
directioo of the vector Bis determined by the right-.hand
screw rule.
lf a magoetic field is produced by several currents, the
magoetic inductioo at a giveo point is the vector sum of
the magnetic ioductions e, of the fields produced at the
point by each current separately (the principie of super-
posiUon of magneUe &elds):
B= i; B 1•

·~·
Magnetic induction and magnetic field strength are
conneeted through the relation
B=µµ,H,
where H is the magneUe fteld strength, tL the permeabJUty
of a medium, and tLo the magnetie coastant.
A curreot-earrying conductor ln a uoUorm. magneUe
fteld is acted upon by the Ampêre foree
F=IlBsina.,
where I is the current in the cooductor, l its length, aod a.
lhe angle belweeo lhe direclions of lhe magnetic induction
vector and the currenl. The direction of lhe Ampàre force
can be determioed by the left-haod rule.
lf a magoetic field is nonuniform, lhe Amp&re force
acting on an elemenl of length dl of the conductor is
dF =IBsina.dl.
The force acting in lhis case on the entire conductor is

F=} /Bsin a.dl.

An electric charge moving in a magoetic field experi-


ences lhe ~ction of lhe Lorentz foree
F=qvBsina.,
where q is the charge, v the velocity of its motion, and a.
the angle betweeo the directions of lhe magnelic induc-
Cb. 3. Electrieity

tion and the velocity vectors. The direction of the Lo-


nntz force can be determined by the Ieft-hand rule (for a
positive charge).
A eurrent loop ln a magneUe &.eld is under the action of
a force couple with a torque
M=PmBsina,

io~~ /it:a~~. 1:0~: ::=\~e~~=~~ d?!~=~


the magnetic induetion veetor and the normal to the
plane of the loop.

509. The currents 11 =11 and1 1 =11 +1 1 pass through


three long straight conduetors arranged in the same
plane parallel to one another at a distance of 3 cm

.
,-
•01
'

e,
\/

y
Fig. 153
(Fig. t53). Determine the position of a straight line sueh
that the magnetic induction of the 6.eld produced by the
currents is zero at each of its points.
Gtven:r=3çm=3xto-am, 11 =11 , 11 =11 +12 •
z-1
Solutlon. Let us suppose that the currents 11 , 1 1 , and
11 Oow in the plane perpendicular to the plane of tbe
310 Selected Problema ou Phyaica

figure awaY from us (the directions of the curren\s are in-


dicated by crosses). The magnetic induction vectors pro-
duced by the currenta are directed, aecording to the right-
hand screw rule, along the tangent at any point of the
magnetic fteld line (the fteld lines are indicated by dasbed
circlea).
The sought straight line on which the magnetic indue-
tion vector is zero can be arranged between the currents
r. and 1. at a distance % from 1•. lndeed, the induction
vectors Ri and B, of the 6.elds produced by the currents
11 and / 1 at point O are directed downwards, while the
induction vector ~ of the lield produced by the current
/ 1 at this point is direeted upwards. According to the su-
perposition principie, we can write

B,+B,+11,=º·
or in projeetioDB on the Y-axis,
B 1 +B1 -B1 =0. (1)
The magnetic induction of the fteld produced by an in6.-
nitely long straight current-carrying conductor is
B= ••,l/(2"r).
This gives

B,= .. ·~·>
Bz=~. (2)

B,= '1:,~~:,~
Subatituting u:pressions (2) into (1), we obtain

.."1*.) +~- "t~~:,·> =0,


or alter the transformation ú 1 + rz - ri= O, we get
-r±'Y;q:tã;I -3xtO-•±t2.4xtO-•
8 a m.
Conaequently, :ti oir: t.2 x to-1 m. The second root of
tbe quadratic equation, z 1 = -t.9 X 10-1 m, corre·
Ch. 3. Electricity 3H

sponds to a point lying between the currents 11 and 11 ,


which is also possible. Therefore, the problem has two
solutions.
510. Currenta of 10 A Oow in the same direction along
two long straight conducton separated by 5 cm. Determine
the magnetic induction at a point lying at a distance
of 3 cm from each conductor.
Giwn: l=5cm-5x10-Zm, 11 =11 =/-=iOA,
r=3 cm=3X t()-1 m.
B-1
Solution. The magnetic induction vector 8 of the 6.eld
at point A is the vector sum of the magnetic inductions
Bi and 8 1 of the fi.elds produced by each current separately
at this point (Fig. 154). The directions of the vectors 8 1
and 8 1 can be determined from the right-hand screw rule.

Fig. tM

The numerical value of the magnetic induction at point


A can be found by using the cosine law:
B-JI B:+.S:+2B1B,cos1&. (1)
The magnetic inductions of the 6.elds produced by each
current at point A are
B1 = µµ.I/(2"r 1), B, = µµ,I/(2nr 1).
Since r1 =r1 =T and B1 =B1 , we have (see Eq. (1))
B= V2'8!+21r,ccs1&=B1 J/2+2cos1&. (2)
312 Selected Probleme ou Phyaic•

Applying the cosine law to ll.. ADC, we find that


l1 = +r: r: -
2r1r 1 COI CI, wbenee COICI= ('! + -11)/ r:
i~·í;~ ~J2:oa~ i;~t<!7q. ~~~!!ºo~~nthe expressions
B=~V2+ 2(~ l'l =~Jf4i'=ii.
B= :::.~::<~rr;,_' 1W~. V4x(3xt0""2) 2 -(5xto-a)ªT
<><66.6~T.
511. A current of 10 A p8888a through a ring made of a
copper wire and having a crou-seetional area of 1 mm1 •
A potential difference of 0.15 V is applied acrou the ends
of tbe ring. Determine the magnetic induction of the 6.eld
at the eentre of tbe ring.
Given: 8=1 mmª=i~ m', /=10 A, U=0.15 V.
B-1
Solution. The magnetic induction of the field at the cen-
tre of a circular current ia
B- ~~ 0 1/(2r), (!)
where ria the radius of tbe ring. Acc.ording to Ohm's law
for a conductor, the potential difference across the encls
of tbe ring is
U=IR. (2)
Considering that R= pUS and l=2nr, wbere Ria tbe
reaistanee of the ring and l its length, we write Eq. (2)
as U = 1p2nr/S, whence
r= US/(l2np). (3)
Substituting expression (3) into (t), we obtain
B= "'"';,71•,
B .f.x3.t.f.xt<r'x0t.~~~~o.nxtO-'xto• T=« p.T.

512. ldentical currents of 2 A Dow in two identical cir-


cular loopa of radius 5 cm whose planes are mutually per-
Ch. a Electricit:J 313

pendicular and whose eentl'88 coincide. Determine the


magnetic induction of the 6.eld at the eentre of the loop1.
Given: R=S em.=5X 10""'I m, /=2 A.
B-1
Solutton. The magnetic induction of the field produeed
by each cireular current at the centre of the corresponding
loop is
B, = B, = µµ 0 //(2R). (1)
Aceording to the right-hand screw rule for the ch088D di~
rections of tbe currents in the Ioop1, the vector S. is di-
reeted away from us and the vector B1 to the right
(Fig. 155). Auording to tbe superposition principie, th&
induction of tbe resultant magnetic 6.eld is
B=B,+B,.
Sinee S. and 8 1 are mutually perpendicular, we can apply
the Pytbagorean theorem to tbe corresponding right tri-
angle and lind that
B=YS:+S:=B, Jf2. (2)
Subatituting Eq. (1) inlo (2), we obtein

B=""'..r'
B= 4xa.1"::rx~;._~2xt.4t T~ 35 _ 4 µT.

Fig. 155
513. A solenoid of lengtb 20 cm and diameter 5 cm should
produce a magnetic induction of t.26 mT on its axis.
314 Select.ed Probleme on Phyeics

Determine the potential difterence that must be applied


aCl'089 the ends of the solenoid winding if a copper wire of
diameter 0.5 mm is used for it.
Giwn: L = 20 cm = 0.2 m, D= 5 cm = 5 x 10-ª m,
B= 1.26mT=1.26x10-1 T, d= 0.5 mm=
5xlo-< m.
U-1
Solution. The potential difference aCl'O!I the ends of the
aolenoid winding is
(1)
Uaing the expreasion B= p.l'oln, we can determine the
c.urrent througb the solenoid:
1= B/(~~,n), (2)
where n= N/L is the number of t.urns per unit length of
tbe aolenoid. Since the total number of tums is N = Lld,
expre88ion (2) can be written in the form
1 = Bd/(~~ 0 L). (3)
The resistance of the winding is
R=pllS.
Since p is the reaistivity of copper, l= nDN = nJJUd,
.and S = :r.dl/4, we have
R = 4pDL/d'. (4)
Transforming Eq. (1) by (3) and (4), we obtain

u-'::;,?
U= "xo.11:~'r.t~!·!~~~~\1$,~5xto-- v~ 2 . 7 v.
514. Two parallel conductors carrying the same cur-
rent are separated by a distance of 8.7 cm and attract each
.other with a force of 2.5 x 10-1 N. Determine the current
in the conducton if the length of each conductor is 320 cm
&nd the currents in them have the same direction.
Ch. 8. Blectrlci&J 3i5

Given: r=8.7cm=8.7x10-1 m, F=2.5x10-s N,


1=320 cm=3.2 m.
1-1
Solutton. Since, by hypotheais, r < l, we can assume
that the wires are inftnitely long. Therefore, the magnetic
induction of the field produced by the conductor / 1
(Fig. 156) is
B, = µµ,I,1(2nr), (!)
Aecording to Ampàre's law, the force acting on the con-
ductor / 1 is F 1 -11 B1 l, or, taking into account Eq. (1),
~:: ~~~{J(~~)~)~hes~:~ 11 = 11 = 1, we have
1-V';f.
I = V 72~x7s~.!"~xx~.ª~.t~~x~x~!::~:~~~~~~;rx~1~0-0~ A ~ 58 A.
515°. A metal rod of length 15 cm is perpendicular to
ao inftnitely long straight wire carrying a current of 2 A.
r:c~dm~;ªti!!ª~;::,:xt~!8!:11 !~: c=:;~~ :~droa~
0.5 A aod the distance between the wire and the nearest
end of the rod is 5 cm.
Given: 1=15cm=15x1()-2 m, / 2 =2 A, r=5 cm=
5x!O"'m, I,=0.5A.
Pi-?
Solution. We divide the rod CD into small elements of
length dz and consider such ao element at ao arbitrary
distance z from the vertical wire (Fig. 157). According to
Ampire's law, the force acting on each element dz is
equal to
dF1 = / 1B 1 dz
and is directed vertically upwards, in accordance with
the left-hand rule for the chosen direc.tions of the currents.
Here
(!)
318 Selected Probleme on Pbyaic1

11 lz

1, df,
li,
0 1,
D

Fig. 158 Fig. 157

is the mapetie induetion of the 6.eld produced by the eur-


rent I 1 at a point lying at the distance z from the verti-
cal wire. Since the forces aeting on other elements of the
rod are also directed vertically upwards, the total force
acting on the rod is
r+l r+l
F1 = ~ dF1 = ~ /JB2 dz. (2)

Substituting Eq. (t) into (2) andintegratingthe obtained


expreuion, we 6.nd that
r+I r+I
Fi= ~ f&t1~1dz = ert 1
1 1 ~. ~
=~lDzl~+l
=~ 1 lnr;z,
4x3.14x io-" xi x2xo.s
F, 2xa.1.f.
X ln 5x105~~~X 10-1 N
o.2.8xt0""' N.
516°. Two long straight parallel conductors are separat-
ed by a distance of 5 cm and carry currents of to and
Ch. 3. Eleclricity 817

20 A. What work per unit length of a conductor must be


tO~"mt~,1~~e:!r~~~selo~~~ºth!~~=di~t~=~uctors to
Given: r 1 =5 cm.=5X10""1 m, 1 1 =10 A, 11 =20 A,
r 1 =10cm=10x10-ª m.
Ali-?
Solution. For the sake of de6.niteness, we assume that
theconductor carryingthe current 11 is stationary, and
the distance r changes due to the displacement of the con-
ductor carrying the current 11 • The second conductor is
in the magnetic 6.eld B1 produced by the conductor with
the current / 1 • Therefore, it is acted upon by the Ampere
force
F,= I,B,I (sin CZ= 1), (1)
which is direeted, according to the left-hand rule, towards
the 6.rst conductor (Fig. 158). This force cannot increase
the distance between the conductors. Therefore, in order
to displace the second conductor, we must apply to it an
externai force F e:u equal in magnitude and opposite to
the force F1 , i.e. Fui= -F1 , or, in scalar form, Fn.t =
F 1 • Since we assume that the 6.rst conductor is long, the
magnetic induction of the 6.eld produced by it is
B,= ~~,I,1(2nr). (2)
Taking Eq. (2) into account, we can write Eq. (t) in the
form
F e:u = j.i.JL01111 l/(2:itr).
Then the externa! force per unit length of the second con-
ductor is

Hence it follows that the force decreues with the dis-


placement of the conductor, and the work done by such a
varying force is

4-="í,, fu.tdr.
Substituting Eq. (3) into the integrand and integrating,
we obtain
318 Selected Prohleml!I on PhJSica

1
"
r, r, F,,. h,
F,

-r,1
" 1
1
D y

Fig. 158 Fig. t59

 _ r~ ~ dr _ l11'al1f1 '~ dr
T-l~r-~lr
,, ,,
= l'fl~1f1 ln~,
4- 4x3.t'tx·~~; 4xt.Ox20 ln~º:.~· J/m
=27.6~1/m.
517. A straigbt conductor of m888 2 kg, carrying a cur-
rent of 4 A, moves vertically upwards in a uniform mag-
netic 6.eld of induction 2 T directed at an angle of
30" to tbe vertical. Three seconds after the beginning of
motion, the velocity of the conductor isto m/s. Determine
the conductor lengtb.
Giuen: B=2 T, a.=30º!:!::!0.52 rad, m=2 kg,
1=4A, t=3s, v=tOm/s.
1-1
Solutton. The conductor CD moving in \he magnetic
field is acted upon by the force of gravity mg and the Am-
pêre force F (Fig. 159). Since tbe conducjor moves at an
angle a to tbe direction of B, the Ampêre force is
F=IB,l,
Ch, 3. Electricity 319

where B"= B sin e& is the z-component of the vector


B. Consequently,
F =IBlllinOI. (1)
Since the motion of the conductor is uniformly accelerat-
ed, we can write Newton's second law in projections of
forces and accelerations on the Z-axis:
F-mg=ma,
or, taking into account expression (1),
/Bl sin e& - mt= ma. (2)
Using tbe formula v = at, we find that a = vlt and write
Eq. (2) in the form
/Bl sin e& - mt """mvlt,
whence
l= ~~!:~> .
2 ~ ~28:~.~S) m ~ 6.57 m.
518. A cyclotron is intended for accelerating protons
to an energy of 5 MeV. Determine the maximum radius of
the proton orbit if the magnetic induction of the field is
t T.
Given: Wk=5 MeV=8x10-ll J, B=1 T.
R-1
Solution. The proton moves in the cyclotron in a heli-
cal orbit consisting of semicircles with gradually increaa-
ing radii. The Lorentz force F = qvB is acting on the
proton. Writing Newton's second law for this particle
in projections on the Y-axis directed to the centre of the
circle, we get
F-ma., (!)
where a,= ac= v21R. Then Eq. (t) can be written in
tbe form qvB = mv'I R, whence
R = mvl(qB). (2)
Transforming expression (2), we obtain
mv = J(2iii JfiiWii2 =V 2mW•• (3)
320 Selected Prohlems on Physice

'Where W k is the kinetic energy of the proton.


Substituting Eq. (3) into (2), we get

R=~·
R Y2x\~!~!g:::~~x10-11 m=0.32 m.
519. An eleetron accelerat.ed by a potential diBerence
-of 300 V moves parallel to a straight conductor at a dis-
tance of 4 mm from it (Fig. 160). Wbat force will act
-on the electron if a current of 5 A is pused througb the
-conductor?
Given: r=4mm=4xt0-3 m, /=5A, U=300V.
F-1
Solution. When the current is pasaed through the con-
ductor, the eleetron is acted upon by the Lorentz force
F=eBv. (1)
Since the electron hu been preliminarily accelerat.ed in
an electric lield, we can write A= âWk, or eU = mv2/2,
wbere m is tbe electron masa. Tberefore,
(2)
The magnetic induction of the &eld produced by a long
.straight current-carrying conductor is
B= µµ,I/(2"r). (3)
Substituting expressions (2) and (3) into (1), we obtain

F=J&~-iu:-Y. 2:' '


4x3.14x 10-1 x t x t.6xto-1ox5
F 2x3.H.x4xto-•
X 1í 2xt.6xto-11 x300 N...., 4 xto-•• N
r 9.ixi0'-1 1 - •

520. An electron moves in a magnetie &eld-of induetion


2 mT in a helix of radius 2 em and pitc.h 5 cm. Determine
the electron velocity.
Cb. 3. F.l.ectricity

Giuen: R=2cm=2xt0-zm. h=5cm=5x1o-tm,


8=2mT=2X10-0 T.
v-1
Solutton. The motion of the electron in a heliJ: can be
repreaented as a motion in a cirele at a veloc.ity "r under

l,,.~
~X
Fig. too Fig. t6t
the action of the Lorentz force in a plane perpendiculu
to the magnetic induction, and a uniform motion along
the field ata veloc.ity "z (Fig. 161). Then the resultant ve-
loc.ity of the electron is
v=Vv:,+u:,. Cll
Let us determine "z and v, separately. Writing Newton's
second law for the electron in projections on the Y-axis,
we obtain
F=ma,,
where F = eBv, and a, = v;IR. Then eBv, = mv'r/R,
whence
v, = eBR/m. (2)
The ,·elocity "z can be determined from the relation
h = VzT:
"z =hlT, (3)
where T is the time during which the electron is translat-
ed in the horizontal direction by a pitch of the heli.x:. On
the other hand, T is equal to the time during which the
21-01170
322 Selecled Problema oa Physica

electron traverses (at a velocity Vr) the distance equal to


the length of the circumference, i.e. T = 2nRlv11 , or,
taking into account Eq. (2),
T = 2nllml(eBR) = 2.nml(eB). (4)
UAing relations (3) and (4), we get
v, = h/T = heBl(2nm). (5)
Suhltituting Eqs. (2) and (5) into (1), we obtain

u=Jf :~,-:: + "11:181 = 2~m )fhZ+4nZR2,


i.8x t.0-11 x2x t.O-ª
v= 2x3.H.x9.1x10'"11
x Y(S x 10-')'+4 ,< 3.14' x (2x10-')' m/s
~7.6J" 101 m/s.

521. An electron beam is directed into a space contain-


ing a uniform electric fieJd of strength 1 kV/m and a
uniform magnetic fiel d of induction 1 mT, which is per-
pendicular to the electric lield. The velocity of the elec-
trons is constant and directed at right angles to vectors
E and B (Fig. t62a). Determine this velocity. What will

)---. ··:1F,,
(•) (b)
Fig. 182

be the motion of the electrons in the absence of an electric


field? What will be the radius of curvature of the trajec-
tory of the elec:trons in this case?
Ch, 3. Elec&ricity 323

Given: E=tOS Vim, B=t mT=10-1 T.


v-1R-1
Solutton. The forces acting on an electron in the elec-
tric and magnetic fields are
Fe1 = eE, Fm = FL = evB (sin ci = 1). (1)
Since the electron moves uniformly in a straight line, the
electric and magnetic forcesaremutually balanced. Writ-
ing the equilibrium condition for the electron in projec-
tions on lhe Y-axis (Fig.162b), we obtain
Fm-Fe1=0,
or, taking into account expressions (1), wB - eE = O,
whence
v = EIB, v = (I0'/10-') m/s = 10' m/s.
If we switch off the electric field, the electron will be
acted upon only by the magnetic field exerting on it the
force
Fm = FL =tVB.
Sioce the electron velocity is perpendicular to the Lorentz
force, the electron will move in a circle. According to
Newton's second law,
FL = mac.

~ª1!!º:i,~~ow:c:~~~}n e:~;~º~JJh,ª!.te!: fact that

R= ;;

R= ::J::~:;~:~. m=5.7xtO-'m.
522. Explain why two conductors carrying current.a in
the same directioo attract each other.
Answer. The 6rst conductor produces in the surrounding
space a magnetic 6eld whose direction can be determined
by the right.-hand screw rule (Fig. t63). The second conduc.-
tor will be in this field. Since it carries a current, it is
acted upon by the AmpQil force F1 whoae direetion can be
determined by the left.-hand rule. Similarly, we can show
21•
Selected Problem.1 on Physic1

Fig. 183 Fig. 16'


that the fi.rst conductor is acted upon by the Ampere
force F 1 equal to F 1 .
523. What will be the behaviour of two curren karrying
conductors arranged at right angles (Fig. 164)?
Answer. A conductor carrying a current / 1 produces in
the surrounding space a magnetic fi.eld of induction B 1 •
According to the left-hand rule, the Ampere force F 1
acting on the segment AO of the second conductor is di-
rected downwards, and the Ampàre force F1 acting on the
segment OC is directed upwards. This couple of forces
rotates the conductor AC about point O counterclockwise.
Similarly, we can show that at the same time the fi.rst
conductor starts to rotate about point O clockwise.

EXERCISES
524.. Determine the magnetic induction of the fi.eld
produced by an infi.nitely long straight wire carrying a
current of 5 A' at• a point lying at 2 cm from it.
525. Determine the magnetic induction of the fi.eld at
the centre of a circular wire loop of radius t cm, carrying
a current of 1 A.
526. Assuming that the electron in a hydrogen atom
moves in a circular orbit of radius 0.53 X to-• cm, de-
termine the magnetic induction of the field at the centre
of the orbit. Consider that a circular current equivalent
to the moving electron is 0.01 mA.
527. Calculate the magnetic induction in a sotenoid
with an iron core if 400 turns of wire are wound over 40 cm
of its length. The current in the solenoid is 8 A and the
permeability of iron is 183.
528. Currents of 30 A Oow in opposite directions
through two long parallel wires separated by a distance of
Ch. 3. Electricity 325

16 cm. Determine t.he magnetic induction oI the field at


a point located at 10 cm from each wire.
529. A force of 0.15 N is acting on a straight conductor
of length 0.5 m which is normal to lines of a magnetic field
of induction 2 X 10-1 T. Determine the current through
the conductor.
530. An electron is directed into a uniform magnetic
field whose induction is 20 mT at right angles to the field
lines at a velocity of 10' cm/s. Calculate the radius of the
circle in which the electron will move.
531. A proton moves at a velocity of 10' cm/s at right
angles to a uniform magnetic field of induction 1 T. De-
termine the force acting on the proton and the radius of
the circle in which it moves.
532. An electron describes a circle of radius 4 mm in a
magnetic field. The electron velocity is 3.6 X 10' mls.
Determine the magnetic induction of the field.
533. What is the direction of the force exerted by the
magnetic field of the Earth on a horizontal current-car-
rying conductor in the northern hemisphere if the conduc-
tor lies in the magnetic meridian plane and the current
Dows through it from north to south?

WORK DONE 1N MOVING A CUBRENT-CARRYING


CONDUCTOR IN A MAGNETJC PIEI.D.
ELECTROMAGNETIC INDUCTJON.
MAGNETIC PIELD ENERGY

The magneUe ftu through a plane surface of area S


in a uniform magnetic field of induction B is
e!>= BS cosa;,
where a; is the angle between the directions of the magnet-
ic induction vector B and the normal n to the plane sur-
face S.
The elemental'J' worlr. done ln movlng a eurrent-e•l'l'J'lng
eonduetor in a magnetic field is
dA = 1 d<I>,
where I is the current in the conductor and dei> the change
in the magnetic Dux.
326 Selected Problema on Phys.ics

The total work done in moTlng a eurrent-earrying eon-


duetor is
...
A=l /d<l>=/(<l>,-<1>1)=/M>.
...
The eml induced ln a solenold is connected with the
rate of change of the magnetic Dux through the relation

11=-N ~~
where N is the number of turns in the solenoid, â<I> =
cn, - cJJI the change in the magnetic Oux, and dt =
t1 - t 1 the time during which this change occun.
The direction of the current induced in a closed conduc-
tor can be determined by using Lenz's law. The emf induced
in a solenoid as a result of change of it.s own magnetic
nux is
l=-L !! ,
where L is the inductance of thesolenoid, M = / 1 - / 1
the change in the current in the solenoid, and ât =
t 2 - t1 the time during which this change takes place.
The lnduetanee oi a solenold is given by
L=JJ.J.1.onªlS,
where f1 is the permeability of the solenoid core, J.1.o
the magnetic constant, n = N ll the number of turns
per unit length of the solenoid, N the total number of
turns in it, l the solenoid length, and S the cl'088-sectional
area of the solenoid.
The energy of the magnetle &.eld of a solenold is
w= Ll'/2.

534°. A straight conductor having a length of 20 cm


and carryiog a current of 2 A moves uniformly in a uniform
magnetic field of induction t T. The velocity of the conduc-
Ch. 3. Electriclty

tor is 15 cm/s and perpendicular to the magnetic induc-


tion vector. Determine the work dona in moving the con-
ductor for 5 s.
Gtven: B=t T, !=20 cm=0.2 m, 1=2 A,
V= 15 cm/s=0.15 m/s, t- 5 s.

A-1
Solution. By definition, the total work done in moving a
conductor in a magnetic field is
...
A=l. , /d!I>, (1)

where d<!> = B dS is the magnetic nux through the sur-


face element dS swept by the conductor during ita motion
for a time interval dt {Fig. 165).
It can be seen. from the figure that dS is the area of the
parallelogram abb' a' with sidea ab = l and aa' = v dt.
Consequently, dS = lv dt, which givea
dll>-Blvdt. (2)
b B //

®
®[®-:.-®]®
® ®®
®® ®®
l dS

a1 -----2a'
Fig. 185 Fig. 188

Substituting expression (2) into the integrand of (1) and


integrating, we obtain
• t
A= lo IB/udt=IB/u 1o dt=IB!vt,
328 Selecled Problema on Pbyaic1

A=2x 1 x0.2x0.15x51 =0.31.


535. A Dat coil having a radius of 25 cm and formed by
75 turns is in a uniform magnetic field of ioduction
0.25 T. The plane of the coil forma an angle of 6Cf witb
the direction of the magnetic induction vector. Deter-
mine the torque acting on the coil in the magnetic field if
the curreot through the coil is 3 A. What work must be
dooe to remove the coil from the magnetic lield?
Gi11<n: B = 0.25 T, R = 25 cm =0.25 m, N = 75,
~-60"=n13 rad, 1=3 A.

M-'IAu.t-'I
Solution. The magnetic field exerts oo the coil contain-
ing N turns a torque
M = NpmB sin cz. (1)
Considering that Pm = 1S, S = nR2 , and ci = n/2 - 'P
(Fig. 166), we transform Eq. (1) u lollows:
M =Nlnll'Bsin (n/2-~)=NlnR'Bcos~,
M = 75 x 3x 3.14 x 0.25'cos(n!3) = 5.5 N ·m.
The work dooe by the magnetic field on the remova! of
lhe coil from it is
A =1(<11,-<!Ji).
On the otber band, in order to remove the coil from the
field, we must apply to it ao externai (say, meehanical)
force whicb will do the work
A.,.=-A=l(<ll,-<11,). (2)
Here «D1 = N BS cosa; aod Cl>1 = O sioce B1 = O. SJlb-
stitutiog these e:z.pressions ioto Eq. (2) aod consideriog
that S = TCR 1 , we obtaio
Âe1.t = 1 NBnJP cos (n/2-q:i) = 1 N&R2 sin q>,
Âut=3 X 75 X 0.25 X 3.14 X 0.25'sin (n/3)=8.6 1.
536. A circular loop haviog a radius of 5 cm aod car-
ryiog a curreot of t A is io a uoiform magoetic faeld of
Cb. 3. Eleetricity 329

induction 4 X to-s T. The loop is arranpd so that its


plane is perpendicular to the magoetic induction. What
work must be done to tum the loop through 90° about an
a:r.is coinciding with its diameter (Fig. t67)?
Given: R- 5cm=5x10""' m, /-1 A, <p=llO'=
n/2 rad, B=4x!O""' T.
A..1-1
Solutton. The initial position of a current loop is th&
position of stable equilibrium since the \arque in this
case is zero. Therefore, the magoetic 6.eld does a negative
work during the rotation of the loop, while the work done
by an e.zternal force is positive.' By de&nition, the work
done by the magoetic 6.eld to rotate the loop is
A =l(<l>,-<I>,), (!}
where cl>1 = BS cos o:1 and cl>1 = BS cos a 1 are the mag--
netic Duns through the plane of the loop in the initial
and 6.nal positions and S .,,, nR1 is the area of the surface
bounded by the current loop. Since a 1 = O and o:1 =
n/2, we can write
cl>1 = BnR1 cos a 1, cl>1 = BnR1 cos a 1 •
Then upression (1) assumes the form
A = /BnR 1 (cos a 1 - cos a 1).

ya

J(
~
®®®
..
181181181 B

®º® ®
y
o 4z X
Pir. t67 Pi1. t68
Selected Problema ou Phyaics

Since A nt = -A, lhe work done by the ext.ernal force


rotating the current loop is
Ânt = 1BnR1 (cos « 1 - cos « 1),
ÂRt = 1X4 X!()-• X 3.14 X (5x1<l-')'x(1-0)1
= 3.14 X 10-• 1.
537. A s\raight cooductor having a lengtb of 1.5 m
and moving wit.h a uniform acceleration of 10 mls1 in
a uniform magnetic field at an initial velocity of 3 m/s
covers a dist.ance of 0.5 m. Determine t.he mean emf induced
in the conductor. The magnetic induction of tbe field
is equal to 0.2 T and perpendicular to the velocity of the
conductor. Determine the instantaneous value of the emf
induced in the conductor at. the end of the path.
Gtven: l=1.5m, v0 =3m/s, a=10m/s", Az=0.5m,
B=0.2T.
(!f)-11-1
Solutton. According to Faraday's law, the emf induced
in t.he conductor moving ata certain velocity in a magnet-
ic fi.eld is
3'=-~ (1)

where AC!> = B AS is tbe magnetic Dux crossed by the


conductor during its motion in tbe magnetic field for a
time interval At (Fig. 168). The figure shows that the area
AS pierced by the magnetic Dux AC!> can be determined as
t.he area of the rectangle Oabc, i.e. AS = l Az. Then
M> = Bl llz and (soe Eq. (1))

l= - B~~ = -Blv. (2)


lf the motion of the conductor is nonuniform in lhe mag-
netic field, taking the mean velocity of tbe conductor for
tbe velocity v and using Eq. (2), we calculate tbe mean
value of the induced emf:
(!!) = -Bl(v).
Ch. 8. Electriclt.y

The mean velocity of a uniform.ly accelerated motion is


defined as
(v) = (v, + v)/2, v' - V: = 2a âz,
v:
whence v = V + 2a 6z is the final velocity of such a
motion. Consequently 1 ' (v) = (v0 + V V: + 2a 6.z)/2.
Theref01'8,
<l>=-Blu,+Y•]+=.
••~+7:~x""'io~x""o~.5 V= -0.99 v.
<I>= -0.2x1.s •+Jf~
Similarly, if v is the instantaneous velocity of the conduc-
tor, the instantaneous induced emf is
r~-Blv, (3)
where v = V v: + 2a 6z is the instantaneous veloci ty at
t.he end of t.he path. Then Eq. (3) becomes

li= -Bqfv:+2a âz,


11=-0.2x1.5 x y·"'3•;-+,-2"'"'x""'1""o'"'x""0"'.5 v = -1.08 v.
538:i. A curront loop rotates uniform.ly in a uniform
magnetic field of induction 4 mT at. a frequency of 20 s- 1 •
The area of the loop is 20 cm'. The rotational axis of the
loop lies in its plane and is perpendicular to the magnetic
induction vector. Determine the maximum magnetic flux
through t.he plane of t.he loop and the emf induced in
the loop during its rotation. Plot the graphs for the time
dependences of t.he magnetic Oux and the induced emf.
G'ven: B=4mT=4x10-ªT, v=20s-',8=20 cm'=
2x10-amt.
Cl>maz-? Faui:r.-?
Soluttan. The magnetic Oux through the plane of the
loop is
li>= BScoacz,
where ci is t.he angle between the normal to t.he plane of
the loop and the direction of the magnetic induction vec-
382 Selecl.ed Problema on Phyaica

tor. lf the loop rotates uniformly, the angle a. ehanges


with time according to the law ci = 6>t. Therefore,
<l>=BSeos6>1. (1)
According to Faraday's law of electromagnetic induction,
the indueed emf is
I=- ~~
or, taking into account Eq. (1),
i= -f..(BSeoaGJl)=BS.,sinUJt. (2)
Expression (1) shows that the magnetic nux through tbe
rotating loop has the maximum value when eos 6>t = t.
ln this ease,
fl> = fl>mu =BS,
fl>mH = 4 X to-1 X 2 X 10-1 Wh = 8 µWb.
Similarly, from expression (2) we can determine the max-
imum emf induced in the loop: lma:11: = BSm or, since
(1) = 2n'Y,

lmu = BS2.n'tJ,
l'mu: = 4 X to-a X 2 X 10-' X 2 X 3.14 X 20 V
"'1 mV.
Using Eqs. (1) and (2), we ean plot the graphs for the
time dependences of fl> and 3' (Fig. 169). Since co = 2n/T,
it. is convenient toconsider the instants of time t corre-
sponding to (1/4)T, (1/2)T, (314)T, and T.
539". A rectangular coil of area 500 cm2 , consisting of
200 tums of a wire, rotates uniformly in a uniform magnet-
ic lield about an axis pasaingthrough its centre and paral-
lel to one of the sides (Fig. 170) at. a frequency of 10 s-•.
The maximum value of t.he emf induced in the coil is
150 V. Determine t.he magnetic induction of the lield.
Gtven: 8=500 cmª=5xt0-Z m2 ,N=200,"Y=-10 r•,
1mai:=-150V.
B-1
Ch. 3. Electricity 883

'1>,pWb s nN
10
2.0
8
1.5
6
4
2
O
-2 T t,s
-4
-8 -1.5
-8
-2.IJ
-10 (a} (b}

Fig. 169

Solution. According to Faraday's law, the emf induced


in a coil consisting of N turns and rotating in a magnetic
field is
(t)

where <I> = BS cos Cilt and Cil = 2nv. Substituting these


expressions into Eq. (t) and difl'erentiating, we obtain
~= -N :, (BScos2nv)=NBS2nvsin2nvt.

Fig. 170
Selected. Problema oa Physics

The induced emf assumes the maximum value at the mo-


ments of time when sin 2nvt = 1:
IR = Mmu = NBS2nv,
whence

B= :~~'
8 200xsxto-!~2xa.t4xtO T~0.24T.
54.0. Ao aluminium ring is placed in a uniform magnet-
ic field so that its plane is perpendicular to the magnetic
induction vector. The diameter of t.he ring is 25 cm and
t.he thickness of the wire of which it is made is 2 mm. De-
termine the rate of change of the magnetic induction of
the field if the current induced in the ring is 12 A.
Given: D=25 cm=0.25 m, d=2 mm=2x 10-a m,
1-12 A.
IJ.B/IJ.t-?
Solution. According to Faraday's law,

IR=-~~.
Here cJ> = BS is the magnetic Oux through the surface
bounded by the ring. Since the area S of the surface is
constant, we can write
IJ.<l>-IJ.(BS)-S IJ.B, 3- -S ':::, (1)

whence l !!t.Bl!!t.t 1 = 'llS. The surface bounded by the


ring is a circle, and hence its area is
S - nD'/4. (2)
According to Ohm's law for a closed circuit,
g -IR,
where R = pl!Sw is the resistance of the ring. Since the
length of the ring is l = nD and the cross-sectional area
of the wire is Sw = ml'/4, the expression for the ring
Ch. 3. ElectricitJ 335

resistance can be written in the form R = 4.pD/d*. There-


fore,
li= 4pD//d'. (3)
Substituting npressions (2) and (3) into (1), we obtain.

I ABI=~
6t n~·

AB1 = 3.Mx0.25x(2xtêFi)•
1Tt tsxt2x0.28xtO-' T
s ~t.6 T/s.
Mt. AsolenoidconsistingofSOturnsand havinga diam-
eter of 8 cm ia in a uniform magnetic lield of induction
60.3 mT. The solenoid is rotated through an angle of 180°
for 0.2 s. Determine the mean value of the emf induced in
the solenoid if its axis ia directed along the field before
and after the rotation.
Given: N=80, d=8 cm=8x1Q-2 m, a 1 =0rad,
'l:i~;~~-~6~~·x ~~~. 2 ''
16-1
Solution. The emf induced in the solenoid ia

l=-N ~~, (!)

where âtl> = tl> 1 - tl> 1 = BS (cos a 1 - cos a 1 ) is the


change in the magnetic Oux piercing the solenoid during
its rotation through an angle of 180°. Since S = nd'/4.,
cos a 1 = -1, and cos a 1 = +1, we have
M> = -2Bnd'/4 = -Bnd'/2. (2)
Substituting Eq. (2) into (1), we find that

I= N::•
I= 80x80.3xto-;~~:~'x(8x t0- 1 ) 1 V~ 0 _24 V.
M2. A coil having a negligibly low resistance and an
inductance of 3 H is connected to a current source with an
emf of 15 V and a negligibly low internai resistance. 1n
338 Selected Problema on PhJBics

-what time will the cummt induced in the coil attain a


'Value. of 50 A?
Giwn: l',=15V, /=SOA, L=3H.
t-?
Solutton. Aceording to Ohm's law for a closed circuit,

IS= l(R + r),


where 1 is the total emf in the circuit, which in the case
under consideration ili the sum of the emf 1 1 of the source
and the emf 1 1 of self-induction emerging in the coil
after it.s connection l9 the source. Consequently,

11=16,+ll,=16,-L*=l(R+r), (1)

where 11 1 = -L~ is the emf of self-induction.


By hypothesis, the resistances R and r are negligibly

L*
low, and hence Eq. (t) can be written in the form 1 1 -
~ O, whence the rate of change of the current Íl!I
(2)

Since the rate of change of the current is eonstant,


l = ~! t, whence t= ~~t or, taking into aecount
Eq. C2),

t= ~~
t= ~~ 3 8=10 s.
543. A solenoid of length 60 cm and diameter 10 cm
-contaios 1000 turns. The current in it increases uniformly
by 0.2 A per second. A ring made of copper wire of crosa-
sectional area 2 mm 1 is put on the solenoid. Determine the
current induced in the ring.
Ch. 3. Electrieity 337

Gtven:l=60 cm'=>I0.6 m,D=10 cm=O.tm, N=t03,


ll//llt=0.2 A/s, S.,..=2mm2 =2 x to-em2.
Ir-?
Solution. The change in the current through the sole-
noid leads to the emergence of an emf of self-induction
i=-L::.
where L = l'P.on11ZS is the inductance of the solenoid.
Since n = Nll and S = n/J 11/4., the inductanee of the so-
lenoid is
L = ~~,nD'N'l(4l).
Therefore,
'I= _ l'llo~~ 1N 1 ~.
The emf ir induced in the ring is smaller than the oh-
tained value of the emf of self-induction in the solenoid con-
sisting of N turns by a factor of N, i.e.
i,=f=- ""':.D•N "*.
According to Ohm 's law for a closed cireuit, the current
(!)

induced in the ring is


l,= li, llR.,
where R r = plrlSw is the resistance of the ring. Since
lr = nD, we have Rr = rmlJISw. Therefore,
I, = 1f,1 Swl(npD),
and using expression (1), we ohtain
Ir= llflo&DS"' ~.

Ir 4x3.1-6~!~~6xx1i.7~01~~2x1Ct-I x0.2A~t.23mA.
544. A solenoid of length 50 cm and diameter 0.8 cm
consists of 20 000 turns of copper wire and is under a
constant volt.age. Determine the time during which the
amount of heat liherated in the solenoid winding is equal
to the magnetic fiel d energy of the solenoid.
'22-0970
338 Seleeted Problems on Physics

Given:l=50cm=0.5 m, D=0.8 cm=0.8ytO-tm,


N=2Y 10'.
t-?
Solutton. According to Joule's law, the amount or heat
liberated in the solenoid winding carrying a direct cur-
rent during a time t is
Q = I'Rt,
where R = pllSw is the resis'8.nce or t.he copper wire used
for the winding. The leng\h of the wire wound on t.he
solenoid is equal to the product of the length or a t.urn by
lhe number of turns in the solenoid: l = nDN. The cross-
sectional area of t.he wire is Sw = :n:~/4. Since the turns
are wound wi\hout gaps, the diameter of the wire is
dw = l/N. Then S.,, = nl1 /(4N1 ) and R = 4pDN1 /l2.
Consequently,
Q = 4I'pDN'tll'. (!)
The magnetic field energy of the solenoid is
W = LI'/2, (2)
where L = Jlflo'J 2lS is the inductance of the solenoid .
.Since the number of t.urns per unit length and the cross-
sectional area of the solenoid are n = N l l and S =
nD''/4., tbe induc\ance of the solenoid is
L = µµ,nD'N'/(41). (3)
Using Eq. (3), we can write Eq. (2) as fol1ows:
W = µµ,nD'N'I'/(81). (4)

~1~~;.:l,,j!~~:~~ii~~~~. b!h~~~:thesis, we obtaio

t= e:;;_:i
4x3.t4x10-1 x t x3.t4xO.Bx tO-Sx0.5
32xt.7xt0:0 x2xt01 5
=l.45x1()-• s.
545. ln wha\ direction will the curreot Oow through an
Ch. 3. Electricity ...
ammeter (Fig. 171) at the moment of diaconnection of the
circuit. by t.he key K?
Answer. Bafore the circuit is disconnected, the current.s

s
J 1 and J 1 Oow through the solenoid and the ammeter

t
Fig. t7t

spectively. After the diaconnection, the emf of self-induc-


t.ion will appear in the solenoid and will tend to main-
tain the current in the solenoid at the previous levei. This
induced current will pass t.h:rough the ammeter, and the
pointer will be diOect.ed in t.he derection opposite to the
initial direction.
EXEllCJSES
566. What work must be done to move by 25 cm a con-
ductor having a length o( 40 cm and carrying a current
of 21 A in a uniform magnetic fi.eld of induction t.2 TI
567. A rect.angular coil of area 40 cm 1 is in a uniform
magnetic lield of induction 0.06 T. The coil consists of
200 turns and can be rotated about an axis perpendicular
to the magnetic lield lines. When a current of 0.5 A is
passed through the coil, it is turned so tbat it.s plane b&-
comes perpendicular to the magnetic lield lines. What
work must be done to rotai.e the coil from this positfon by
1/4 of a tum; by 1/2 of a tum; by a complete t.um?
548. A Dat loop of area 10 cm1 is in a uniform magnetic
li:eld perpendicular to its plane. Determine the current in
the loop if the fi.eld decreases ata constant rate of O.t Tis
and the resistance of the loop is 10 Q.
569. The current in a coil having a length of 50 cm, a

...
diamet.er of 10 cm, and consisting of 1000 turns increases
... Selected Problema on Physies

uniformly hy 0.1 A per second. A ring made of copper wire


of cross-sectional area 2 mm 2 is put on tbe coil. Deter-
mine tbe current in the ring, assuming that t.he magnetic
nw: through the solenoid and the ring is the same.
550. A square loop made of copper wire and having an
area of 25 cm 2 is placed in a magnet.ic field of induction
0.1 T. The normal to the loop is parallel to the magnet.ic
induction vector. Tbe cross-sutional area of the wire is
1 mm2 • Wbat charge will pass througb tbe loop after
awitehing off the fiel d?
551. A coil baving 30 turns rotatu about. a horizontal
axis lying in its plane and perpendicular to the magnetic
meridian plane at a frequency of 10 s- 1 • The magnet.ic
fi.eld strengt.b of the Earth is 40 A/m. The ma:r.imum emf
induced in the coil is 0.001 V. Determine the area of the
coil.
552. A fi.eld of induction 0.7 Tis produced between the
poles of a dynamo (d.e. generator). The arm.ature of the
generator coosists of 100 turns having an area of 500 cm1
each. Determine the rotational frequency of the armature
if the maximum emf induced in it is 200 V.
553. A current of 2 A is passed through a 20-em long
coil ha ving a diameter of 3 cm and eonsisting of 400
turns. Determine the inductanee of the eoil and tbe mag-
netic Ou:r. pierc.ing it.
554. If a current passing tbrough a solenoid changes by
50 A per second, an emf of self-induction of 0.08 V appears
across the ends of the winding. Determine the induct.ance
of lhe solenoid.
555. A coil eonsistingof 100 turns is sbort-eircuited and
plaeed in a magnetic fi.eld of strength 9.6 kA/m. The area
of a turn is 5 cm1 and the planes of the turns are perpen-
dicular to tbe magnetic fi.eld lines in thecoil. Whatcharge
will pass tbrougb tbe coil as a result of its removal
from the field? The coil resistanee is 2 O.
556. Tbe winding of an electromagnet bas an induct.ance
of 0.5 H. a resist.ance of 15 r;J, and is under a constant
voltage. Determine the time during wbicb tbe amount of
heat liberated in the winding is equal to the magnetic field
energy of tbe core of the electromagnet. ·
557. A closed solenoid with an iron core of lengtb
Ch. 3. Elec&ricity 341

150 cm and cross-sectional area 20 cm 1 contains 1200


turns. Determine the magnetic lield energy of the solenoid
if the current through it is 1 A and the permeability of
iron is 1400.
QUESTIONS FOR REVISION

&ion. H. Formu te Lenz


tromape&ic induction.
expreuion for the emf of &he • 15. Defme inducLance.
!.t·.,w:~ fs' i~= :1'~!d1~1::t;~to1i1 ~h':'!=eu~76e\d
energy of a solenoid?
Chapler 4
OPTICS

4.1. Basic Quantltles


and Laws ln Photometry
Lumlnous ftux is defined as the radlant energy W emit-
ted by a point source per unit time:
<l>-Wlt.
Luminous intensity is de6.ned as the luminous Oux <b
emitted per unit solid angle (!) by a point source in a given
direction:
I - <!>/.,.
Solid angle i9 a spatial angle bounded by a conical sur-
face whose vertex coincides with a point source and the
area S of the base is a part of a spherical surface of radius
R:
(!) = SIR 1 •

Illuminance is de6.ned as a luminous Oux incident on a


given surface S per unit area of the surface:
E= 4> 1n/S.
The illuminance produced by a point source at a dis-
tance r from it is
E = I cos a./r2 ,
where ex is the angle between the incident ray and the nor-
mal to the surface at the point of incidence.
Lumlnous emittanee of a source is defined as the lumi-
nous ftux <b emitted by a bright surface S per unit area
of the surface:
R-<l>IS.
Lumlnance (brlghtoess) of a source in a certain direc-
tioo qi is de.6.ned as the luminous intenaity 1 of the souree
Ch. 4. Optics ...
in this direction per unit area of the surface S of the
source:
B = Il(S cos ~),
where q:i is the angle between the normal to the surface and
the direction of observation.
The luminous emittance and the luminance of a source
are connected through the relation
R =nB.

558. Calculate the luminous Oux incident on an area


element of 10 cm1 placed at a distance of 2 m from a
source of luminous intensity 200 cd.
Gtven S=tO cmZ=f0-3 mz, r=2 m, /=2X toz cd.
'G>-1
Solutton. We assume that the source is at the centre of
the sphere of radius r. The area element Sisa part of the
spherical surface. Then the illuminance of the area ele-
ment is
E =llr1 (!)
since a. = O. On the other hand,
E= G>/S. (2)
Equating the right-hand sides of relations (1) and (2),
we obtain Ilr" = 0/S, whence

oi>=* o;
Clt= 2 x 1 x i<r* lm=0.05Im.
1

559. Two electric lamps placed close to each other illu-


minate a screen located at a distance of t m from the
lamps. One of the lamps is switched oi. To what dis-
\8.nce should the other lamp be brought closer to the screen
for it.s illuminance to remain unchanged?
Gtven: r 1 =1m.
àr-1
... Seleeted Problems on Physics

Solutton. The illuminance of t.he screen produced by


the two lamps is
E 1 = E 1 +E, = 211~.
where E 1 and E 1 an tbe illuminances of the screen dueto
each lamp separately.
After one lamp hu been switched oft and the other has
been broughtcloserto t.he screen, its illuminance becomes
E 11 = llr:,
wbere r 1 is a new distance from the lamp to tbe screen.
By hypothesis, E 1 = Ew and hence 21/r:_ =/Ir:,
from which we get r 1 = Jf;:t2 = r1IV2. Consequent.ly,
tJ.r = "• - ,., =ri - r11V2 = r1 c1 - t/Jf2},
âr = 1 (1 - 111.41) m "'0.3 m.
560. A screen and a plane mirror whose surfaces are par-
allel are placed at t.he same distance of t m on both
Screer1

• ..
~
1f----10---<'---b - ,.,.._,.,,,,,.
-
2 s•
1

Fig. t72
sides of a pointsouroeof2cd(Fig.172). What is t.he illu-
minance at the centre of the screen?
Giuen: r=tm, /=2cd.
E-1
Solution. The illuminance of t.he screen is produced by
the rays directly emitted by the source S (e.g. ray 1)
and the rays incident on the screen after t.he refiection by
the mirror (e.g. ray 2). Tbe rays incident on the screen
after the reOection by the mirror cao be regarded 88 the
rays emitt.ed by a source S', viz. the virtual image of the
source S formed by the mirror and located at a distance r
behind it. Since the solid angle io which the rays propa-
gate does not change 88 a result of reOection by a plane
mirrar, the luminous iotensity of the sourçe S' is the same
Cb. '- Optics

as that of the source S. Then tbe illuminance of the


scre~m is
E = E 1 + E 1 = /Ir: + Itr:,
where r 1 = r and r 1 = 3r. Consequent.ly,

E-=-fs-+ 9~ = ~~ ,

E= ~: 12 h: ~ 2.2 Ix.
561. Through what angle should an area element be
turned for it.s illuminance to decrease by half in compari-
son with tbe value correaponding to the normal incidence
of the rays?
Given: E 1 =0.5E1•
a-1
Solu.Uon. The illuminance of the area element for the
normal incidence of the rays is
E, - Ilr'. (1)
Tbe illuminance of the same area element for an oblique
incidence of the rays is
E, - I cos a.Ir'. (2)
By hypothesis, E 1 = 0.5E1 • Using expressions (1) and
(2), we find that 1 cos a./r8 = 0.5//r2, wbence
cosa. = 0.5, a.~ t.05 rad.
562. A lamp suspended at a beigbt of 5 m illuminates
an area element on the ground. At what distance from tbe
centre of tbe area element is the illuminance of tbe ground
surface smaller than that at the centre by a factor of two
(Fig. 173)1
Glven: h-5m, E 0 -2E.
!-1
Solution. Tbe illuminance of the ground surface at the
centre of the area element ÚI
E 0 =llh1 •
Seleeted Prohlems on Physica

The illuminance of the ground aurface at a diatance


from the centre of the area element is
E= /cosa/ri.
The fi.gure shows that coa a = klr and r = Vh2 + 12 •
Then E= Jhl(}Íh 2 +1 2) 1 • Considering that E 0 = 2E,
we obtain J/h1 =2/h/(}Íh2 +11 ) 1 , whence
l=h Vv:1-1,
l=5 V V4-tm<><3.83m.
563. A dome in the form of a hemiaphere of radius t m
is illuminated by two identical lampa su.spended at a
height of 2 m above the ground and separated from each
.other by the same distance (Fig. 174). Determine the illu-
minance at the points of the hemiaphere located at the
minimum diatance from the sources if the total luminoua
flux produced by each lamp ia 1():111 lm.
Given: R=1m, h=2m, l=2m,<1>=1():111Im.
E-1
Solution. Point C lying on the straight line connecting
a source A with the centre O of the hemiaphere ia at the
minimum diatance from the aource A (.see Fig. 174). The

Fig. 173 Fig. 174


Ch. 4. Optics 3'7

illuminance at point c is produced by the sources located


at points A and B and separated from it by the distances
1AC 1 = r 1 and 1BC 1 = r 1 • Consequently, the illu-
minance at point c is
E = I cos a 1/r: + 1 cos a 1fr:, (1)
where
1 = <l>/(4n), "• = O (2)
since the ray AC propagates along the radius of the hemi-
sphere and coincides with the normal to its surface at the
point of incidence. The angle a 1 can be determined from
D.ABC by using the cosine law:
cos a 2 = (r: + r: - P)/(2r r
1 1 ). (3)
lt follows from the fi.gure that
r, - 1AO 1- 1OC 1, (4)
where 1OC 1 = R. From D.AOD, we find that
1AO1-Y1 ADI'+ 1OD I'- Vl'l4+h'.
Since, by hypothesis,
l = h = 2R, (5)
we have 1AO1 = RV5 ~ 2.24R. Substituting the ex-
pressions for 1AO 1 and 1OC 1 into Eq. (4), we obtain
r, - 2.24R - R "' 1.24R. (6)
Applying the cosine law to D.ABC, we fi.nd that
r.= r! + P - 2r1l cos Jl. (7)
The value of cos p can be determined from D.AOD by con-
sidering that 1AD 1 = l/2 = R. This gives
cos ~ = 1AD Ili AO 1 = R/(2.24R) <><0.45. (8)
Substituting expresaions (6), (5), and (8) into (7), we ob-
tain
r: = (1.248)1 + 4R1 - 4 X 1.24R1 X 0.4.5 ~ 3R 1 ,
whence
r 1 s:rw1.73R. (9)
Seleeted Problema oa Physics

Subst.U.uting expressions (5), (6), aml (9) into (3), we get


cos ª• < 1 ·~~:~·~3 :!;;'8' - 0.12. (tO)
Substituting expressions (2), (6), (9), and (10) into (1),
we fi.nally obtain

E=! [ (1 .:!a,. + <1 ~7~,. J~ ~~ ~o.055;.,


E= o.~x101 l:r.=55lx.
564. Determine the illuminance of tbe Eartb 's surface
produced bysolar rays incident along tbe normal to t.he
surface. Tbe Sun's luminance is t.2 X t0 11 cd/m 1 • The dis-
t.ance between tbe Eartb and tbeSun is t.5 X to• kmand
t.be Sun's radius is 7 X t0 6 km.
Gtven: B=t.2xt0' cd/m•, r=t.5xt08km=
t.5 X tOlt m, R=-1X105 km =-7 X to• m.
E-?
Solution. Since tbe distance from the Earth to t.be Sun
is very long, we assume tbat the rays emit.ted by lhe Sun
are incident. on t.be Eart.b in a parallel beam. Assuming
tbat. tbe Sun is a plane luminous disc, we fi.nd tbat its
luminance is
B - 21/S,
wbere S = nR 1 • The coefficient 2 is int.roduced since t.be
fiai. diBc emits in two directions. Tben B = 2//(nR 1),
whence
I -nBR'/2. (1)
By bypot.besis, coa ci = t. Tben t.he illuminance of t.he
Eart.b 's surface is
E= Ilr1 • (2)
Substituting expression (1) into (2), we obtain
E=~·
E a.14x21~2,;~~~Jl,~10I')' 1x~sxt()f.Ix.
Cb. 4. Optic1 349

565. Ao eleetric bulb whose luminous intensity is


100 cd is enclosed in a froated 1pherical dome of diame-
ter 5 cm. Determine tbe luminous emittance and the lu-
minance of tbe bulb, neglecting the light absorption by the
glass of which the dome is made.
Given: /=100 cd, D=5 cm=5x 10-2 m.

.. R-18-1
Solution. Tbe Iuminous emittance of a source is de.fi.ned

R-11>/S,
where clJ = /fJ> is the emitted luminous Oux, fJ> = 4n
tbe total solid angle, and S = nD 1 • Therefore,

R= !~ = ~,
R= c::11:1)I !:. =1.6x 1()11Im/mZ.
The luminance of the bulb is

B=~,
B=1.:.~4 i0' ~=5.1 x tO'cd/m1 •
566. A lamp of luminous intensity 60 cd is used for
printing photographs. If the lamp is placed at a distance
of 1.5 m from a photograph, the exposure time is 2.5 s.
Determine the exposure time for a lamp of luminous inten-
sity 40 cd located at a distance of 2 m from tbe pboto-
graph.
Given: / 1 =60 cd, r 1 =1.5 m, t 1 =2.5 s, / 1 =40 cd,
r 2 =2m.
t,-?
Solution. The radiant energy received by a photograph-
ic paper illuminated for the time t is equal to the prod-
uct of the luminous Oux clJ and the exposure time t:
W-lf>t-ESt.
350 Selected Problema on Physics

Consequently, for the two cases we ean write

The quality of the photographs is the same if the same ra-


diant energy W 1 = W 1 falis on the pbotographie paper,
or, taking into aeeount Eq. (1), E 1St 1 = E 1 St 1 , whence
t 2 =E1t 1!E 1 •
Aeeording to the law of illuminanee, E 1 = J 1/r: and
E,, = 111,-:. Therefore,

t, -!.d!!
- tf/1'

t 1= 60 ~
1 5!~x~.s s=6.64s.

567. Why is it diffi.eult to see in tbe daytime lhe inte-


rior of a room through a window glau from outside
without approaehing the face close to tbe gl888?
Answer. Two luminous Ou:r.es propagate from the win-
dow outside: the lirst is produced hy the rays reDected by
the window glass and lhe second by the rays pU!ing
through the window from the room. Since tbe second Dux is
produeed hy tbe rays multiply reOeeted by lhe room walls,
its intensity is mueh loW81' than that of the first Dux due
to absorption and seattering of light upon reOeetion. For
this reason, the observer sees the glittering glass rather
than the interior of the room.
568. The snow on the sloping surfaces of roofs usually
melts sooner than on the ground. Why?
Amwer. ln medium latitudes, the Sun at noon is not at
the zenith, and hence the solar rays are incident on a hor-
izontal surface at a certain angle. If a roof has a slope,
the angle of incidence of the rays on it is smaller than the
angle of incidence on the horizontal surface of the ground.
Since the illuminance of a surface is defined as E =
1 cos a.Ir", the illuminance of the roof is larger than that
of the ground, whicb causes a rapid melting of snow on the
roofs.
Cb. 4. Optics 351

EXERCISES

569. A lamp of luminous intensity 400 cd is suspended


on a pole of height 6 m. Calculate the illuminance of the
ground at 8 m from the foot of the pole.
570. A source of light should be plaeed between two
sereens so that the left screen must be illuminated twice
stronger than the right sereen. At what distance from tb&
left screen must the source of light be placed if the sep-
aration between the screens is 100 em?
571. The light from an eleetric bulb of luminous inten-
sity 200 cd is ineident on a small horizonia.I area ele-
ment at an angle of 45°, produeingan illuminance of 141 lx.
Determine the distanee r between the lamp and
the area element and the height h at which the lamp is sus-
pended (see Fig. 173).
572. Two lamps of luminous intensity 200 cd each are
suspended on a pole at 3 and 4 m above the ground. De-
termine the illuminance of the ground at 2 m from the
foot of the pole.
573. A lamp is suspended from the ceiling and is be-
tween a vertical picture anda plane mirror. Determine the
luminous flux ineident on the picture of area 0.5 m1 if the
distance from the lamp to the picture and to the mirror is
4 and 2 m respectively. The luminous intensity of the
lamp is 96 cd.
574. Two point sources are 2 m apart. A small area ele-
ment is arranged at an angle !l on the perpendicular pass-
ing through the midpoint of the line segment eonneeting
the sources at t m from the line. At an angle !l = 15°,
the illuminance of both sides of the area element is the
sarne and equal to 20 lx. Determine the luminous inten-
sities of the sources.
575. A lamp in whieh light is emitted by an ineandea-
cent sphere of diameter 3 mm has a luminous intensity
of 85 cd. Determine the luminanee of the lamp if its spher-
ieal bulb of diameter 6 em is made of a transparent glass.
576. Determine the illuminance of the edge of a round
table of diameter 1 m if it is illuminated by a lamp sus-
pended ata height of t m from the eentreof thetable. The
total luminous Oux of the lamp is 600 b:.
Selected Problema on Phyaica

577. A lamp of luminoUlil intensity 1000 cd is SUlilpended


at 8 m from the surface of the ground. Determine the area
-0f the surface whose illuminance is not lower than t lx.
QUESTIONS FOR REVISION

t. Wbat ia luminoua ftm:? ID what units ia it meuured? 2. What


ia lumiJloWI iDteDSity? ln what uuits ia it meuu.red? 3. Define
aolid angle • .fi. Wbat ia illumiDuce? 5. Write a formula for cal-
eulating the illumiJlance produced by a point 80\lrce, 6. Define 11.t-
miDoua emittauce. ln what units ia it e:z:preued? 7. What is Lhe
lumiD811Ce of a aource? ln what unita is it meuured? 8. Write a
formula connecting the luminoUB emitlaDce and the lumiDuce of
a aource ..

4.2. Geometrlcal Optlcs


REFLECTIOlf OF UGBT. MIRROBS

The reftectfon oi llght at the interface between two me-


dia obeys the following law:
The incident ray AO, the refiected ray OB, and the nor-
mal OC to the reDecting surface at the point of incidence
-0f the ray lie in the sarne plane. The angle of incidence t
is equal to the angle of reDection ti (Fig. t 75).
lf the rays incident on the interface between two media
are parallel, they remain parallel after reOection. ln

Fig. 175 Fig. 176


Ch. 4. Oplies 353

this case, we have a specular refteeUon, and the reOecting


surface :Ui a plane minor.
When constructing the image of an object formed by a
f~:::ie am;~~· ~n:h:~~~~t(:;~:rn~i::~: 1ir:.y116).e~~~~
agate after reDection so that their continuations will
intersect behind the mirrar at point A' which is a virtu-
al image of point A. Similarly, we construct an image

Fig. 177

B' of point B. Therefore, the image of the object is erect,


virtual, and equal in size to the object wbich is symmetri-
cal to it about the plane of the mirrar.
If a reOecting surface is spherical, we have a spherieal
mirror. Tbe centre of the spherical surface (point C)
is the opUcal centre of lhe mirror (Fig. 177). The vertex O
of the spberical segment is known as the pole of the mir-
ror. Tbe straight line passing through the optical centre
and the pole of the mirrar is the principal optical axfs oi
lhe mirror. Any straight line passing through the optical
centre of the mirrar is called an auxiliary opUeal axis.
Spherical mirrors can be concave or convex.
A parallel bundle of rays incident on a eoneave mir-
ror converges at a single point, viz. the foeus of the mir·
ror, after reflection. Tbe focos F lying on the principal ap-
licai axis is called the principal foeus of the mirror
(Fig. 178a). Tbe geometrical locus of ali foci forms a foeal
plane.
A parallel bundle of rays is reOected by a eonvex mir-
rar so that the continuations of the reOected rays intersect
behind the mirrar at a single point F known as the virtu-
al foeus of the mlrror (Fig. 178bl.
23-0970
h*
35' Selected Problems on Physics

--.
·~ F (a}
/
' , (b)

Fig. 178

White constructing images formed by spherical mirrors,


it is convenient to make use of the following rays inci-
dent on a mirror:
(a) a ray parallel to the principal optical axis of the
mirror, which after refiection passes through the princi-
pal focus;
(b) a ray passing through the principal focus, which
after reflection propagates parallel to the principal opti-
cal axis:
(e) a ray passing through the optical centre of the mir-
ror. which after reflection propagates along the sarne line
in the backward direction.
The focal length 1OF1 = F (focus) of a mirror having
a radlus R of curvature is
F = R/2.
lf an object is at a distance d from the mirror and its
image is at a distance j from it, we have
±lld ± li/ = ±1/F.
ln this formula, lhe distances from the mirror to real
points are taken with the plus sign and thedistances from
the mirror to virtual points with the minus sign.
The ratio of the linear size A' B' of the image to the lin-
ear size AB of an object is called the magnifi:cation of .a
mlrror:
r = IA'B' lilABI =//d.
Ch. 4. Optics 355

578. A plane mirror AB can rotate about a horizontal


axis O. A light ray is incident on the mirror at an angle
~he ~~t:i1::! ~~e ':!:!r!~et~:!=:~:nr:~t: ;°~~~dt :L}?
Given: a, p.
y-?
Solutton. As the mirror is rotated through the angle
p, the normal to the mirror will also he rotated through

;·~:~: laJ
Fig. 179
(b)

s
M

Fig. 180

the angle p, and bence after the rotation of the mirror,


the angle of incidence is a + p, while the augle between
the incident and reftected rays is 2 (a + p). Before the
rotation of the mirror, the angle between the ineident and
reDected rays was 2a. Consequently, the reflected ray
will he rotated through the angle (Fig. 179b)
y = 2 (a + ~) - 2a = 2~.

in:l!~fo~i~h~!~c.'i=~~~':J aª~:iethi!~:i~:np~~r.:d
Selected Problems .on Phys1cs

on the path of the ray, the light spot on the screen is dis-
placed upwards by 3.5 cm (Fig. t80). Determine the an-
gle of incidence of the ray on the mirror if the distance
hetween the mirror and the screen is 50 cm.
Gtven: h=3.5cm-3.5 X tO-Zm, l=50cm = 0.5m.
t-1
Solution. Let us consider the path of the ray SO inci-
dent at point O of the plane mirror MN. According to the
construction, DO is the perpendicular dropped on the
mirror, CO the perpendicular to the horizontal line SA,
and OB the reOected ray. The figure shows that LDOC =
:r.12 - t, LCOB = :r.12 - ci, and LDOB = i. Then
(n/2 - !) + ("12 - <>) = t, whence
t= (n-e>)/2. (1)
On the other hand, we can write
tan<> =IABi/IOAI =hll"""· (2)
Substituting expreuion (2) into (t), we find tbat

I= n~h/l

i= 3.14-3.S:to-ito.5 t.535rad.

ca:~\:::~~o i:i~ !~:::::-:~a~r:l~f~~!~!º:~:h


other?
Given: a= 45º ~O. 785 rad.
n-1
Solution. If the bright point A 0 is placed between mir-
rors 1 and 2 (Fig. t81), the rays emerging from it will be
incident on the mirrors and after undergoing multi.pie
reOection will form virtual images of the sources on their
continuations. Constructing consecutively the images of
point A 0 formed by mirrors 1 and 2, we find that A1 is
the image of point A 0 formed by mirror J, A 2 and As
are the images of points A 0 and A1 formed by mirror 2,
A, and A 5 are the images of points A 1 and A 1 formed by
Cb. 4. Optics 357

mirror J, and 6.nally, A 7 and A8 are the imagea of poiots


A 4 and A 6 formed by mirror 2. Ali the subsequent imagea
will coincide with the ones obtaioed earlier. Thus, seveo
images will be formed by the mirrors arranged at 45°.
581. The radius of curvature of a concave mirror is
80 cm. At what distance from the mirror must ao object
be placed for its real image to be twic:e as large as the ob-
ject?
Given: R~80cm~0.8m, \A'B'i~21ABI.
d-?
Solution. We place the object AB between the focus
and the optical centre of the mirror and construct its
image (Fig. 182). For tbis purpose, we consider the rays
emerging from point A of the object AB. Of all these
rays, incidentonthemirror, wechoosethe ray parallel to
the principal optical axis. After reOection by the mirror,
it will pass through the principal focus F. For the second
ray, we take the one propagating from poiot A through
~~i9 t~~b! ~ri~!f;~lreo~et~~~}º~x~~~ Th~ ;~li~~~f8:rª~:r:!=
tion of the reOected rays will be a real image of point A.
The images of all other points of the object AB will lie
on the perpendicular dropped from point A' onto the
principal optical axis. Consequently, A' B' will be a
real, magnified, and inverted image of the object AB.

Fig. 181 Fig. 182


358 Selected Problema .on Physics

ln order to determine the distance from the object to


lhe mirror. we ean use the formula for a eoneave mirror:
1/d + 1/f = 1/F.
Considering that F = R/2 and 1A' B' 111 AB 1=
fld = 2, i.e. f = 2d, we find that 1/d + 1/(2d) = 2/R,
whence
d= 3R/4,
d = (3 X 0.8)/4 m = 0.6 m.
582. Determine lhe principal focal length of a mirror
if a bright spot and its image lie on the principal optical
axis of the concave mirror at t6 and 100 cm respectively
from the principal focus.
Given: a=16cm=0.16m, b=100em=1m.
F-?
Solution. Let us eonstruet the image of the bright spot
S lying between the centre of the mirror and its principal
focus. For this purpose, we consider two rays, viz. lhe
ray 80 propagating along lhe principal optical axis and
an arbitrary ray SA (Fig.183). After reDection by lhe mir-
ror, the ray 80 will propagate in the opposite direetion
along lhe principal optical axis. ln order to fi.nd the path
of the ray SA after reDection by the mirror, we draw the
focal plane M N and an auxiliary optical axis CD paral-
lel to Lhe ray SA, which will intersect the focal plane at

Fig. 183
Cb.. 4. Optics 359

point B. The ray SA must pass through this point after


reflection by the mirror (aecording to the property of the
bundle of rays parallel to an auxiliary optieal axis).
The image S' of the bright spot S will lie ou the prineipal
optical axis at the point of intersection of the continua~
tions of the refleeted rays.
ln order to determine the principal focal length F, we
can use the formula for a concave mirror:
tld +li/ = 1/F,
where d = 1OF1 + 1FS1 = F +a, and / = I OF 1 +
1 FS' 1 = F + b. This gives
1/(F +a) + 1/(F + b) = llF,
whence
F=]/iiii,
F=]/0.16x1 m=0.4 m.
wnr'i!letb!e!: fi18th!ª~f3t ~r:,v: :resdeti~!:~~tha~ee~d1;
principal focus of the mirror.
583. A concave mirror is placed on the path of a
converging bundle of rays so that the point of intersection
of the rays is behind the mirror at 20 cm from its pole
(Fig. 184). The rays reftected by the mirror converge at a
point at a distance equal to half the focal length of the
mirror. Determine the radius of curvature of the mirror.
Given:IA0i=20cm=0.2m, IOA,I =F/5.
R-?

u.: _____ o~ 1
---
,,
Fig. tM
360 Selected Problems on Physics

Solution. If we place a point source of light at point


A 1 , its virtual image will be at point A (according to the
principie of reversibility of paths). Consequently, if we
assume that point A 1 is ao image, point A will be the vir-
tual source corresponding to the image. ln this case, the
formula for a concave mirror has the form
-1/d + li/ -1/F,
where d - 1 AO 1. / - 1 A,O 1 - F/5, and F - Rl2.
Therefore,
-1/d + 1/IR/(5 x 2)1 - 1/(R/2),
whence
R -Bd,
R - 8 X 0.2 m - t.6 m.
584. A screen MN is placed infrontof a convex mirror
at 5 cm from its pole (Fig. 185). An object AB of height
H

Fig. 185

3 cm is situated at 5 cm from the screen. At what posi-


tions of the eyewill ao observersee the image of the entire
object? What is the maximum size of the object (for a
given arrangement of the object, mirror, and screen) for
which the mirror will form the image of the entire object?
The diameter of the mirror is 10 cm.
Ch. 4. Op\ies 361

Gtven: b=5cm=5xto-111m, a=5cm=5xtO-Z m,


IABl=3cm=3Xlo-"m, D=IOcm=O.tm.
IAKl-1
Solutton. Let A 1 8 1 he the virtual image of the object
AB. Obviously, 8 1 will he the point of intersection of the
continuations of the rays reftected by the mirror and lying
within the cone CB1G. Similarly, A 1 will be the point of
intersection of the continuations of the rays reflected by
the mirror and contained in the cone AA 1E. ln order to
see the image of the entire object AB, the observer's eye
must he in the space in front of the mirror between the rays
lC and A 1 E. The maximum size of the object is deter-
mined from the similarity of triangles LON and NAK
(the curvature of the segment OL can be ueglected):
1 AK lil W 1 = alb, whence
IAKl=f IWl=+f.
IAKI = 55xxt'io-:xº2t m = 5 x 10-z m.
585. At what distance from a convex spherical mirror
must an object be located for its image to be closer to the
mirror than the object by a factor of 1.5? The radius of
curvature of the mirror is 1.6 m. Construct the image oí
the ohject.
Given: f=dll.5, R=l.6m.
d-1
Solution. Using the formula for a convex mirror, we
obtain
!/d - til= -1/F.
Considering that f= d/1.5 and F = R/2, we find that
!/d - t/(d/1.5) = -1/(R/2),
whence
d= O.SR/2,
= (0.5 X 1.6)/2 m = 0.4 m.
d
ln order to construct the image of point B, we shell use
two rays (Fig. 186): the ray propagating along ao auxilia-
362 Se1ected Problema on Phyaics

ry optical axis BC, which after reOeetion will pass io the


directioo of poiot B, and the ray propagating parallel to
the principal optical axis, which after reOection will pass

Fig. 186

80 that its continuation behind the mirror will pass through


the principal focus F. The poiot of interseetion of the
cootiouatioos of the reflected rays will give the image B1
of point B. Dropping a perpendicular from point 8 1
onto the principal optical axis, we obtain the image
A 1B1 of the object AB. The obtained image is diminished,
virtual, and erect.
586. A bright point S is on the principal optical axis
of a concave mirror of radius 40 cm at 30 cm from its
pole (Fig. 187). At what distance in front of the concave
mirror should a plane mirror be placed for the rays re-
flected by the mirrors to return to point S?

s,

Fig. 187
Cb. 4. Optics

Given: R=40cm=0.4m, d=30cm=Ü.3m.


x-?
. Solution. ln the absence of the plane mirror, point S1
would be the image of tbe bright point S formed by the
concave mirror (a similar construction of the image was
carried out in Problem 582). If the plane mirror is placed
on tbe path of the rays reDected by the concave mirror
and propagating to point S 1 , the rays will be incident on it
in a converging bundle and after reDection will form the
final image at point S 2 • Point S 1 can be regarded as a vir-
tual source for the plane mirror (see Problem 583). The
image S 2 of the virtual source formed by this mirror will
be real and symmetric to the source S 1 about the plane of
the mirror.
Since the plane mirror divides the distance between the
source sl and its image st by half, and, by hypothesis,
point S 2 must coincide with point S, the plane mirror
must be placed at equal distances from points S and S 1 •
Let us determine tbe distance from tbe concave mirror to
the image of the bright point formed by it using the for-
mula for a concave mirror:
1/d + 1// = 1/F.
Considering that F = R/2, we fmd that 1/d + 1// = 2R,
whenee
/ = Rd/(2.d - R). (1)
The figure shows that the distance between the plane and
concave mirrors is
r=d+(j-d)l2=(d+n12. (2)
Substituting expression (1) into (2), we obtain
d'
x= 2d-R '

x= 2 xo~'t-o. 4 m=0.45m.
587. Two identical concave mirrors are arranged oppo-
site to each other so that their foci coincide. A point source
of light is placed at 50 cm from the fi.rst mirror on the
... Selected Problema on Phyaics

--- s,

Fig. 188

common axis of the mirrors (Fig. 188). Where will the


image be formed after the refiection of the rays by the
mirrors? The radius of curvature of each mirrar is 80 cm.
Given: d1 =50cm=0.5m, R=80cm=0.8m.
x-?
Solution. ln the absence of the second mirror, point
8 1 would be the image of a bright point S formed by the
first mirror. Ir we place the second mirror on the path of
the rays reflected by the first mirror, the rays will be in-
cident on it in a converging bundle and after reOection
will form the final image at point 8 2 • Point 8 1 can be re-
garded as a virtual source for the second mirrar. The
image 8 1 of the virtual source formed by thesecond mirrar
is real and is ata distance x from the second mirror. Ap-
plying the formula for a concave mirror to the first mir-
rar, we can determine the position of point 8 1 :
lld, + 11/, = l/F, or !id, + li/, = 21R,
whence
/, = Rd,1(2d, - R). (1)
Similarly, applying the same formula to the second mir-
ror, we can find the position of point 8 2 :
-!ld, + li/, = llF,
Oh. 4. Optics 365

or, considering that F = R/2, and / 2 = z, we obtain


-1/d2 + 1/z = 2/R, whence
• = Rd,1(24, + R). (2)
The figure shows that d 1 = /1 - 2F. Therefore (see
Eq. (!)),
~= 2d~~R -~= Rii~=:1) • (3)
Using Eq. (3), we can reduce Eq. (2) to the form

x= 2:;~~~1~i~~~~-;~iR R-di,
•=(0.8-0.5) m=0.3m.
It follows from the solution that the positions of points
S and 8 1 coincide.
588. A point source of light of luminous intensity
75 cd lies on the principal optical axis of a concave mir-
ror of radius 50 cm at 35 cm from its pole. Determine the
maximum illuminance of a screen placed at 2.5 m from
the mirror and perpendicular to the principal optical
axis.
Given: R=50cm=0.5m, d=35cm=0.35m,
l = 75cd, L=2.5m.
E-1
Solution. The illuminance is maximum at tbe centre of
the screen and equal to the sum of the illuminances from
the sources S and S' (its image formed by the concave mir-
rar, Fig. 189):
E 0 = E + E 1 = llr2 + l 1tr;_, (1)
where r = L - d is the distance from the source S,
r 1 = L - f tbe distance from the source S' to tbe screen,
1 the luminous intensity of the source S, and 11 the lumi-
nous intensity of the source S'. ln contrast to Problem 560,
the luminous intensity of the source S' is not equal
to tbat of the source S since the solid angle in which the
rays reOected by the concave mirror propaga te is not equal
to tbe solid angle for the rays incident on the mirror. ln
order to calculate the luminous intensity of the source
386 SeJecled Problema on Phy1ic1

Sc~l'fBfl
'
.,
d

Fig. 189

S', let us consider tbe luminous fiux cl> = 16> incident


on the mirror from the source. Ir we neglect the energy
tosses during reftection, tbe same Dux will propagate within
the solid angle U11 : cl> = 1 1 fll 1 • On the other band,
wd2 = ooif (see Fig. 189). From these relations, we ob-
tain
1, = I Uldl'. (2)
Substituting expression (2) into (1) and taking into ac-
count the expressions for r and r 1 , we obtain

E,= I [ (L~d)' + (f )' (L~f)' ] • (3)


The distanee Crom the image S' of the source to the mirror
is
1/d +
llf = l/F,
where F = R/2. Tberefore,

f= 'ld"!.R'
2 ~t~s°"3g. 5 m = 0.875 m.
Finally, we can write

Eu =75 ( (2.5-t0.35)• +( ~~1; )2(2.5-;.875)•] b: ~ t941x.


589. At what angle must a ray be incident on a plane
mirror for the reOected ray to be perpendicular to the in-
cident ray?
Ch. 4. Optica
'""
Answer. The ray must be incident at an angle of n/4 rad.
ln this case, according to the law of renection, the
angle of reftection is n/4 rad, and hence the angle between
the incident and reDected rays is 'IC/2 rad, i.e. the rays
are mutually perpendicular.
590. If the surface of water vibrates, the images of ob-
jects acquire intricate forms. Why?
Answer. The vibrating surface of water can be regarded
as a combination of concave and convex mirrors of differ-
ent radii, which form different images.

EXEB.CISF.S

591. A person stands in front of a plane mirror and then


moves away from it by t m. What is the increase in the
distance between the person and bis image?
592. An object is between two parallel plane mirrors.
How many images of the object are formed?
593. A light ray reHected by the mirrar of a galvanome-
ter is incident on the central~division of the scale separat-
ed by t .5 m from the mir:or and perpendicular to the
incident ray. When a current is passed through the galva-
nometer, the mirror tums so that the light spot on the
scale is displaced by 2 m. Determine the angle of rotation
of the mirror.
594. A concave mirror forms a 3 x inverted image of
an object. The distance from the object to i ts image is
28 cm. Determine the principal focal length of the
mirror.
595. The image formed by a concave mirrar is smaller
than the object by a factor of four. 1f the object is moved
closer to the mirror by 5 cm, its image will have a size half
that of the object. Determine the principal focal length
of the mirrar.
596. The focal length of a concave spherical mirror is
1 m. At what distance from the mirror must a point source
of light be placed for its image to coincide with the
source?
597, At what distance from the face should a convex
mirror of diameter 5 cm be held to see the face complet.ely
368 Selected Problems on Physics

if the focal length of the mirror is 7.5 cm and the length


of the face is 20 cm?
598. A bright spot is at 1 m from the pole of a convex
mirror, and its image bisects the segment of the optical
axis between the pole of the mirror and i ts focus. Deter-
mine t11e radius of curvature of the mirror.
599. Concave and convex spherical mirrors of the same
radius of curvature equal to 60 em are arranged so that
their principal optical axes coincide. Where should an
object be placed at right angles to the principal optical
axis for its images in the mirrors to be identical? The
distance between the poles of the mirrors is 150 cm.
600. Construct the image of an object formed by a con-
vex mirror. Does the type of the image depend on the
distance from the object to the mirror?
601. Construct the image of ao object formed by a con-
cave mirror if the object is (1) behind the optical centre of
the mirror; (2) between the focus and the optical centre;
(3) between the focus and the pole of the mirror.

REFB.ACTION OP IJGHT.
TRIN LENSES. OPTICAL INSTRUMENTS
The refraction of Ught at the interface between two me-
dia obeys the following law:
The incident ray AO, the refracted ray OB, and the
normal OC to the refracting surface at the point of inci-
dence of the ray lie in the same plane (Fig. 190). The ratio
of the sine of the angle of incidence t to the sine of the ao-

n,

n,

Fig. 190 Fig. 191


389

gle of refraction r is a c.onstant equal to the relativa índex


of the second medium with respeet to the first:
sin t/sin r = n11 •
The relaUve refraeHve lndex n11 is connected with
aheolute refraeUn indlees n 1 and n 1 and with the veloci-
ties v1 and 111 of propagation of light in two contacting me-
dia through the following relat.ions:
n 11 = n 1/n1 = v /v
1 1•

The velocily of propagaUon of Ught ln ~•euum is


e= 3 x 10' m/s. The veloclty of propagatlon of light ln
alr is v~ e.
The total reftecUon of light is observed when light prop-
agates from ao optically more dense medium to an opti-
cally less dense medi um and con-
sists in the complete reDection of
light from the interface between
two media if the angle of inci-
dence of ali the raya is larger
than the criticai angle.
Fig. t9.2 The erltleal angle i 0 (Fig.191)
is the angle of incidence for whieh
the refracted ray passes along the
interface between two media. The value of the angle can
be found from the relation
sin i 0 = n,/n1 • ,,,

When light passes through a prism, it is denected to-


wards its base (Fig. 192). The angle y is known as the
prism angle and fi the angle oi defleeUon between the in-
cident ray and the ray emerging from the prism. ln order
to rotate a ray through n/2 and n rad, a right-angle
iso.sceles prism known as a totally refteeUng priSDI is used.
A transparent body bounded by two spherical surfaces
or a spherical surface and a plane is called a lens. The
point through which any ray propagates without changing
ite direction is called the prinelpal opUeal centre O of a
lens (Fig. t93a). The straight line FF pusing through the
vertices 0 1 and 0 1 of the spherical surfaces is known as the
2f.-Ot70
370 Selected Problema OD Phyalc1

(a}
N
+f- lbl
Fig. 193

prlnelpal Opt1eal His of a tens. Any stnight line AB


passing through the optical centre of a lens is called an
auxillary optlcal Hls.
Lenses converting a parallel bundle of rays incident on
them into a converging bundle of rays are called eon-
verglng lenses.
Lenses converting a parallel bundle of rays incident on
them into a diverging bundle of rays are known as dl·
verglng lenses.
Schematic diagrams of converging and diverging lenses
are presented in Fig. 193b.
A bundle of rays parallel to the principal optical axis
converges at the principal foeus F after refraction. For a
converging lens, the principal focus is real, while for a
diverging lens, it is virtual. Each lens has two foci, viz.
front and rear foel. The plane MN drawn through the
:~:~t:!1f:~heª:oc~f~\a~~gt: ~g~h1e9áatn~i:C~ r:~i;:~
aliei to the optical nis and refracted by the lens passes
through the same point lying in the focal plane of the
lens.
ln order to construct images formed by lenses, it is con-
venient to use the following rays from the entire bundle
incident on a lens:
(a) a ray parallel to the principal optical axis of the
Cb. 4. Optic1 311

lens, which aft.er refraction passes through the princi pai


foCUSi
(b) a ray passing through the principal focus, which af-
ter refraction propagat.es parallel to the principal optical
axis;
(e) a ray passing through the optical centre of the lena,
which after refraction does not change its direction.
The formula for a lens and the sign convention are the
same as thoae for spherical minors, i.e.
±t/d ± tlf = ±1/F.
The focal length F of a lens, the radii R 1 and R 1 of
eurvature of the spherical surfaces, and the absolut.e re-
fractive indices n1 and n1 of the substance of the lens and
of the snrrounding medium are connected through the
following relation:

7-=(*-1) (±-k-±*)·
where the sigos in front of the terms containing R 1 and
R 1 are positive for convex surfaces and negative for con-
cave surfaces.
The quantity reciprocai to the focal length is known as
the focal (lens) power:
D= llF.
The focal power of a system of lenses in contact with
one another is equal to the sum of the focal powers of the
lenses constituting the system:
D = D, +D, + D, + ....
un~~: J!:A:B~?ü!:e~!::e º~ª !h:~i!:.!11:i::~~º!r'::
object:
r = 1A,B, Ili AB 1 = fld.
The total magnlfieatlon of an optlcal &)'Item of lemes
is equal to the product of the magni&cations produced by
each lens separately:

... r - r1r.r•....
372 Selected Problems on Physics

602. A light ray is incident on a glass plate whose re-


fractive index is 1.5. Determine the angle of incidence of
the ray if tbe angle between the renected and refracted
rays is 90" (Fig. 194).
Given: n=l.5, y=90•,.t.57rad.
i-?
Solution. lt can be seen from the figure that i y + +
r = :n:, whence r = :n: - y - i. Substituting the value
of y, we obtain
r = 'lfÍ2- i. (1)
On the other hand, according to Snell's law,
sin Usin r = n. (2)
From expression (1), we lind that sin r = sin (n/2 - i) =
cosi. Then Eq. (2) can be written in the form sin l/cos t =
tan i = n. whence
i = arc.tan n,
i = arc.tan 1.5 :::::= 0.98 rad.
603. The absolute refractive índices of diamond and
glass are 2.42 and 1.5. What is the ratio of the thicknesses
-0f these materiais if the time of propagation of light in
them is the same?
Given: n1 = 2.42, n2 =1.5.
12111 -?
Solution. The absolute réfractive indices Ri and n 1 of
diamond and glass are connected with the veloci\ies v1
and v1 of propagation of light in them through the fol-
lowing relation:
n 1/n1 = v2/v1 • (1)
Since light propagates in a homogeneous medium at a
COllStant velocity, we can write
v, = l,lt,v, = l,Ji, (2)
Ch. 4. Optics 373

~ r
2
. n,
"
Fig. 194 Fig. 195

where t is the time of propagation of light through a sub-


stance, and li and l 2 are the tbicknesses of the diamond
and glass.
Dividing expressions (2) termwise, we obtain
v1/v1 = l 2/l 1 • (3)
Comparing Eqs. (1) and (3), we find that
l1 /l1 = n1 /n 1 ,
l,ll, - 2.42/1.5 - 1.61.
604.. Determine the criticai angle of incidence of a ray
on the interface between glass and water (Fig. 195).
Giuen: n 1 = 1.5, n2 =1.33.
io-?
Solution. The criticai angle of incidence at which total
internai refiection is observed can be determined from the
condition sin i 0 = n 1/np whence
i 0 = arcsin (n1 /n1 ).
i 0 = arcsin (1.33/1.5) ~ 1.08 rad.
605. Light rays emerge from turpentine into air. The
criticai angle for these rays is 4.2º53'. Determine the ve-
locity of propagation of light in the turpentine.
Gtven: i 0 =42º53' ~0.64rad.
v-?
37• Selected Problema on Phy1ic1

Solution. The refractive indicea n 1 and n 1 of turpentine


and air are connected with the velocities of propagation
of light in these media t.hrough the relation
n11n, = vlc. (1)
On the other hand, the criticai angle of incidence at which
total internai reDection is observed can be found from the
condition
(2)
Comparing Eqs. (1) and (2), we find that sin t0 = vlc,
whence
11 = C!IÍD io,
• = 3 X 10' X 0.68 mi• <>< 2.01 X 10' mi•.
606. A scratch is made on the lower face of a plane-
parallel glass plate. An observer looking from above aees
tbe scrat.ch at 4 cm from the upper face of the plate. What
is the thickness of the plate?
Gt11en: h=4 cm=4xto-z m.
H-1
Solutlon. Let the scratch be at point A on the lower sur-
face of the glass plate. We construct the image of point A
which is seen by the observar (Fig. 196). For tbis purpose,
we consider two raya, viz. the ray AC incident at right
angles on the upper surface of the plate and the ray AD
incident on the upper surface at a small angle t. The fig-
d

Fig. t96
Ch. 4. Optics 375

ure shows that point B is a virtual image of poiot A. ln


order to determine the thickness H of the plate, we con-
sider .6.ACD: 1AC 1 = 1CD l/tan i, or, since 1AC 1 =
Haod ICDl=d,
H = d/tan i.
The segment d can be found from .6.BCD: 1 CD 1 =
1CB 1tan r, or, considering that 1CD 1= d and 1CB1 = h,
we obtain d= h tan r. Then H = h tan r/tan i.
Since the angles r and i are small, we can replace
the ratio of their tangent.s by the ratio of their sines, i.e.
tan r/tan i t:::: sin r/sin i. Consequently,
H = h sio r/sin i.
But according to the law of refraction, sin r/sin i """
n11/na1r = ng sioce n 81 r = 1. Therefore,
H=hn,,
H = 4 X 10-2 X 1.5 m = 6 X tQ-t m.
607. Determine the angle of deOection of a ray by a glass
prism with a prism angle of 3º if the angle of iocidence of
the ray on the front face of the priSID is zero.
Gtuen: 7=3º~0.052 rad, i 1 """0.
6-?
Solution. Figure t 97 shows that LAKM = L BCD =
L NKE = y. We denote LN KF = « aod LEKF = 6.
Then
6 = «-y. (t)

Fig'. t97
Selected Problema OD Physics

According to the law of refractioo, for the face CD we


have sin y/sio cz = 1/n, Sioce the llDglea a. llDd y are small,
we cao write aio y/sin cz ~ y/cz. Theo y/cz = 1/n, wheoce
,. = yn. (2)
Substit.ut.ing relatioo (2) iot.o (1), we obt.aio
6 = yn - y = y (n - 1),
6 = 0.052 (1.5 - 1) rad = 0.026 rad.
608. A Ught ray is iocideot oo a triangular glass prism
with a prism angle of 45º aod emerges from it at. ao angle
of 30°. Determine the angle of incidence of the ray.
Given: y=45º~0.79rad, r2 =300~0.52rad.

t.-?
Solutton. Let us consider 6.DKE formed by the ray
DE aod the perpendiculars KD aod KE to the faces AC

Fill'. t98

and BC of the prism (Fig. 198). According to the well-


known theorem io pometry, the exterior angle P, of this
triangle is equal to the sum of the interior aogles r1
aod i 1 which are oot adjaceot. t.o it., i.e. P, = r1 + t 1.
But P, = y as angles with mutually perpendicular sides.
Coosequently, y""' r1 + i 1, wheoce
r, = y-1,. (1)
Accordiog t.o t.he law of refract.ion, for the face CB we
have sin i 1/sin r 1 ""' 1/n, whence sin t 1 = aio r1 /n =
0.5/1.5 = 0.333, t 1 = 19.5°. Therefore, accordiog to
Eq. (1), r, = 45º - 19.5° = 25.5º.
Ch. .. Optics 377

According to the law of refraction, for the face AC


we have sin t 1/sin r1 = n, whence sin t 1 = n sin r 1;
sia 1, = 1.5 X 0.43 = 0.645,
t, = 40'12' "' 0.698 rad.
609. A converging lens forms a real twofold magnifted
image of ao object. Determine the focal length of the Iens:
if the diatance between the lens and the image of the object
is 24 cm. Construct the image formed by th.e leD.!1,
Gtwn: r=2, 1=24cm=0.24m.
F-1
Solutton. ln order to conslruct the image of the upper
point A of the object AB, we consider the paths of two
raya (Fig. 199). After refraction, the fi.rat ray propagating

2 B

Fig. t99

parallel to the principal optical axis will pus through the


principal focua of the lens. The second ray passing througb
the principal opt.ical centre of the lens will not change its
direction. The point A 1 of intersection of the raya will
be a real image of point A. Dropping a pel'pendicular from
point .A. 1 onto the principal optical axis, we shall obtain
a real, magni&ed, and inverted image A 1B 1 of the object
AB. ln order to determine tbe focal length, we sb.all use
the formula for a thin lem t/d + tlf = t/F, wbence
F = d//(d + 1). (1)
Tbe linear magni&cation of the lena is r = 1A1B1 V
1 AB 1 = fld, wheaee
d =ftr. (2)
378 Selected Problema ou Phyaica

Substituting expression (2) into (t), we obtain

F= ::~~t = rit '


F= ~~ m=8x10--tm.
610. Determine the focal leogt.h of a double-eonvex
_glass lens immersed io water if ita focal length in air is
known to be 20 cm.
Giwn: F 1 =20 cm=0.2m.
Fz-?
Solution. The focal leogth of a double-convex lena is
connected wi\b. the abaolute refractive indices n1 aod n 1
of the leoa material and of t.b.e surrounding medium
through the following relation:

f=(~-1)(-k-+f.-).
For the leos in air, we have

*=(n -t)(*+f.-).
1

Similarly, for the lena in wat.er, we have


(1)

i-=(-i;-t) (i;-+i;-)
= ( ··::··) hl;+k). (2)

Dividing relation (t) by (2) termwise, we obtain F 1 /F1 =


(n1 - 1) n 1 /(n1 - n 1 ), wheoce
p _ F1n1 (n1-0
z- n1-n.. '
Fa = 0.2 X :.:a ~-~35-1) m C!:! 0.8 m.
611. Determine the principal focal lengt.h of a plano-
.convex glasa lens in t.urpent.ine if the r"dius of curvat.ure
.of its convex surface is 25 cm.
Gtuen: R 1 """25 cm"""0.25 m.
F-1
Ch. 4. Optics

Solutton. The focal length of the lena can be determined


from t.he formula

-}=(~-1)(-},-+-},). (!)

Since for t.he plane surface of the lens we have 1/R1 =O,
Eq. (1) can be reduced to the form. t!F = (n 1/n 1 - 1)/R 1 ,
whence
F--R_,_ _ R...
- A1/A1-f - Ai-41'

F = ~·~5~1~4~7 m = 12.25 m.
612. Conatruct the image of a bl'ight apot lying on the
principal opt.ical axis of a thin converging lena at. the
midpoint between t.he lens and ilci focua. Characterize t.be
image.
Gi11tn: d= O.SF.
f-1
Solutton. Using the formula for a converging lena 1/d -
1/f = 1/F, we obtain
f = Fdi(F - d),
or, conaidering that d = 0.SF,
f = F·O.SFl(F - 0.5F) = F.
Consequently, the image of the bl'ight spot will be virtual
and located ata diat.ance F from the lena. ln arder to con-
atl'uct the image of the bright apot S, we shall use two
rays, viz. the ray 80 propagat.ing along the principal opti-
cal axis and an arbitrary ray SA (Fig. 200). Tbe ray SO
passes through the lena without being refracted. ln arder
to determine the path of the ray SA after its refraction in
the lena, we draw the focal plane M N and an auxiliary op-
tical axis DO parallel to the ray SA, which will intersect
the focal plane at point B. The ray SA muat pau through
t.his point after refraction in the lena (in accordance with
the property of the bundle of raya parallel to an auxiliary
optical axis). Tbe image S' of the bright spot S will lie
Selecled Problema on Pbysice

:
380

-~.;-.

N
Fig. 200
on the principal optical axis at the point of intersection
of the continuat.ions of the refraeted rays.
613. Wbat is the magnification of a magnifier whose
lens power is 16 D? Construct the image of an object
formed by t.he magnifier.
Glven: D=16D.
~
Solution. The magnification produced by the magnifier is
r=LIF=LD,
where F is the focal length of the magnilier and L the
dist.ance of normal vision (which is equal to 0.25 m for a
normal eye). Consequently r = 0.25 X 16 = 4.
The object. to be examined through a magnifier is
placed bet.ween the magnifier and its focos (Fig. 201). ln
order to construct the image of point A of the object, we
shall use two rays emerging from it, viz. the ray parallel
to the principal optical axis, which after refract.ion will

Fig. 201
Ch. 4. Optics

pa8S through the focus, and the ray passing through the
principal optical centre of tbe lens, which will not change
it.s direction. The image A 1 of point A will be formed at
tbe point of intersection of tbe continuations of the re-
fracted rays. Similarly, we can obtain tbe image B1 of point
B. Consequently, the image A 1B 1 of the object AB is vir-
tual, magnified, and erect.
614. A converging bundle of rays is incident on a diverg-
ing lena. After passing through tbe lens, the rays inter-
sect at a point lying at 15 cm from the lens. lf we remove

Fig. 202

the lens, tbe point of intersection will be shifted by 5 cm


towards the previous position of the lem. Determine the
focal power of the lena.
Given: d=15cm=0.15m, l=5cm=0.05m.
D-1
Solution. lf we place a point source of light at point A,
point A 1 will be its virtual image (Fig. 202). Using the
formula for a diverging lens 1/d - 1// = -1/F, we can
write
F = /dl(d -/), (1)
where I = d - l. By definition, the focal poW81' of a di-
verging lens is D = -1/F or, using Eq. (1),

D=-'ft, 1 =-.(LI).
D= o.15x(O~t~ o.05) º"' -3.3D.
615. A diverging lens with a focal length of 12 cm is
placed between two point sources so that it is twice as
Selected Problems on Physics

Fig. 203

close to one of them than to the other. The dist.ance be-


tween the images of the sources is found to be 7.8 cm. De-
termine the distance between the sources.
Given: F=12 crn=0.12 m, d2 =2d1 , l=7.8 cm=
0.078m.
L-1
8olution. Let us construct the images 8~ and 8; of the
bright spots 8 1 and 8 1 (Fig. 203) (a detailed construction
is described in Problem 612). Using the formula for a di-
verging lens to each source separately, we obtain
tld, - li/, - -llF, lld, - li/, - -llF .
. By hypothesis, d 1 = 2d1 , l = / 1 + /1 , and L = d1 +d 1•
Therefore,
L- -9F(F-l)t~:,.~~
-9 x 0.12 (O.t2-0.078) +3 X 0.t2y'8 (O.t2)1+(0.t2-0.078)2
L 4 (2 X O.t2 0.078)
-0.6'5+0.t23
0.648
Ch. 4. Optic&

wbence L °' 0.12 m. (The negative root does not 1atisfy


the hypotbesis.)
616. A bundle of rays parallel to the pl'incipal optical
axis is incident on a double-convex lena whose focal length
is 12 cm. Another double-convex lens having a focal length
of 2 cm is at 14 cm from the first leD8. Tbe principal opti-
cal axa of tbe lenses coincide. Where will tbe image be
formed?
Given: F 1 =12 cm=0.12 m, l=14 cm=0.14 m~
F 2 a::::2 cm=0.02 m.
1,-1
Solutton. We shall construct the paths of the rays in
the given optical sy1tem (Fig. 204). By hypothesis, the-
foci F1 and F,. of the lenses coincide (l = F1 + F1).
Consequently, the bundle of rays emerging from the sec-
ond leoa will be parallel to the principal optical axis.
No image will be formed in this case (the image is at in-
finity).
617. The objective of a photographic camera bas a fo-
cal length of 50 mm. What must be the exposure time for
a motor car moving uniformly at a velocity of 72 km/h
at 2 km from the camera at right angla to its optical axis
for its image to be displaced on the picture by 0.005 mm
during tbis time? CoD8truct the image.
Given: F=50mm=5x 1G-2m, d=2km=2X 101 m,
v= 72 km/h= 20 m/s, 1 1 =0.005 mm =5 X to-e m.
t-1
Solutton. ln order to COD8truct tbe image of the car
(object AB), we consider two rays (Fig. 205a), viz. the-
ray parallel to the principal optical axis, which after re-

Fig. 204
Seleeied Problema on Physies

fraction will pus through the focm F of the objective,


and the ray passing through the opt.ical centre O of the
.ohject.ive, which will not change ita direct.ion. The point.
A 1 of inteneet.ion of t.he refracted rays will be a real im-
age of point A, Dropping a perpendicular from point A1
onto the principal optical a:r.is, we shall obtain a real,
diminished, and inverted image A 1B1 of the object AB.
During t.he exposure time t, the car is displaced by a
distance s = vt, whence
t = slv. (1)
It follows from Fig. 205b t.hat s/s1 = dlf, whence
.• = s,dlf. (2)
Using the formula for a converging lens 1/d + 1// = 1/F,
we find t.hat
dlf = (d - F)IF. (3)
Subst.itut.ing expression (3) into (2), we obt.ain
s = s, (d - F)IF.
Then Iram Eq. (1) we get
t= •1 <~-;F) ~ ;: .
t- S~~;c1 ~~()8 S=1fr"2s.
618. The height of the image of an object, formed on
the opaque glass of a camera, is 30 mm when the object

(aJ
B,

d
1
1 1
: 1
;~s,
1
Fig. 205
38•

is at t5 m and 51 mm wben the ohject is at. 9 m. Deter-


mine the focal length of tbe objective.
Given: d1 :::af5m, h.1 """30mm=3 X tO-Zm, cfs=9m,
hz-51mm-5.I X 10-Sm.
F-1
Solutton. Using the formula for a converging lem for the
distances "1. and d 1 , we obtain
lfd, + li/, - l/F, lfd, + lff, - 1/F. (1)
Using the formula for the magnification of tbe lena for
the sarne distances, we find that h/h.1 = "1.1/1 and h/h. 1 =
dali 1 , whence
f, - h,.d,/h, f, - h;l,lh, (2)
where h is the beight of the object.
Substituting Eq. (2) into (t), we obtain
lfd, + hlh,d, - llF, l/d1 + hlh;I, - l/F. (3)
Solving Eqs. (3) togsther, we gst
F= dt:-:;"1'
F = 9 x 55~1 ~ t:;:',- t:; tª;. to-• m ~ 0.43 m.
619. Using a telescope with a focal length of 50 cm of
the objective, an observar can clearly see objecla at 50 m
from tbe objective. ln what direction and by what dis-
tance should the eyepiece be shifted to accommodate
the telescope to infinity? Con.struct. the image.
Gwen: p.,,,,,50 cm=0.5 m, d=SO m.
1-1
Solution. Using the formula for a converging leos t/d +
1.1/1 = f!F. we can determine the distance between the
objecti ve and the image formed by it

/1= d'!!_P'
li=::~:: m=0.505 m.
U-0970
386 Selected Problems OD Phy11ic11

When the telescope is ad;usted to infi.nity, the image must


be formed by t.he objective in its focal plane, i.e. / 1 =
0.5 m. Conaequently, the eyepiece must be shifted towards
the objective by a distance
l = f, - t..
l = (0.505 - 0.5) m = 0.005 m.
Let us eonstruct the image of an object formed by the
telescope (Fig. 206). For t.his purpose, we shall take two
rays emerging from point A of the object AB.
The 6.rst ray pasaing through the focus F 1 of the objec-
tive will propagate parallel to the principal optical axis

Fig. 206

after refraction until it is incident on the eyepiece at point


D. After refraction in the eyepiece, it will propagate so
tbat its continuation will intenect the optical axis at
the focus F 1 •
The second ray passing through the optical centre 0 1
of the objective will not. ehange its direct.ion and will be
incident on t.b.e eyepiece at point E. ln order to determine
the path of the ray after refraetion in the eyepieee, we
draw the focal plane M N tbrough the foeus F 2 and an
auxiliary optieal axis parallel to the ray and inteneeting
the focal plane at point K. The seeond ray is refraeted so
that its eontinuation will pa88 tbrough point K.
Ch. 4. Optica 381

The point A 1 of intersection of the continuations of the


tirst and second rays emerging from the eyepieee is a vir-
tual image of point A. Dropping a perpendicular from
point A 1 onto the principal optical axis, we shall obtain
a virtual, diminished, and erect image .A..B 1 of the
object AB.
620. A mieroseope consista of ao objective and an eye-
piece arranged so that the distance between their princi-
pal foci is 18 cm. Determine the magnilication produced
by the microscope if the focal lengths of the objective and
the eyepiece are 2 and 40 mm respectively. Construct the
image of ao object formed by the microscope.
Given: l=18 cm=0.18 m, F 1 =2 mm=2:< JO-•m,
F 1 -40mm=4X10-:1 m.
r 1
Solution. We shall construct the image of ao object
A B which is normally placed near the focal plane of the
objective. For this purpose, we take two rays emerging
from point A of the object AB (Fig. 207).
The first ray passing through the focus F 1 of the objec-
tive will propagate parallel to the principal optical axis
after refraction until it is incident on the eyepiece at
point D. After refraction in the eyepiece, the ray will
pass through it.s focus F 1 •
The second ray passing through the optical centre 0 1
of the objective will not change it.s direction and will be
incident on the eyepiece at point E. ln order to determine

... Fig. 207


... Selected Problem1 on Physica

the path of the ray alter refraetion in the eyepiece, we


draw the focal plane M N through the foeus F 1 and an
auxiliary optieal axis parallel to the ray and intersecting
the focal plane at point K. The second ray must pua
through this point alter refraetion in the eyepieee.
The point A 1 of intersection of the eontinuations of the
first and second rays emerging from the eyepiece is a vir-
tual image of point A. Dropping a perpendicular from
point A 1 ont.o the principal optieal axis, we shall obtain
a virtual, magnified, and inverted image A sfJ 1 of the
objeet AB.
Sinee tbe mieroscope consista of two Ienses (objective
and eyepiece), its magnifieation is given by
r = r,r,, <t>
where r 1 and r 1 are the magnilieations of Lhe objective
and the eyepiece respeetively. By definition, the magni-
fieat.ion of the objecti ve is
r, = 1,1rt,. (2)
Since / 1 ~ l and d1 :::t:I' F 1 , we have r 1 ~ l/Fl' The eye-
pieee operates as a magnifier, and hence
r, = LIF 1 , (3)
where L = 0.25 m is the distance of normal vision. Sub-
stituting expressions (2) and (3) into (1), we obtain

r= ).~.
r= 2 x~~::2~o-• 562.
621. A bundle of rays is ineident on a diverging lens of
focal length F 1 parallel to ita principal optieal axis. At
what distanee from the centre of tbe diverging lens must
a eonverging lens be plaeed for the rays emerging from it
to propagate, as before, parallel to the principal optieal
axis? The focal length F1 of the eonverging lens is twice as
J.arge as that of the diverging lens (Fig. 208).
Given: F1, F 2 =2F1.
l-?
Ch. 4. Optics ...
,, --- o,
º•
f,
ª'
Fig. 208

Solution. For tbe diverging lens, we can write


-ti/,+ t/d, - -llF,. (t)
Tbe souree of a parallel bundle of rays is assumed to be
at infinity so that 1/d1 =O, and from Eq. (1) we obtain
tlF, - ti/,.
whence F 1 = / 1• Consequently, the virtual image of tbe
source formed by tbe diverging lens is at its virtual f~
cus and is a souree for tbe converging lens. For the con-
verging lens, we have
lld, +li/, - t!F,. (2)
The figure shows that d 1 = li + l. By hypothesis, the
rays emerging from the converging lens must propagate in
a parallel bundle, i.e. 1// 8 = O. Substituting the expres-
sions for F 1 , d1 , and / 1 into Eq. (2), we obtain
1/(2F,) - 1/(F1 + l),
whence l = F 1 , i.e. tbe required distance is equal to the
focal length of tbe di verging lens.
622. Water is poured into a concave mirror whose radi-
us of curvatura is 50 cm. The focal power or the obtained
system is 5.3 D. Caleulate the focal length of the water
lens.
Gi.,.n: R-50 em-0.5 m, D-5.3 D.
F-1
390 Selected Prohlems on Physica

Solution. The optical system consista of the concave


mirror and the plano-convex wa\er lens brought in con-
tact. Therefore, its focal power is
D =D, + 2D,, (1)
where D 1 = 1/F1 = 2/R is the focal power of t.he concave
mirror and D 1 the focal power of the water lens. The coef-
licient 2 is introduced into Eq. (1) since the rays pus
twice t.hrough t.he water leM. It. follows from this equa-
tion that D 2 = (D - D 1 )/2, or, taking into account the
expression for D 1,
D, = (D - 2/R)/2. (2)
By defmition, the focal length is F 1 = 1/D 2 , or, t.aking
into account expression (2),

Fi= D!_2fR'

F2 = 2
5.3 _ 2/0.S m ~ 1.54 m.
623. A double-convex lens is obtained from two iden-
tical thin watch glasses the space between which is filled

Fig. 209

wit.h water. The focal power of such a lens is 4 D. Deter-


mine the focal power of a plano-conca ve lens consisting
of a thin watch glass t.ouching the bottom of a thin-walled
cylindrical glass vessel if the space between the watch
glass and the bottom of the vessel is filled with water.
Given: D0 = 4 D.
D2-?
Solution. ln order to determine \he focal power of the
plano-concave lens, we shall li.li the watch glass plaeed
at the bottom of the cylindrical vessel with water
(Fig. 209). We shall obtain a thin plane-parallel layer of
Ch... Op... 391

water whieh can be treated u a system consisting of t.wo


water lenses: plano-conve.x (upper lens) and plano-eoncave
(lower lena). The focal power of tbe plane-parallel wa-
ter layer is
(1)
On the other hand,
D~D,+D,, (2)
where D 1 and Dt are the focal powers of the plano-eonvex
and plano-concave lenses. Tbe plano-convex lena is half
the double-convex leu cut along the line AB. There-
fore, its focal power is
D,~D,12 ~ 2 D. (3)
Substituting e.xpressions (t) and (3) into (2). we find that
O = 2 D + D 1 , whence D 1 = -2 D.
624. Determine the focal power of a pair of spectaeles
for a long-sighted person whose distance of normal vision
is 40 em.
Glven: Li-1 =40 cm=0.4 m, Ln=25 cm=0.25 m.
D-1
Solution. Without. the spectacles, the image S' of
point S is form.ed on the retina R of the eye if point S
is at the distance SO equal to the distance L1_. of normal
vision for a long-sighted eye from the eye lem EL
(Fig. 210). For the long-sighted eye to be able to examine

Fig. 210
392 Selected Problelll8 on Physics

eloser objeets, the lena L of the spectaclea should be


used. Then the image S' of point s• lying at a diatance
La of normal viaion for a normal eye from the lerui of the
speetacles will, as usual, be formed on tbe retina R of
the eye, and the pel'Son sees not the poiot. s• itself but
its virtual image S lying at the distanee L 1_. of normal
visioo for the long-sighted eye from the eye lena EL. Tbe
focal power of t.he lens-eye optieal system (we negleet
the distanee between the eye and the lena) is
(1)
wbere DL and DEL are the focal powers of the lens of the
spectaeles and tbe eye lena. On the other hand,
lld + li/= llF =D, (2)
wbere f is the distaoce from the retina to the eye
lens.
Since the lens of the spectaeles ia at a small distance
0 10 from the eye lena, we ean negleet this distance and
write d = La. Tbe focal power of tbe eye lena can be de-
termined by applying t.he formula for a t.hin lena without.
t.be apectaclea:
l/d' + l/f = !/FEL = DEL• (3)
Here d' = Li-a· Taking into account expreaaiona (2)
and (3), we ean writ.e Eq. (1) in the form
llL, + li/ = DL + llL,_, + li/,
whence
D _ L1-1-La
L- L1-1Ln '

DL- U;;Z:~ D=l.5 D.

625. A abort.-aighted person diat.inguiahu amall object.a


at. a di.st.ance not. longer t.ban 15 cm. Determine t.bia di.s-
t.ance of normal viaion in apect.aelu with a focal power
of-30.
Ch. 4. Optica

Given: d1 =15 cm=0.1.5 m, D1 = -3 D.


-1
Solution. The focal power of the eye without spectacles
in the case of myopia is
D, - !/d, + !//, (!)
where d 1 = 1 80 1 ia the distance between ao object and
the eye lens and f = 1OS' 1 the distance between the eye-
lens and the retina (Fig. 211).
The focal power of a shOl't-sighted eye with spectacles
is
D - D, +D, - lld, +!//, (2)
whel'e D 1 is the focal power of the spectacles and d 1 ~
1S ·o 1 the diBtance between the objoot and the eye lena

s•

Fig. 2ti
(we neglect the diat.anee between the leDB of t.he spectacles
and the eye lens).
Solving Eqs. (1) and (2) together, we obtain 1/d1 +
D 1 = 1/d1 , whence

els= 1+11~1'
da= t-X·!5o.t5 m~0.27 m.
The short-sighted person with spectacles sees the virtu-
s
al image of the object s·
located at a longer distance-
from tbe eye than without the spectacles.
... Selec&ed Problema OD Physics

626. Wby does the outline of objects above heated soil


seem vibrating on a hot day?
Answer. On a hot day, heated air currents from tbe soil
circulate in the vertical direction. Tbe air density changes
continuously and randomly due to nonuniform beat-
ing, and so does the refractive index which is related to
density. For this reason, the outline of objects seems vi-
brating.
627. How can you explain tbe lustre of precious stones?
Answer. The lustre of precious stones is explained by
total internai reOection which is attained by special
facetting.
628. Why are plants not watered on a hot sunny day?
Answer. Drops of water left on the leaves of plants af-
ter watering play the role of small lenses which may fo-
cus solar rays and burn the leaves.

EXEllCISE!I

629. What is the troe depth of a ri ver if it seems to be


2 m along the vertical according to a rougb visual esti-
mate?
630. A ligbt ray propagatea from glass into water. The
angle of incidence of the ray at tbe interface between tbe
two media is 40º. Determine the angle of refraction and
tbe criticai angle.
63t. A light ray is incident on a trihedral prism made
of quartz glass at an angle of 36º. The prism angle is 400.
At what angle will the ray emerge from the prism and
what is the angle of it.s deOection from lhe initial di-
rection?
632. Where will the image of an object placed at 30 cm
from a converging leos with a focal length of 60 cm be
formed? Wbat is the type of tbe image?
633. The distance between a lamp and a screen on ao
optical bench is 1 m. A converging lens placed between
them forms a diminished image of the lamp on the screen.
lf the lens is shifted towards the lamp by 60 cm, a magni-
fied image is formed on the screen. Determine the focal
length of the lens.
Ch. 4. Opt.ica 395

634. Construct. the image of an ohject. formed hy a con-


verging lens if the ohject. is (1) heyond twice the focal
length; (2) hetween the focus and a point at t.wice t.he fo-
cal length from the lens; (3) hetween the focus and the
lens.
635. The radii of curvature of the surfaces of a douhle-
convex glass lens placed in water are 50 cm each. Deter-
mine the focal power of the lens.
636. Construct the image of an ohject produced hy a
diverging lens. Does the type of the image depend on the
distance hetween the ohject and the lens?
637. Determine the focal length of a di verging lens if
a converging hundle of rays incident on it intersect.'t the
principal opt.ical axis after refraction at a point lying at
25 cm from the lens. ln the absence of the lens, the point
of intersection of the rays will he at 7 cm from the point
of convergence through the lens.
638. A terrain must he photographed from an aeroplane
Oying at an altitude of 4 km to obt.ain pictures on the
scale 1:5000. Determine tbe focal power of the ohjective.
639. ln order to ohtain good-quality photographs, of
size 12 X 9 cm, photographic paper must be exposed for
8 s. ln what proport.ion should the exposure t.ime he in-
creased for obtaining a 48 X 36 cm photograph?
640. A telescope with a foclll length of 50 cm is adjusted
to infinity. After the eyepiece has heen shifted hy a cer-
tain distance, the ohjects at 50 m from the objective he-
come focussed. By what distance has lhe eyepiece heen
shifted during focussing?
641. The focal lengths of the ohjective and the eyepiece
of a microscope are 3 and 50 mm respectively. The dis-
tance hetween the ohjective and the eyepiece is 135 mm
and the distance hetween the ohject and the objective is
3.1 mm. Determine the magnification of the instrument.
642. One surface of a douhle-concave lens is silver plat-
ed. The radius of curvature of the lens surfaces is 20 cm.
An ohject of height 5 cm is at 50 cm from the lens. Deter-
mine the height of the image formed by the optical sys-
tem.
396 Selected Problems on Phy!ics

QUBSTIONS FOR REVISION

1. What is meaat by abaolute and relative relraetive indices?


2. Formulate the law oi rellection oi light. 3. Define lhe pole, the-
optical cent.re, and lhe focus oi a mirror. 4. Write a formula for cal-
f:!•~j~l~t~C:~lW::. ;~ ~=~b!cal mi~ror. 5. Form~late th~
you know? t e optaca centre a
9. What is the foca n what units is it expressed?
IO. Write a formula or ca cu aUng the focal length of a lens.
~:~,:x,t~i:r~iPl: :ln:!::U8:u~':-S !:ri:~Y1:r:!!rby 1
:·1!!:
4.J. Quantum Propertles oi Llght
Some phenomena like photoelectric effect or light pres-
sure lead to a new concept of light according to which a
light Oow is a flow of elementary particles, viz. photons
(quanta of light). One of the characteristics of a pboton is
its energy. A monochromatic ligbt flow consists of pho-
tons with the same energy
• - hv - hc/1.,
where h is Planek's constant, " lhe frequency of light, e
the velocity of light in vacuum, and Â. the wavelengtb.
The phenomenon consisting in tbe emis.sion of electrons
by metal bodies exposed to light radiation is known as
exlrlnsie pholoelectrie effect (photoelfeet).
Photoeffect oheys Einstein's law
e =h-v =A +mv212,
where 8 is the energy of a photon, A the work funeUon for
a given metal, mv1 /2 the maximum kinetic energy of an
emitted electron, and m the electron mass.
Photoelectric effect is observed only when a metal is
irradiated by ligbt at a frequency larger than or equal
to the criticai frequency -v0 known as photoelectrie lhresh-
old. ln other words, the photoelectric tbreshold corre-
sponds to the pboton energy equal to the work function
for a given metal:
h-vo =A.
Ch. 4. Op\ics 397

ln this case, the velocity v of electrons, and hence their


kinetic energy m.v'-/2, are zero.
Light incident on bodiea exerts a pressure on them,
which is determined by the luminoua intensity and reOec-
tivity of the surface of a body. Llght pressure is given by
p - I (1 + p)/c,
where p is the light pressure, 1 the luminous int.ensity,
i.e. the radiant energy incident on a unit area of the body
surface per second, e the velocity of light in vacuum, and
p the refteeUon eoef&elent.
lf a body reOects light rays specularly, then p = t and
p = 21/c. lf a body absorbs light completely (black-
hody), then p =O and p =fie.

643. Determine the maximum wavelength of light for


which the photoeffect can be observed in platinum.
Givt1i: A=8.5x10-" J.
Amu-?
Solution. Using the formula A = hv 0 , we can deter-
mine the photoelectric threahold for platinum:
v 0 - Alh. (1)
This frequency corresponds to the required maximum
wavelength
Âmu: = c/v 0,
or, taking into account Eq. (1),

Amu:=T,

Amu 3 X 10;.~ ~-:-~, tO-ll m ~ 2.34 X 1()-7 m.


644. Determine the maximum velocity of an electron
emitted by ceaium exposed to light of wavelength 400 nm.
Gtr:en: A.=400 nm=4xt0-7 m.
"mu:-?
... Selec&ed Problema ou Physics

Solutton. From Einstein's law for photoelectric effect


hv =A + mvl/2, we bave llmn = 1'2 (hv - A)lm. Con-
sidering that v = c/'J,., we obtain

llmu=V{-(~-A),
ll111u:= Jf 9.1x~cr-1 (6.62x!~:~ãxtó•
X 3.2xtO-") m/s
-6.5x10'm/s.
645. The maximum wavelength of light at which the
photoeflect for potassium is observed is 6.2 X t0- 6 cm.
Determine the work function for potassium.
Gtven: Â.m•'( = 6.2 X 10-5 cm= 6.2 X 10-7 m.
A-1
Solutton. The maximum wavelength at which the pho-
toeffect is obaerved for a metal is connected with the pho-
toelectric threshold for the metal through the relation

Vo = c/Ã.mn• (1)
The work function for the metal is A = hv 0 , or, using
Eq. (1),

A-~1'
A= 6.62x6 ~[:~~~xto• J~S. 2 x 10_ 19 J.

64.6. The maximum wavelength of light at which the


photoeffect for tungsten is observed is 0.275 p.m. De-
termine the work function for tungsten, the maximum ki-
netic energy of emitted electrons, and tbe maximum ve-
locity of the electrons knocked out by light of wavelength
0.18 µm from the metal.
Given: "-mu: "'""0.275 µm = 0.275 x 10-e m,
1.-0.18µm-0.18x10.., m.
A ? Wma:i: ? Vmu: ?
Ch. 4. Optics 3911

Solutton. The work function for tungsten is

A= ~:z'
A 6 · 82 0.~~;_.x 10 • l!::!=:7.2xt0-19 J.

Using Einstein 's 1aw for photoeleetrie effect ao d con-


sideriog that "' = c/1, we obtain hc/1 = A + m~lll/2.
whence the maximum kioetie energy is
Wmu= mv'~H =lf-A,
Wmu 6.82~-~C::~.xtoª 7.2 x to-1e J
""3.8 X 10-" J.
Knowing the maximum energy of the emitted electrons.
we can determine the maximum velocity corresponding to-
it:
11mu=Jf"~,
11mn = JI 2 ~.:! ~;~ 11 : ~ 9. 1 X 105 m/s.
647. The energy of a photon is equal to the kioetic ener-
gy of an electron having an initial velocity of 10' m/s
and accelerated by a potential diDerence of 4 V. Deter-
mine the wavelength of the pboton.
Given: e-=Wk, v0 =10' m/s, U=4 V.
A-?
Solution. The energy of the photon is e = h"' = hcl'J..,
whence
A=lu:le. (1)
By hypothesis,
e= W1r.= mv2/2, (2)
where m is the mass of the elettroo' and v its final velocity
as a result of acceleration by the electric fi.eld.
The work done by the electric lield is equal to the
change in the kinetic energy of the eledron: rrw'-12 -
mV:12=A, whence mv212=mv'/2+A, or, sioce A =eU,
mv'/2 = mV:/2 + eU. (3)
Selected Problema OD Phyaica

Here mv'J2 is the initial kinetic en.ergy of the electron


and e its ehlll'I!"· Solving Eqs. (2) and (3) togsther, we
lind that
• -mv',12 +eu. (4)
Substituting expression (4) into (1), we obtain

Ã.= mvlf~+.u'
9.ixio!'Gc7iW,:;~3i~6~ 1io-1•x4 m~ 1.8x 1Q-7 m.
648. The elec:.trons knocked out of the surfac.e of a met-
al during photoelec:.trie eftec.t by a radiation of frequen-
.cy 2 X 1011 Hz are eompletely retarded by a decelera-
tion field with a potential difterence of 7 V. When the ra-
diation frequeney is 4X1011 Hz, the potential diflerenee
~=~u:~~:ul~te'~f.n~t~s e!:C!:a-:t.is 15 V. Using these da-
Gtven: v 1 =2X1015 Hz, U1 =7 V, v1 =4xto11 Hz,
u,-15 V.
h-1
Solutton. Let us write Einstein's law for the two cases
of photoell'ec.t c.onsidered in the problem:
kv1 - A + mV:/2, hv 1 - A + m0:/2. (1)
Since the electrons escaping from the surface of the metal
are c.ompletely retardedby the deceleratingelectrie field,
the change in their kinetie energy is equal to the work
done by the elec:.trie field:
mv1 /2 =eU. (2)
Using Eq. (2), we e.ao write (1) in the form
hv1 =A + eU1 , hv 1 =A+ eU1 •
Solving this system of equations, we find that

h= '!~'.;v~1) '
h= \6:i~~ 1 ·~~;;171 J s=6.4X1()-16 J s.
0 0
Ch. 4. Opt.ics

649. What is the number of photons incident per sec-


ond on a human eye if the eye perceives light of wave-
length 0.5 p.m at a luminous power of 2 x 10-17 W?
Given: t=f s, Ã.=0.5 µm=5 X 10-Tm, N=2 X 10-11 W.
n-1
Solution. The total radiant energy incident on the eye
is W = Nt. The energy ora photon is e = hc/A. Then the
number of photons incident on the eye is
W Nt'J,,
n=g=-;;;--·

n ~~~~~~~~!~' 50.
650. A water drop or volume 0.2 ml is heated by light
of wavelength 0.75 p.m, absorbing 1010 photons per
second. Determine the rate of heating of the drop.
Given: V=0.2ml=0.2x10-e m3 , Ã.=0.75 µm=
7.5x10-1 m, n= 1010 s-1.
IJ.T/IJ.t-1
Solution. The amount or heat received by the drop is
Q = c.m IJ.T, (!)
where m is the mass of the drop, Cw the speciii.c heat for
water, and !J.T the change in the temperature of the water
during its heating. The energy given away by light during
the time .t\t is
W= ne!J.t, (2)
where e is the energy of a photon. Neglecting all possible
energy losses, we assume that the entire energy received
by the drop is spent on its heating, i.e. W = Q, or,
taking into account expressions (1) and (2), ne !J.t =
mcw fl.T, whence we can determine the rate of heating:
IJ.T/IJ.t = no/(mc.). (3)
Observing that m = p V, where p is the density of water
and e = hei').. we can write expression (3) as follows:
61 iahc:
ài""=).pVc:w'
Zl-0970
402 Selected Problema oo Physics

AT tOHx6.82xtO-"xSxtO' K
àe 7.5xtO-'xtCPxó.2xtó'"ox4.2xtO* s
=3.15 x 10-0 K/s.
651. Determine the light pressure exerted on the walls
of an electric bulb of power 100 W. The bulb is made in
the form of a sphere of radius 5 cm, and its walls reftect
10% of the incident light. Assume that the entire power
consumed by the bulb is converted into radiation.
Gtven: N..,,,.toz W, R=5 cm=Sx10-z m, p=0.1.
p 1
Solution. The light pressure is given by
p = I (1 +
p)lc. (1)
Since 1 = N IS, where S = 4nR1 is the surface area of
the bulb, Eq. (1) can be transformed as follows:

p= b~lc (1+p),

p= 4xB.t4x(s!~o-t.)ixsxtO' (1+0.1)Pa=12 µPa.


652. A light beam of wavelength 0.49 µm is incident
at right angles to a surface and exert.s a pressure of
5 µPa on it. How many photons are incident on a square
metre of the surface per second. The reOection coefftcient
of light from a given surface is 0.25.
Given: A=0.49 µm=4.9xlll"' m, p=5~Pa=
5x 10-0 Pa, p-0.25.
n-1
Solution. Using the formula p = l (1. + p)/c for light
pressure, we can determine the energy of all the photons
incident on a square metre of the surface per second:
I = pc/(I + p).
The energy of a photon is
e=hv=lu:I'>•.
Consequently, the number of photons incident on asquare
metre of the surface per second is
Cb. 4. Optics

n = .!_ = ___é:__
g 1&(1+p}

6.:::r~!·~r::r~) m-s.s- 1=2.9X1()2t m-Z·s-1.


653. A radiant energy now of 63 J is incident on a sur-
face of area 100 cm• per minute. Determine tbe ligbt pre&-
sure for tbe cases wben the surface completely reftects and
completely absorbs the entire radiant Dux.
Given: 8=100 cm.1 =10-Z m1 , t=1 min=60s, W=63 J.
p-?
Solution. 1. Since p = 1, the ligbt pressure is
p ~ 2//c.
By de&nition, I = W/(St), and hence
2W
P=-C§í"''

P- sxt0t2:16J..xeo Pa=0.7 ,...Pa.


2. Since p=O, p=llc, or
w
p= tS'f"'

P ãxto•x~O-Sx60 Pa=0.35 ,...Pa.


654. Why are photographic films developed in red light?
Answer. Red light does not act on the &lm in view of the
small energy of photons at t.bis frequency, wbicb is insuf-
ficient to initiate a chemical reaction in thc emulsion
layer.
655. On which surface (black or white) does ligbt exert
a higher pressure?
Answer. Tbe ligbt incident on a white surface produces
on it a pressure p = 21/c. Since tbe light is practically
absorbed by a black surface, its pressure is p = I /e (see
Problem 653). Hence it follows tbat a bigher pressure is

...
exerted on the wbite surface .
Select.ed Prohlems OD Physics

EXERCISES

656. Determine t.he energy of a photon if the wavelengt.h


corresponding to it is 17 X 10-11 m.
657. Determine the work function for zinc if t.he
wavelength corresponding to its photoelect.ric threshold
is 300 nm.
658. Determine the photoelectric threshold for lit.hi-
um.
659. The photoelectric threshold for potassium cor-
responds to a wavelengt.h of 0.577 llm. What. must be the
potential difference between the electrodes to stop the
electron emission from t.he surface of the potassium if the
cat.hode is exposed to light. with a wavelength of 0.4 llm?
660. Determine the photoelectric threshold for cesium
if the retarding potential for a radiation of wavelength
º·~f.~a~·t;a~ion of t.he energy of a photon causing
a photoeffect is spent for the work function if t.he maxi-
mum velocity of electrons knocked out from the surface
of zinc is 101 m/s? The photoelectric threshold for zinc
corresponds to a wavelength of 290 nm.
662. A radiant Dux of power 5 p.W and wavelength of
0.36 p.m is incident on the surface of a metal. Determine
the photoelectron saturation current if 5% of the incident
photons knock out. electrons from the metal.
663. A photon Dux of intensity 1018 photons/s is inci-
dent at. an angle of 45° on a mirror surface of area 10 cm'.
The wavelength of inciden t light is 400 nm. Determine the
light pressure exerted on the surface if the reflection coeffi-
cient for the surface is O. 75.
664. A radiant flux of power 1 1-LW is incident along the
normal to a surface of area 1 cm2 • Determine the light.
pressure if the reDection coefficient is 0.8.
665. Determine the light pressure of solar radiation
exerted on a square metre of the Earth's surface normal
t.o t.he direction of the radiation if the solar constant is
8.38 kJ/(cm2 ·min). The coefficient of refiection of light.
from the Earth's surface should be neglected.
666. A parallel bundle of rays of wavelength 0.5 p.m
is incident along the normal on a blackened surface pro-
Oh. 4. OpUcs

ducing a pressure of 10-• N/em• on it. Determine the


number of pbotons contained in a cubic metre of incident
light Oow.
QUESTIONS FOR REVISION

~r2~ih-:t ~ ~=~~~ftec';·a~A:tbet;;!r:l'~~
tric eflect. 4. lxplain Lhe working prmciple of a ~tocell.
5. Formulate the laws of photoelfect. 6. Wr1te Eins\eiD s law for
photoeBect. 7, What is photoelectric threahold? 8. What is ret.ard-
lng J)!>tential? 9. Write a formula for calculating light pressure.
10. What is the value of light pressure exerted on the surface of a
blackhody? on a mirror surface?
Chapler 5
OSCILU.TIONS AND WAVES

5. t. Mechanlcal Vlbrllllons
and Waves
Rarmonlc osclllaUons occur under the action of a
force F which is proportional to the displacement x of a
body and directed to the equilibrium position:
F = -kx,
where k is the proportiooallty factor.
The law of motion for harmonic oscillations is
:t=Asin((llt+ cp 0 ),
where x is the displacement of a body from the equilib-
rium position ata given instant of time, A the amplitude
of oseillations, cot + qi0 the phase of oscillatlons, cp 0
the iniUal phase, and w the cyclic frequency.
Cyclic frequeocy Ol is connected with the frequency v
of oscillatlons and the period T through the relations
oo = 2nv = 2n/T.
The perlod of natural oselllations oi a simple pendulum
is
T-2nVUg,
where l is the length of the pendulum and g the free-fall
acceleration.
The perlod of natural vlbnlloos of a slmple oacillator
is
T-2n\fmlk,
where rn is the mass of a vibrating body and k the spring
const.ant.
Ch. 5. Oscillations anel Waves 407

The instantaneous velocity of • hannonieally oscil-


latlng body is
V=~ =Â<OCOS (<0t +wo) = Vmu: COS(<Ot +fPo),

where A<0 = "mn: is the veloelty amplitude.


The aecelentlon of a harmonle•ll)' oselllatlng body at
a given instant is

a=*= ~:z = -Aw2 sin(<0t+qi0)

= -am 1xsin (<0t+cp0),

where .&o1 = 4mu is the •eeeleratlon ampllfude.


The force eallling harmonie osetllations
F = ma = -mA<01 sin (<0t + qi0 ) = -m<01.z,

where mAw1 = F mu: is the force •mplltude and m the


mass of an oscillating body. Since F = -k.z, we have
k=mw 1 •
The total energy of a harmonleally osclllatlng body is
W = mA 1 w1/2 = kA 1/2.
Besides natural oscillations, a body can perform forced
oscill•tions under the action of ao externai force. lf the
frequency of the externai force coincides with the fre-
quency of natural oscillations, reson•nee is observed.
A body can simultaneously participate in several vibra-
tions. ln this case, vibrations are added. Let us consider
two particular cases of addition of vibrations.
t. Two vibrations occurring along the same straight
line in the same direction with the same period but with
different amplitudes and initial phases are added. The
equations for component vibrations have the form
z 1 = A 1 sin (mt + q>01), .z1 = A 1 sin (wt - Wo 1).
The resultant vibration is expressed through the equa-
üon
Seleeted Problema OD Physics

where A = V A: + A: + 2A1A1 cos (tpo1 tpo1) is the


amplitude of the resultant vibration and cp 0 =
arctan ~::::t~:::: is its initial phase.
2. Two mutually perpendicular vibrations of the same
period but withdiflerent amplitudes and initial phases are
added. The trajectory of the resultant vibration is given
by
~~ +1\--2 ;:i COS(1Poz-CJ101)=sinZ(tpoz-qi
1 01).

Depending on the phase diflerence and the ratio of ampli-


tudes, this will be either a straight line, or an ellipse, or
a circle.
The process of propagation of vibrations in an elastic
medium is known as a wave. If the direction of vihrations
coincides with the direction of propagation of a wave, the
wave is referred to as longilwllnal (e.g. an acoustic wave
in air). If the direction of vibrations is perpendicular to
the direction of propagation of a wave, we have a lrans-
verse wave. The wavelengtb is defioed as
).. = vT,
where v is the velocity of propagation of a wave and T
the period of vibratioo.
The equation for a plane wave has the form
x = A sin wlt - (r/v)l = A sin (mt - kr),
wbere k = 2:n:/A is the wave number and r the distance cov-
ered by the wave from the source of vibrations to the
point under consideratioo (Fig. 212).

Fig. 212
Ch. S. Oscillat.iODS and Waves 409

The phase dHlerenee for two vibl'llting pofnts located at


distances r 1 and r 2 from the source of vibrationa is
âqi = qi 2 - qi1 = 2n (r1 - r 1 )/'A..
When a plane wave is incident on the interface between
two media, a reftected wave is formed. Being superim-

~
O

Fig. 213
r

posed on the incident wave, it forms a standiog wave.


The equation for a slanding wave is
x = 2A cos kr sin o>t,
where A (r) = 2A cos kr is the amplitude of the stand-
ing wave.
The amplitude of a standing wave has its maximum
value at the points satisfying the condition
r = 2nA/4
and known as the antinodea of lhe standing wave. Here
n =O, 1, 2, ... (Fig. 213, points A, C, E, ...).
The amplitude of a standing wave has its minimum
value at the points satisfying the condition
r = (2n + 1) l./4
and called the nodes of lhe standing wave. Here n =
O, 1, 2, . . . (Fig. 213, points B, D, F, ... ).

667. A material particle of mass 10 g vibrates accord-


ing to the law z = O.OS sin (0.6t + 0.8). Determine the
maximum force acting on the particle and its total ener-
gy.
Gi1Jen: m = 10 g = 10-• kg, z = 0.05 sin (0.61 + 0.8).
F, ? W ?
410 Selected Problema oo Physics

Solution. Comparing the general form of equation for


harmonic oscillations z= A sin (0>t+ tp 0) with the equa-
tion z= 0.05 sin (0.6t+0.8) given in tbe problem, we
fi.nd that A = 5 x 10-11 m, 0> = 0.6 rad/s, and tp0 =
0.8 rad. Using the expression F = -m.Aci.1 sin (0>t + qi0 )
for the force causing harmonic oscillations, we obtain
F 0 =mAw11 ,
F 0 =IO-• X 5X10-• X (0.6)' N=1.8 X 10-•N.
The total energy of the vibrating partiele is
H'-= mA;:i•
W t0-1 X (5 X til)I X (0.8)1 J = 4 .5 f'J •

668°. Write the equation for a harmonic oscillation


whose amplitude is 10 cm, the period is tos, and the ini-
tial phase is zero. Determine the displacement, velocity,
and acceleration of an oscillating body 12 s after the be-
ginning of the oscillation.
Given: A=10cm=0.1. m, T..:tos, Cflo=O, t 1 ='12s.
z-? v1 -? a,-?
Solution. Writing the equation for harmonic oscilla-
tions, we obtain
z =A sin (0>t + op,) = A sin (2nt/T + op,).
Substituting the given quantities into this equation, we
gel
z=0.1sin ( 2 xt~· 14 t}=O.isin0.6281.

For the instant t 1 , we can write


z = 0.1 sin (0.628 X 12) m <>< 0.095 m.
The veloci ty of the oscillating body is

v=~=Awcoa(wt+q1o) 1
Cb. 5. Oscillation1 and Wave1 4U

or, substituting the values of the given quantities, we ob-


tain
v = 0.1 X 0.628 cos 0.628t = 0.0628 coo 0.628t.
For the instant t 1 , we have
Vi = 0.0628 cos (0.628 X 12) m/s ~ 1.95 X 10-1 m/s.
The acceleration of the oscillating body is
a=*= -A0>Zsin(0>t+q1o),
or, substituting the values of the given quantities, we ob-
tain
a = -0.1 X 0.628' sin 0.628t = -0.0393 sin 0.6281.
For the instant ti, we have
a, = -0.0393 sin (0.628 X 12) mis'
= -3.73 X 10-1 m/s 3 •
669. ln what minimum time after the beginning of os-
cillations will the displacement of a point from the equi-
librium position be equal to half the amplitude if the pe-
riod of oscillations is 24 s and the initial phase is zero?
Given: .ci=A/2, T=24 s, 'fo=O.
ti-?
Solutton. Writing the equation for harm.onic oscilla-
tions, we obtain
z = A sin (<0t + cp0) = A sin (2n.t/T + cp 0).
By hypothesis, z 1 =A sin (2n.t1/T)=(1/2)A. Cancelling
out A, we get
sin (2nt,/T) = 1/2.
The value of the sine is 1/2 wben its argument is n.16,
i.e. 2n.t/T = n/6, whence

ti= -&- '


ti-=~•=2 8.
412 Select.ed Problema on Pbysics

670. Write t.he equation for a barmonic oscillation if


it.s amplitude is 5 cm, the period is 4 s, and the initial
phase is n/4 rad. Plot the time dependences of displace-
ment, velocity, and acceleration.
Giuen: A=S cm=0.05 m, T=4 s, Q>o=Jtf4 rad,
•-•m 1•-•m 1•-•00 i
Solution. Substituting the given values into the equa-
tion z=A sin (2nt/T +
cp0 ), we obtain

•-0.05sin(-}1+-T)· (1)
ln order to plot the time dependence of the displacement
z, we compile the table for the values of the displacement
z at various instants t. For the initial instant t 0 =0,
from Eq. (1) we obtain
00 -0.05 sin (n.14) m "" 0.035 m.

~;:~i:gqÍo(1lhe V:.~~!.u~~~:e!1:ntti;:eut!'Ã. :corre-


0,05=0.05 sin (~tmu+T),

whence sin (-}t u:+ f


111 )=1, ~tmu+T= T,
tmn: = 0.5 s. We can determine the values of z at imtants
t 1 = fmu +T/4., t1 = t 1 +
T/4, etc. The obtained
value.s are used for compiling the following table:

e, 11 1 o 0.5 1.5 1 2.5 1 3.51 4.5

s, m 1 0.005 0.05 1--0.051 o 1 0.05

Using the table, we plot the time dependence of the dis-


plaeement z=z(t) (Fig. 214a). ln order to plot the velocity
graph, we malte use of the equation
v-A.,cos(2nt/T +'Po).
z,m

5 t,s

a, m/st

t, s

Fig. 214
Select.ed Problema on Physic11
or, substitut.ing the given values of the quantities,
u=0.05 2: co•(~1+-T)<><0.08cos(~•+-T)·
ln analogy witb the previous case, we compile the fol-
lowing table:

'" 1 o 1 0.5 1 1.5 1•.51 •.5


v, m/s 1 0.06

Using the table, we plot the time dependence of the ve-


locity v=v(t) (Fig. 214b). ln order to plot the accelera-
tion graph, we make use of the equation
a = -Aw:t. sin (2rr.t/T + qi0 ),
or, substituting the given values of the quantities,
a= -0.05( 2: )2 sin (~•+-T)
~-0.12sin (~t+f ).
ln analogy with the previous cases, we compile the fol-
lowing table:

'· • 1 1 0.5 11.51 2.5 3.5 1 4..5

a, m/s• l -o.oe 1-0.121 o 10.12 J-0.12


Using the table, we plot the time dependence of the
acceleration a= a (t) (Fig. 2t4c).
671. Write the equation for a harmonic oscillation of a
body if its total energy is 3 x 10-15 J and the maximum
force acting on the body is 1.5 mN. The period of oscilla-
tions is 2 s and the initial phase is 60°.
G'ven: W=3x10-5 J, Fmu.""'1.5 mN=t.5xt0-3 N,
T=2 s, 1J1o=6D°=n/3 rad.
z=z(t)-1
Ch. S. Oscillat.ions and Waves .f.15-

Solutfon. The displacement of an oscillating body is


given by
z =A sin (2nt/T + ~.). (1)
ln order to determine the amplitude, we shall write the
expressions for the total energy and the maximum force:
W = mA 1 oo1/2, F 111 n = m.Am1 • (2)
Oi viding Eqs. (2) termwise, we obtain

Â=F::r.'
A= 2 ;:::t: m=4 X to-t m.
Then Eq. (1) beeomes
z = 4 x to- 1 sin (nt + n/3).
672. A pendulum consists of a heavy bali of mass 100 g
suspended on a string of length 50 cm. Determine the pe-
riod of oscillations of the pendulum and it.s energy if
the maximum angle by which it is deftected from the-
equilibrium position is 15°.
Given: m=100 g=0.1 kg, l=50cm=0.5 m,
«=15° ~0.26 rad.
T-?W ?
Solution. Assuming that the bali is a simple pendulum
and considering that it oscillates harmonically, we can
determine the period of it.s oscillations:
T=2nylig,
T=2x3.14V0.5/9.8s<><l.42 s.
The total energy of the pendulum deDected through an
angle a is the potential energy
w, = mgk. (1)
lt follows from Fig. 215 that k = 1AO 1- 1OB I· Since
1OB 1 = l cos a, we have
k = ! - ! cos a. = ! (1 - cos a.). (2)
416 Select.ed Problems oa Physics

Substituting expresaion (2) into (t), we obtain


w, = mgl (t - cosa),
W, = 0.t X 9.8 X 0.5 X (1 - 0.97) J "" 15 mJ.
673, A bali is suspended on a long string. It is 6.rst
lifted along the vertical to the point of suspension, and
then deDect.ed through a small angle.
ln what case will the ball retum to

14
the initial position sooner after
having been released?
cc Given: l.
t,!t,,-1
Solution. Let us consider the first
e
- :'e~~!'1d::e!!~n~q:.i•:i~~~ t7ori::~
fali of the ball from a height equal
to the length l of the string:
Fig. 215
t,=V2111. (1)
ln the second case, the time t 1 of motion of the bali from
the deDected position can be determined from the equation
z = A sin (mt + qi 0 ) for harmonic oscillations. Since at
the initial instant the pendulum suffers the maximum
deOection from the equilibrium position, its initial
phase is filo = n/2. Since .x = O in equilibrium, we can
write O =A sin (mt 1 + n/2). Hence, sin (cot 2 + :n:/2) =O
and ftlt 1 + :n:/2 = :n:, wbence
t, = n/(2co) = T/4. (2)
The bali is a sirnple pendulum, and hence its period of
oscillations can be determined from the formula T =
2:n:Jfiil. Substituting this expression into (2), we obtain
t, = T/4 = (n/2) V"ffi. (3)
Dividing Eq. (1) by (3) termwise, we get
.!!,= Y2't1 = 2]f2 ,,, 0 .9 .
'• n Vfii/2 n
Consequently, the bali returns to the initial position
.sooner in the fint case.
Ch. 5. Oscillation11 aud Wave11 417

674.. A small ball is suspended on a string of lengtb 1 m


from the ceiling of a carriage. At wbat velocity of tbe
carriage will the bali oscillate witb tbe maximum am-
plitude under tbe action of impacts of tbe wheels agaiost
the joints of the rails? Tbe lengtb of a rail is t2.5 m.
Given: l=f m, s=f2.5 m.
v-1
Solution. Tbe bali performs forced oscillatioos at
a frequency " equal to tbe frequency of impacts of the
wbeels against the joints:
"=vis. (1)
If the size of the bali is small as compared witb tbe lengtb
of thestring, itcan be trea.ted as a simple pendulum wbose
period of oscillations is T0 = 2n Jfüf. Then tbe natural
frequency of oscillations is
•o= llT, = (1/2n) Viif. (2)
Tbe amplitude of undamped forced oscillations attains
it.s maximum value in resonance, i.e. when " ~ "o·
Substituting this condition into Eqs. (t) and (2), we &nd
that vis = (1/2") VKii. whence

v=ii-Jlf,
v = 2 !23~14 Jf ~ 2;- ~ 6.2 m/s.

675. A copper ball suspended on a spring performs ver-


tical vibrations. What will be tbe cbange in tbe period of
vibrations if an aluminium ball of the same radius is sus-
pended on the spring?
Given: p1 =8.9x tos kg/m 3 , p1 =2.7x toa kg/m3 •
T,IT,-1
Solution. Since the balis suspended on the spring an
simple osrJ.llators, tbe periods of their vibrations are
T1 =2nVm,lk, T2 =2nViii;ik,
27-0970
418 Select.ed Probleme on Physics

where m1 = (4/3):nR1p1 and m1 = (4/3)nR 1p1 are the masses


of the copper and the aluminium ball. Therefore,
..!!..= 211 Vm1Jk .,,. 1/.:fl.
r, 2n Vm,Jk ,., Pt '
Ti 1/s.sxtoa 18
r;-= Y 2.1xtoa ~ · •
i.e. lhe period of vibrations will decrease.
676. Two vibrations of the sarne direction and frequen-
cy have amplitudes of 20 and 50 cm. The second vibration
leads in phase the first vibration by 30°. Determine the
amplitude and the initial phase of the sum of the vibra-
tions if the initial phase of the first vibration is zero.
Gtven: A1 = 20 cm= 0.2 m, A1 = 50 cm= 0.5 m,
fPo 1 =0, fP02.-1f01=300~0.52 rad.
A-1 ~0 -1
Solution. When two vibrations occurring in the sarne
direction are added, lhe amplitude of the resultant vibra-
tion is
Â= 1'A:+A:+2A A,eos(1J1oz-Ql01),
1

A= Y(0.2) 2 +(0.5) 2 +2 x 0.2 X 0.5 X 0.87 m ,,.o.68 m.

From the expression tan cp0 = ~::!:::!~:~~~!, we


can find the initial phase of the resultant vibration:

cp 0 =arctan !:::::!!:~~:::,
fPo=arctan g;:~::::g::1 arctan0.394~0.38 rad.
677. A point takes part simultaneously in two mutually
perpendicular vibrations with multiple periods, equal
amplitudes, and zero initial phases. Plot the trajectory of
the point if the period of vibrations along the Y-axis is
twice the period of vibrations along the X-ai:is.
Gtven: Â1 =Âz=Â, ~01 =po.=0, T2 =2T1:
y(z)-1
Ch. 5. OscillatiODI and Waves 'i.9

Solu.tton. The equat.ions for harmonic vibnt.ions along


t.he X- and Y-axes have t.he form
• - A aio (2nt/T1), (1)
y - A aio (2nl/T,). (2)
Since, by hypot.hesis, Ts = 2T1 , Eq. (2) can be reduced
t.o t.he form
r - A oin (2nt/2T1) - A sin (nl/T1). (3)
Since t.he equat.ion for the result.ant. t.rajectory in this
case is tnnscendental, we shall solve this problem nu-
y

Fig. 2i6

merically. Using Eqs. (1) and (3), we compile the t.able


of t.he values oh: and y at. various inst.ants t expresaed in
fractions of t.he period T1 :

l·lr.:~
o o o .• 5T1/8 -0.7A ..... I
••
T,!8 0.7A o.u 311/' -A 0.7A
T /4 A 0.7A 7T1/8 -0.7A o.u
~~~8 0.7A 0.9A ' T, o o
o A
d
' '
Using t.he table, we plot t.he t.rajectory abcdefghi of the
point. (Fig. 216). ln view of symmet.ry, we can supplement.

"'
Selecl.ed Problema on Phyaics

the trajectory with the region a./t. We leave it to the


reader to verify the correctnesa of thia assumption.
678. The displaeement from the equilibrium position
of a point lyiog at 4 em from the souree of vibrations ia
equal to half the amplitude in a time T/6. Determine the
wavelength.
Gtuen: r=4 em.=0.04 m, t-.T/6, z=A/2.
1
Soluiion. We transform the wave equation
z=Asino:i(t-r/v),
eonsidering that o:i = 2n/T and 1.. = vT:

z=A sin{ 2;' - ~r )=Asin( ~'- 2n{ ).


Then{ =A sin ( 2: : - 2n·~·Of.), and after transformations
we obtain Í=sin { -j-- 0 ·~ ). whieh shows that the
argument of the sine is n/6, i.e. -j-- O.~n =T.
Consequently, Â. = 0.48 m.
679. A transversa wave propagates along an elastie
cord at a veloeity of 15 m/s. The period of vibratioo of
points on the eord is 1.2 s and the amplitude of vibration
is 2 em. Determine the wavelength, the phase, and the
displaeement of a point separated by 45 m from the souree
of vibration in 4 s.
Gtven: v=15 m/s, T=t.2 s, .4=2 em.=2x10-2 m,
r=45 m, t=4 s.
1.-1 q>-1 •-l
Solution. The wavelengtb is
1.. = vT,
1. - 15 x 1.2 m - 18 m.
The phue and the displacement of any point ean be deter-
mined from the wave equation
z =A sin o:i (t- rlv).
Ch. 5. Oecillatiom and Waves 42i

The phue of vihration is equal to the argument. of the


sine in the wave equat.ion:

q>=OJ(t--;-)=~(1-7).
q>= ~ (4- ~ )rad"'5.24 nd.
The displaeement. of t.he point. is
z: = 2 x to-1 -sin 5.24 m ~ -1.73 x to-1 m.
680. Vibrations of period 0.25 s propagate along a
straight. Une at. a veloeity of 48 m/s. Ten seconds after
the emergenee of vibrationa at the init.ial point.8, t.he dis-
plaeement of a point. at 4.3 m from it. is found t.o be
3 em. Determine t.he phase and t.he displaeement. of a
point. separated by 4.5 m from the souree of vihrat.iona at
the same inatant..
Giuen: T=0.25 s, v=4B m/s, f=10 s, r 1 =43 m,
r 2 =4.5 m, z:1 =3 em,,,..3xto-a m.
'h-1 .. -?
Solutlon. Writ.ing t.he equations deseribing the vihra-
tions of the point.s separated by t.he distanees r 1 and r 1
from the souree, we ohtain
z:1 =Asinw(t-r1/v), z.=Asinw(f-rJv).
Considering that w""' 21'/T, we have

z:1 =Asin ~ (t-2!-)• :r::1 =Asin ~ (t-7)·


The amplitude of vihrations can be determ.ined from the
equation for z:1:

A 1in ((2n/T~(t r1/u}] '

A ain ((2n~~)1~~ 43/48)] m ~ 6 X 10-S m.


Using the equat.ion for z:1 , we can determine t.he phase at
t.he point separated by the dist.ance r 1 from the source of
422 Selected Problems on Physics

vibrations:
fP2=~(t-7).
cp 2 =o~ (10-~)rad=145Trad~227.65 rad.
The displacement at the point separated by the distance
r 2 from the source of vibrations at the instant t is
x 2 =Asinqi2 ,
x2 = 6 X 10-2 sin { 145 -T) m = 6 X 1Q-2 sin -T
=6x10-2 m.
681. Two points are 6 and 12 m away from a source of
vibrations. Determine the phase difference for the points
if the period of vibrations is 0.04 s and the velocity of
propagation of vibrations is 300 m/s.
Given: r 1 =6 m, r 2 = 12 m, T=0.04 s, v=300 m/s.
a~-1
Solution. The equations for vibrations of the points
are
x1 =Asin i (t-~). Xz=AsinT(t--?-).
whence the phases of vibrations of the points are
1P1=-}(t-~). 1Jl2= i (t--?-). (1)
Taking into account expressions (1), the phase diflerence
Õ.q:t=1P1-1P2is

drp=~{t-7")-~ (t-~)= ~ (ri-r1),


.ó.rp=o.042; 3õõ (12-6) rad =n rad.
The points vibrate in antiphase.
682. The distance between the second and sixth anti-
nades of a standing wave is 20 cm. Determine its wav&-
length.
Gtven: .ó.r6 ,i = 20 cm= 0.2 m.
A-?
Ch. 5. Oacillations and Waves 423

Solution. By hypothesis, .dr8, 1 = r 8 - r 1 , where r 8 is


the distance from the source of vibrations to the sixth
antinode of the standing wave and r 2 the distance from
the source to the seeond antinode. But the distance r
from the sourçe to an antinode is connected with tbe wave-
length through the relation r 11 = 2n)J4, where n is the
number of the antinode. Therefore,
r,-2·6A/4=3A, r 2 =2·2A/4-A, Ar,,,=3A-A=2A,
whence
Ã.= 6~··'
Ã.= º;/' m-0.t m.
683. Standing waves are excited in a cord of 3 m
whose one end is fastened to a wall and the other end

Fig. 2t7

vibrates at a frequency of 5 Hz. Six nodes are formed bet-


ween the source and the wall. Determine the velocity of
propagation of tbe wave in the cord.
Given: l=3 m, v=5 Hz.
v-1
Solutton. The velocity of propagation of a wave is
v = AIT = Av. Figure 217 shows that l = tU/4, whence
A = 41111. Therefore,
...
V=""iT'"•
4 x ~x 5 m/s=5.45 m/s.
v= 1
6M. Will tbe period of oscillatioos of a swing change if
a load is placed on the board?
Selected Problema on Physics

Answer. The swing sbould be regarded as a simple pen-


dulum. Therefore, the period of oscillationa will not
cbange aince it does not depend on the lll8SS of the load.
685. If a person carries a load tied to one end of a string,
the amplitude of the load becomes very large at a cer-
tain pace. Explain the phenomenon.
Amwer. If tbe frequency of periodic jerks during
walking coincides with the natural frequency of the load,
resonance will take place, which leads to a considerable
increase in tbe amplitude of the load.

EXERCISES

686. The period of vibrations of a matel'ial particle is


2.4 a, the amplitude ia 5 cm, and the initial phase of vi-
brations is n/2. What are the displacement, velocity, and
acceleration of the vibrating particle 0.4 a after the be-
ginning of the vibrationa?
687. A body performa harmonic oscillations according
to the law :z = 50 ain (n/3)t cm. Determine the amplitude
of the force and the total energy of the body if its mass is
2 kg.
688. ln wbat time does a body vibrating harmonically
according to the law z = A ain (l)t cover (t) the entire
trajectory from the equilibrium poaition to the extreme
position; (2) tbe fi.rat half of the trajectory; (3) its second
bali?
689. Plot on the same graph two harmonic oscillationa
of the sarne amplitude 3 cm and with a period of 8 s,
having the phaaes n/4 and 3n/2.
690. Determine the maximum velocity and the maxi-
mum accel81'ation of a vibrating point if its amplitude is
5 cm and the period is 4 s.
691. A material particle vibrates harmonically with a
period of 2 s, an amplitude of 50 mm, and a zero initial
phase. Determine tbe velocity of the particle at the instant
when its displacement from the equilibrium position ia
25 mm.
692. The amplitude of harmonic vibrationa of a mate-
rial particle is 2 cm and its total energy of vibrations is
Ch. 5. Oscillaüone and Wavee 425

3 x tO-' J. At what displacement from the equilibrium


position does a force of 2.25 X t0- 6 N act on the particle?
693. A pendulum clock is accurate when the length of
the pendulum is 55.8 cm. Wbat will be the lag of the clock
in 24 h if the pendulum length is increa.sed by 0.5 cm?
Assume that the pendulum is simple.
694. Determine the period of vibrations of a simple os-
cillat.or if its mass is t96 g and the spring constant is
2 X 10' N/m.
695. Two harm.onic oscillations of the same direction,
t.be same period of 8 s, and the same amplitude of 0.02 m
are added. The phase difference in oscillations is rc14
and the initial phase of one of them is zero. Write the
equation for the resultant oscillation.
696. A point participates in two mutually perpendicu-
lar vibrations simultaneously. The equations for the vib-
rations are :e = 2 sin wt and y = 2 cos wt. Determine the
traject.ory of the point.
697. A wave propagates from a source of vibrations along
a straight line. The displacement of a point at the moment
of time 0.5T is 5 cm. The point is separated from the
source by '>J3. Determine the amplitude of vibrations.
698. A source performs undamped vibrations according
to the law :e=0.05 sin 500nt m. Determine the displace-
ment of a point 60 cm from the source O.Ot s after th&
beginning of the vibrations. Tbe velocity of propagation
of the vibrations i.s 300 m/s.
699. Determine the phase difference for two points sep·
arated by 20 cm for a wave propagating at a velocity of
2.4 m/s at a frequency of 3 Hz.
700. A tuning fork (a source of sound waves) is placed
in front of ao observer's ear and another identical tuning
fork is placed ata distance of 47.5 cm from the 6.rst. The
observer does not hear any sound with such ao arrange-
ment. Determine the frequency of vibrations of the
tuning fork.
QUESTIONS FOR REVISION

1. What vibratiom are called undamped harmonie vibratiom?'


2. Write expz_eulom for the diaplacement, ih.e velotity, ud the
acceleration of a vibrating point. 3. Define the amplitude, the pe-
Selected Problema on Phy1ic1

r1od, and phue of an oscillation. 6. Write formulas for the pe-


riods tions of a simple ~ndulum anda simple oacillator.
f~:4y;:;:~~~,~i~º~,:W:.~r.:!i:hl':,i:::i:~ t:r:;:.1
:1r:dd::::?=:l :.· rP.~":~~~y;; :~~::1 ~-:~
tO. What is wavelength? tt. Write a formula connecting the wave-
length, the velocity of propagation of a wave, and its ~riod.
!~Xf~tew=:e i!'To'~~0~t~ W~~e~fi:~:d~!!ial!:!i!:d!°:Í !
standing wave? How can their po,sitions he determined relative
to the source of vihrations?

5.2. Electromagnetlc Osclllatlons


and Waves
When a plane frame of area S, consisting of N turns, ro-
tates uniformly in a magnetic field of induction B at an
angular velocity a>, the emf induced in the frame is
1 = 5 0 sin (a>t + cp0),
where 3 0 = NSBa> is the emf amplitude.
If the voltage supplied to a circuit without inductance
and capacitance varies according to the law
U= U0 sina>t,
the current I varies according to a similar law
1 = / 0 sin a>t.
The resistance of such a circuit is known as (ohmlc)
reslstanee and is given by
R = U,11 0•
The direct current whicb liberates in a circuit witb an
ohmic resistance tbe same energy per unit time as an al-
ternating current is known as tbe elfectlve value leu
:!Uetr·t::eTbe
·~~=:i~:1c:::t·i~:1t~· ~b!~Ç~~:SP:~':!~
effective voltage and current are connected with
tbe amplitude values of tbe alternating current tbrougb
the following relations:
u,. = U,1}'2. Ioff. = I,JJf2.
Ch. 5. Oacillatione and Waves .Ui

The parameters of an a.e. circuit with an ohmic resi-


stance are calculated in the same way as for a d.e. circuit
by using effective currents and voltages.
Let us consider a circuit consisting of a small ohmic
resiatance R and a capacitor of capacitance C (Fig. 218a).
If the voltage across the capacitor varies according to
the law
U = U0 sin (l)t,
we can neglect tbe ohmic resistance R and write
q = q0 sin (l)t, 1 = / 0 cos (l)t,
where q0 = CU 0 and / 0 =- q0 (1) = CU 0 (1).
The reactance of such a circuit (eapaeitive reaetanee)
can be determined from Ohm's law:
Rc = U 0 110 = 1/(l)C.
Let us consider a circuit consisting of a small ohm-
ic resiatance R and a coil of inductance L (Fig. 218b).
lf the voltage applied to the circuit varies according
to the law
U = U0 sin (l)t,
we can negleet the ohmic resistance R and write
I = -/0 cos (l)t,
where / 0 - U,l(wL).
The reactance of such a circuit (tndueUve reaetanee)
can be determined from Ohm's law:
RL = U 0/ /0 = (l)L.
Tbe lmpedanee oi ao a.e. eireuit containing an ohmic
resistance and capacitive and inductive reactances

·o ·O ·O·
(a) (6}
Fig. 218
(C}
Select.ed Problema on Physlcs

(Fig. 21&) is

Z= VR'+ (0>L- ~e)'.


A system consisting of \wo windinp coupled \hrough
a core is called a tr&D1former. If the primary winding con-
tai na N 1 \ums and \he secondary winding haa N 1 \urna,
\he trallllformaUon raUo is
k = N 1 /N1 = W1/l 1 ,
where 1'1 and 1 1 are the emf's induced in the primary
and secondary windings.
If the volt.age drop in the ohmic resistance of \he pri-
mary winding of a transformar is negligibly small, and
if \he secondary winding is disconnected, then 1 1 =
U1 and 1 1 = U1 , which gives
N1IN1 = u.1u9.
The efftcleney of a transformer is the ra\io of \he power
P 1 developed by \he secondary winding \o \he power P 1
applied to the primary winding:

1}=~100%.
ln a circui\ consisting of an inductance L anda capaci-
tance C, there can be induced natural electromagoeUc
oseJllaUons whose period is determined by TbOIDSOD. 's
formula

Such a circuit is a somce of eJectromagnellc waves which


involve \he propagation of oscilla\ions of electric and
magnetic &elds.
The equaUons for 8 plane eleetromagneUc wave bave
the form
E = E 0 sin w (t - r/c), B = B0 sin w (t - r/c),
where e = 3 x to• m/s is the veloelty of eJeetromagnetlc
wave1 ln vacuum..
Ch. 5. Oscillations and Waves '29

701. Ana.e. circuit with an effective voltage of 110 V


contains a series-connected capacitar of capacitance
50 Jl.F' and a coil of inductance 200 mH and ohmic re-
sistance 4 D. Determine the amplitude of tbe current in
the circuit if tbe frequency of the altemating current is
100 Hz. Find the a.e. frequency at which the voltage reso-
nance is observed in the circuit.
Gtven: u,.=110 V, C=50 µF=5xt()-• F,
L=200 mH=0.2 H, R=4 Q, v=IOO Hz.
lo-? Vr-?
Solution. According to Obm's law, the amplitude of the
current is
I,
u, (1)
JfRtT(tiJL-t/(IOC))t"
Considering that U0 = U, 11V2 and Ci> = 2:rtv, we can
write Eq. (1) in the form

lo Uerrt1"2
J.ÍR•+(2nvL t/(2mlC))'

10 Jf4•+12x3.t4xtooxo~:o~;c~4~3.t4x too x sxto-•n•


0<1.65 A.
At the voltage resonance, the amplitudes of voltage
across the capacitor and the coil are equal: Uoc = UoL·
Since U0 c = lof(2:rtv.,C) and UOL = 2stvrL10, we obtain
t
v,=2n VLC'

v, 2x3.t4V0~2x5xto-• Hz~SOHz.
702°. An electric circuit with a low resistance contains
a capacitor of capacitance 0.2 µF and a coil of inductance
t mH. During the resonance, the current in the circuit
varies according to the law J = 0.02 sin wt (Fig. 219).
430 Selected Prohlems on Physics

Determine the instantaneous value of current and the


instantaneous values of voltages across thecapacitor and tbe
coil one-third of the period after the beginning of oseilla-

Fig. 2t9
tions. Plot lhe time dependences for tbe current and the
voltages.
Given:C=0.2 µF=2xlQ-T F, L=I mH=10-' H,
t=T/3.
l 1 U0 -1 UL-1
Solutlon. Sinee tbe resistance R ~O, tbe cyclic fre-
quency w = 2n!T, or, using Tbomson's formula,

"'=,..:-u= /rc ·
(l)= Vto-a: 2 xtO-' ~d -=7.1x10' rad/s.
According to tbe law of current variation, we can write
l = 0.02 sin (2nt/T). (1)
For t = T/3, lhe instantaneous vaÍue of the current is
1=0.02sin(~~T)A=l.73xt0-Z A.
The voltage across the capacitar is Ue = q/C, where q
is the charge on tbe capacit~r. By definition, the current
i
is I = dq/dt, whenee q = J dt. Substituting Eq. (t) into

q-
. o
lhe integrand of this formula and integrating, we obtain

io 0.02sio (l)tdt= - º-:2 COS(l)t.


Ch. 5. Oscillations and Wnes 43t

Then tbe voltage aeross the eapaeitor is


Uc=- o~~ coswt= 0: :2 sin (wt--j-)
= º:: 2 sin ( 2; t-T).
For t=T/3, we get

Uc ?.txt~~xto-1 sin (~f-T) V=0.7 V.


The voltage aeross the coil is
UL=Lf.-=Lf.-(0.02sinGll)=0.02mLcosGll

=0.02mLsin ( G11+-j-).
For t = T/3, we ohtain
Ui.=0.02x 7.1xtf}lox10""3 sin ( ~f+T) V
:.-0.7 V.
The time dependenees for the eurrent and the voltages ae-
ross the eapaeitor and the eoil are shown in Fig. 220a
and b.
(a}
I

º~r't
(6)
Uc.UL

~
o~
Fig. 220
Selected Problems oa Phyaica

703°. The voltage ac.rosa the eapac.itor in the elec.tric.


c.ireuit shown in Fig. 219 varies according to the law
Uc =O.Oi sin <i>t. Determine the instantaneousvaluesof
the c.urrent and the voltages ac.ross the eapac.itor and the
c.oil after on&-aixth of the period if the eapac.itanc.e of the
c.apac.itor is 0.02 µF and the induc.tanc.e of the c.oil is
tO mH. Plot the time dependences for the c.urrent and the
voltages ac.ross the c.apac.itor and the c.oil.
Given: C-0.02 pF-2x !()-• F, L-10 mH=llr' H,
t-T/6.
Uc-? /-? UL-?
Solution. ln analogy with Problem 702, for R = O,
we have
t
.,_ vu'
Vto-•xzxto-• 7 ~7.txtO• rad/s.
The instantaneous value of the volt.age ac.ross the capaci-
tar is
Uc =O.Oi sin <i>t. (1)
For t = T/6, we have
Uc=0.01sin (~{-) v~s.6 mV.
The charge on the capacitar is q =CU e, or, taking into
acc.ount Eq. (1), q = O.OtC sin wt. The c.urrent is
I= i =-i; (0.01Csinwt)=0.01Cwcoswt
=0.01C<i>sin ( wt+f). (2)
For t = T/6, the instantaneous value of the c.urrent is
/-O.OI X 2x t()-lx 7.1 x t<l'sin ( ",;' +r)A-7.lµA.
The voltage ac.ross the c.oil is UL=L~, or, taking
into account Eq. (2),
U• - L f,- (O.OIC0> cos "") - -0.0ILC.,Z sin 0>t.
Ch, 5. Oacilla~a and Wavea 433

Since (1) 1 = 1/(LC), we have


UL = -0.01 sin (l)t = 0.0t sin ((l)t + n).
For t = T/6, the instantaneous value of the voltage aeross
the coil is
UL=O.Otsin(-}f+n) V~-8.6 mV.
The time dependences for the current and tbe voltages ean
be constructed in analogy with the previous problem (we
leave this to the reader).
704. Determine the amplitude of tbe emf indueed during
tbe rotation of a rectangular frame at a frequeney of
50 Hz in a uniform magnetic Jield of induction 0.2 T
if the area of the frame is 100 cm1 and the induction veetor
is perpendicular to the rotational axis of the frame. Tbe
initial phase is zero.
Gtvsn: v ..... 50 Hz, S=tOO cm2 =10""2 m2 , B=0.2 ·r,
'Po~o.
t,-1
Solu.tion. Aeeording to Faraday's law, the emf indueed
in the frame is
I=-~~. (1)

Here .àcJ> = cJ> 2 - cJ>1 is the ehange in tbe magnetie Oux


during the time .àt, where cJ> 1 = BS eos (l)t and cJ>1 =
BS cos [(I) (t + .àt)} are the magnetic Ouxes piercing the
frame at the instants t and t + ât. Therefore,
à<I> ~ <!>, - <1>1 ~ BS {e.. 1'" (t + àt)l - coo ..1). (2)
Using the trigonometrie formula cos (a. + P) =
cos a. cos p - sin a. sin p, we traosform expression (2)
as follows:
AcJ> = BS (cos (l)t cos (1) At - sin (l)t sin (1) At - cos (l)t).
Since w .àt is small, cos (1) At ~ 1 and sin (1) At ~ (1) ât.
Consequently,
AcJ> = BS (cos (l)t - (1) ât sin (l)t - eos (l)t)
= -BS w ât sin wt. (3)
28-0970
,. Selected Problems on Physies

Substituting expression (3) into (1), we obtain 1 =


BSw sin wt, which shows that the emf amplitude is
i 0 = BSw, or, since '-'> = 2nv,
~o= 2:r:vBS,
i, - 2 X 3.14 X 50 X 0.2 X to-• V - 628 mV.
The emf can be determlned by differentiation if we wrlte Fara-
day's law in the form

I=-~~.
705. The voltage across a subcircuit carrying an alter-
nating current varieB with time according to the law U =
U0 sin (cot + n/6). At the moment t = T/12, the instan-
taneous voltage is 10 V. Determine the voltage ampli-
tude.
Giuen: t-Ttt2, U,-tO V.
u,-1
Solution. Substituting the values of U1 and t into the
equation
U = U0 sin (wt +
:n/6)
and considering that w = 21t/T, we obtain
10=U0sin (~*+f), or 10=U011in-j-,
whence
Uo= sin~~t3) =~ Ve::H.5 V.
706. An eleetric furnace whose resistance is 22 Q is fed
by ao a.e. generator. Determine the amount of heat lib-
erated by the furnace in 1 h if the current amplitude is
10 A.
Given: R=22 !l, t~t h-3.6x til' s, / 0 -tO A.
Q ?
Solution. According to Joule's law, the amount of heat
liberated in ao a.e. circuit is
Q - IlirRt - (I,tV°2)' Rt,
Q - (t0tV2)' X 22 x 3.6 X t01 1 - 39.6 M1.
Ch. 5. Oscillat'°ns and Waves 435

707. The current in the primary winding of a transform-


er ia 0.5 A and the voltage acroas its ends is 220 V. The
::rr::ti~n :!eisª:0:.~ªV 'J::~i! ;i~e~:!i!!:/~~t~
traosform.er.
Given: / 1 =0.5 A, U1 =220 V, / 2 =11 A, U,=9.5 V.
~ 1
Solution. The efficiency of tbe traosform.er is defined as
TJ=.;;-100%.
Since P 2 =12U2 and P 1 =/1U1, we can write
~-~100%,
~=o·~~~i, 100%= 95%.
708. During what time will a neon lamp glow if it is
connected for 1 min to an a.e. circuit with an effective
voltage of 120 V and a frequency of 50 Hz? The lamp
starts and stops glowing at 84 V.
Gtven: t0 =1 min=60 s, Uet1=120 V, i1=50 Hz,
U•=84 V.
tl.-?
Solution. Wben the lamp is connected to the circuit, tbe
voltage across it.8 electrodes varies according to the law
U = U 0 sin 2ni1t, (1)
where
u, - v2u... (2)
The neon lamp starts and stops glowing at Ue < U0 •
Coosequently, tbe duration of its glow during half a
period ia
4t=t,-t,. (3)
where t 1 and t 1 are the instant.8 wben the lamp start.8 and
stops glowing. Since the interval t0 contain.s 2t0v time in-
t.ervals ât, the lamp glows during the time

... t, = 2t0v 41. (4)


Selected Problems on Pbysica

From Eqs. (1) and (2), we 6nd that U - }"2u,, sin 2n•t.
lf t = ti and t = t 1 , we have U = U1 , and hence
U1 =\12U1 rrsin2n"Yti, U1 - V2u1 rrsin2nvt,,,
whence

sin2nvti= y~~err ,
sm(2"·50t1) - t.<~l20 <><}.
Consequently, 100nti = n/6, whence ti= 1/600 s. Sim-
ilarly sin (2n· 50t,) !::!!: 1/2, i.e. 100nt 1 = 5n/6, whence
t 1 = 5/600 s. Subst.it.uting t.he values of ti and t 1 into
Eq. (3), we obt.ain
llt - (5/600 - 1/600) s - 1/150 s.
The t.ime during which t.he lamp glows can be determined
by subst.it.uting the values of .6.t, t 0 , and 'Y into Eq. (4):
t, - (2 X 60 X 50 X 1/150) S - 40 s.
709. The potent.ial diflerence across t.he plat.es of a ca-
pacit.or in an oscillatory circuit. varies according t.o t.he
Iaw U = 50 cos (10'nt). The capacitance of the capacitor
is 0.9 µ.F. Determine t.he inductance of the circuit., the
law of time variation of t.he current. in the circuit., and the
wavelengt.h corresponding t.o t.he circuit..
Gtven: C-0.9 µF-9x1Q-7 F.
L-? 1.-1 l(t)-?
Solu.tton. Using Thomson's formula
T - 2ny'LC, (1)
we can determine L = T3/(4n 2C), or, considering t.hat.
T = 21'/w, L = 1/(Cw2). Jt follows from the equation
U = 50 cos 10'nt that w = 10'n rad/s. Consequently,
t.he inductance of t.he circuit is

L"""exto-1x~O'x3.U)1 H!::!!:1.12 mH.


Cb. 5. Osclllations and Waves 437

Tbe wavelengtb is A=cT, or, taking into account


expresaion (t),
A=2;c'
,.._ zx:~~xs~t~to• m=6x 10' m.
By defi.nition, the current is
I = - /0 sin (l)t. (2)
Since / 0 = U0 /(!i1L), we have (aee Eq. (2))
l= - ~1 sinwt.
Tbe values of U0 and (1) can be determined from the equa-
tion U ,.. 50 cos 101:id: U0 = 50 V and (1) = tOªn rad/s.
Consequently,
1- tCl'xS.H.~.tzxm-a sintD'nt=-t.42sint()int.
710. Tbe capaeitance of tbe variable capaeitor of the
oscillatory eircuit of a receiver varies between C1 and
C1 = 9C1 • Determine the waveband of the reeeiver if tbe
capacitance C1 of tbe capacitor corresponds to a wave-
length oi 3 m.
Giwn: C1 =9C\, 1..1 =3 m.
Â.1-?
Solution. We denote by A,, and A1 the wavelengths
limiting the waveband. Tben
A,=cT,=2...,VLC,. (!)
l.,=cT,-2...,VLC,, (2)
wbere T1 and T1 are tbe minimum and maximum periods
of oscillations of tbe cireuit. Considering tbat C1 = 9C1 ,
we can write Eq. (2) in tbe form
l.,=6...,VLC,. (3)
Comparing Eqs. (!) and (3), we find that A, = 3!.,, i.e.
Ã.t = 3 X 3 = 9 m. Consequently, tbe wavehADd of the
Selected Problema ou Physics

oscillatory circuit is limited by the wavelengths 3 and


9 m.
711, A direct and an alternating correnta under the
same voltage are passed in turn through lhe &.lament of an
electric lamp. Will the amount of heat liberated by the
lilament be the same io the two cases?
Answer. Consideriog that lhe filament bas purely ohmic
resistance, we can say that it is heated in tbe two cases
to the same exteot since, by bypothesis, the d.e. voltage
is equal to the effective voltage Uen of the alteroating
current. According to Joule's Iaw, for direct and alter-
oating currents we have
Q1 = u 1t/R, Q1 = u:ntlR
respectively. Compariog these e:r.pressions, we &.nd that
Q11'2?Jiow is energy transferred from the primary wiod-
ing of a transformer to the secondary winding, although
the windings are not connected to each other through a
conductor?
Answer. When an electric curreot passes through the
primary wiodiog of the transformer, it produces a mag-
netic flux which entirely passes through the core and
pieroes tbe turos of the secondary winding. Since we are
dealing with an alternating current io the primary win-
iog, the magnetic Dux varies with time and an emf is
induced in lhe secondary winding.
713. Why are skip (sileot) zones observed in radiocom-
munication on short waves?
Aoswer. ln view of nonhomogeneity of the atmosphere
for electromagnetic waves of this range, they are re-
fracted, which leads to the emergence of skip zones.

EXERCI SES
714. A resistor of resista.nce 15 O anda coil of inductaoce
50 mH are conoected in series to ao a.e. circuit under
a voltage of 120 V. Determine the frequency of the cur-
rent if the amplitude of the curreot in the circuit is 7 A.
715. An oscillatory circuit bas an inductaoce of 1.6 mH
and a capacitance of 0.04 µF. The maximum voltage ac-
Ch. 5. Osclllat.ions and Waves 439

ross the terminais is 200 V. Determine the maximum cur-


rent in the circuit, neglecting the resistance of the circuit.
716. Determine the instantaneous and effective values
of the emf in an a.e. circuit 0.002safterthe beginning of
oscillations if the amplitude value of the emf is 127 V.
The frequency of tbe alternating current is 50 Hz and its
initial phase is zero.
717. The primary winding of a transformar with a
tranaformation ratio of 8 is connected to a circuit under
a voltage of 220 V. The resistance of thesecondary winding
is 2 í.l and the current in it is 3 A. Determine the volt-
age across the terminais of the secondary, neglecting the
losses in t.he primary.
718. A coil of length 50 cm and cross-sectional area
3 cm' contains 1000 toros and is connected in parallel to
an air capacitor. The capacitor consists of two plates of
area 75 cm' each, .separated by 5 mm. Determine the pe-
riod of oscillations of the obtained oscillatory circuit.
719. Determine the period of oscillationa of an oscilla-
tory circuit emitting an electromagnetic wave of wave-
le~~ Jet~:1~ine the period of oscillations in an oscilla-
tory circuit containing a capacitor of capacitance SOO pF
and a coil of inductance t mH.
721. The inductance of the coil in an oscillatory circuit
varies between 50 and SOO H and the capacitance of the
capacitor varies between 10 and 1000 pF. What frequen-
cy waveband can be obtained by tuning such a circuit?
722. Determine the natural frequency of electrical os-
cillations in a circuit containing a coil of inductance
3 ~~. EJ!c:Ji:~:~\~~ :!a~::;~:;::; i~~ homogene-
ous medium at a veloeity of 2 X 108 m/s. What is the
wavelength of electromagnetic oscillations in the medium
if their frequency in vacuum is t MHz?
QUESTIONS FOR REVISION
1. Describe the constJuct.ion of a simple oscillatory cin:uit.
2. What are tbe natural (requency and period of osclllat.ions of a
cin:uit? 3, Write an expression for the emf induced in a frame ro-
tating unifonnly at. a certain angular velocity in a magnet.ic field.
Selected Problems OD Physics

4.. What •re tbe elfective values of voltage and eurrent? 5. What
are lhe valuea of resisLulce and capacitive ud induetive re•e~

;~t.~· d:~: i:Ueaifldre:'!~~r::r~mr.~ ~! :a:Jõr~':ti!i


ratio? 9. Define tbe elftciency of a tramformer. tO. Write an equa-
tion for • pl•ne eleetromagnetic wave. li. What 11 tbe velocity of
8D eleetromagnetie wave in vacuum?

S.J. Wave Properlles oi Llght


Phenomena like interference. diBraction, and disper-
sion of light determine its wave properties.
li~~r:.~r:~: aº!~~~~lofi~:ic~ur~~:!~:i. :!d c~~:~t
in light intensity are formed in certain regions of space.
!r~~l~:~0e1:::::; i~8 s:::.iri~~t~!~~h::C:~i!~~~~:,~
enl if their phase difference remains constant in time.
The condllion for lhe mHlmum lntensfty of Ught is
â=2k (1./2),
wbere ô.=n 1 r 1 - n1 r1 is the oplfcal path dlllerence for
waves, k=1, 2, 3, ... is an integer, and 1.. the wave-
length.
The condllfon lor lhe mlnimum inleDBlty has the form
â=(2k + 1) (1./2).
When interference is observed in thin plane-parallel
plates in reDected light, the conditions for the maximum
and minimum intensities have the form
li.= 2dn cos r - (1./2) = 2k (1.12),
â=2dn coor- (1./2)=(2k + 1) (1./2\,
where d is the thickness of a thin plate, n the refractive
índex of the material of the plate, and r the angle of re-
fraction of light in it.
For interference in the transmitted Jight, the condi-
tions for ma:s:imum and minimum change places.
DUJraclion of llght is a phenomenon observed when
light propagates in a medium with clearly manifested
inhomogeneities and consisting in the light propagation
Ch. 5. Oscillatiions and Waves -Kt

to the region of umbra. Diffraction of light can be ob-


served by using a diflracUon grating.
The condiüon for the principal maxhna for a normal
incidenee of light on a di8'raction gratlng has the form
dsinqi=k'A,
where d is the grating constant, qi the angle of deftection
of a ray from the initial direction, k = 1, 2, 3, ... is
an integer (spectrum order), and ). the wavelength of
light incident on the grating.
Disperslon of llght con.sists in the dependence of the re-
fractive iudex on the wavelength. An experimental evi-
dence of dispersion is the decomposition of white light
:~i:ts~::n~ii:,b!'e~!!ih;~:~i~:io~:: !t~~~ º!::i~:~
uously transformed into one anot.her. Consequently,
white light is compound. Red-hot solids and liquida give
continuous spectra, while incandescent gaaes produce
line spectra typical of a given element.
ln optics, a concept of Iight ray is often used. A ray is.
a Une along which light transfers ao energy.

724. A Iight wave of wavelength 700 nm propagates.


in air. What is its wavelength in water?
Gtven: ).1 = 700 nm = 7 x 10-7 m.
A,-?
Solution. The wavelengths ).1 and ~ of the light waves.
in air and in water are connected with the velocities
v1 and v1 of propagation of the waves in air and in water
through the following relations:
(1)
where 'Y is the frequency of light oscillations, which does
not change upon a transition of light from one medium
to another. Dividing Eqs. (1) termwise, we obtain
(2)
Select.ed Problems oo Physjcs

The velocities of propagation of lighl. in air and in water


are conneeted with the absolute refractive indices "1
d and n 1 of the media through the relation
S, Sr v1/v1 = n fni.
1 (3)

Comparing expressions (2) and (3), we lind


that lwi/'>.,, = n 11ni, whence

~= 1~:· 1

Âa= 7xt1:;x1 m!:!:::5.26x 1()-1 m.

725. Two coherent sources 3 1 and 3 1


emitting at a wavelength of 0.5 ~ are
separated by 2 mm. A screen is placed par-
Fig. 221 allel to the line connecting the sources
at 2 m from them. What will be observed
at point A oi the soreen (Fig. 22i)1
Givtn: A-0.5J11D=5X1()-T m, d=2 mm=
2x10-1 m, l=2 m.
à-1
Solutton. At point A of the sereen, there will be the in-
tensity maximum if the path difference for the rays ema-
nating from the sources 3 1 and 8 1 is equal to an integral
numher of wavelengths, and the intensity minimum if
the diBerence is equal to an odd number of half-waves.
Let us calculate the path diBerence (see Fig. 221):
à= IS,A l-IS1A 1.
where iS,Al=Vl'+d' and 1s1A1-1. Coneequently,
à= VP+d'-l =IVl+(d/l)'-1.
Since dll < 1, using the formula of approximate calcula-
tions, we obtain
bl[t + i-( f )']-1 =4i-.
â!:!::: c2 ~!~'> 1 m=1Q-f m.
Ch. 5. Oscillation1 and Wave1 "3

Comparing the values of 4. and A.1 we find that. tbe path


dilerence 4. is equal to an integral number of wavelengths
(two). Consequently, at point A tbere will be the intensi-
ty maximum.
726. A vertical soap film is first observed in reDected
light through a red glua (l., = 6.3 x 10-• m). The dis-
tance bet.ween two neighbouring red Unes is 3 mm in
this case. Then the lilm is observed through the blue glass
(1. = 4 x 10-1 m). Determine the separation between
two neighbouring blue Unes, assuming that lhe shape of
the lilm does not change during the observations.
Given: A.1 =6.3 X 1o-" m, z 1 =3 mm=3 x 10-S m,
l.,=4x11t"' m.
"z-1
Solu.tUm. The rays which get into an observer's eye

B
are reDected from a thin wedge at right angles to its sur-

.!!
z,
1

(b)
Fig. 222

face. The opt.ical path dilerences for the kth and (k + 1 )th
red lines are respectively
à,=2h,n -l.,12=kl.,,
(1)
à,.,= 2h,.,. - l.,/2 = (k + 1)1.,
(cos r = 1 in both cases). Here h11 and h111 +1 are lhe thick-
nesses of the vert.ical soap film whose section is a wedge
(Fig. 222a), corresponding to the lines. From Eqs. (1),
we obtain
4.111+1 -4.111""" +in-i--( 2h n-1t)
2h11 111

=(k+1)l.,-kÃ,,
Selected Problema on Phy11ics

whence
2n (h,+1 - h.) - 1.1. (2)
Similarly, for the blue lines (Fig. 222b)
2n (h,..1 - hm) - ).,. (3)
Dividing expression (2) by (3) termwise, we find that

::::=t =i;-. (4)


On the other hand, it follows from the similarity of tbe
hatebed triangles (see Fig. 222) that
(5)

~~:·~~/l ~e :1if:c:~1:b!:!des of Eqs. (4) and (5), we ob-

z2 = z1 i;-•
x2 =3x1Q-3x 6~3xxi'(;.., m=1.9x10""'m.
727. Determine the radius of curvature of a lens used
for observing Newton's rings if tbe distance between tbe

Fig. 223

second and third bright rings is 0.5 mm. The instrument


is illuminated by light of wavelength 5.5 x 1()-7 m, and
observations are canied out in reOected light.
Ch. 5. Oscillatioos and W aves "5

Given: 4 3 ,2 =0.5mm=0.5x1.0-3 m, A=5.5x 10-'m,


n=1.
R-1
Solution. From ll.OAB (Fig. 223), we have 1BA I" =
I BO 1' + 1AO1'. or R' = ri + (R - h)', whence
rl - 2Rh + h1 = O. Neglecting h" which is small in
comparison with the other terms, we obtain r-.
=
YJl2ji. On the other hand, the path difference for the
kth bright ring in reOected light is
4,=2kn - W2=2k (W2),
whence 2k= (2k + l)/()J2n). Therefore,

r,=y(2k+tl4"{-.
For k=2 and k=3, we have

r1 =]Í(2x2+1)-*=Ji~,
r,=y(2x3+1) ~ =J/ 1; : .
This gives

.Ó.T3,,,=T3-T2=]Í 1 ::-v ~
whence

R= :.:. '

R ~~~Ys'!'1~~ m=5.7 m.

728. Determine the maximum spectrum order for the


yellow line of sodium with a wavelength of 5.89 X t0- 7 m
if the diffraction grating constant is 2 p.m.
446 Selected Problema ou Physics

Gtven: A= 5.89 x tl)-7 m, d= 2 µm= 2 x 10-- m.


kmu-?
Solution. Using the formula d sin cp = kl. for a diffrac-
tioo grating, we obtain
k=á sin ~/A. (1)
Expressioo (1) shows that for given d and A, the order k
of tbe spectrum will be maximum when sin cp = 1, i.e.
at an angle of deDect.ion q> = 1.57 rad. Consequently,

kma:r.={,

kmas= s.~:~~;1 ~a.


729. At what distance from a diffraet.ion grating
should a screeo be placed to obt.ain a distaoce of 50 mm

-~,
Fig. 224

bet.ween the zero-order maximum and t.be fourth-order


spectrum for light with a waveleogth of 500 nm? The
diffract.ion grating constant is 0.02 mm.
Given: k=4, l=50mm=5X10... m, Ã.=500 nm=
5x10-1 m, d=0.02mm=2x10-s m.
z-1
Solutton. Using t.be formula d sin cp = kA for a diffrac-
tion grat.iog, we obtain
sio ~=k)./á. (1)
Cb. 5. OscillatJons and Waves .W.7

On the oth91' hand, sin cp can be determined from ô.ABC


(Fig. 224) in which the side BC is a part of the sereen lo-
eated at a distanee z = 1AB 1 from the diffraetion grat-
ing. At point B we have the zero-order maximum (un-
deDeeted image), while at point C the image of the fourth-
order speetrum is ohserved:
sinq>=IBCi/IACl=l/Vl'+z'. (2)
Comparing Eqs. (1) and (2), we find that kl.Jd=
l/V zz+r. whenee
z=Jf 11•1•;.~>.az• =-tt-VackW,
z= .. ~~~~' J/(2x10~)Z-42x (5x1Q-7) 2 m~0.5m.
730. Determine the diffraetion angle for the seeond-
ord91' speetrum for light emitted by aodium with a wave-
length of 589 nm if t mm of the diffraetion grating eon-
tains tive lines.
Gi.ven: k=2, 1-589nm=5.89x10""'m,
N 0 =5 mm-t-5x toa m-1.
q>-1
Solutl.on. Using the formula d ain q>=kA for a diffrae-
tion grating, we find that
sin q> = kA/d. (1)
Siilee the numher of lines per unit length of the grating
ia eonneeted with the grating eonatant through the rela-
tion N 0 = t/d, Eq. (1) ean he written in tbe form sin cp =
k'AN 0 , whenee
cp = arcsin k'AN0 ,
cp = aresin 2 x 5.89 x tO-T x 5 x toa rad
~ 5.8 x to-1 rad.
731. Determine the maximum order of the apeetrum
that can be formed hy a diffraction gra.tinr with 500
... Selecf.ed Problema on Physiea

lines in t mm if the wavelength of incident light is 590 nm.


What is the maximum wavelength that can be observed
in the spectrum of the grating?
Gtwn: N 0 =500 mm-1 =5 x 105 m- 1, Ã.=590 nm=
5.9x!CJ-7 m.
km.u.-? ).mu-?
Solu.tton. Using the formula d sin cp = k). for a diffrac-
tion grating. we find that
k ~ dsinq>/~. (!)
Considering that d= i!N. we transform Eq. (1) as fol-
lows:
k~sinq>/("1.N 0 ). (2)
Expresaion (2) shows that for given ). and N 0 , the maxi-
mum order kma:.: of the spectrum can be observed for the
maximum value sin fPma:r. = 1, i.e.

klllu = si~TJ:n: = À1o '


kma:r.= 5.9X1Ó}XSX10' "' 3 •

The maximum wavelength that can be observed by using


this grating is

).mo:r.= ds~:~u = km!ii;No •

).mu= axs1xt0i m=6.67X10-7 m.


732. Water is illuminated by red light. What colour
does a diver see under the water surface?
Answer. The diver sees red light since the colour per-
eeived by the eye is determined by the frequency of oscil-
lations that does not change upon a transition of light to
water and not by the wavelength that changes.
733. While observing a soap film formed in a plane ver-
tical frame, one can notice that interference fringes are
displaced with time downwards. The upper part of the
Ch. 5. Oscillatton11 and Waves 4'9

6.lm gradually becomes black, and the film is ruptured.


Explain tbe phenomenon.
Answer. Water in the film gradually Oows down, as a
result, the lower part of the 6.lm becomes tbicker and the
upper part thinner. The conditions for interference are
different on tbe regiona witb difterent thicknesses, which
leads to the appearance of dark and bright fringes on tbe
surface. As tbe water Dows down, tbe tbickness of vari-
ous regions changes, bringing about tbe changes in tbe
conditions for interference, wbich is manifested in a
downward shift of tbe fringes. When the thickness of the
upper part becomes very small (d~ O), tbe patb diDer-
ence will be 'JJ2. The waves at various parts of tbe film
suppress one another, and the film becomes black.
734. A pinbole camera forms the image of an object
witb the belp of a small orifice. As the diameter of the
ori&ce decreases, tbe sharpness of the image is first im-
proved and then deteriorated. Why?
Anrirer. As long as tbe diameter of the ori&ce is much
larger than the wavelength of the light incident on it,
diDraction can be disregarded, and the image is sharp.
1t can be constructed by using the laws of geometrical
optics. But wben tbe diameter of the ori&ce becomes com-
mensurate with tbe wavelength, diffraction cannot be ne-
glected any longer (i.e. tbe light will propagate in the
region of umbra). The image is blurred.
735. Are the spectra of the Sun, the Moon, planeta,
and stars identical?
Answer. Tbe spectra of the Moon and planets are the
same as the spectrum of the Sun since tbey glow due to
reflected solar rays. The spectra of stars may differ.
736. Wby is red ligbt used for stop signals in traffic?
Answer. Red light has a larger wavelength in compari-
son with light of the other colours constituting the spec-
trum. The rays with a larger wavelength are scattered
by the atmosphere (dust particles and water vapour con-
tained in it) to a smaller extent and bence are seen bet-
ter from a long distance.

29-0970
l50 Selected Problems on Physics

EXERCISES

737. The wavelength of some rays in water is 435 nm.


What is the wavelength of these rays in air?
738. ln the experiment with Fresnel's mirrors, the dis-
tance between the Yirtual images of a light source is
0.5 mm, and the distance to the screen is 5 m. The inter-
ference fringes formed in green light are separated by
5 mm. Determine the wavelength of green Jight.
739. Light from a projection lantern passes through a
small bole covered by a blue glass, faIJs on a screen with
two small boles separated by 1 mm. and then falis on an-
other screen placed 1. 7 m behind lhe lirst. The distance
between the interference fringes on the second screen is
found to be 0.8 mm. Determine the wavelength of blue
light.
740. White light falis on a soap lilm with a refractive
index of 1.33 at an angle of 45º. What must be the mini-
mum thickness of the IHm for the reflected rays to be yel-
low and have a wavelength of 6 x 10-r. cm?
741. Newton's rings are formed between a plane glass
anda lens with a radius of curvature of 8.6 m. Monochro-
matic Jight is incident along the normal to the surface.
lt was found as a result of measurements that the di-
amet.er of the fourth dark ring is 9 mm. Determine the
wavelength of incideot light.
742. Violet light of wavelength 0.45 µm is incident along
the normal to a diffraction grating with a grating con-
stant of 2 l'm. What is the maximum order of the spec-
trum that can be observed by using the grating?
743. A diffraction grating has 500 lines in 1 mm. At
what distance from the central nondecomposed line wiH
the beginning and end of the visible first-order spectrum
be on a screen placed at 2 m from the grating if the grat-
ing is paraBel to the screen and illuminated by light
incident along the normal to its surface?
Cb. 5. Osdllat.ions and Wavea 4SI

iom can
diflraction of lifr t o 7. Deacribe the comtruction of a
diflraction ctmg. 8. Write a formula for a diflraction grating.
~ '!1-.:!lbehradeüie r!~:!:~ :l•:;::. iL 1if.! ~ ~~!,i
speetra YoU know. 12. Describe the cODStruction or a spectroscope •

...
Chapler 6
STRUCTURE OF ATOMS
AND ATOMIC NUCLEI

6.1. Structure oi Aloms


An atom of any element consists of a positively charged
nucleus and electrons moving around it. The total
charge of ali the electrons constituting the atom is equal
to the charge of i ts nucleus.
The simplest atom as regards the structure is the hy-
drogen atom consisting of a nucleus and an electron mov-
ing around it. According to Bohr's fint postulate, it
moves in one of n circular orbits without emitting any
energy. AB the electron goes over from one orbit to an-
other, thehydrogen atom emitsor absorbs an energy quan-
tum (Bohr's second postulate):
hv=W1 -W1 ,
where h is Planck's constant, v the frequency of tbe emit-
ted light, and W1 and W 1 the total energy of the electron
in the atom on the corresponding orbit, which can be cal-
culated by the formula

Wn = - S::~n• •
where e is the electron charge, m the electron mass, e0
the eleclrlc eonstanl, and n = 1, 2, 3, . . . the number of
an electron orbit.
The wavelength ). of light emitted by the hydrogen
atom as a result of an electron transition from one orbit
to another can be delermined from the formula

+=R({-i)•
where R is the Rydberg eonstant.
Ch. 6. Structure of Atom!I and Atomic Nuclei 453
For visible light, n 1 = 2 is the number of the orbit to
which the electron jumps and n 1 = 3, 4, 5, 6 is the num-
ber of the orbit from which the electron jumps.

745. Determine the energy emit.ted by the hydrogen


atom as a result of an electron t.ransition from the third
to the first orbit.
Gt111n: n1 =1, n1 =3.
e-1
Solutton. According to Bohr's second postulat.e, the
energy emitt.ed by the atom as a result of an elect.ron tran-
sition from the third to the first orbit is
e= Wn 1 - Wni• (1)
Here Wna= -e'ml(ShªS:n:) and Wn 1 = -e'ml(8h1 S:n:>
are the energies of the electron on the third and ftrst or-
bi t.s. Substituting tbe expressions for Wn 1 and Wn 1 int.o
Eq. (t), we obt.ain

e= - :~I + :.::JJ1 = ::: (*--;ij-) •


8 x:tZtx•:;2;.~9(~.~ ~~11)1 ( t - ft-) 1
=1.94 X 10-11 J.
746. By what amount has the electron energy in the
hydrogen atom changed upon the emission of a phot.on
with a wavelength of 4.86 X 10-1 m?
GiVen: 1=4.86 x tQ-7 m.
AW-?
Solutton. The change in the energy of the at.om upon
the emission of the photon is AW = hv, or, considering
that v=c/'A.,
AW=~
AW 6.62~.:-:~cAxto• 1=4.4x to-11 J.
... Select.ed Problema OD Pbys.lca

74.7. Calculate the total energy of the eledroo on the


secood orbit of the hydrogeo atom.
Gtven: n-2.
W-?
Solu.tion. The energy of the elect.roo oo the secood orbit.
of the hydrogen atom is

W=-a:'~.,
W= 8X (8.~~~~:;:;:)~~8~~ ~ ~~tl)I X 21
= 5.44 X 10-11 J.
74.8. Determine the maximum aod mioimum wave-
lengths in t.he visible regioo of the emissioo spectrum of
the bydrogen atom.
Given: 11s=2.
- -----
"-uaai:-? Â.iain-?
Solu.tton. The wavelength of light emitt.ed by the hydro-
gen atom as a result of ao electron transition from one
orbit to another is determined from the formula
(1)

The mioimum energy is emitted by the atom upon a trao-


sition of t.he electroo to the secoodorbit from the third or-
bit (n 1 = 3), which is the cloaest to the second orbit.. This
correapoods to the emission of light with the maximum
wavelength ).uau:· Consequently,

Âmax t.097xUJi~(t/ 21 f/S•) m=6.56x 10""' m.


The maximum energy is emitted by the at.om when t.he
eleetroo goes over to t.he secood orbit from ao iofioitely
remote orbit (n. = oo), whieh eonespoods to the emis-
sioo of light with the mioimum waveleogth Â.min· Con-
sequeotly,
Â.m.1n""" t.097 X tCP ~ (t/21 t/oo) m= 3.65 X 10-7 m.
Ch. 8. Structure of Aloms alld Alomic Nuclei 45S

However, this wavelengt.h does not belong t.o the visible


light. region (>..m1n < 400 nm). Therefore, using the condi-
t.ion Am 1n;;;> 400 nm and Eq. (t), we can choose n and .6.nd
that "-min = 4.t X t0- 7 m for n= 6. Thu.a, only four
lines can be seen in the visible region of t.he emission
spect.rum of t.he hydrogen atom.
74.9. Asanelect.ron goesoverfrom some orbit to the sec-
ond orbit, the hydrogen atom emits light with a wave-
length of 4.34 x 10-7 m. Determine t.he number of t.he
unknown orbit.
Given: A=4.34x 1()-7 m, n1 =2.
nl-?
Solution. We shall use the formula for the wavelength
of light emitt.ed by the hydrogen at.om as a result of an
elect.ron t.ransition from one orbit t.o another:

+=R(f-nf)'
whence
.. Y t/nl-t/(AR} '
1
l't121 t1(4.Mx t0- 7 xt.097xU>'} -= 5 ·

750. A beam of light emit.ted by a gas-discharge tube


filled wit.h atomic hydrogen is incident along t.he normal
to a diflraction grating with a grating constant of 5 x
10-" cm. From what orbit must the electron come to the
second orbit for the spectral line in the ftfth-order spec-
trum to be observed at an angle of 4tº?
Gtven: d=5x10-t cm=5x10-•m, k=5, n,=2,
~~41º<><0.72 rad.

n,.-?
Solution. Using t.he formula d sin fl) = kA for a diftrao-
tion grat.ing, we can determine the wavelengt.h of the
emitted light, corresponding to the given spect.ral line:
>..=dsinfl)/k.
... Selected Problems ou Physics

From the formula =R ( f- ,ij--nf} , we can determine


tho number of the unknown orbit:

n11=}"t/nfi~1,'(l.R)' (1)

..
or, using expression (1),
1
t/11111-k·(dsincpR) '

n1i = -=v7 1"12"'•=;•"'1c"'•"'x"'10-"'1•x"o"'


" .658"""x"1".097"""x"1"0'"> """" 3 •
751. The structure of an atom (nucleus + electrons)
resembles the structure of the solar system (the Sun +
planets). What is the difference between them?
Answer. ln the atom, electric forces of attraction act
between the electrons and the nucleus, while in the solar
system, gravitational forces of attraction act between tbe
Sun and the planets.
Besides, the motion of an electron in the atom does not
obey the principies and laws of classical mechanics and
is governed by the laws of quantum mechanics.
752. What is the diflerence between an atom in the
ground state and an atom in ao excited state?
Answer. The main difference between tbese atoms con-
sists in that the electrons in the atom in the ei:cited state
move in orbits which are furtber from tbe nucleus and
possess a higher energy in comparison with the electrons
in tbe atom in tbe ground state.

EXERCISBS

753. When a photon is emitted by tbe hydrogen atom,


the total energy of the a tom changes by 2.56 eV. Deter-
mine the wavelength of the emitted light.
754. What must be tbe m&J:imum energy of electrons
bombarding hydrogen atoms for the emission spec-
trum of tbe bydrogen atoms excited by these electrons
to contain only one spectral line?
755. Calculate tbe total energy of the electron on the
tbird orbit of the hydrogen atom.
Ch. 8. Structure oi Atom! and Atom1c Nuclei 457

756. Determine the wavelength corresponding to the


th~rd spectral line in the visible region of the spectrum
emitted by the hydrogen atom.
757. As the elec troo in the hydrogen a tom goes o ver
from the fourth stationary orbit to the second orbit, the
green line is emitted in the hydrogen spectrum. Deter-
mine tbe wavelength corresponding to the line.
758. The electron in the hydrogen atom can move in
circular orbits of radii 0.5 x 10-1 and 2 x 10- 10 m.
What is the ratio of the angular velocities of the electron
moving in these orbits?
759. The radius of the electron orbit in the hydrogen
atom is 2 x 10- 10 m. What is the wavelength of photons
that may cause the ionization of the atom?
760. What is the frequency of rotation of the electron
in the hydrogen atom if it moves in a circular orbit of ra-
dius 5 x 10-11 m?
761. When mercury vapour is bombarded by electrons,
the energy of a mercury atom increases by 4.9 eV. What
is the wavelength emitted by the atom as a result of the
transition to the ground state?

QUESTIONS FOR REVJSION

~~~"!fi'3:heth~f.!iS:a~? !'d~Í~ 0~.anF::!i:ia~ :'c!::.;9:~ut=:


4. Write a formula for the total energy oi the hydrogen atom.
5. Write a formula for lhe angular velocity oi an electron moving
in a etationary orhit. 8. Derive a formula for lhe radius of a sta-
tionary orbit. 7. What 1s a quantum numher? 8. Write a formula
for calculating the wavelength in the viaihle region of the emiuion
spectrum oi lhe hydrogen atom.

6.2. Structure oi Afomlc Mudei


An atomle nueleus eonsists of nucleons, viz. protons
and neutrons. The number of nueleons in a nueleus is
equal to the mass number A (the atomie mass of an ele-
ment in atomic mass units (amu), rounded to an integer).
The number Z of protons in a nucleus is equal to the atom-
ie number of the element in Mendeleev's Periodic
458 Selected Problems oa Physics

Table and corresponds to the charge of tbe nucleus in the


units of elementary charge (electron charge). Conse-
quently, the number of neutrone io a nucleus is N =
A -Z.
For any element and elementary particle we have a no-
tation: iX, where :p stands for a proton, :n :ci
tron, -~e for an electron, ~e for a positron, and
for a neu-
for an
alpha-particle.
Atoms of the same element may have different numbers
of neutrons in a nucleus. Such atoms are known as lso-
topes of a given element.
The rest mass of a nucleus is smaller than the rest mass
of a neutral a tom by the mass of the electrons constitut-
ing the electron shell of the atom:
Mnuc=Ma-Zm11 ,
where m.. is the electron mass.
The mass defect oi a nucleus is the di B'erence hetween
the sum of the rest masses of the nucleons and the rest
mass of the nucleus:
âm= Zmp +(A - Z) mn - Mnuc•
where mp is the rest mass of a free proton (which is not
bound to the nucleus) and mn the rest mass of a free neu-
tron.
The bfndlng energy of a nucleus 1s deflned as the work
that must be done to split the nucleus of an atom into
lieparate nucleons and to remove them at distances beyond
the range of nuclear forces without imparting a kinetic
energy to them. The binding energy of a nucleus is ex-
pressed by
Wb=âmt2 ,
where âm is the mass defect of the nucleus and e the
velocity of light in vacuum.
When atomic nuclei are bombarded by elementary par-
ticles, atomic nuclei of one element may be tran&formed
into atomic nuclei of another element. The reactions of
transformation of elements (nuclear reactions) are sche-
matically written in the form
~X+ ;a-1:ttY. ~X+ _1ºe - z-1Y.
Ch. 6. Structure of Af.oms and Atomic Nuclei 459

ln these reactions, the law of con.servation of electric


charge and of the number of nucleons must be observed.
The investigation of the properties of atoms Ied to the
discovery of radloactlvlty. Radioactive substances can
emit three types of radiation: a-radiation, P-radiation,
and y-radiation.
Alpha-radlatlon is the Oow of positively charged parti-
cles, viz. doubly ionized helium atoms, beta-radlaHon is
an electron Dow, while gamma-radlatlon is formed by
electromagnetic waves of very short wavelength.
The number of radioactive (nondecayed) atoms de-
creases with time during a radioactive decay according to
the law
N=Noe-M,
where N 0 is the number of radioactive atoms at the ini-
tial instant, À the radloaetlve deeay eonstant, and t the
decay period.
The half-ltfe of a radioactive isotope is de.lined as the
period of time during which half the radioactive substance
existing at the initial moment undergoes a decay:
T,,, - ln 2/A - 0.693/A.

762. What is the structure of the nucleus of the lithi-


um isotope ~Li?
Given: !Li.
----
Z-1 N-1
Solu.tion. It follows from the notation for the lithium
isotope ;Li that its nucleus consists of seven nucleons
(A = 7): three protons (Z = 3) and four neutrons (N =
7 - 3 -4).
763- What is the difterence between the nuclei of the
nitrogen isotopes ·~N and 1 ~N?
Gl.ven: •:N, •:N.
Z-1 N-1
Solution. The notation for tbe nitrogen isotopes '!N
and ·~ shows that the number of protons in tbeir nuclei
... Selected Problems OD Phyaies

is the same and equal to seven (Z = 7), and the number of


neutrons is respectively N 1 '""' 14 - 7 = 7 and N 2 =
15 - 7 = 8. Consequently, tbe nuclei of the isotopes
differ in the number of neutrons in them.
764. Calculate the mass defect of the nucleus of the
isotope ::Ne.
Given: ::Ne, m,..=1.6724x1.0-2.T kg, mn=1.6748x
10-17 kg, Mnuc = 33.1888 X to-u kg.
ll.m-?
Solu.tion. The mass defect of a nucleus is
Am= Zm,.. +(A - Z)mn - M 111 uc• (1)
Tbe notation of the element ::Ne shows that A = 20
and Z= 10. Then Eq. (1) can be written in the form
Am= 10mp + (20 - 10)m11 - Mnuc
+
= 10 (m,.. mn) - Mauc·
ll.m = (10 X (1.6724 X 10-n + 1.6748 X 10-")
- 33.1888 X 10-"] kg = 2.882 X 10-u kg.
765. Determine the binding energy of the nucleus of
the lithium isotope :u.
Given: ;Li, m,..= t.6724 X f()-27 kg, mn= 1.6748 X
10-Z7 kg, Mauc= 11.6475 X 10-:r.7 kg.
Wb-1
Solution. The binding energy of a nucleus is given by
w,-11.mc•. (1)
Since Am= Zmp + (A - Z) mn. - Mnuc• Eq. (1) c1.n
be written in the form
Wb= (Zmp +(A - Z) mn. -Maual c2.
It follows from tbe nota.tion for the lithium isotope:Li
that A = 7 and Z = 3. Suhstituting lhe values of A
and Z into Eq. (1), we obtain
Wb= (3mp + 4mn. - Maucl c1 ,
W,= (3 X 1.6724 X 10-n + 4 X 1.6748 X 10-"
- 11.6475 X 10-H) (3X 10') 1 1=6.201 X 10-" 1.
Ch. 8. Structure of Atoma and Atomic Nuclei '81

766. An unknown element and a proton are formed as a


reault of the capture of an a-particle by the nucleus of the
nitrogen isotope 1 :N. Write the nuclear reaction equation
and. ide~~ify 1~he ~nknown element.
Given. 1a, ,N, 1p.
1X-?
Solutlon. We write the nuclear reaction equation
'!N+~a-:p+1x. (1)
Since the sums of the mass numbers and charges on the
right- and left-hand sides of Eq. (1) must be equal, we
have 14+4=1 +A and 7+2=1 + Z, whence
A= 17 and Z = 8. Consequently, the obtained element
can be presented as '!X. From Mendeleev's Periodic
Table, we 6.nd that it is the oxygen isotope 1 :0.
767. As a result of the capture of a neutron by the nu-
cleus of the isotope 1 ~N, an unknown element and an
a-particle are formed. Write the nuclear reaction equa-
tion and identify the unknown element.
Given: ~n. 1:N, :U·
1X-?
Solutton. We write the nuclear reaction equation
1
:N+!n-1X+:U·
According to the law of conservation of masa number and
charge, we have 14+1 =Â +4 and 7 +O=Z+2,
whence A= 11 and Z = 5. Consequently, we can write
the unknown element in the form •:x.
Using Mendele-
ev's Periodic Table, we find that it is the boron isotope
·:e.
siJ:S~f ~~:~~~~:::s~ ~!u~ali~~e d:;•~dh;~hi!hf 4~8t
Calculate the number of atoms decomposed in 1 mg of
the given radioactive sample during to h.
Given: :!Na, T11z=14.8 h~5.33x10' s, t=to h=
3.6xlO's, m=I mg=IO-Okg.
IJ.N-?
•62 Selected Prohlem1 on Physic1

Solution. The number of atoms that have decayed


during the time t is
IJ.N - N, -N, (1)
where N 0 is the number of undecayed atoms at the initial
instant, equal to thenumber of ali atoms in t mg of ~!Na,
and N the number of undecayed atoms during the time
t.Since N= N 0 e-u, Eq. (1) can be written in the form
flN=N 0 -N0 e-U=N0 (1-e-U). (2)
Considering that À= ln 2/T112 , we transform expression
(2) as follows:
llN=N0 (1-e- 1" 21 fT1/t)= N 0 [1-(eln 2)-l/T1ltJ

-N,(1-2-lfT, 1 ~. (3)
Since the number of atoms contained in a mole of :!Na
is equal to Avogadro's constant NA• the given mass m
contains the number N 0 of atoms equal to the product of
the number m/M of moles and Avogadro's constant:
N,-mN.IM, (4)
where M is the molar mass of sodium. Subst.ituting ex-
pression (4) into (3), we obtain
õ.N =-i-NA (t-2-t/T1/1),

õ.N= 24~:r, x6.02x t02ªx(t-2-ux10'11$.33xto'1)


~9.3x 1011 •
769. Determine the half-life of radon if Crom 10' atoms
175 000 atoms decay during 24 hours.
Given: t=i day=8.64x10' s, N0 =10',
IJ.N-1.75x 10'.
T11z-?
Solu.tion. The half-life of radon is
T 11, - 0.693/A. (1)
The radioactive decay coostaot A can be determined from
the relation /!J.N = N 0 (t - e-11) (see Problem 768),
Ch. 6. Structure of Atoms anel Atomic Nuclei 463
wbence
"-= tl~e log N0 1!.._0AN · (2)
Substituting e:ipression (2) into (1), we obtain

T111 log ~!~(~:'~) ~N)) •

T111 =- i~~~(~Õ~ax~-.~~~';ll s~3.3x10 5 s.


770. Why cannot a-particles emitted by radioactive
samples initiate nuclear reactions in heavy elements?
Answer. The energy of such a-particles is insuflicient
to overcome the repulsive electric forces exerted by the
nucleus of a heavy element to approach it to a distance
of 10-u m starting from which lhe forces of interaction
between nucleons considerahly exceed the force of elec-
trostatic interaction.
771. What is lhe difference in the composition ofatom-
ic nuclei of radioactive elements and ordinary atomic
nuclei?
Answer. The number of neutrons in atomic nuclei of
radioactive elements is considerahly larger than the num-
ber of protons in ordinary atomic nuclei. For exemple,
the nuc1eus of the uranium isotope S::U contains 146
neutrons and 92 protons.

EXERCISES

772. Describe the structure of the nucleus of the potas-


sium isotope ::K.
773. What is the difference between the nuclei of the
oxygen isotopes •:o. '!0, and 1:01
774. What minimum energy must be spent to split the
nucleus of lhe helium isotop:J ~He?
775. Determine the mass defect of the nucleus of the
hydrogen isotope :H.
776. How are the mass number and the atomic number
of an element change upon a proton radioactive decay?
777. Determine the reaction product for lhe bombard-
ment of tbe nuclei of the magnesium isotope :!_Mg by
... Selected Problems ou Physica

a-particles if oeutrons are koown to be liberated in the


nuclear reaction.
778. Write the nuclear react.ion equation and identify
the unknown element formed as a result of t.be bombard-
ment of nuclei of the aluminium isotope ~:AI by a-parti-
cles if one of the reaction products is neut.ron.
779. How many nuclei of the iodine isotope 1 :~1 from
109 nuclei decay per second?
780. The initial mass of a radioactive isotope dec.reases
in 8 b to one-third. ln what proportion will it. decrease
in 24 h from the initial instant?
781. Explain wby electrons are emittea by an atomic
nucleus as a result. of P-decay.

QUESTIONS FOR REVISION


APPENDICES

l. Dariv.Lives of Some Functions

Functlon Derlvatlve Punctlon Derlv1.tlve

....
........ ...........
e=const o sin:r

...
nzn-l -sin.:r
a"lna 1
õõiiZ
.!. 1
-lliD1 .:r

2. lnde.6.nite Integrais of Some Functiona

Punctlon Jntqnl Functlon Inutrat

.. ::~+e
sin.:r -coaz+C

.!.
:.
lnl•l+C
e<+C
........ sin:r+c
-In/coa:rl+C
lnlsiDll'l+C

3. SI Unit.s of Physical Quanti1.ies


untt

'-.
Ouantlt:r
notatlon

Bau unlt1
Leogth m m
MH• kll...... kr kg
... Appendices

(Tabk 1 conUnu~

1-
Unlt

.
Qu1ntlty
notatlon dlmenalom

Current
T;m ampere 'A A
Temperature kelvin K K
Luminous intensity candeia od od
Amount oi sub- mole mol mol
stance
Supplm&tntary unH1
~lidª ::i~e /::::ian 1
,..i 1 dimensionless
d imensionless

.
Dtnuttl 1&111t1
AN• square metre m• m•
vo1- cUbic metre m• m'
Period "'°"d
Frequency of pe- hertz '
H• .-•
riõdic process
Rotational
quoncy
Ire- 98CODd inverse .-• .-•
m-•.kg
Density
ki~~~m::e kg/m•
Veloeity metre persecond mi• m-s-'
Acceleration metre per second m/s1 m·s-•
squaNd
newkln N m·kg·s--s
'º""
PreMure pascal Pa m-1 ·kg·s-s
W:~~UD~~yheat Joule 1 m1 ·kg·s-•
Power watt w m1 ·ke·r'
Eleetric charp coulomb e "A
(quantityofelec-
tricity)
V m•.kg·r'·A-1
El:::.~ial YoJ:~ volt
ence, emf
Electric field volt per metre V/m m·kg·S-1 ·A-1

.....
strengtb
Electric capaci- faNd
Electric resistance ohm
Magnetic Dux weher
g
Wb
m-•.q.-•.s•·A1
m1 ·ke·s""'·A-•
m'·kg·r'·A-1
MagneUc induction tesl• T ke·ir'·A-1
Maguetic &eld ampere per Alm m-1 .A
strengt.h malN
Inductance hao" H m-1 ·Jre·r'·A-t
Appeadices 467

Tabü 8 (conUniud)

Qaantltr

Lum.ioous Dus.
lllwninance
L,..Pl>ft'

4. Approximate Values of Some Fundamental Physical Constants

Phl81c•1 conatant Numerlcal ·nlue

Earlh's radius R 6.37X10' m


Earlh's mau M 5.97 X i()H kg
9.8 m/s'
s:::!i=:-i'ª:!!:!'ration ~ 6.67X10-11 m•t(kg·s')
Avo1adro's constaat NA 6.02 X 10U mol-1
~h!ªBo~m':1:!.&ant f i:~~'I~J-~JK
Volume of a mole of a ias un- V1 22 .41 m 1/mol
der aormal coad.itions
Elemeatary charge 1.6X10-1º C
~ ::~~1~~i:o.
Electro11 IQU8
Faraday'• coqataqt
3x10 mJs
~r.:!k~! ~!a~t vacuum. 1

The Rydherg coqatant •' 6.62x10-" l·s


R 1.097 X 10' arl
Proton lllU8 mp t.6724 X 1Q-17 q.
Neutron mass mn 1.6748 X 10-" kg

5. Dellsity of Stme Suhataqcea, 1QI kgtm•


(p=1.ot X10I Pa, T=273K)
Solld• L'q•ltll
Alumiai um.
"'......
2.7 Alcohol 0.8
Brick 1.8 0.8
Copper 8.9 Mercury 13.6

·-
loe 0.9 011 0.9
lron 7.8 Petrol 0.7
Leod U.3 Water
Nickel 8.8 1.0
1.03
'88 Appendice11

ª""'
Silver 10.5
Gan•
Air :t.29x:to-•
Stool 7.8 Helium o.tsxt.o-•
Wood 0.8 Hyd- 0.089x to-a
O:r.1gen t.43xto-•

6. Longitudinal Elastlc Modulua, 1011 P•


Aluminium 0.7 Iron 2.t
Brass 0.9 Le•d 0.:1.7
Copper :1..2 Steel 2.2

7. ~tN,~ension of Some Liquids at Room Temperatura,


Alcohol 2.2 Mercury 47.t
Aniline 4.3 Soap solution 4..0
Kerosene 3.6 Water 7.4

s. Speci.8.c Heat, toa J/(kg•K)


Air
Alcohol
1.005
2.42
......
Lwl º·"'
0.13
Aluminium 0.88 Nitrogen 1.05
Brau
~~
0.38 0.92
Carhon dioxide 0.83 0.46
0.23
~$'.,:....
0.38 Tln
14.20 Water 4.:1.9
Ice 2.10
The 1pecilic beata for gases are gi'rl!n at constant voluma.

9. Molar Mus of Some GUH, t0-1 kg/mol


Air
Carhon dioxide 001
:;~:mHeA.
..•
29

2
~il::aºA.
Water vapour HsO
32
28
18

iO. Melting Point for Some Solids, K


Alumiai um 933 lron 1803
Brasa H73 Lwl 800
Copper :1.356 Silver 1233
Ice 273 Tm 505
Appendicea ...
H. Laten\ Heat of MeUing, 10' J/kg

AlumiDium 3.90 Load 0.25


Coppor 1.80 Sil't'er !.OI
loe 3.35 TID 0.58

12. Temperature of Vaporizatiou, K


(p=t.OtxtOI Pa)

Alcohol 351 Mercury 630


Ether 308 Water 373

13. Latent Hee.t of Vaporization, tOt J/kg

Alcohol 9.05 Mercury 2.82


Ether 3.68 Water 22.60

H.. Heat of Ccmbuatiou, 107 J/kg

Alcohol 2 .93 Petrol 4..61


Coal 2.93 Petroleum 4.61
Keroeene 4..61 Wood 1.26

15. Coefficient of Linear E:r.paneion for Some Solide, t()-1 K- 1

Aluminiam 2.40 ID't'ar O.t5


Brus 1.90 Irou t.:1:1
Copper 1 .70 Lead 2.90
Glau 0.90 Steel t.tO

16. Coef&cient of Volume E:r.pansiou for Some Liquida, 1~ K-.1

Alcohol H.O Petroleum rn.o


Keroaene rn.o Sulphuric acid 5 .6
Mercury t.8 Water t.8
•70 Appeadice1

17. Deuity of Satunted Water Vapour at Various Tem.pen.tum

T, K 1p,to-•-1 T, K

283 2.tol 283 9.40


28ol 2.33 28ol 10.00
265 2.Sol 285 t0.70
266 2.76 288 H.40
287 2.99 287 t2.t0
288 3.24 288 t2.80
289 3.51 289 13.60
270 3.81 290 H.50
271 4.13 291 15.40
272 6.47 292 18.30
273
274
u•
5.:!0
293
294
17.30
18.30
275 5,80 295 19.40
276 6.00 296 :!0.80
277 6.40 297 21.80
276 6.80 296 23.00
279 7.30 299 24.40
280 7.80 300 25.80
281 8.30 30I 27.:!0
282 8.80 302 28.70

18. Relative Permi.Uivity

Ebonite 3.0 Mica 7.0


Glus 7.0 Parafline 2.0
GlyceriDe 39.t Wate< 81.0
Keroeene 2.0

p, tO-TD·m a., 101 X-1

....
Alumini.um
"""""
0.28
O.t'1
1.l!O
3.8
•.2
6.0
Appendices 471

Tab~ 19 (eoraUrarud)

p, IO-'lil·m

Leod 2.tO 4.3


u.o
Nichrome
Silver
1'uDgMen
O.te
0.55
º·'
3.8
5.2

20. Abeolute ftelract.ive Index


Alr 1.00029 Ice 1.31
Alcohol 1 .38 Quart.z 1.54
Dlamond 2.42 Turpeo.Une 1 .47
Glass t.50 Water t.33

2t. Work Fuact.iOD for Some Metais, tO-U J

Cesium 3.2 Pot.asslum 3.2


LUhium 3.8 Tunpteo. 7.2
Plat.iaum. 8.5 Zlnc 8.8

22. Basic Properties oi Some Elemantary Particles

Par\lcle Symbol 1Cbarge,10-u e1 M-.10-11q

Alpba-parLlclo
Elict.roo.
I ...
-1•
3.2
-1.8
8."'8
0.0009U
Neut.roa
.......
Poait.roo. r, o
1.8
1.8
1.8748
0.0009t1
t.8724
IP 1

23. MU!I oi t.he Naclei. oi Some Llght. Isotopes, tO-•' kg

~H
1 He
t .8728 ~ 18.2787
3.3438 N 23.2481

~iie 8.6"8
9.9855
U.8475 Ne
28.55Zl
28.2202
33.t888
472 Appendlce•

24. Approximate Calculat.iona

ima~bi~u:!;::!\ ~.~~~ ~c~~~~C:· :::!&enco~:l.:'~tb .~:fro!;


r = 9.8 mls1 •
Modem students can use various calculaton which give a large
number of signilicant digits. A student has to decide ahout the num-
ber of diEl'its tbat must be retained in calculations and disregard
lhe remaioing digits. Tbe rules for appro:r.imate calculatiODS are
A"iven below.
t. ln addition and subtraction, the reeult is rounded so tbat it
does not contain signilicant digits that are not present at leut in
one of the given quantities.
E i:ample. 3.351 + 2.45 + t .2534 ::::- 7 .05.

e::.
2. ln multitication, the facton are rounded so that each of
~=iºJi: mi~'::!!: ~~:e;&:;~~ dii~:Ue:id~I i~. tbat in
Example. 2.St X t.2 X 5.245 ~ t.2 X ~2.
2.?x
the

The li.Dai result must contain tbe same number ol signilicant


digits as that in the rounded factore.
3. ln division, the same rule as in multiplication should be
observed.
Eumple. 6.24+2.t24 ~ 6.24+2.t2.
4. Wliile raising to the second ~or tbird) power, the result mus!
con~:!:m~ie~ur~r ~f t~~Ificant igits equal to that io the bue.
5. Wliile e.ztracting a square (or cube) root, tbe result muet con-
tain tbe same numher of signilicant digits as tbat in tbe radicand.
Example. Jf8.82 ~ 2.61.
~~=- rules must also be applied wbile calculating compln n-
ANSWERS TO PROBLEMS

.f., 111 =8 m/s. 5. s=-t.5X101 m, 11=4.5X101 m.


6. z = t9 x to• m, 11 = 2 x 10' m. 7. S = 5 m, Sz = 4 m, s, =
- 3 m. 30. 11 ~ 0.05 m/s1 , e ~ 53 a. 31. i = 20 a. 32. a= 0.8 m/a1 ,
8 10 = 7.6 m. 33. a= 1.6 miai, 110 = 5 mla. 34. 111 19.8 mia, =
i = 3 s, u, = 9.9 mia. 35. t ~ 0.5 a. 36. {11) ~ 8.2 mia. 37, {11):::::
16.7 m/s. 38. S - -200 m, l = 2.2 X tOS m. 39. a= 2 m/al,
(111) = 20 m/a, {~ = 28.6 mia, {v) = 23.5 m/a. 40. h. = 20 m,

~: kf:1'inia. 1 ~. ~1:·211 ""aJ~." 6s6.1'~· ~54io ':i~s~~~


twofold. 58. 1 ~ 8.7 X IQI m, I !:!:! 101 a. 59. 11 = 1'2i"A + i;t.
60. 1!::!!! 5.t m. 61. a= arctan {f-i). 62. 1 ~6.3 m.
63.u~tt.5m/s.64.e~t7.3xt0 1 s.75. l=0.37 m. 76. 11~
7.6 x toa m/a. 77. (1) ~ 7.2 x 10-1 radia, 11 ~ 3.25 x 101 m/s.
18. 11 ~ 32 m/1. 79. N = 90 rev, e ~ 0.14 radia•. 80. e ~
3.2 radia•. St. 1 = to a. 82. R ~ 6.t m.

180. N = 123 kW. 181. 110 ~ 7.5 m/a. 182. (F) = 12 kN.
~8:'3%"."-fa8. 51i ~ª2.a 1 ':: "187.20• :·o~&:·~~ lJ~·:~ r]•:_
189. Al = 0.03 m. 190. Wn = 50 J.
Answen to Problema

~·{/'xiõ.!2 ~~- :: l ==.1·~ s':6ºx~6..:, Hkg=: 2!~80!mio.:2~


10-1 kg. ZZ9. p = 0.75 X 101 ~/m•. 230. Yc = 4.2 X 10-• m•.
131. p = 0.75 X U)I kg/m1 •

~·k~~ =a:_ ~·O!Qb=-:t ~.T~~~kJ.-J! ~:, k~Ai ~


485kJ.304.. 1J = 20%, A = t.26.kJ. 305. A = -j; R (T1Ttl ln ~.
306. Ã = 48 kJ. 307. Ã = 136 J.

317. p = 5 x 10_. kg/m1 . 318. p = 10.24 x 10-1 Q/m•.


319. m O! 2.8 X 10.... kg. 320/
10.... kg. 312. B O! 58.7'(. 323.
O! 99.7 kPa. 321. m = 1.'16 X
O! 3.3 X 10". 334.. V= 0.1 m1•
32$. 111 = 41.Z kg.
Answers lo Pftlblems

59t. :1& = 2 m. 592. In&nitely large number. 593. 'P = 2.9 X


fO-fo rad. 594. F ~ O.t m. 595. F = 0.25 m. 596. :1& = t.26 m.
597. :1& = O..f.5 m. 598. R ""' 2 m. 599. :1& = t.05 m (bom the con-
eave mirror). 629. li = 2.66 m. 839. r = 0.82 rad, i. = 1.tS rad.
631. r 1 ~ O.tt rad, 6 ~ O.M rad. 632. / = o.e m, virtual image.
.,. Answers to Problems

833. F = O.t6 m. 635. D= 0.42 D. 637. F""' 6.4 m. 638. D=


t.25 D. 639. '=
50 •. MO. '""' 5 X to-• m. 64.t. r = t87.5.
612. 1 Ã1B1 1 = 9 X 10--a m.

688. :r = 2.5 x w-1 m, v""' O.tt m/s, a= - 0.17 m/s1 •


~:: :1o~ ~09 N~ 'f.s5
;·~t?· m67:.' t1 _ ; ~4fa~J '; [~!~· ::,;
69t. 11 = O.dlrm/s. 692. :r = -i.5 X têr-
m. 693. /U = 3.46 X
tO' s. 894. T ~ 0.2 s. 895. :r = 3.7 X to-1 ein (0.785t +
0.392).
696. :r1 /4 +gl/4 = t. 697. A ~ 5.8 em. 698. :r = O. 899. Aqi =
1.57 rad. 700. v=3'8 Hz.

=
7t4. 'Y = 6t Hz. 715. lmu = t A. 7t6. l'en 50 V, S = 70 V.
717. U = 2t.5 V. 718. T = 6.28 X 10-1 s. 719. T = 10-• s.
720. T ~ 4. 7 X 10.... s. 'TJI. Fmm 2.3 to 7 .t kHz. 722. v = 7t kHz.
723. Â = 200 m.

737. 1 = 5.79 X 10-1 m. 738. 1 = 5 X 10-1 m. 739. Â = 4.7 X


~2;: ::_ 17~='\.~_=;1 1,.! 0~4 t~~' 11m,;., i~-m~ = 5.89 x 10-1 m.

753. ). = 4.86 X to- 7 m. 7M. W = t.9' X 10-tt J. 755, W =


-2.42 X to-11 J, 756. 1=4.MX10-1 m. 757, ). = 4.88 X
10-7 m. 758. 0> 1/w1 = 8. 759. Â = 3.45 x 10-1 m. 780, 'Y =
3.6 X t()I• Hz. 76t. Â = 25.3 X 10-1 m.

772. A = 39, Z = t9. 773. ln the number of neutmns.


774. W ~ 4.53 X 10-11 J. 775. àm ~ 2.44. X to-li kg. 776,
àA=t, àZ=t. 777. flSi. 778. ttP. 779. àN = t()I. 780. n = 27.
TO THE READER

Mir Publishers would be grateful for your


commenta on the content, tranalation and
design of thia book. We wouLi also be pleued
to rec:eive any other suggestions you may
wiah to make.
Our address ia:
Mir Publbhers
2 Pervy Rizhaky Pereulok
1-tto, GSP, Moteow, t29820
USSR
AUIO FROM MIR PUBLISBERS

N. 1. Goldfarb
Physlcs Problems and Questiona

This is a colleetion of questione and problems covering


ali the sections in the curriculum for entrance exam-
inations to university levei educational establishments
with a strong emphasis on physics.
The collection contains many interesting and original
problems, which were set in exams for a variety of techni-
cal and physics institutes.
The eompilation is aimed at those preparing to enter
university or college, or about to take part in problem
solving competitions, and for physics lecturers.
ISBN &«l-001785-2
Elementary Textbook on Physies
Edited by G.S. Landsberg

This three-volume edition forms a course on elementa-


ry physics that has become very popular in the Soviet
Union. Each section was written by an authority in the
appropriate &eld, while the overall unil.y and editiog was
supervised by Academician G.S. Landsberg (1890-1957).
A feature of this course is the relatively small number-
of formulas and mathematical manipulations. Instead,
~!~~!~! ;~~ f:!U:~oa.=b~~: 1:::Kch~~:! !!dn:f!':
understandable to sehool children. Another aspeict of
the text is the technological application of the physical
laws.
These features make the text a world-class textbook.
For students preparing to enter universities and col-
leges, and for those at highschoolsspecializing in physics.
A.I. Buzdin, et ai.
Aptitude Test Problems in Physics

This collection contains about 250 problems in physics


:set for boys and girls from Moscow schools participating
in physics olympiads from 1968 to 1985. The most inter-
esting and original problems from these examinatiom have
been included togetheJ.' with their solution.
lntended for schoolchildren and teachers of phys1cs,
this book can also be used for preparation to entrance ex-
aminations to the higher educational institutions, where
ao advanced knowledge of physics is required.
ISBN 5-03-001488-3

Das könnte Ihnen auch gefallen